You are on page 1of 252

Unit 1

1)
0/1
You are seeing an otherwise-healthy 28-year-old man who is presenting with a 4-month
history of epigastric discomfort and heartburn symptoms that are usually exacerbated
after meals, especially after eating spicy foods. He denies dysphagia, weight loss, or
decreased appetite. He has an active lifestyle and takes no medications. His vital signs or
normal and physical examination is normal except for mild epigastric tenderness to deep
palpation. Routine laboratory studies are normal including liver function studies and
lipase. Bedside ultrasound shows no evidence of gall stones. What is most appropriate
treatment at this time?

(Select 1)( 1pts extra credit)

Upper Endoscopy

Esophageal manometry

Ambulatory 24-hour esophageal pH monitoring

Trial of acid-suppression therapy

None of the above

The answer is D, Typical GERD symptoms include chest discomfort (heartburn) and
regurgitation. Symptoms occur most often after meals, especially fatty meals. Lying
down, bending, or physical exertion often aggravate symptoms, and antacids provide
relief. Patients with classic symptoms rarely require testing to confirm the diagnosis
because of the high positive predictive value of classic symptoms (1). When heartburn
(89% specificity, 81% positive predictive value) and regurgitation (95% specificity, 57%
positive predictive value) occur together, a physician can diagnose GERD with greater
than 90% accuracy.
Performing diagnostic tests for all patients presenting with symptoms that might indicate
GERD would be costly and is not necessary to arrive at a sufficiently accurate diagnosis.
Response to an empirical trial of acid-suppression therapy is considered a sufficiently
sensitive and specific method for establishing a GERD diagnosis among patients with
classic symptoms of heartburn or regurgitation. read more
Feedback on your answer
Collapse
2)
0/1
You are consulted to see a 22-year-old black female in the ER who has a history of sickle
cell anemia. She presented with worsening lower back and leg pains along with some
shortness of breath that has progressively worsened since she was discharged 5 days ago
following an RBC transfusion ( 2 units of matched RBCs) for anemia following a prior
sickle cell crisis 1 week ago. Her current medications are hydroxyurea and folic acid.
On physical examination, she is in obvious distress and appears jaundiced. Her oral
temperature is 36.5°C, blood pressure is 125/80 mm Hg, pulse rate is 110/min, and
respiration rate is 22/min. Chest X-ray is shows normal lung fields and cardiac size. Her
labs are below. Which of the following laboratory findings would best explain this
patient's current clinical presentation?
Laboratory studies:

Current value Values at hospital discharge


Hemoglobin 7.0 g/dL 10.0 g/dL

Leukocyte count 14,000/µL 7000/µL

Platelet count 170,000/µL 200,000/µL

Reticulocyte count 2.1% of erythrocytes 5.4% of erythrocytes

Bilirubin, total 5.2 mg/dL Not ordered

Bilirubin, direct 0.4 mg/dL Not ordered

(Select 1)( 1pts extra credit)

Antineutrophil antibodies

HLA antibodies

IgA deficiency

New alloantibodies

The answer is D, Delayed hemolytic transfusion reactions (DHTR) are potentially life-
threatening complications observed in patients with sickle cell disease. The clinical
presentation of a DHTR in SCD may be quite similar to that of a sickle cell pain crisis,
with or without features of an “aplastic crisis. The manifestations of acute or delayed
hemolysis, patients typically exhibit symptoms of a pain crisis, marked reticulocytopenia
(a decrease from the patient’s usual absolute reticulocyte count), and may develop a more
severe anemia following transfusion than was present before. is almost always a result of
complement-mediated intravascular hemolysis caused by preformed antibodies in the
recipient's plasma to the donor's red blood cells (RBCs). This medical emergency results
from the rapid destruction of donor RBCs by preformed recipient antibodies, usually anti-
A or anti-B but occasionally anti-Rh or anti-Jka, capable of fixing complement. Rapid
intravascular hemolysis may lead to disseminated intravascular coagulation (DIC), shock,
and acute renal failure due to acute tubular necrosis.
The presence of antibodies against recipient neutrophils present in donor plasma is
known to cause transfusion-related acute lung injury (TRALI), which may mimic
noncardiogenic pulmonary edema, including radiographic evidence of pulmonary edema
and pulmonary infiltrates. Patients may also have fever and hypotension. This
constellation of symptoms and findings is not consistent with this patient's presentation,
which is most characteristic of a pain crisis. In addition, TRALI occurs during or soon
after a transfusion, and this patient's findings were delayed several days after the
transfusion.
Platelet refractoriness is an inappropriately low increment in the platelet count following
a transfusion, generally defined as an increment of less than 10,000/µL (10 × 109/L).
HLA alloimmunization can cause platelet refractoriness, but this patient did not receive
platelets, and HLA alloimmunization would not explain her current findings. Read
more here or more from Up to Date
Feedback on your answer
Collapse
3)
0/1
You are seeing a 45-year old male who presents with acute abdominal pain following a
high speed motor vehicle collision. On exam he has a seat belt mark on his abdomen.
How much more likely does having a seat belt sign on physical exam increase
the likelihood of having an underlying abdominal injury?

Image gallery (click for full size)

(Select 1)( 1pts extra credit)

Doesn't change the likelihood at all

1 x higher

5-10 x higher
10-20 x higher

The answer is B, 5x higher. This sign increases the likelihood of having some intra-
abdominal injury .
According to a systematic review of 12 studies involving 10,757 patients, the physical
examination findings most strongly associated with intra-abdominal injury following
BAT are the following [1]:
●Seat belt sign (likelihood ratio (LR) range 5.6 to 9.9)
●Rebound tenderness (uncommon but substantially increases risk when present; LR 6.5,
95% CI 1.8-24)
●Hypotension (defined as SBP <90 mmHg; LR 5.2, 95% CI 3.5-7.5)
●Abdominal distension (LR 3.8, 95% CI 1.9-7.6)
●Abdominal guarding (LR 3.7, 95% CI 2.3-5.9)
●Concomitant femur fracture (LR 2.9, 95% CI 2.1-4.1) (femur fractures are significant
distracting injuries, and may indicate BAT among pedestrians stuck by
automobiles). read more

Feedback on your answer


Collapse
4)
0/1
You are seeing a 43-year old black female who presents with right upper quadrant pain.
You are suspecting gallstones but your ultrasound and CT machines are not operating
right now. The only imaging you can do are plain films. What percentage of Cholesterol
gallstones are visible on plain radiographs?

(Select 1)( 1pts extra credit)

<2%

3%-5%

10%-30%

25%-50%
The answer is C. Plain radiographs have little role in the diagnosis of gallstones or
gallbladder disease. Cholesterol and pigment stones are radiopaque and visible on
radiographs in only 10%-30% of instances, depending on their extent of calcification.
For more on the work-up of a patient with gallstones, read here.

Feedback on your answer


Collapse
5)
0/1
A 48-year old white female presents with sudden onset of abdominal pain that radiates
to her back. She appears in moderate distress and is vomiting up coffee ground material
upon exam. She denies any past medical history and denies the use of alcohol or drugs.
She takes over the counter ibuprofen for arthritis in her knees.

On exam she is diffusely tender in both upper quadrants and is guarding. Her chest xray
is shown below. Her labs show a normal lipase and slightly elevated amylase. Her NG
lavage showed clear fluid on aspirate without blood or bile. What is this patient's
definitive treatment?

(Select 1)( 1pts extra credit)

Admit, keep NPO and put on IV Proton Pump Inhibitor and IV Octreotide
To the OR for closure with a piece of omentum (Graham patch) for a
perforated gastric ulcer

Upper Endoscopy to cauterize the bleeding ulcer

To the OR for closure with a piece of omentum (Graham patch) for a


perforated duodenal ulcer

The answer is D. She has a duodenal ulcer that has perforated, the tip off in this scenario
is that the NG lavage should no blood or bile. Also the xray shows free air. There are
some non operative treatment pathways for patients but in this case our patient needs
surgery. Perforation occurs in 2–10% of patients with PUD and accounts for more than
70% of deaths associated with PUD. Perforation is often the first clinical presentation of
PUD. The incidence of duodenal perforation is 7–10 cases/100,000 adults per year . The
perforation site usually involves the anterior wall of the duodenum (60%), although it
might occur in antral (20%) and lesser-curvature gastric ulcers (20%) . Duodenal ulcer is
the predominant lesion of the western population, whereas gastric ulcers are more
frequent in oriental countries, particularly in Japan. Gastric ulcers have a higher
associated mortality and a greater morbidity resulting from hemorrhage, perforation and
obstruction . PPU used to be a disorder mainly of younger patients (predominantly
males), but recently the age of PPU patients is increasing (predominantly females). The
current peak age is 40–60 years . The need for surgery for PPU has remained stable or
even increased and the mortality of peptic ulcer surgery has not decreased since the
introduction of H2 receptor antagonists and peptic ulcers are still responsible for about
20,000–30,000 deaths per year in Europe . This may be due to an increase in use of
aspirin and/or NSAIDs . One can say that nonoperative treatment is limited to patients
<70 years of age who are not eligible for surgical repair due to associated morbidity, with
documented contrast studies showing that the perforation has sealed completely. When
the patient is in shock or when the time point between perforation and ‘start of treatment’
is >12 h, simple closure should be the first treatment of choice. Perforated Peptic Ulcer
Disease: A Review of History and Treatment

Feedback on your answer


Collapse
Unit 2
6)
0/1
Which of the following is not an example of referred pain?
(Select 1)(1pts)

Ureteral stone causes ipsilateral testicular pain


Ruptured hemorrhagic cyst causing lower back pain

Acute MI causing jaw pain

Acute Appendicitis causing periumbilical pain

All of the above

None of the above

The answer is D, as Acute Appendicitis causing periumbilical pain is an example of


visceral pain.
Visceral Pain
•Caused by the stretching of fibers innervating the walls or capsules of hollow or solid
organs
•A steady ache or vague discomfort to excruciating or colicky
•Visceral afferents follow a segmental distribution and pain is localized by the sensory
cortex to an approximate spinal cord level determined by embryological organ of the
organ involved.
•Biliary, duodenum, stomach produce epigastric pains
•Most small bowel, appendix, cecum cause periumbilical pain
•Hindgut structures, such as the bladder, distal two-thirds of the colon, and pelvic
genitourinary (GU) organs, usually cause pain in the suprapubic region.
•Pain is often reported in the back for retroperitoneal structures, such as the aorta and
kidneys
•Deep musculoskeletal structures (especially of the back) are innervated by visceral
sensory fibers with similar qualities to those arising from intra-abdominal organs.
•Intraperitoneal organs are bilaterally innervated causing pain to be located in the midline
regardless of which side it is coming from. Classic example is early appendicitis causing
periumbilical pain

Visceral Pain
•Caused by the stretching of fibers innervating the walls or capsules of hollow or solid
organs
•A steady ache or vague discomfort to excruciating or colicky
•Visceral afferents follow a segmental distribution and pain is localized by the sensory
cortex to an approximate spinal cord level determined by embryological organ of the
organ involved.
•Biliary, duodenum, stomach produce epigastric pains
•Most small bowel, appendix, cecum cause periumbilical pain
•Hindgut structures, such as the bladder, distal two-thirds of the colon, and pelvic
genitourinary (GU) organs, usually cause pain in the suprapubic region.
•Pain is often reported in the back for retroperitoneal structures, such as the aorta and
kidneys
•Deep musculoskeletal structures (especially of the back) are innervated by visceral
sensory fibers with similar qualities to those arising from intra-abdominal organs.
•Intraperitoneal organs are bilaterally innervated causing pain to be located in the midline
regardless of which side it is coming from. Classic example is early appendicitis causing
periumbilical pain
Feedback on your answer
Collapse
7)
0/1
What is the most common surgical cause of abdominal pain in adults?
(Select 1)(1pts)

Appendicitis

Cholecystitis

Small bowel obstruction

Perforated peptic ulcer

All of the above

None of the above

The answer is A, appendicitis. The value of detailed epidemiologic knowledge


notwithstanding, it is worthwhile to keep in mind the truism that common things are
common. Regarding which things are common, the most extensive information currently
available comes from the ongoing survey begun in 1977 by the Research Committee of
the OMGE. As of the last progress report on this survey, which was published in 1988,
more than 200 physicians at 26 centers in 17 countries had accumulated data on 10,320
patients with acute abdominal pain [see Table 3].23 The most common diagnosis in
these patients was nonspecific abdominal pain (NSAP)—that is, the retrospective
diagnosis of exclusion in which no cause for the pain can be identified.24,25 NSAP
accounted for 34% of all patients seen; the four most common diagnoses accounted for
more than 75%. The most common surgical diagnosis in the OMGE survey was acute
appendicitis, followed by acute cholecystitis, small bowel obstruction, and gynecologic
disorders. Relatively few patients had perforated peptic ulcer, a finding that confirms the
current downward trend in the incidence of this condition. Cancer was found to be a
significant cause of acute abdominal pain. There was little variation in the geographic
distribution of surgical causes of acute abdominal pain (i.e., conditions necessitating
operation) among developed countries. In patients who required surgery, the most
common causes were acute appendicitis (42.6%), acute cholecystitis (14.7%), small
bowel obstruction (6.2%), perforated peptic ulcer (3.7%), and acute pancreatitis
(4.5%).23 The OMGE survey's finding that NSAP was the most common diagnosis in
patients with acute abdominal pain has been confirmed by several srudies12,13,25; the
finding that acute appendicitis, cholecystitis, and intestinal obstruction were the three
most common diagnoses in patients with acute abdominal pain who require operation has
also been amply confirmed [see Table 3].1,12,13

Table 3. Frequency of Specific Diagnoses in Patients with Acute Abdominal Pain

Frequency in Individual Studies (% of Patients)

Wilso Brew
n et er et
OMGE23( Irvin13( de
Diagno al (N
82 al12(N Hawthorn83
N= N= Dombal1(
sis = = (N = 496)
10,320) 1,190) N = 552)
1,196 1,000
) )

Nonspecifi
c
34.0 45.6 34.9 41.3 50.5 36.0
abdominal
pain

Acute
appendicit 28.1 15.6 16.8 4.3 26.3 14.9
is

Acute
cholecysti 9.7 5.8 5.1 2.5 7.6 5.9
tis

Small
bowel
4.1 2.6 14.8 2.5 3.6 8.6
obstructio
n

Acute
gynecolog 4.0 4.0 1.1 8.5 — —
ic disease

Acute
pancreatiti 2.9 1.3 2.4 — 2.9 2.1
s

Urologic
2.9 4.7 5.9 11.4 — 12.8
disorders
Perforated
pepticulce 2.5 2.3 2.5 2.0 3.1 —
r

Cancer 1.5 — 3.0 — — —

Diverticula
1.5 1.1 3.9 — 2.0 3.0
r disease

Dyspepsia 1.4 7.6 1.4 1.4 — —

Gastroenter
— — 0.3 6.9 — 5.1
itis

Inflammato
ry bowel — — 0.8 — — 2.1
disease

Mesenteric
— 3.6 — — — 1.5
adenitis

Gastritis — 2.1 — 1.4 — —

Constipatio
— 2.4 — 2.3 — —
n

Amebic
hepatic 1.2 — 1.9 — — —
abscess

Miscellane
6.3 1.3 5.2 15.5 4.0 8.0
ous

OMGE = World Organization of Gastroenterology.


2014. Scientific American Surgery. Hamilton, Ontario & Philadelphia, PA. Decker
Intellectual Properties Inc. ISSN 2368-2744. STAT!Ref Online Electronic Medical
Library. http://online.statref.com/Document.aspx?fxId=61&docId=803. 10/16/2014
1:15:18 PM CDT (UTC -05:00).

Feedback on your answer


Collapse
8)
1/1
You are seeing a 28-year old white male who was brought in by EMS after he was
assaulted while at a night club. On exam his GCS is 7, he is breathing spontaneously and
localizes pain. He has a large scalp laceration to his right parietal region. He is rushed to
the CT scanner and the image is shown below. What is the diagnosis?

(Select 1)(1pts)

Subdural Hematoma

Epidural Hematoma

Intraparenchymal Hematoma with extension

Subarachnoid Hemorrhage

The answer is B. Epidural hematoma (EDH) is a traumatic accumulation of blood


between the inner table of the skull and the stripped-off dural membrane. The inciting
event often is a focused blow to the head, such as that produced by a hammer or baseball
bat. In 85-95% of patients, this type of trauma results in an overlying fracture of the skull.
Blood vessels in close proximity to the fracture are the sources of the hemorrhage in the
formation of an epidural hematoma. Because the underlying brain has usually been
minimally injured, prognosis is excellent if treated aggressively. Outcome from surgical
decompression and repair is related directly to patient's preoperative neurologic
condition. more from Read more
Feedback on your answer
Collapse
Unit 3
9)
0/1
A 22-year old male presents with severe epigastric pain that radiates into his chest after
vomiting over a dozen times following a night of heavy drinking. Vital signs show an
oraltemperature of 100.7 F, blood pressure 147/97, heart rate 127beats/min, respirations
24/min. Physical exam shows crepitus in the suprasternal notch, clear lung exam
bilaterally, and severe epigastric tenderness on abdominal exam. What is most likely
causing this patients symptoms?
(Select 1)(1pts)

Mallory Weiss tear

Spontaneous Pneumothorax

Perforated Duodenal Ulcer

Esophageal Perforation

The answer is D. This patient has evidence subcutaneous air (crepitus) and has likely
ruptured his esophagus. Effort rupture of the esophagus or Boerhaave's syndrome is a
spontaneous perforation of the esophagus that most commonly results from a sudden
increase in intraesophageal pressure combined with negative intrathoracic pressure
caused by straining or vomiting. It was first described by Dr. Herman Boerhaave, a
physician from Leiden, the Netherlands.

In patients suspected to have an esophageal perforation based upon clinical presentation


and chest radiography, computed tomography (CT) scan or esophagram should be
obtained promptly.
Computed tomography — Computed tomography (CT) scan shows esophageal wall
edema and thickening, extraesophageal air, periesophageal fluid with or without gas
bubbles, mediastinal widening, and air and fluid in the pleural spaces, retroperitoneum or
lesser sac (image 2).
Esophagram — The diagnosis of esophageal perforation can also be confirmed by water-
soluble contrast esophagram (Gastrografin), which reveals the location and extent of
extravasation of contrast material. However, false negative results occur in as many as 10
percent of cases. The sensitivity of contrast studies depends upon the size and location of
the perforation, and the technique used for the study.
Although barium is superior in demonstrating small perforations, it causes an
inflammatory response in mediastinal or pleural cavities and is therefore not used as the
primary diagnostic study [13]. If, however, the water-soluble study is negative, a barium
study should be performed for better definition (image 3).
Upper endoscopy — The role of upper endoscopy in the diagnosis of spontaneous
perforation remains controversial. Both the endoscope and insufflation of air can extend
the perforation and introduce air into the mediastinum [14]. However, in the appropriate
setting, often in the operating room with a patient prepared for surgical intervention,
endoscopy performed by a skilled endoscopist can be a useful tool for diagnosis and
treatment planning.
TREATMENT — Boerhaave's syndrome is rare and there is little evidence to guide
treatment. As a general rule, treatment depends upon the size and location of the
perforation, whether it is a contained perforation in the mediastinum or between the
mediastinum and visceral lung pleura, how rapidly it is diagnosed, whether the esophagus
has underlying disease, and the patient’s underlying comorbidities (algorithm 1).
Surgery is generally required for thoracic perforations [15], while cervical perforations
can often be managed without surgery.More from UptoDate
Feedback on your answer
Collapse
10)
0/1
You are seeing a 64-year old male in clinic for a follow up visit. He has a past medical
history of GERD and has had 2-3 EGDs in the past with no evidence or signs of Barrett's
esophagus. He takes (omeprazole 40 mg BID) but is always forgetful about taking his
medication. Which of the following options is an indication to consider surgery for the
treatment of GERD?
(1pts)

Older than 50 years

Noncompliance with drug therapy

Prefers a single surgical intervention to long-term drug treatmen

Experiences prominent symptoms of regurgitation, even with medical


control of heartburn symptoms

All of the above

None of the above

The answer is E, Consider surgery as an option for patients with well-documented GERD
who require long-term PPI maintenance therapy but show satisfactory relief of symptoms
and who:
 Are older than 50 years
 Consider long-term medication a financial burden
 Are noncompliant with drug therapy
 Prefer a single surgical intervention to long-term drug treatment
 Experience prominent symptoms of regurgitation, even with medical
control of heartburn symptoms
Nissen fundoplication is the most common surgical intervention for GERD. This
procedure aims to restore the physiology and anatomy of the gastroesophageal junction
by wrapping the gastric fundus around the distal esophagus. The FDA has also approved
several endoscopic procedures for treatment of GERD, including endoscopic suturing and
radiofrequency ablation of the lower esophageal sphincter. Read more

Feedback on your answer


Collapse
11)
0/1
Which of the following is NOT associated with an increased risk of esophageal
adenocarcinoma?
(Select 1)(1pts)

Cigarette smoking

Gastroesophageal reflux disease

Obesity

Helibactor pylori infection

All of the above

None of the above

Answer D is not correct. Esophageal cancer has two main subtypes — esophageal
squamous-cell carcinoma and esophageal adenocarcinoma; their precursor lesions are
esophageal squamous dysplasia and Barrett’s esophagus, respectively. Although
squamous-cell carcinoma accounts for about 90% of cases of esophageal cancer
worldwide, the incidence of and mortality rates associated with esophageal
adenocarcinoma are rising and have surpassed those of esophageal squamous-cell
carcinoma in several regions in North America and Europe. In the United States, more
than 18,000 new cases of esophageal cancer and more than 15,000 deaths from
esophageal cancer were expected in 2014. Esophageal carcinoma is rare in young people
and increases in incidence with age, peaking in the seventh and eighth decades of life.
The main risk factors for esophageal adenocarcinoma are gastroesophageal reflux
disease, obesity, and cigarette smoking; H. pyloriinfection is associated
with a reduced risk. Cigarette smoking and alcohol consumption constitute the
main risk factors for esophageal squamous-cell carcinoma. High intake of red meats, fats,
and processed foods is associated with an increased risk of both types of esophageal
cancer, whereas high intake of fiber, fresh fruit, and vegetables is associated with a lower
risk.
N Engl J Med 2014; 371:2499-2509December 25, 2014DOI:
10.1056/NEJMra1314530

Feedback on your answer


Collapse
Unit 4
12)
0/1
A 48-year old black male presents to you with sudden onset of vomiting and diffuse
abdominal pain that hurts to breathe two hours prior to arrival. He denies past medical
history and his only medications are ibuprofen for chronic right shoulder pain. He smokes
1 pack per day and occasionally drinks alcohol.
His vitals show an oral temperature of 99.3 F, heart rate of 129 beats/min, respiratory rate
20/min, blood pressure 151/100, O2 saturation of 98% on room air.
Physical exam shows him to be uncomfotable. Cardiac exam shows tachycardia, his
lungs are clear to auscultation, and abdominal exam shows diffuse tenderness to light
palpation greatest in the right upper quadrant along with guarding. Rectal exam shows
heme positive brown stool. An upright chest x-ray which is shown below. What is the
diagnosis?

(Select 1)(1pts)
Pneumomediastinum

Pneumoperitoneum

Intestinal Malrotation

Small Bowel Obstruction

Pneumothorax

The answer is B, Pneumoperitoneum which describes gas within the peritoneal cavity,
and is often the harbinger of a critical illness. The most common cause of a
pneumoperitoneum is from the disruption of the wall of a hollow viscus. Read more
An erect chest x-ray is probably the most sensitive plain radiograph for the detection of
free intraperitoneal gas. If a large volume pneumoperitoneum is present, it may be
superimposed over normal aerated lung with normal lung markings.
 subdiaphragmatic free gas
 cupola sign (in supine film)
Free gas within the peritoneal cavity can be detected on an abdominal radiograph. These
signs can be further divided by anatomical compartments in relation to the
pneumoperitoneum:
 bowel related signs
 double wall sign (also known as Rigler's sign or bas-relief
sign)
 telltale triangle sign (also known as triangle sign)
 peritoneal ligament related signs
 football sign
 falciform ligament sign
 lateral umbilical ligament sign (also known as inverted
"V" sign)
 urachus sign
 right upper quadrant signs
 lucent liver sign
 hepatic edge sign
 fissure for ligamentum teres sign
 Morison's pouch sign
 cupola sign
The patient above likely has a perforated ulcer from underlying NSAID use. Ulcer
perforation should be suspected in patients who suddenly develop severe, diffuse
abdominal pain. Perforations complicate 2 to 10 percent of peptic ulcers [70]. Duodenal,
antral, and gastric body ulcers account for 60, 20, and 20 percent of perforations due to
peptic ulcer disease (PUD), respectively. If imaging is required, plain x-rays are typically
obtained first. Careful interpretation of upright chest and abdominal films can detect
diagnostic free air in many cases of perforated gastric and duodenal ulcers [76]. The
presence of free air on abdominal imaging is highly indicative of a perforated viscus
(image 1 and image 2 and image 3), although about 10 to 20 percent of patients
with a perforated duodenal ulcer will not have free air [76]. If free air is found, no other
diagnostic studies are necessary. Leakage of water soluble oral contrast may be useful in
selected cases. Once the oral contrast is given, the patient should be rotated 360 degrees
and placed on the right side to fill the antrum and duodenum with contrast. However,
many perforations have already sealed spontaneously by the time of presentation [77], so
the absence of a leak does not exclude the diagnosis of a perforated ulcer. Read More
Feedback on your answer
Collapse
13)
0/1
You are seeing a 45-year old male in clinic who has a past medical history of
osteoarthritis for which he takes daily alleve. He denies tobacco use or alcohol use. He
has been having recent early satiety, bloating and epigastric discomfort over the last
month. You suspect peptic ulcer disease. Which of the following ulcer locations is more
associated with H. Pylori infection?
(Select 1)(1pts)

Duodenal

Gastric

Esophageal

Jejunal

None of the above

The answer is A, duodenal ulcers. H. pylori, a gram-negative, helical, rod-


shaped bacterium, colonizes the gastric mucosa of approximately one-half of the world
population and an estimated 30% to 40% of the U.S. population. H. pylori is present in
95% of patients with duodenal ulcers and in 70% of those with gastric ulcers. It is
typically transmitted via the fecal-oral route during early childhood and persists for
decades. The bacterium is a known cause of gastric and duodenal ulcers and is a risk
factor for mucosa-associated lymphoid tissue (MALT) lymphoma and gastric
adenocarcinoma. Am Fam Physician. 2015 Feb 15;91(4):236-242.

Feedback on your answer


Collapse
14)
0/1
You are seeing a 44-year old female who presents with worsening epigastric pain
following meals and "heartburn" that has been getting more persistent over the last few
days. She has a past medical history of hypertension. Her only medication is naprosyn
500 mg BID and HCTZ 25mg daily. On exam she has mild epigastric tenderness to
palpation. Her rectal exam shows her stool is brown but guaiac positive for blood. Her
portable upright chest xray shows no signs of free air or perforation.Based on her history
of current NSAID use, where would you likely expect the location of her ulcer?

(Select 1)(1pts)

Gastric

Duodenal

Jejunal

Esophageal

NSAID use confers around a 40 fold increase in the development of gastric ulcers and an
8 fold increase risk for the development of duodenol ulcers and . Therefore in this patient
you would expect gastric ulcer more likely than a duodenal ulcer.
The use of NSAIDs is the most commonly identified risk factor for peptic ulcer bleeding,
especially in the elderly. Studies have found relative risks for bleeding ranging from 2.7
to 33.9 [11]. Studies have also shown that the risk is drug-specific and dose-dependent.
As an example, in a study of 2777 patients, the overall relative risk (RR) of bleeding
associated with NSAID use was 5.3 (95% CI 4.5-6.2). However, the risk varied by drug
and was lowest for aceclofenac (RR 3.1, 95% CI 2.3-4.2) and was highest for ketorolac
(RR 14.4, 95% CI 5.2-39.9) [18]. The risk was higher in patients taking high-dose
NSAIDs compared with those taking medium- or low-dose NSAIDs (RR 6.8, 95% CI
5.3-8.8 versus 4.0, 95% CI 3.2-5.0). There was also an increased risk of bleeding with
aspirin use (RR 5.3) that again was dose-dependent (RR 7.5 with 500 mg per day versus
2.7 with 100 mg per day). The concurrent use of aspirin and NSAIDs conferred an even
greater risk of bleeding than was seen with either agent alone (RR 12.7). Finally, the risk
was highest in the first 30 days of NSAID use, with a RR of 7.6 (95% CI 6.0-9.5). The
risk remained high between days 31 and 90 days (RR 7.3, 95% CI 4.0-13.2), but dropped
after 91 days (RR 2.6, 95% CI 1.6-4.1).
NSAID use has also been identified as a risk factor for ulcer perforation [16,19]. In a
study of 176 patients from Spain, NSAID use was the only risk factor that was
significantly associated with perforation (odds ratio [OR] 3.6). Read more
Feedback on your answer
Collapse
15)
0/1
What is the most common complication of peptic ulcer disease?

(Select 1)(1pts)

Gastric outlet obstruction

Perforation

Bleeding

Cancer

The answer is B, Bleeding. Bleeding is the most common complication of peptic ulcer
disease requiring hospitalization. In a study of the National Inpatient Sample (NIS),
bleeding occurred in 73 percent, perforation in 9 percent, and obstruction in 3 percent
[4]. H. pylori, nonsteroidal antiinflammatory drugs (NSAIDs), and the use of low dose
aspirin are the most common etiologies of ulcer bleeding [7] and ulcer perforation [8-
10]. In observational studies, duodenal, antral/pyloric, and gastric body ulcers
account for 60, 20, and 20 percent of perforations, respectively [8,9]. Among these
complications, perforation had the highest mortality rate, followed by obstruction, then
hemorrhage.
Bleeding peptic ulcer — Upper gastrointestinal bleeding due to peptic ulcer disease
is a common indication for emergency management of peptic ulcer disease [11]. Most
patients with acute bleeding can be managed with fluid resuscitation and transfusion, acid
suppression therapy, and endoscopic intervention. For those who fail these efforts,
surgery may become necessary. The specific indications for surgery for patients with
bleeding peptic ulcer are reviewed separately.
Perforated peptic ulcer — Ulcer perforation may be suspected in patients with a
history consistent with peptic ulcer disease who develop the sudden onset of severe,
diffuse abdominal pain. Once a diagnosis of perforation is established, surgical
intervention is indicated.
Gastric outlet obstruction — Gastric outlet obstruction is the least frequent ulcer
complication in developed countries. Most cases are associated with duodenal or pyloric
channel ulceration; gastric ulceration accounts for only five percent of cases.
Surgical consultation should be obtained for patients with chronic partial gastric outlet
obstruction that is refractory to medical treatment, and those found to have complete
gastric outlet obstruction, or those readmitted with gastric outlet obstruction after recent
“successful treatment.” Surgery is indicated if the patient fails to respond to conservative
medical management and endoscopic therapy.
Surgical treatment of gastric outlet obstruction is almost never an emergency. Because
most patients present with some degree of malnutrition and frequently have electrolyte
imbalances (ie, hypokalemic, hypochloremic metabolic alkalosis secondary to vomiting
or nasogastric suctioning), it is important to correct any derangements and optimize the
patient’s medical status prior to proceeding with surgery. Failure to identify and correct
these issues increases perioperative morbidity.
It may also be reasonable to try and improve the patient’s overall nutritional status with
nutritional support, as these patients are frequently malnourished and can have significant
postoperative delayed gastric emptying due to their atonic stomach.
Many experienced gastric surgeons advocate preoperative nasogastric sump
decompression to decrease gastric dilation and, hopefully, gastric atony. Read more

Feedback on your answer


Collapse
16)
0/1
What % of benign appearing gastric ulcers are found to be harbor malignancy?
(Select 1)(1pts)

<1%

5-11%

15-21%

24-30%

The answer is B, 5-11%. Given the low predictive value of the signs and symptoms of
PUD, the diagnosis of uncomplicated peptic ulcers is difficult to make solely on a clinical
basis. The diagnosis of PUD is typically made by endoscopy and upper gastrointestinal
(UGI) radiography. Endoscopy is the most accurate method of establishing the diagnosis
of peptic ulcers, with a reported sensitivity of 92% and specificity of 100%.7 In addition
to identifying the ulcer and its features, location, and size, endoscopy provides an
opportunity for biopsies to test for H. pylori and exclude malignancy and for therapeutic
interventions for bleeding ulcers. Duodenal ulcers are most often benign and do not
require routine biopsy. Multiple biopsies are indicated for all gastric ulcers as even
lesions with a benign appearance harbor malignancy in 5 to 11% of cases. Read more
Feedback on your answer
Collapse
Unit 5
17)
0/1
You are seeing a 27-year old male in the trauma bay shortly after being stabbed in this
back with a 6 cm buck knife by his disgruntled neighbor after an argument.
He arrived via EMS sitting up, awake and alert with a GCS of 15. His BP is 140/90, HR
120 and O2 sat is 85% on a non re-breather.
On physical exam there is a 3 cm non bleeding wound just left of his mid thoracic spine.
Upon auscultation there is decreased breath sounds on his left side. There are no signs of
tracheal deviation or JVD.
What is the next best step in management?
(1pts)

Immediately to the OR for surgical exploration

Upright Chest Xray followed by a chest tube

Immediate Needle thoracotomy on the left

Pericardiocentesis under US guidance

The answer is A, Upright Chest Xray followed by a chest tube. This patient is awake and
talking. He is stable enough for a chest xray followed by chest tube placement. A needle
thoracotomy to should be done if he has evidence of tension pneumothorax which would
be respiratory distress and hypotension. He will likely then need a chest tube and can be
further managed. Read More
Feedback on your answer
Collapse
18)
0/1
What is the most common abdominal organ injured in the setting of blunt abdominal
trauma?
(Select 1)(1pts)

Spleen

Liver

Kidneys

Small Intestines
The liver is the most commonly injured solid organ in blunt trauma, comprising 5% of all
trauma admissions, and because of its size is frequently involved in penetrating trauma.
Following blunt trauma, the most commonly injured structures are the parenchyma and
hepatic veins. Blunt forces dissipate along segments of the liver and along the fibrous
coverings of the portal triad structures; the hepatic veins, however, are not so insulated.
Given its size and location within the abdomen, the liver is also commonly involved in
penetrating trauma. Stab wounds typically result in direct linear tears, whereas gunshot
wounds or shotgun wounds result in significant cavitary injuries attributable to blast
effect and the "tumbling" of the missile within the liver parenchyma. Thus, arterial injury
is more common with penetrating trauma.
Over the past 20 years, nonoperative management (NOM) of liver injuries has evolved to
become the prevailing therapeutic strategy for blunt hepatic trauma. Several concurrent
changes resulted in this paradigm change. First was the realization that diagnostic
peritoneal lavage (DPL) was sensitive but not specific for identifying intraperitoneal
hemorrhage that necessitated operative management. Surgeons recognized that many
laparotomies undertaken for a positive DPL were associated with liver injuries that did
not require intervention for bleeding.1Second, trauma surgeons noted that nonbleeding
hepatic venous injuries, if manipulated at laparotomy, often resulted in more hemorrhage
and sometimes even death.2 Furthermore, it became conspicuous that with hemostasis
achieved in the operating room, recurrent postoperative bleeding was rare. Therefore,
surgeons queried whether hepatic venous injuries, which are low-pressure system
injuries, could heal without intervention. Finally, computed tomography (CT) provided a
reliable method for diagnosing and grading liver injuries.

The spleen is the second most commonly injured abdominal organ in blunt trauma
patients. Historical studies have reported a 10% mortality with all splenic injuries;
however, isolated splenic injury mortality is less than 1%. The mechanisms of injury are
similar to those seen with liver injuries: motor vehicle collisions, autopedestrian
accidents, and falls. Similar to penetrating trauma to the liver, stab wounds to the spleen
typically result in direct linear tears, whereas gunshot wounds result in significant
cavitary injuries.

Until the 1970s, splenectomy was considered mandatory for all splenic injuries.
Recognition of the immune function of the spleen refocused efforts on splenic salvage in
the 1980s.38,39Following success in pediatric patients, NOM of splenic injuries was
adopted in the adult population and has become the prevailing strategy for blunt splenic
trauma.

Duodenal and pancreatic injury continues to challenge the trauma surgeon. The relatively
rare occurrence of these injuries, the difficulty in making a timely diagnosis, and high
morbidity and mortality rates justify the anxiety these unforgiving injuries invoke.
Mortality rates for pancreatic trauma range from 9 to 34%, with a mean rate of 19%.
Duodenal injuries are similarly lethal, with mortality rates ranging from 6 to 25%.
Complications following duodenal or pancreatic injuries are alarmingly frequent,
occurring in 30 to 60% of patients.1-3 Recognized early, the operative treatment of most
duodenal and pancreatic injuries is straightforward, with low morbidity and mortality.
2014. Scientific American Surgery. Hamilton, Ontario & Philadelphia, PA. Decker
Intellectual Properties Inc. ISSN 2368-2744. STAT!Ref Online Electronic Medical
Library. http://online.statref.com/Document.aspx?fxId=61&docId=2329. 10/27/2014
1:39:15 PM CDT (UTC -05:00).

Feedback on your answer


Collapse
19)
0/1
You are seeing a 42-year-old female following in the trauma bay following a high-speed
motor vehicle collision. She was was a restrained passenger and her car flipped over after
she lost control. She had to be cut out of the car and is complaining of shortness of breath
and upper abdominal pain.
She is awake and alert and her GCS is 15. Her pulse is 120/min, respirations are 28/min,
and blood pressure is 80/40 mm Hg. Breath sounds are decreased at the left lung base. An
x-ray of the chest shows opacification of the left lower lung field. You decide to place an
emergent chest tube that yields a small amount of air followed by greenish fluid. Which
of the following is the most appropriate next step in management?
(Select 1)(1pts)

CT scan of the abdomen and pelvis

CT scan of the chest

Thoracoscopy

Laparotomy

The answer is D. This patient suffered a ruptured diaphragm and chest tube placement
appears to have ruptured the bowel as evidence from the greenish fluid that was returned.
She needs an emergent laparotomy to control bleeding and repair her diaphragm. Read
More
Feedback on your answer
Collapse
Unit 6
20)
0/1
A 46-year obese black female presents to you with 2 days of worsening of right upper
abdominal pain and vomiting. She feels hot and cold chills and says that she sometimes
feels this pain after eating but now its much worse.
Vitals: Temp is 39.0 C, heart rate 125 beats/min, blood pressure 124/90. Physical exam
reveals inspiratory arrest upon palpation of her RUQ to deep palpation.
Her labs show a WBC of 17,000. Her ALT, AST, bilirubin, and lipase are within normal
limits. What is the most likely diagnosis?
(Select 1)(1pts)

Cholelithiasis

Choledocholithiasis

Klatskin Tumor

Acute Cholecystitis

The patient presents with the triad of acute cholecystitis which is fever, vomiting,
jaundice and RUQ pain. This triad has been reported to be present in as high 70% present
of cases to as little as 20% of cases. Acute cholecystitis is the result of cystic duct
obstruction leading to gallbladder edema, mucosal sloughing, and, potentially, ischemia.
Signs and symptoms of acute cholecystitis include right upper quadrant pain, nausea, and
vomiting. Unlike the self-limited pain associated with biliary colic, pain associated with
acute cholecystitis generally lasts more than 6 to 8 hours. In addition to the signs of local
inflammation (pain and tenderness in the right upper abdomen or epigastric area),
patients also exhibit signs of systemic inflammation, including fever, leukocytosis, and
elevated C-reactive protein. Mild elevations in bilirubin and liver enzymes may be
present. A positive Murphy sign on physical examination is suggestive of a diagnosis of
cholecystitis (likelihood ratio of 2.8). Necrosis of the gall-bladder wall can occur due to
prolonged inflammation and decreased gallbladder perfusion. Gallbladder perforation
occurs in 5 to 10% of patients with acute cholecystitis.
Renoylds pentad adds sepsis and altered mental status to the triad and is less common and
can be easily missed. A Klatskin tumor presents with painless jaundice. While she might
have cholelithiasis and choledocholithiasis her symptoms are more indicative of acute
cholecystitis. Some cases of acute cholecystitis have mildly elevated bilirubin but severe
elevation suggests either Choledocholithiasis, cholangitis, or obstruction of the CBD.
2014. Scientific American Surgery. Hamilton, Ontario & Philadelphia, PA. Decker
Intellectual Properties Inc. ISSN 2368-2744. STAT!Ref Online Electronic Medical
Library. http://online.statref.com/Document.aspx?fxId=61&docId=942. 10/20/2014
8:07:24 PM CDT (UTC -05:00).

Feedback on your answer


Collapse
21)
0/1

Which of the following imaging studies is most sensitive for the diagnosis of acute
cholecystitis?

(1pts)

Ultrasound

HIDA scan

CT scan

MRI

The answer is B, HIDA scan. US is about 88% sensitive where as HIDA Scanning is
slightly better and more senstive but US is still the recommended as the first line
diagnostic imaging study. Cholescintigraphy using 99mTc-hepatic iminodiacetic acid
(generically referred to as a HIDA scan) is indicated if the diagnosis remains uncertain
following ultrasonography. Technetium labeled hepatic iminodiacetic acid (HIDA) is
injected intravenously and is then taken up selectively by hepatocytes and excreted into
bile. If the cystic duct is patent, the tracer will enter the gallbladder, leading to its
visualization without the need for concentration. The HIDA scan is also useful for
demonstrating patency of the common bile duct and ampulla. Normally, visualization of
contrast within the common bile duct, gallbladder, and small bowel occurs within 30 to
60 minutes (image 3). The test is positive if the gallbladder does not visualize. This
occurs because of cystic duct obstruction, usually from edema associated with acute
cholecystitis or an obstructing stone (image 4).
Cholescintigraphy has a sensitivity and specificity for acute cholecystitis of
approximately 90 to 97 percent and 71 to 90 percent, respectively [23,26,28,29].
Cystic duct obstruction with a stone or tumor in the absence of acute cholecystitis can
cause a false positive test. Conditions that can cause false positive results despite a non-
obstructed cystic duct include:
 Severe liver disease, which may lead to abnormal uptake and excretion of
the tracer.
 Fasting patients receiving total parenteral nutrition, in whom the
gallbladder is already maximally full due to prolonged lack of
stimulation.
 Biliary sphincterotomy, which may result in low resistance to bile flow,
leading to preferential excretion of the tracer into the duodenum without
filling of the gallbladder.
 Hyperbilirubinemia, which may be associated with impaired hepatic
clearance of iminodiacetic acid compounds. Newer agents commonly
used in cholescintigraphy (diisopropyl and m-bromotrimethyl
iminodiacetic acid) have generally overcome this limitation.
False negative results are uncommon since most patients with acute cholecystitis have
obstruction of the cystic duct. When they occur, they may be due to incomplete cystic
duct obstruction. Rear more

Feedback on your answer


Collapse
22)
0/1
Which of the following complications of cholelithiasis can present with fever, persistent
tachycardia, hypotension, or jaundice?
(Select 1)(1pts)

Cholecystitis

Cholangitis

Pancreatitis

All of the above

None of the above

The presence of fever, persistent tachycardia, hypotension, or jaundice necessitate a


search for complications of cholelithiasis, including cholecystitis, cholangitis,
pancreatitis, or other systemic causes.
For more on the physical examination of patients with gallstones, read here.
Feedback on your answer
Collapse
Unit 7
23)
0/1
The incidence of TRALI (Transfusion-related acute lung injury) has declined in recent
years due to which of the following choices below?

(1pts)

The use of plasma from female donors only


The use of plasma from male donors only

The use of whole blood instead of plasma

Different storage techniques

The answer is B, The use of plasma from male donors only. Transfusion of blood
products is not without risk. According to the Annual Summary of the FDA for 2012,
there were 38 transfusion-related fatalities in the United States, as opposed to 46 in
2008. Since the number of adverse outcomes is small in comparison to the overall
number of transfusions (an estimated 24 million in 2008), many emergency physicians
are unfamiliar with the potential complications of blood product administration. As an
example, TRALI is unfamiliar to many physicians and not universally discussed with
patients during the consent process,63 despite the fact that it was the leading cause of
transfusion-related fatality in both 2008 and 2012.
As early as the 1950s, noncardiogenic pulmonary edema was recognized as a potential
complication of blood transfusion, although it was not until 1983 that Popovsky and
colleagues coined the term “transfusion-related acute lung injury.” These investigators
reported that transfusion of whole blood, RBCs, and plasma from multiparous women
was found to be associated with the rapid onset of hypoxemia and pulmonary infiltrates,
either during transfusion or within several hours of administration. The association with
multiparous donors suggested as the mechanism of lung injury the passive transfer of
antibodies in the transfused blood product, with subsequent activation of recipient white
blood cells. Just as pregnant women can become immunized to Rh antigen present on the
RBCs of the fetus during periods of feto-maternal hemorrhage, exposure to fetal white
blood cells can give rise to antileukocyte antibodies. This idea has been supported by the
identification of antileukocyte antibodies in transfused blood products derived from
multiparous women. Although other factors may contribute to the pathogenesis of
TRALI, exposure to plasma or plasma-containing blood products containing
antileukocyte antibodies is still believed to be the primary mechanism for this
transfusion-related syndrome. Both the American Association of Blood Banks and the
American Red Cross have implemented changes to reduce this risk, including the
preferential use of male plasma, especially in high-plasmacontent products (FFP and
platelets). A national survey in 2008 revealed that > 90% of blood banks had instituted
collection of male-only, malepredominant, or never-pregnant female plasma.
Retrospective studies support these practices, confirming a significant reduction in rates
of TRALI following their adoption.67 Unified definitions of TRALI have been proposed
by the National Institutes of Health (NIH) and a Canadian Consensus Panel. The
Consensus Panel defined 4 criteria, as well as a separate diagnostic category, “possible
TRALI,” for patients in which the development of acute lung injury is temporally related
to both transfusion and another potential cause of acute lung injury. Read more
Feedback on your answer
Collapse
24)
0/1
When geographically and logistically feasible, critically injured patients should be taken
directly to a designated level I trauma center or to a level II trauma center if a level I
trauma center is more than 30 minutes away. The currently available field trauma scores,
however, are not reliable in making this triage decision. Because the potential harm
associated with undertriage is high and could result in death or substantial morbidity and
disability, criteria were initially chosen to err on the side of minimizing undertriage rather
than minimizing overtriage. What are the target levels for undertriage rates within a
trauma system to a level I or II trauma center?
(Select 1)(1pts)

<5%

5-10%

10-15%

15-20%

The answer is A, <5%. When geographically and logistically feasible, critically injured
patients should be taken directly to a designated level I trauma center or to a level II
trauma center if a level I trauma center is more than 30 minutes away. The currently
available field trauma scores, however, are not reliable in making this triage decision.
The development of field triage criteria has paralleled the development of trauma centers.
The decision is based on a practice algorithm called a "decision scheme." The first field
triage decision scheme was published by the American College of Surgeons in 1986, with
subsequent updates in 1990, 1993, 1999, 2006, and 2011.6 The decision scheme has four
steps that the emergency medical service (EMS) provider proceeds through after
measuring vital signs and level of consciousness to decide if the patient should be
transported to a higher-level trauma center [see Table 2]. Because the potential harm
associated with undertriage is high and could result in death or substantial morbidity and
disability, criteria were initially chosen to err on the side of minimizing undertriage rather
than minimizing overtriage. However, overtriage results in an overuse of financial and
human resources, overcrowding of trauma centers, and increased EMS transport times.
Target levels for undertriage rates within a trauma system range from 0 to 5% of patients
requiring level I or level II trauma center care, whereas levels of overtriage vary from 25
to 50%. As field triage continues to evolve on the basis of new research findings,
overtriage rates might be reduced while maintaining low undertriage rates. Read more
Feedback on your answer
Collapse
25)
0/1
In the absence of a major surgical insult or concomitant coagulopathy, what platelet count
is required for normal coagulation?
(1pts)
10,000/μL

20,000/μL

50,000/μL

100,000/μL

The answer is B, Patients with a normal INR and a normal aPTT who exhibit unexpected
bleeding may have impaired platelet activity. Inadequate platelet activity is frequently
manifested as persistent oozing from wound edges or as low-volume bleeding. Such
bleeding is rarely the cause of exsanguinating hemorrhage, although it may be life-
threatening when it occurs in certain locations (e.g., inside the cranium or the
pericardium). Inadequate platelet activity may be attributable to an insufficient number of
platelets, platelet dysfunction, or a platelet inhibitor. In the absence of a major surgical
insult or concomitant coagulopathy, a platelet count of 20,000/μL or higher is usually
adequate for normal coagulation.14,15 There is some disagreement regarding the
threshold for platelet transfusion in the absence of active bleeding. Patients undergoing
procedures in which even small-vessel oozing is potentially life-threatening (e.g.,
craniotomy) should be maintained at a higher platelet count (i.e., > 20,000/μL). Patients
without ongoing bleeding who are not specifically at increased risk for major
complications from low-volume bleeding may be safely watched with platelet counts as
low as 10,000/μL. Read more
Unit 1
1)
1/1
You are seeing a 26-year old college student in the ER following a roll over MVC. He
was wearing a seat belt and was not ejected. Upon arrival his complaining of abdominal
pain. His vital signs show a heart rate of 120/min, blood pressure is 134/98, RR 20/min,
O2 98% on room air. Physical exam shows he has a seat belt sign across his abdomen and
has tenderness greatest in his upper abdomen with no guarding. His CT scan of the
abdomen/pelvis with IV contrast only shows a liver laceration.

True or False: Most liver lacerations/injuries are managed operatively as they


often fail non-operative management.
( 1pts extra credit)

True

False Correct

False: The current approach to hepatic trauma has evolved to nonoperative management
in more than 80% of cases. Several contributing factors have been recognized: realization
that more than 50% of liver injuries stop bleeding spontaneously, the precedent of
successful nonoperative management in pediatric patients, knowledge that the liver has
tremendous capacity to heal after injury, and improvements in liver imaging with CT.
Criteria for nonoperative management include foremost, hemodynamic stability, absence
of other abdominal injuries that require laparotomy, immediate availability of resources
including a fully staffed operating room, and a vigilant surgeon. In general, any patient
who is stable enough to have a CT scan performed is likely to be successfully managed
nonoperatively. Grade I and II hepatic injuries should be observed in a monitored setting
with serial hematocrit evaluations and bed rest. Higher-grade injuries in stable patients
should be observed in an intensive care unit setting with optimization of all coagulation
factors. Read More
Feedback on your answer
Collapse
2)
0/1
A 55-year-old white female presents with pain and redness to her right breast at the scar
site from a lumpectomy that was performed for stage I cancer four months ago. She has
also since completed multiple rounds of radiation treatments to her breast as well.
Currently her vitals are normal and she is afebrile. Physical exam reveals 5-6 cm of
erythema surrounding her surgical scar that is painful to touch, there is no axillary
adenopathy, no wound drainage, crepitance, or bullous lesions. Which one of the
following organisms would be the most likely cause of cellulitis in this patient?

(Select 1)( 1pts extra credit)

Non–group A hemolytic Streptococcus

Pneumococcus pneumoniae

Clostridium perfringens

Escherichia coli

The answer is A. Cellulitis in patients after breast lumpectomy is thought to be related to


lymphedema. Axillary dissection and radiation predispose to these infections. Non–group
A hemolytic Streptococcus is the most common organism associated with this infection.
The onset is often several weeks to several months after surgery. Pneumococcus is more
frequently a cause of periorbital cellulitis. It is also seen in patients who have bacteremia
with immunocompromised status. Immunocompromising conditions would include
diabetes mellitus, alcoholism, lupus, nephritic syndrome, and some hematologic cancers.
Clostridium and Escherichia coli are more frequently associated with crepitant cellulitis
and tissue necrosis. Pasteurella multocida cellulitis is most frequently associated with
animal bites, especially cat bites. Ref: Swartz MN: Cellulitis. N Engl J Med
2004;350(9):904-912.
Feedback on your answer
Collapse
3)
1/1
What is the most common cancer among woman?
(Select 1)( 1pts extra credit)

Ovarian

Lung

Colon

Breast

The answer is D, Breast Cancer. Approximately 230,480 American women are diagnosed
with breast cancer annually, and 39,520 women die from this disease. Global cancer
statistics show that breast cancer is the most frequently diagnosed cancer and the leading
cause of cancer death among females, accounting for 23 percent of total cancer cases and
14 percent of cancer deaths. Breast cancer is now also the leading cause of cancer death
among females in economically developing countries.
Globally, breast cancer is the most frequently diagnosed cancer and the leading cause of
cancer death in women. In the United States, breast cancer is the most commonly
diagnosed cancer and the second most common cause of cancer death in women. In
addition, breast cancer is the leading cause of death in women ages 40 to 49 years.Breast
cancer is treated with a multidisciplinary approach involving surgical oncology, radiation
oncology, and medical oncology, which has been associated with a reduction in breast
cancer mortality. Read more from Up to date

Feedback on your answer


Collapse
4)
1/1
You are seeing a 48-year old white female who has no past medical/family history in
your office. She is complaining of right sided bloody nipple discharge over the last 2
days. She denies any trauma or palpation of any mass. On exam you note a reddish
discharge from her right breast that is guaiac positive. What it the most likely cause of
her findings?

(Select 1)( 1pts extra credit)

Mammary Duct Ectasia


Intraductal papilloma

Nonpuerperal mastitis

Ductal carcinoma in situ (DCIS)

The answer is B, Intraductal papilloma. The old concern over cancer is the issue, and the
way to detect cancer that is not palpable is with a mammogram. That should be the first
choice. If negative, one may still wish to find an resect the intraductal papilloma to
provide symptomatic relief. Intraductal papillomas consist of a monotonous array of
papillary cells that grow from the wall of a cyst into its lumen. Although they are not
concerning in and of themselves, they can harbor areas of atypia or ductal carcinoma in
situ (DCIS). Papillomas can occur as solitary or multiple lesions. The standard approach
to a papilloma diagnosed by core needle biopsy (CNB) is to perform a surgical excision,
particularly if atypical cells are identified [14,16-21]. In a meta-analysis of 34 studies
that included 2236 non-malignant breast papillary lesions, 346 (15.7 percent) were
upgraded to malignancy following a surgical excision [21]. Because of a risk of
malignancy, these require surgical excision.Read More
Feedback on your answer
Collapse
5)
0/1
You are seeing a 31-year-old woman who presents with worsening sore throat, dry
cough, fever, and severe neck pain over the last week. She is otherwise healthy and takes
no medications.
Her vitals show a temperature is 102.5 °F, blood pressure is 99/68 mm Hg, pulse rate is
125/min, and respiration rate is 24/min. On exam she appears ill and her neck is tender to
palpation along the left side with overlying erythema, mild induration, without
lymphadenopathy. Her pharynx is erythematous, with tonsillar enlargement and no
exudates or ulcers. Her lungs are clear to auscultation. The remainder of the examination
is normal. Her chest x-ray shows multiple bilateral infiltrates. Labs show a leukocyte
count is 19,700/µL with 16% band forms. Hgb 10.8 g/dl, BUN 34 mg/dL, serum
creatinine level is 1.8 mg/dL. Which of the following tests is most likely to establish the
diagnosis?

(Select 1)( 1pts extra credit)

Computed tomography (CT) angiography of the chest with contrast

CT of the neck with contrast

Soft Tissue Neck xray


Transthoracic echocardiography

The answer is B, The patient should undergo computed tomography (CT) of the neck
with contrast. She has fever, leukocytosis, sore throat, unilateral neck tenderness, and
multiple densities on chest radiograph, suggestive of septic emboli. The combination of
these factors points strongly toward Lemierre syndrome, which is septic thrombosis of
the internal jugular vein. The diagnosis should be suspected in anyone with pharyngitis,
persistent fever, neck pain, and septic pulmonary emboli. CT of the affected vessel with
contrast would confirm the diagnosis. Treatment includes intravenous antibiotics that
cover streptococci, anaerobes, and β-lactamase-producing organisms. Penicillin with a β-
lactamase inhibitor and carbapenem are both reasonable choices (eg, ampicillin-
sulbactam, piperacillin-tazobactam, ticarcillin-clavulanate).
Chest CT would better characterize the pulmonary infiltrates, but this information would
not provide specific diagnostic information that would guide therapy.
Soft tissue radiography of the neck cannot detect jugular vein filling defects or
thromboses, which are diagnostic of septic thrombophlebitis.
Echocardiography would be helpful to exclude right-sided endocarditis as a cause of
septic emboli. However, there is nothing in the history or on cardiac examination to
suggest a cardiac source of septic emboli. Centor RM, Samlowski R. Avoiding sore
throat morbidity and mortality: when is it not “just a sore throat?” Am Fam Physician.
2011;83:26, 28. PMID: 21888123
Feedback on your answer
Collapse
Unit 2
6)
0/1
Nonoperative Management of solid-organ injuries can be pursued in hemodynamically
stable patients who do not have overt peritonitis or other indications for laparotomy.
According to contemporary data, what percentage of patients with splenic injuries are
candidates for nonoperative management?
(Select 1)(1pts)

<10%

25%

45%

60%

The answer is D, 60%. NOM of solid-organ injuries can be pursued in hemodynamically


stable patients who do not have overt peritonitis or other indications for laparotomy.5-9
Contemporary data suggest that this will be approximately 60% of the overall splenic
injury population.6 Failure of NOM in this selected population has been reported at only
10 to 20%. It is important to understand that this rate represents an average and may vary
widely across differing grades of injury and across varying patient populations. For
example, the NOM failure rate reported in the Eastern Association for the Surgery of
Trauma multiinstitutional study from 2000 was 11%.10 However, those with highgrade
injuries (III, IV, V) had a NOM failure rate of 20%, 33%, and 75%, respectively. In
addition to increasing grade, contrast extravasation, older age, and high injury severity
score (> 25) have been linked to higher risk of failure of NOM.11-16 NOM may be time
and resource intensive and is not safe without the supporting resources such as a setting
and staff for adequate observation, rapid operating room mobilization if needed,
immediately available anesthesia, and an adequate blood bank. If such resources are
unavailable, patients should be transferred to a higher level of care or early operation
should be considered.
The role of angioembolization of the injured spleen continues to evolve. The
identification of contrast extravasation within the splenic parenchyma (pseudoaneurysm)
as a risk factor for failure of NOM led to liberal use of angioembolization and improved
rates of splenic salvage in this population [see Figure 1 and Figure 2].17 Contrast
extravasation into the peritoneal cavity outside the splenic parenchyma is also commonly
cited as an indication for angiography. However, the use of angioembolization in such
patients has met with less success.18 In our experience, such patients rarely remain stable
for long and often require laparotomy if angiography cannot be performed rapidly
In addition to use in contrast blush, angiography is increasingly used in all higher-grade
spleen injuries undergoing NOM. Multiple institutions have addressed this concept, and a
meta-analysis published in 2011 and a subsequent prospective study in 2014 highlighted
the higher splenic salvage rate during NOM when angioembolization is employed in all
grade IV and V injuries.19,20 This is echoed in a Level 2 recommendation in the Eastern
Association for the Surgery of Trauma practice management guidelines from 2012.10
Subcapsular hematomas also may be at higher risk for delayed rupture, and special
consideration should be given to patients with this injury pattern. Also currently
controversial is the superiority of proximal embolization versus distal embolization.18,21
With such emphasis placed on reporting techniques for increasing NOM success and
splenic salvage, it is important to keep in mind the role of early, rapid operation.
Approximately 20 to 30% of patients with splenic trauma deserve early splenectomy, and
failure of NOM often represents poor patient selection.22,23 In adults, indications for
prompt laparotomy include hemodynamic instability and initiation of blood transfusion
within the first 12 hours considered to be secondary to the splenic injury. In the pediatric
population, blood transfusions up to half of the patient's blood volume are used prior to
operative intervention. Following the first 12 postinjury hours, indications for laparotomy
are not as black and white. Determination of the patient's age, comorbidities, current
physiology, degree of anemia, and associated injuries will determine the use of
transfusion alone versus intervention with either embolization or operation. Delayed
hemorrhage or rupture of the spleen can occur up to weeks following injury, but the
majority who fail do so while still hospitalized.24 Overall, nonoperative treatment
obviates laparotomy in more than 90% of appropriately selected cases. Read more
Feedback on your answer
Collapse
7)
0/1
What is the reported mortality rate of isolated traumatic splenic injuries?
(Select 1)(1pts)

<1%

5-10-%

10-20%

20-30%

The answer is A, The spleen is one of the most commonly injured abdominal organs in
blunt trauma patients. Historical studies have reported 10% mortality with all splenic
injuries; however, isolated splenic injury mortality is less than 1%. The mechanisms of
injury are similar to those seen with liver injuries: motor vehicle collisions, automobile-
pedestrian collisions, falls, and any type of penetrating injury. Stab wounds to the
abdomen are less likely to cause spleen injury compared with liver injury due to the
spleen's protected location. Stab wounds to the abdomen are less likely to cause injury to
the spleen than to the liver, due to its protected location.
Until the 1970s, splenectomy was considered mandatory for all splenic injuries.
Recognition of the immune function of the spleen refocused efforts on splenic salvage in
the 1980s.1,2 Following success in pediatric patients, nonoperative management (NOM)
of splenic injuries was adopted in the adult population and has become the prevailing
strategy for blunt splenic trauma. Read more
Feedback on your answer
Collapse
8)
0/1
In cases of blunt trauma to the diaphragm, the injury is on the ____ side __% of the time?
(Select 1)(1pts)

Right; 90%

Left; 75%

Left; 25%

Right; 10%
The answer is B, left;75%. In cases of blunt trauma to the diaphragm, the injury is on the
left side 75% of the time, presumably because the liver diffuses some of the energy on
the right side. Blunt diaphragmatic injuries result in a linear tear in the central tendon,
whereas penetrating injuries are variable in size and location depending on the weapon. It
is important to identify the trajectory of penetrating injuries to determine the likelihood of
diaphragm injuries. With blunt and occasionally with penetrating injuries, the diagnosis is
suggested by an abnormality of the diaphragmatic shadow on a chest radiograph [see
Figure 5]. In patients without clear imaging results in the trauma bay, a CT scan may
identify a diaphragmatic injury.
Regardless of the etiology, acute injuries are repaired through an abdominal incision.
Thoracoscopy or laparoscopy may be used if concomitant injuries requiring laparotomy
have been ruled out. Following delineation of the injury, the chest should be evacuated of
all blood and particulate matter, and tube thoracostomy should be placed if not previously
done. Using Allis clamps to approximate the diaphragmatic edges, the defect can be
closed with a running permanent suture [see Figure 6]. Occasionally, large avulsions or
shotgun wounds with extensive tissue loss will require mesh to bridge the defect.
Alternatively, transposition of the diaphragm cephalad one to two intercostal spaces may
allow repair without undue tension. Read more
Feedback on your answer
Collapse
9)
1/1
You are seeing a 20-year-old male in hospital 12 hours after he was admitted for
observation following an MVC. Initially he was complaining of left sided chest wall pain
after his car was t-boned on the passenger side. His initial physical examination showed
bruises on the anterior chest wall and upper abdominal wall. X-rays revealed fractures
of his sixth and seventh ribs on the left (but no pneumothorax or pleural effusion). A
FAST Exam did not show any free intraperitoneal fluid. Currently, he complains of
worsening epigastric pain, left shoulder pain and mild nausea. His current vitals show his
blood pressure is now 95/60 and pulse rate 115 beats/min, and O2 saturation is 96% on
room air. Which of the following is the next best step in treatment?
(Select 1)(1pts)

Repeat ECG and cardiac biomarkers

Repeat PA and lateral chest x-ray

Abdominal CT scan with intravenous contrast

Transesophageal echocardiogram

This patient presents with blunt abdominal trauma with the delayed onset of hypertension
and signs and symptoms worrisome for likely splenic injury. Most common injuries are
to the spleen, liver, and less common injuries are to the hollow viscous organs in the
abdomen. Symptoms and signs suggesting splenic injury include left upper quadrant pain,
abdominal wall contusion, left lower chest wall tenderness, hypotension, and left
shoulder. Pain referred from splenic hemorrhage, hitting the phrenic nerve and diaphragm
(Kehr sign). The initial examination after blunt abdominal trauma can be unremarkable
and the symptoms can occur hours later, indicating ongoing splenic injury. The best
choice here would be an abdominal CT scan with intravenous contrast, only (no oral
contrast is needed because as little utility). This will define organ injury, assess for
presence of bleeding in all abdominal compartments, determine the need for surgery.
The spleen is the second most commonly injured abdominal organ in blunt trauma
patients. Historical studies have reported a 10% mortality with all splenic injuries;
however, isolated splenic injury mortality is less than 1%. The mechanisms of injury are
similar to those seen with liver injuries: motor vehicle collisions, autopedestrian
accidents, and falls. Similar to penetrating trauma to the liver, stab wounds to the spleen
typically result in direct linear tears, whereas gunshot wounds result in significant
cavitary injuries.
Until the 1970s, splenectomy was considered mandatory for all splenic injuries.
Recognition of the immune function of the spleen refocused efforts on splenic salvage in
the 1980s.38,39Following success in pediatric patients, NOM of splenic injuries was
adopted in the adult population and has become the prevailing strategy for blunt splenic
trauma.40

INITIAL EVALUATION AND INJURY GRADING

Addressing the patient's ABC's, examining the patient's abdomen, and performing
adjunctive imaging with FAST and CT are the initial steps of diagnosing a patient's
splenic injury. Hypotension with a positive FAST scan should prompt emergent
laparotomy. For patients with an identified blunt splenic injury on a CT scan, the injury
should be graded according to the AAST injury grading scale [see Table 1].3

Similar to liver injuries, the grade of splenic injury predicts failure rates and complication
rates of NOM. Other findings that should be searched for on a CT scan include contrast
extravasation (is the contrast blush contained within the spleen, or does it spill into the
peritoneum?), the amount of intra-abdominal hemorrhage (is it isolated to the splenic
fossa, or does blood extend into the pelvis?), and the presence of pseudoaneurysms.

NONOPERATIVE MANAGEMENT

Indications for Nonoperative Management

NOM of solid-organ injuries is pursued in hemodynamically stable patients who do not


have overt peritonitis or other indications for laparotomy.41-45 There is no age cutoff for
patients for the NOM of solid-organ injuries.46,47 High-grade injuries, a large amount of
hemoperitoneum, contrast extravasation, and pseudoaneurysms are not absolute
contraindications for NOM; however, these patients are at high risk for failure.48-51 The
identification of contrast extravasation as a risk factor for failure of NOM led to liberal
use of angioembolization. The true value of angioembolization in splenic salvage has not
been rigorously evaluated. Patients with intraparenchymal splenic blushes who are
otherwise asymptomatic may be considered for a period of observation rather than
empiric angioembolization52; it is thought that the contained hemorrhage within the
splenic capsule may result in tamponade of the bleeding

It is clear, however, that 20 to 30% of patients with splenic trauma deserve early
splenectomy and that failure of NOM often represents poor patient selection.53,54 In
adults, indications for prompt laparotomy include initiation of blood transfusion within
the first 12 hours considered to be secondary to the splenic injury or hemodynamic
instability. In the pediatric population, blood transfusions up to half of the patient's blood
volume are used prior to operative intervention. Following the first 12 postinjury hours,
indications for laparotomy are not as black and white. Determination of the patient's age,
comorbidities, current physiology, degree of anemia, and associated injuries will
determine the use of transfusion alone versus intervention with either embolization or
operation. Unlike hepatic injuries, which rebleed in 24 to 48 hours, delayed hemorrhage
or rupture of the spleen can occur up to weeks following injury. Algorithms for the
management of pediatric splenic injuries exist,55 and the patient's physiologic status is
the key determinant. Rapid mobilization in patients who are hemodynamically stable with
a stable hematocrit and no abdominal pain is generally successful. Overall, nonoperative
treatment obviates laparotomy in more than 90% of cases.

Follow-up Imaging

Out of concern over the risk of delayed hemorrhage or other complications, follow-up CT
scans have often been recommended; unfortunately, there is no consensus as to when or
even whether they should be obtained. Patients with grade I or II splenic injuries rarely
show progression of the lesion or other complications on routine follow-up CT scans; it is
reasonable to omit such scans if patients' hematocrits remain stable and they are
otherwise well. Patients with more extensive injuries often have a less predictable course,
and CT may be necessary to evaluate possible complications. Routine CT before
discharge, however, is unwarranted. Outpatient CT, however, in patients who participate
in vigorous or contact sports should be performed at 6 weeks to document complete
healing before resuming those activities. A more convenient and less expensive
alternative to follow-up CT is ultrasonographic monitoring of lesions.

OPERATIVE EXPOSURE AND HEMORRHAGE CONTROL

In penetrating abdominal injuries not suitable for NOM and in blunt abdominal injuries
when NOM is contraindicated or has failed, exploratory laparotomy is performed.

To ensure safe removal or repair, the spleen should be mobilized to the point where it can
be brought to the surface of the abdominal wall without tension. An incision is made in
the peritoneum and the endoabdominal fascia, beginning at the white line of Toldt along
the descending colon and continuing cephalad 1 to 2 cm lateral to the posterior peritoneal
reflection of the spleen; this plane of dissection is continued superiorly until the
esophagus is encountered [see

Figure 15a]. Posteriorly, blunt dissection is performed to mobilize the spleen and
pancreas as a composite away from Gerota fascia and up and out of the retroperitoneum;
this posterior plane may be extended to the aorta if necessary [seeFigure 15b].
Additionally, the attachments between the spleen and the splenic flexure of the colon may
be divided to avoid avulsion of the inferior splenic capsule. Care must be taken not to
pull on the spleen; otherwise, it will tear along the posterior peritoneal reflection, causing
significant hemorrhage. It is often helpful to rotate the operating table 20° to the patient's
right so that the weight of the abdominal viscera facilitates viscera retraction. Any
ongoing hemorrhage from the splenic injury may be temporarily controlled with digital
occlusion of the splenic hilar vessels. Once mobilization is complete, the spleen can be
repaired or removed without any need to struggle to achieve adequate exposure.

DEFINITIVE MANAGEMENT OF INJURIES

Splenic injuries are treated operatively by splenectomy, partial splenectomy, or splenic


repair (splenorrhaphy), based on the extent of the injury and the physiologic condition of
the patient. Splenectomy is indicated for hilar injuries, pulverized splenic parenchyma, or
any grade II or higher injury in a coagulopathic or multiply injured patient. We employ
autotransplantation of splenic implants [see

Figure 16] for partial immunocompetence in younger patients.56 Drains are not used.
Partial splenectomy can be employed in patients in whom only the superior or inferior
pole has been injured. Hemorrhage from the raw splenic edge is controlled with a
horizontal mattress suture, with gentle compression of the parenchyma [see

Figure 17]. Similar to hepatic injuries, splenorrhaphy techniques to achieve hemostasis


include topical agents (electrocautery, argon beam coagulation, thrombin-soaked gelatin
foam sponge, fibrin glue, BioGlue), enveloping the injured spleen in absorbable mesh,
and pledgeted suture repair.

POSTOPERATIVE CARE AND COMPLICATIONS

Enthusiasm for splenic salvage was driven by the rare but often fatal complication of
overwhelming postsplenectomy sepsis (OPSS). OPSS is caused by encapsulated
bacteria,Streptococcus pneumoniae, Haemophilus influenzae,
and Neisseria meningitidis, which are resistant to antimicrobial treatment. In
patients undergoing splenectomy, prevention against these bacteria is provided via
vaccines administered optimally at 14 days but definitely prior to hospital
discharge.57 Vaccines to be administered include Pneumovax (Merck & Co., Inc.,
Whitehouse Station, NJ), Menactra (Sanofi Pasteur, Swiftwater, PA), and Fluvirin
(Novartis, East Hanover, NJ). Revaccination remains open to debate, but some argue for
revaccination every 6 years.
An immediate postsplenectomy increase in platelets and white blood cells (WBCs) is
normal; however, beyond postoperative day 5, a WBC count above 15,000/μL and a
platelet to WBC ratio less than 20 are highly associated with sepsis and should prompt a
thorough search for underlying infection.58,59 A common infectious complication
following splenectomy is a subphrenic abscess, which should be managed with
percutaneous drainage. Following splenectomy or splenorrhaphy, postoperative
hemorrhage may be attributable to loosening of a tie around the splenic vessels, a missed
short gastric artery, or recurrent bleeding from the spleen if splenic repair was used.
Additional sources of morbidity include a concurrent but unrecognized iatrogenic injury
to the pancreatic tail during rapid splenectomy, resulting in pancreatic ascites or fistula.

2014. Scientific American Surgery. Hamilton, Ontario & Philadelphia, PA. Decker
Intellectual Properties Inc. ISSN 2368-2744. STAT!Ref Online Electronic Medical
Library. http://online.statref.com/Document.aspx?fxId=61&docId=2297. 10/17/2014
4:45:39 AM CDT (UTC -05:00).

Feedback on your answer


Collapse
Unit 3
10)
0/1
According to contemporary data, what percentage of blunt hepatic injuries may be
managed nonoperatively?
(Select 1)(1pts)

<10%

25%

50%

>80%

The answer is D, > 80%. Over 80% of patients with liver injuries may be managed
nonoperatively. The liver is the most commonly injured solid organ in blunt trauma,
comprising 5% of all trauma admissions, and because of its size is frequently involved in
penetrating trauma. Following blunt trauma, the most commonly injured structures are
the parenchyma and hepatic veins. Blunt forces dissipate along segments of the liver and
along the fibrous coverings of the portal triad structures; the hepatic veins, however, are
not as resilient. Stab wounds typically result in direct linear tears, whereas gunshot
wounds or shotgun wounds result in significant cavitary injuries attributable to blast
effect and the "tumbling" of the missile within the liver parenchyma. Thus, arterial injury
is more common with penetrating trauma. Biliary trauma is more common with
central/hilar trauma, either blunt or penetrating.
Nonoperative management (NOM) of liver injuries is now the prevailing therapeutic
strategy for blunt hepatic trauma and can be employed for isolated right subcostal
penetrating wounds. Several concurrent changes resulted in this paradigm change. Over
80% of patients with liver injuries may be managed nonoperatively. One of the early
studies to test the application of NOM in 1995 supported its broad application, with an
overall success rate greater than 85% in hemodynamically stable patients, despite
substantial hemoperitoneum documented by CT.11 Of the 8% of patients who failed
NOM, half required operation as a result of associated injuries (i.e., enteric or pancreatic
injuries), whereas half underwent laparotomy for hepatic-related hemorrhage. Patients
who require intervention for hemorrhage typically fail NOM in the first 24 to 48
hours.2,11,17 Patients who fail NOM due to associated enteric or pancreatic injury have a
more variable time frame to presentation17; half manifested symptoms within 48 hours,
with the remainder becoming symptomatic up to 3 weeks later. Perhaps not surprisingly,
those patients who failed NOM had failure rates associated with increasing grades of
hepatic injury, with grade V injuries having a greater than 20% failure rate. Subsequent
studies have reported failure rates of 14% in grade IV injuries and 23% in grade V
injuries.12 The most recent analysis of the National Trauma Data Bank of severe blunt
liver injuries (grade IV and V) identified that initial NOM occurred in 73%, with a failure
rate of 7%.16 Interestingly, failure of NOM was associated with higher mortality.
Predictors of failure of NOM included increasing age, male sex, increasing Injury
Severity Score, decreasing Glasgow Coma Scale score, and hypotension. A similar study
of high-grade liver injuries identified a similar pattern with NOM initiated in 66%
patients with a failure rate of 9%.18 The amount of hemoperitoneum evident on a CT
scan appears to correlate with successful management; patients with a large amount of
hemoperitoneum (i.e., blood extending into the pelvis) are more likely to fail NOM.
However, predicting which patients will ultimately require laparotomy has yet to be
accomplished. Read more
Feedback on your answer
Collapse
11)
0/1
The liver is the most commonly injured solid organ in blunt trauma, comprising 5% of all
trauma admissions. With that being said, over 80% of patients with liver injuries may be
managed nonoperatively. Which of the following is not a predictor of nonoperative
management?
(1pts)

Female sex

Increasing age

Decreasing Glasgow Coma Scale score

Hypotension
Increasing Injury Severity Score

The answer is A, female sex. Over 80% of patients with liver injuries may be managed
nonoperatively. One of the early studies to test the application of NOM in 1995 supported
its broad application, with an overall success rate greater than 85% in hemodynamically
stable patients, despite substantial hemoperitoneum documented by CT.11 Of the 8% of
patients who failed NOM, half required operation as a result of associated injuries (i.e.,
enteric or pancreatic injuries), whereas half underwent laparotomy for hepatic-related
hemorrhage. Patients who require intervention for hemorrhage typically fail NOM in the
first 24 to 48 hours.2,11,17 Patients who fail NOM due to associated enteric or pancreatic
injury have a more variable time frame to presentation17; half manifested symptoms
within 48 hours, with the remainder becoming symptomatic up to 3 weeks later. Perhaps
not surprisingly, those patients who failed NOM had failure rates associated with
increasing grades of hepatic injury, with grade V injuries having a greater than 20%
failure rate. Subsequent studies have reported failure rates of 14% in grade IV injuries
and 23% in grade V injuries.12 The most recent analysis of the National Trauma Data
Bank of severe blunt liver injuries (grade IV and V) identified that initial NOM occurred
in 73%, with a failure rate of 7%.16 Interestingly, failure of NOM was associated with
higher mortality. Predictors of failure of NOM included increasing age, male sex,
increasing Injury Severity Score, decreasing Glasgow Coma Scale score, and
hypotension. A similar study of high-grade liver injuries identified a similar pattern with
NOM initiated in 66% patients with a failure rate of 9%.18 The amount of
hemoperitoneum evident on a CT scan appears to correlate with successful management;
patients with a large amount of hemoperitoneum (i.e., blood extending into the pelvis) are
more likely to fail NOM. However, predicting which patients will ultimately require
laparotomy has yet to be accomplished. Read more
Feedback on your answer
Collapse
12)
0/1
What is the most common abdominal organ injured in the setting of blunt abdominal
trauma?
(Select 1)(1pts)

Spleen

Liver

Kidneys

Small Intestines
The liver is the most commonly injured solid organ in blunt trauma, comprising 5% of all
trauma admissions, and because of its size is frequently involved in penetrating trauma.
Following blunt trauma, the most commonly injured structures are the parenchyma and
hepatic veins. Blunt forces dissipate along segments of the liver and along the fibrous
coverings of the portal triad structures; the hepatic veins, however, are not so insulated.
Given its size and location within the abdomen, the liver is also commonly involved in
penetrating trauma. Stab wounds typically result in direct linear tears, whereas gunshot
wounds or shotgun wounds result in significant cavitary injuries attributable to blast
effect and the "tumbling" of the missile within the liver parenchyma. Thus, arterial injury
is more common with penetrating trauma.
Over the past 20 years, nonoperative management (NOM) of liver injuries has evolved to
become the prevailing therapeutic strategy for blunt hepatic trauma. Several concurrent
changes resulted in this paradigm change. First was the realization that diagnostic
peritoneal lavage (DPL) was sensitive but not specific for identifying intraperitoneal
hemorrhage that necessitated operative management. Surgeons recognized that many
laparotomies undertaken for a positive DPL were associated with liver injuries that did
not require intervention for bleeding.1Second, trauma surgeons noted that nonbleeding
hepatic venous injuries, if manipulated at laparotomy, often resulted in more hemorrhage
and sometimes even death.2 Furthermore, it became conspicuous that with hemostasis
achieved in the operating room, recurrent postoperative bleeding was rare. Therefore,
surgeons queried whether hepatic venous injuries, which are low-pressure system
injuries, could heal without intervention. Finally, computed tomography (CT) provided a
reliable method for diagnosing and grading liver injuries.

The spleen is the second most commonly injured abdominal organ in blunt trauma
patients. Historical studies have reported a 10% mortality with all splenic injuries;
however, isolated splenic injury mortality is less than 1%. The mechanisms of injury are
similar to those seen with liver injuries: motor vehicle collisions, autopedestrian
accidents, and falls. Similar to penetrating trauma to the liver, stab wounds to the spleen
typically result in direct linear tears, whereas gunshot wounds result in significant
cavitary injuries.

Until the 1970s, splenectomy was considered mandatory for all splenic injuries.
Recognition of the immune function of the spleen refocused efforts on splenic salvage in
the 1980s.38,39Following success in pediatric patients, NOM of splenic injuries was
adopted in the adult population and has become the prevailing strategy for blunt splenic
trauma.

Duodenal and pancreatic injury continues to challenge the trauma surgeon. The relatively
rare occurrence of these injuries, the difficulty in making a timely diagnosis, and high
morbidity and mortality rates justify the anxiety these unforgiving injuries invoke.
Mortality rates for pancreatic trauma range from 9 to 34%, with a mean rate of 19%.
Duodenal injuries are similarly lethal, with mortality rates ranging from 6 to 25%.
Complications following duodenal or pancreatic injuries are alarmingly frequent,
occurring in 30 to 60% of patients.1-3 Recognized early, the operative treatment of most
duodenal and pancreatic injuries is straightforward, with low morbidity and mortality.
2014. Scientific American Surgery. Hamilton, Ontario & Philadelphia, PA. Decker
Intellectual Properties Inc. ISSN 2368-2744. STAT!Ref Online Electronic Medical
Library. http://online.statref.com/Document.aspx?fxId=61&docId=2329. 10/27/2014
1:39:15 PM CDT (UTC -05:00).

Feedback on your answer


Collapse
13)
0/1
You are seeing a 34-year old female in the trauma following an MVC. He is complaining
intense diffuse abdominal pain after his car was hit from behind causing him to spin out
and crash into a wall. Initially he had a GCS of 15. His vitals showed a BP of 125/76, HR
95, RR 18, O2 of 98% on room air. On physical exam his lungs are clear and his
abdomen shows diffuse tenderness to light palpation and ecchymosis over his
epigastrium. His initial fast exam is negative but technically limited and you order a CT
scan with IV contrast of his abdomen and pelvis as you suspect a possible liver
injury. What description from the options below accurately describes a grade IV liver
laceration on CT imaging?
(Select 1)(1pts)

Parenchymal distruption involving 25 - 75% hepatic lobes

Parenchymal distruption involving >75% helpatic lobe

Intraparenchymal laceration < 10cm diameter

Sub capsular hematoma, > 50% surface area, or ruptured with active
bleeding

The answer is A. Parenchymal distruption involving 25 - 75% hepatic lobes. Read


more
The AAST Liver injury grading system is as follows
 grade I :
 haematoma: sub capsular, < 10% surface area
 laceration: capsular tear, < 1cm depth
 grade II :
 haematoma: sub capsular, 10 - 50% surface area
 haematoma: intraparenchymal < 10cm diameter
 laceration: capsular tear, 1 - 3cm depth, < 10cm length
 grade III :

haematoma: sub capsular, > 50% surface area, or ruptured with
active bleeding
 haematoma: intraparenchymal > 10 cm diameter
 laceration: capsular tear, > 3 cm depth
 grade IV :
 haematoma: ruptured intraparenchymal with active bleeding
 laceration: parenchymal distruption involving 25 - 75% hepatic
lobes or
 1 - 3 Couinaud segments (within one lobe)
 grade V :
 laceration: parenchymal distruption involving >75% helpatic
lobe or
 > 3 Couinaud segments (within one lobe)
 vascular: juxtahepatic venous injuries (IVC, major hepatic vein)
 grade VI : vascular: hepatic avulsion

Feedback on your answer


Collapse
Unit 4
14)
0/1
You are seeing a 24-year old female who is 4-weeks post postpartum in your clinic. She
is complaining of bilateral nipple pain over the last two days along with a nipple rash.
She notes pain both with breastfeeding and even when she is not feeding. The pain is so
bad that she didn't breastfeed her baby this morning because of the pain. Physical exam
shows erythema to both breasts along with cracking around both areolas. There are no
signs of an abscess and no induration present. What is the most likely cause of this
patient's symptoms?
(Select 1)(1pts)

Engorgement

Mastitis

Deep space abscess

Candida infection

None of the above

The answer is D, Candida infection. In breastfeeding women, bilateral nipple pain with
and between feedings after initial soreness has resolved is usually due to Candida. Pain
from engorgement typically resolves after feeding. Mastitis is usually unilateral and is
associated with systemic symptoms and wedge-shaped erythema of the breast tissue.
Improper latch-on is painful only during feedings. Eczema isolated to the nipple, while a
reasonable part of the differential, would be much more unusual.

Ref: Jatoi I, Kaufman M (eds): Management of Breast Diseases. Springer, 2010,


pp 95-96.
Feedback on your answer
Collapse
15)
0/1
You are seeing a 25-year old white female who is presenting to your clinic with bloody
nipple discharge over the last 3 days. She denies any history of a breast lump, she is not
breast feeding and denies family history of breast cancer. Her exam shows no palpable
masses but there is scant bloody discharge coming from her left nipple. You order a
mammogram that does not show any suspicious lesions. What is the most likely
diagnosis?
(Select 1)(1pts)

Fibroadenoma

Intraductal papilloma

Cystosarcoma Phyllodes

Mammary dysplasia

The answer is B, Intraductal papilloma. The old concern over cancer is the issue, and the
way to detect cancer that is not palpable is with a mammogram. That should be the first
choice. If negative, one may still wish to find an resect the intraductal papilloma to
provide symptomatic relief. Intraductal papillomas consist of a monotonous array of
papillary cells that grow from the wall of a cyst into its lumen. Although they are not
concerning in and of themselves, they can harbor areas of atypia or ductal carcinoma in
situ (DCIS). Papillomas can occur as solitary or multiple lesions. The standard approach
to a papilloma diagnosed by core needle biopsy (CNB) is to perform a surgical excision,
particularly if atypical cells are identified [14,16-21]. In a meta-analysis of 34 studies
that included 2236 non-malignant breast papillary lesions, 346 (15.7 percent) were
upgraded to malignancy following a surgical excision [21]. Because of a risk of
malignancy, these require surgical excision.Read More
Feedback on your answer
Collapse
16)
1/1
A 25 -ear obese black female presents to you with a painful breast lump along with
tenderness in both her breasts. She notes recurrent pain and multiple lumps on both
breasts that seem to “come and go” at different times during her menses. Now she has a
firm, round, mass that has not gone away for the last 5 weeks. What is the most likely
diagnosis?

(Select 1)(1pts)

Cystosarcoma Phyllodes

Intraductal papilloma

Fibrocystic disease

Fibroadenoma

The answer is C. This patient has fibrocystic disease which is a nonproliferative epithelial
lesions that are generally not associated with an increased risk of breast cancer [1]. It
should be noted that terms such as fibrocystic changes, fibrocystic disease, chronic cystic
mastitis, and mammary dysplasia refer to nonproliferative lesions and are not useful
clinically, as they encompass a heterogeneous group of diagnoses [5,11]. The most
common nonproliferative breast lesions are breast cysts. Other nonproliferative lesions
include papillary apocrine change, epithelial-related calcifications, and mild hyperplasia
of the usual type [5]. Apocrine metaplasia (also referred to as a "benign epithelial
alteration") is also a nonproliferative change that is secondary to some form of irritation,
typically associated with a breast cyst. Read More
Feedback on your answer
Collapse
17)
1/1
A 30-year old white female presents to your office with right breast pain. She is currently
breast feeding her healthy 4-week old infant and has been having focal tenderness,
swelling and redness to her right breast near the nipple over the last 2 days. She reports
no past medical history and is a non smoker. Given the patient’s history and physical,
which of the following inflammatory disorders is the most likely diagnosis?
(Select 1)(1pts)

Acute mastitis

Granulomatous mastitis

Mammary duct ectasia

Benign severe breast engorgement


The answer is A. Lactational mastitis is a localized, painful inflammation of the breast
associated with fever and malaise that occurs in breastfeeding women and has been
estimated to occur in 2 to 10 percent of breastfeeding women Acute mastitis almost
always occurs during the first month of breastfeeding. The diagnosis of mastitis is made
clinically. Lactational mastitis typically presents as a hard, red, tender, swollen area of
one breast associated with fever >38.3ºC in a nursing mother. Other systemic complaints
may variably include myalgia, chills, malaise, and flu-like symptoms. In the early stages
of breast infection the presentation can be subtle with few clinical signs, while patients
with advanced infection may present with a large area of breast swelling with overlying
skin changes (eg, erythema). Reactive lymphadenopathy can also cause axillary pain and
swelling. Septic shock rarely occurs.In a lactating woman, severe engorgement can be
distinguished from mastitis because engorgement is bilateral with generalized
involvement [1]. Mastitis associated with erythema and edema during is uncommon and
a diagnosis of inflammatory breast cancer must be excluded The most common
organisms causing the infections are S. aureus and streptococcal species. It may also be a
complication of nipple piercing. “D” is incorrect because women often present with
inversion of the nipple and over 90% of women affected are smokers. Read more
Feedback on your answer
Collapse
Unit 5
18)
0/1
Which of the following is considered a benign lesion that usually does not require cancer
treatment?
(Select 1)(1pts)

Lobular carcinoma in situ

Ductal carcinoma in situ

Medullary carcinoma

Mucinous carcinoma

The answer is A, Cancer that is confined to the lumen of the duct or


lobule of the breast and has not penetrated the basement membrane
is termed in situ cancer. This generally refers to ductal carcinoma in
situ (DCIS) but also encompasses a benign entity called lobular
carcinoma in situ (LCIS). Small uniform cells confined to the lobule of
the breast characterize LCIS. It is generally a clinically and
mammographically occult lesion that is identified only incidentally
when a biopsy is performed for calcifications or a mass that proves to
be some other benign lesion. LCIS is actually not cancer but rather is a
benign lesion and does not require cancer treatment. The primary
issue with LCIS is that it conveys an increased lifelong risk of
subsequent invasive cancer quantified at 0.5 to 0.75% per year. In
addition, when LCIS is identified on a core-needle biopsy, there is a 10
to 20% chance of DCIS or invasive cancer in the surrounding tissue;
therefore, surgical excision is warranted. Long-term follow-up shows
that the large majority of women with LCIS never develop invasive
breast cancer. Therefore, ablative surgical therapy and radiation for
LCIS are not necessary. Previously, LCIS was considered in and of
itself an indication to consider bilateral mastectomy. However,
mastectomy is generally not indicated in women with LCIS and should
be performed only in the context of risk reduction for those at very
high risk related to factors such as inherited susceptibility. Because
women with a diagnosis of LCIS are at increased risk for subsequent
invasive cancer, they should be counseled regarding that risk and may
benefit from consultation with genetics professionals if they have a
family history of breast or ovarian cancer. Women with a biopsy
showing LCIS may also consider risk-reducing chemoprevention with
one of the selective estrogen receptor modulators (SERM's), tamoxifen
or raloxifene. These reduce the risk of subsequent invasive cancer by
about 50%, with an acceptable toxicity profile.20,21 Raloxifene is the
preferred agent in postmenopausal women. Read more on Breast
Cancer
Feedback on your answer
Collapse
19)
1/1
True or False: A clinically suspicious breast mass should be biopsied regardless of
imaging findings such as a negative mammogram.
(1pts)

True Correct

False

True: Algorithms for clinical and imaging evaluation of palpable masses are stratified by
the age of the woman. Even in the setting of palpable masses, image guidance may
improve diagnostic accuracy. A clinically suspicious mass should be biopsied regardless
of imaging findings, as 10 to 15 percent of such lesions can be mammographically
occult. Read more

Feedback on your answer


Collapse
20)
0/1
You are seeing a 43-year old Hispanic female who is presenting with an enlarging mass
in her right breast. It has been present for several years and growing to it’s present size.
On examination of her right breast you note a large 10 cm mass that is mobile, firm
and rubbery. There are no palpable axillary nodes. What is the most likely diagnosis?

(Select 1)(1pts)

Fibroadenoma

Fibrocystic disease

Intraductal papilloma.

Cystosarcoma Phyllodes

The answer is D. This is a Phyllodes tumors, which are an uncommon fibroepithelial


breast tumors that are capable of a diverse range of biologic behavior. In their least
aggressive form, they behave similarly to benign fibroadenomas, although with a
propensity to recur locally following excision without wide margins. At the other end of
the spectrum are tumors that metastasize distantly, sometimes degenerating histologically
into sarcomatous lesions that lack an epithelial component. Phyllodes tumors account for
fewer than 0.5 percent of all breast malignancies. The vast majority occur in women, in
whom the median age at presentation is 42 to 45 (range 10 to 82 years).and there was a
higher incidence in Latina whites, as compared to non-Latina whites, Asians, and African
American women. Read More
Feedback on your answer
Collapse
21)
0/1
True or False: When approaching a patient with an abnormal mammogram for a
suspicious breast mass the next best step is a surgical excision is preferred over
percutaneous needle biopsy.
(1pts)

True Incorrect

False

False: Breast lesion suspicious for malignancy requires tissue biopsy.


Percutaneous needle biopsy is preferred over surgical excision in all
circumstances. Surgical excision as a diagnostic procedure is not a
justifiable alternative simply because of "patient choice" and should be
performed only when needle biopsy cannot be performed for specific
technical reasons, when a needle biopsy is either nondiagnostic, the
result is not concordant with the imaging findings (i.e., the needle
biopsy is benign, but the lesion is of high suspicion), or in highly select
other cases. Technical reasons that may preclude needle biopsy
include anatomic location of the lesion on mammography directly
opposed to the chest wall or in the far periphery of the breast so that
it cannot be visualized on stereotactic imaging devices. Breast Cancer
Feedback on your answer
Collapse
Unit 6
22)
0/1
A 23-year old male patient presents with a human bite to his right hand after an
altercation just prior to arrival. He has no past medical history and is up to date with
tetanus. Exam shows there are two small puncture wounds overlying the dorsal aspect of
his right hand over his 1stmetacarpal and does not seem to involve any
tendons or ligaments. The wound does not appear amenable to sutures for
closure and you decide to copiously irrigate it with saline and discharge him home. What
antibiotic should he receive for wound care prophylaxis?
(1pts)

Clindamycin

Flagyl

Oxacillin

Augmentin

You didn't make any selections


The answer is D. Augmentin. The most common bacterial etiology in human bites is strep
viridans. You are however covering for Eikenella corrodens which is a gram negative rod
which is susceptible to pcn but resistant to flagyl, clindamycin, first generation
cephalosporins and erythromycin. It is susceptible to flouroquinolones, bactrim and
augmentin. The most important treatment is good irrigation and initial wound
cleaning. Read more
Feedback on your answer
Collapse
23)
0/1
Which of the following statements regarding soft tissue infections is not true?
(Select 1)(1pts)

As many as 20% of necrotizing soft tissue infections are primary


(idiopathic) and occur in previously healthy patients who have no
predisposing factors and no known portal of entry for bacterial inoculation
Crepitus is noted in the majority of patients with necrotizing soft tissue
infections

Physical findings characteristic of a necrotizing infection include


tenderness beyond the area of erythema, crepitus and cutaneous
anesthesia

Fewer than 40% of patients with Patients with Necrotizing Soft Tissue
Infections exhibit the classic symptoms and signs described

The answer is B, crepitus is noted in only 30% of patients with necrotizing soft tissue
infections. Patients with necrotizing soft tissue infections often complain of severe pain
that is out of proportion to their physical findings. Compared with patients who have
nonnecrotizing infections, they are more likely to have fever, bullae, or blebs [see Figure
1]; signs of systemic toxicity; hyponatremia; and leukocytosis with a shift in immature
forms. Physical findings characteristic of a necrotizing infection include tenderness
beyond the area of erythema, crepitus, cutaneous anesthesia, and cellulitis that is
refractory to antibiotic therapy.6 Tenderness beyond the borders of the erythematous area
is an especially important clinical clue that develops as the infection in the deeper
cutaneous layers undermines the skin. Early in the course of a necrotizing soft tissue
infection, skin changes may be minimal despite extensive necrosis of the deeper
cutaneous layers. Bullae, blebs, cutaneous anesthesia, and skin necrosis occur as a result
of thrombosis of the nutrient vessels and destruction of the cutaneous nerves of the skin,
which typically occur late in the course of infection.
Clinicians should be mindful of certain diagnostic barriers that may delay recognition and
treatment of necrotizing soft tissue infections.7 In particular, these infections have a
variable clinical presentation. Although most patients present with an acute, rapidly
progressive illness and signs of systemic toxicity, a subset of patients may present with a
more indolent, slowly progressive infection. Patients with postoperative necrotizing
infections often have a more indolent course. Moreover, in the early stages, underlying
necrosis may be masked by normal-appearing overlying skin. As many as 20% of
necrotizing soft tissue infections are primary (idiopathic) and occur in previously healthy
patients who have no predisposing factors and no known portal of entry for bacterial
inoculation. Finally, crepitus is noted in only 30% of patients with necrotizing soft tissue
infections. Overall, fewer than 40% of patients exhibit the classic symptoms and signs
described.7,8Accordingly, it is imperative to maintain a high index of suspicion for this
disease in the appropriate setting.Read More on Soft Tissue Infection
Feedback on your answer
Collapse
24)
0/1
You are seeing a 30-year-old male wrestler who is presenting with a painful lesion on his
upper back. He first noted a small painful area a few days ago, and the lesion has since
enlarged and became more red. He notes that others on his wrestling team have similar
lesions and have sought care elsewhere. He has no past medical history and takes no
medications.
Physical Examination of the upper back reveals a 1 × 1 cm erythematous, raised pustule
that is tender to palpation, with a 4 × 4 cm area of surrounding erythema. The remainder
of the physical examination is normal. You elect to perform an incision and drainage, and
culture. Of the following, which is the most appropriate empiric treatment?
(Select 1)(1pts)

Levofloxacin

Doxycycline

Dicloxacillin

Cephalexin

Ciprofloxacin

All of the above

None of the above

The answer is B, Purulence is more often present in staphylococcal cellulitis. Specimens


from wounds, carbuncles, or furuncles may reveal gram-positive cocci in clusters,
consistent with S. aureus. Clinicians must judge whether MRSA or MSSA is likely.
Appropriate choices for the oral treatment of MSSA include cloxacillin; dicloxacillin;
cephalexin; and, for penicillin-allergic patients, clindamycin or a macrolide
(erythromycin, clarithromycin, or azithromycin). However, patients treated with
cephalexin should be observed for treatment failure, as some studies have reported high
failure rates in adults (29), possibly due to poor absorption (30). Appropriate orally
administered choices for the treatment of MRSA include trimethoprim–
sulfamethoxazole, doxycycline, minocycline, or linezolid. This patient likely has a
community acquired MRSA as others on his wrestling team have similar lesions and thus
MRSA targeted therapy is best. Read More on Cellulitis and Soft-Tissue
Infections
Feedback on your answer
Collapse
25)
0/1
Which of the following choices best differentiates cellulitis from erysipelas?
(Select 1)(1pts)

Presence of fever
Presence of leukocytosis

Depth of inflammation

Presence of purulence

All of the above

None of the above

The answer is C, depth of inflammation. Cellulitis and erysipelas are diffuse spreading
skin infections that not associated with underlying suppurative foci. Clinically, there is
often some degree of overlap between the two different entities. Erysipelas is
differentiated from cellulitis by the depth of inflammation; erysipelas affects the upper
dermis, including the superficial lymphatics, whereas cellulitis affects the deeper dermis
and subcutaneous fat (Mayo Clin Proc, Vol. 89;1436). Read More onCellulitis and
erysipelas from Up to Date

Unit 1
1)
0/1
You are seeing a 42-year old male who is presenting with flank pain following
an MVC. On physical exam he has a large degree of flank ecchymosis along with gross
hematuria. You are suspecting a kidney injury and he appears stable for CT at this time.
What renal injuries in the setting of trauma require surgical management?

(Select 1)( 1pts extra credit)

Vascular Injury

Expanding Hematoma

Shattered Kidney

All of the above

None of the above

The answer is D. All of the above. Generally about 80-90% renal injuries treated
conservatively with remarkable resolution! Injuries requiring surgery: vascular injury,
shattered kidney and an expanding hematoma.
The goal in the management of renal trauma is to safely preserve the maximal number of
renal units, avoiding unnecessary explorations, repairs, and nephrectomies. Increasing
numbers of renal injuries, including grade IV and V injuries, are being been managed
nonoperatively. The accuracy and rapidity of helical CT, combined with the
improvements achieved in resuscitation methods, have reduced the number of renal
explorations performed.8 Currently, 36% of all penetrating renal injuries and fewer than
5% of blunt injuries necessitate operative management.9 All grade I and II renal injuries,
regardless of the mechanism of injury, can be managed with observation alone because
the risk of delayed bleeding is extremely low. Most grade III and IV injuries, including
those with devitalized parenchymal fragments and urinary extravasation, can be managed
nonoperatively with close monitoring, serial hematocrit measurement, and repeat imaging
in selected cases. Active arterial bleeding, in the absence of other associated injuries, can
be treated with emergency arteriography and selective angioembolization.

The only absolute indications for renal exploration are pedicle avulsion, pulsatile or
expanding hematoma, and hemodynamic instability resulting from renal
injury.6 Significant numbers of shattered kidneys (grade V) and renal vascular injuries
(grades IV and V) are now managed nonoperatively. The strongest predictor for
nephrectomy is severity of renal injury; however, roughly one third of all penetrating and
44% of blunt grade IV and V renal injuries are now managed nonoperatively.9,13,14In
patients who require laparotomy for associated injuries, renal exploration and
reconstruction of grade III and IV injuries may reduce the likelihood of delayed
complications. However, the need for laparotomy and surgery for other intra-abdominal
organs is associated with a higher likelihood of nephrectomy. Thus, exploration of
suspected kidney injuries (as determined by previous imaging or on-table evaluation) in
patients undergoing laparotomy for major splenic or bowel injury should be attempted by
surgeons experienced in repairing an injured kidney.9 In reality, the success of
nonoperative management for most grade III and IV injuries means that operative
intervention in cases of blunt trauma is typically limited to patients with the most severe
renal injuries, in whom conservative management fails either because of bleeding or
because of ongoing urinary extravasation despite ureteral stenting.15

A significant number of patients with a penetrating injury and a minority of those with
blunt trauma require immediate laparotomy before radiographic
evaluation.9,13 Hematuria should alert the surgeon to the possibility of renal injury, and
the presence of a perinephric hematoma visible through the mesocolon should prompt
further evaluation. If a major renal injury is suspected on the basis of the size of the
hematoma or an abnormal intraoperative intravenous pyelogram (IVP), exploration is
indicated.
2014. Scientific American Surgery. Hamilton, Ontario & Philadelphia, PA. Decker
Intellectual Properties Inc. ISSN 2368-2744. STAT!Ref Online Electronic Medical
Library. http://online.statref.com/Document.aspx?fxId=61&docId=2366. 10/20/2014
1:04:32 PM CDT (UTC -05:00).

Feedback on your answer


Collapse
2)
0/1
You are seeing a 30-year old male in the trauma bay who was an unrestrained driver
involved in a rollover MVC who is complaining of lower abdominal pain. His GCS is
15, vitals are stable and he is awake and talking. The rest of his exam shows he has
moderate right lower abdomen and flank tenderness to palpation without guarding.
His iStat labs show a Hct 45.7, BUN 15, Cr 1.2. Upright CXR is shows no signs of
trauma and there is a normal sized medastinum. CT scan of his abd/pelvis demonstrates a
small renal subcapsular hematoma with 0.5 cm superficial parenchymal laceration that
does not involve the collecting system. According to the American Association for the
Surgery of Trauma (AAST), what grade injury does this demonstrate?

(Select 1)( 1pts extra credit)

Grade I

Grade II

Grade III

Grade IV

The patient above has a grade II renal laceration as he has a


<1cm renal laceration,Read more
Renal trauma grading is often done using theAmerican Association for
the Surgery of Trauma (AAST)3-4 according to depth of damage and
involvement of the urinary collecting system and renal vessels.
grade I - contusion or non enlarging subcapsular haematoma, but no

laceration ;
 grade II - superfical laceration < 1cm depth and does not involve the
collecting system ; non expanding perirenal haematoma
 grade III - laceration > 1cm, without extension into the renal pelvis
or collecting system and with no evidence of urine extravasation
 grade IV - laceration extends to renal pelvis or urinary extravasation.
 grade V - shattered kidney ; devascularisation of kidney due to hilar
injury.
Read more from Up to Date

Feedback on your answer


Collapse
3)
0/1
You are seeing a frail 78-year old male after he fell getting out of bed striking his left
chest wall on the nightstand. He is now complaining of severe left sided chest wall pain.
His exam shows him to be in discomfort with a blood pressure of 110/68, HR 95
beats/min, RR 20, O2 of 95% on room air. His left chest wall has ecchymosis and is very
tender to palpation without crepitus. His chest x-ray shows 5 non displaced rib fractures
of the 5th-9th ribs on the left without of pneumothorax or pulmonary contusion. You
have decided to admit him for pain control and observation as he is only getting minimal
relief from IV ketorolac and morphine. Out of the options below what is the best
treatment option for this patient while he is an inpatient?

(Select 1)( 1pts extra credit)

Supplemental Oxygen via non rebreather

Intercostal block

Chest wall binding to limit respiration

Surgical fixation

The answer is B, this patient needs sufficient analgesia to prevent respiratory


hypoventilation, atelectasis and pneumonia. Read More
Choice of analgesia
We prefer to use epidural analgesia for patients who do not have
contraindications to placement of a thoracic epidural catheter.
 Patients who cannot have an epidural should be managed with parenteral
opioid agents administered intravenously or via patient-controlled
analgesia (PCA), if possible, with consideration of supplementation with
non-steroidal antiinflammatory drugs, if not contraindicated.
 For patients failing to achieve adequate pain control with intravenous
agents, local anesthetic blocks are a useful option.
Pain control is fundamental to the management of rib fractures to decrease chest wall
splinting and alveolar collapse. Patients with pain due to rib fractures seek to minimize
their chest wall motion by reducing their tidal volume and coughing effort. Adequate pain
management improves patient tolerance for deep breathing and coughing, which
improves lung volume and clears secretions, thereby decreasing the risk for pneumonia.
Options for analgesia — A number of strategies have been proposed to optimize
pain control in patients with rib fractures. The involvement of a dedicated pain service is
critical to provide tailored therapy and monitor its effectiveness.
Regional anesthesia — Regional anesthesia techniques available for the
management of multiple rib fractures include continuous epidural infusion, paravertebral
block, intrapleural infusion, and intercostal nerve block [38]. Of these, we prefer to use
epidural infusion in those patients who have no contraindications.
Continuous epidural infusion — The use of epidural catheters in patients with
multisystem trauma is often limited due to contraindications for epidural catheter
placement (eg, spine fractures, coagulopathy) [39-41]. When used, epidural analgesia is
associated with a shorter duration of mechanical ventilation and decreased incidence of
nosocomial pneumonia. All patients, but particularly older patients, managed with
epidural analgesic infusion should be closely monitored for side effects.
The most widely studied approach uses epidural catheters to infuse local anesthetics with
or without the addition of narcotic agents. A systematic review and metaanalysis of eight
randomized trials did not find any differences in the need for mechanical ventilation,
length of intensive care unit (ICU) stay, or mortality in patients managed with epidural
analgesia compared with those who were not [40]. However, the duration of mechanical
ventilation was less for those managed with thoracic epidural but significantly higher
rates of hypotension were found with a thoracic epidural compared with no epidural or
epidural at the lumbar level. Three of the eight included studies found superior pain
control associated with epidural use; however, a pooled analysis of all the data was not
performed.
Other trials not included in this metaanalysis have found improved pulmonary function
and better pain scores for epidural analgesia compared with intravenous narcotics [42-
46]. A review that evaluated the use of epidural catheters reported that for eligible
patients epidural catheters were more likely to be used in trauma centers than nontrauma
centers [47]. The mortality risk was significantly reduced up to one year after injury for
patients with three or more rib fractures who received an epidural catheter relative to
those who did not (odds ratio [OR] death 30 days 0.08, 95% CI 0.01-0.43, OR 60 days
0.09, 95% CI 0.02-0.42, OR 365 days 0.12, 95% CI 0.04-0.42).
Paravertebral block — Paravertebral block is only useful for patients with unilateral
rib fractures, but is associated with a lower rate of systemic hypotension. For clinicians
experienced with this technique, placement of continuous infusion catheters may be
technically easier to perform compared with epidural catheter placement. Two studies
have found this technique to be as effective as epidural for controlling pain associated
with unilateral rib fractures [48,49].
Intercostal nerve blocks — Intermittent intercostal nerve block can also be used to
control pain associated with rib fractures, but is limited by the duration of the block and,
for patients with multiple rib fractures, the need to perform the procedure at multiple
intercostal levels. Repeated blockade is needed for prolonged relief.
A continuous infusion device is available that requires surgery for placement, which has
been performed at the bedside in intubated patients [50]. As such, given other options for
pain control, this approach is generally not applicable to the majority of multiple rib
fracture patients.
Intrapleural infusion — Another approach involves infusion of local anesthetics
directly into the pleural space. The catheter used to administer the anesthetic can be
placed adjacent a thoracostomy tube at the time of tube placement. One placebo
controlled trial study failed to find clear benefit with this approach [51]. Another study,
which compared intrapleural with epidural analgesia, found the epidural approach
associated with better pain relief [52].
Intravenous narcotics — Intravenous narcotics (eg, morphine) are preferred over
subcutaneous or intramuscular injection due to a rapid and predictable onset of action.
The management of pain with intravenous narcotics is discussed in detail elsewhere.
Although there is no evidence specific to the management of rib fractures, intravenous
nonsteroidal anti-inflammatory drugs (eg,ketorolac) can be used to supplement and
reduce narcotic usage [53]. However, these agents are avoided in patients who have a
significant bleeding risk (eg, hemothorax, solid organ injury) and those with renal
insufficiency or hypovolemia.
Patient controlled analgesia (PCA) is advocated for patients with rib fractures because of
more timely access to pain medication by the patients and a reduced risk for excessive
sedation.

Feedback on your answer


Collapse
4)
1/1

What percentage of patients will have thoracic injuries on CT with no physical


examination findings but with a significant mechanism of injury alone (ie, motor vehicle
crash at > 35 mph, fall from > 15 ft, automobile hitting a pedestrian with pedestrian being
thrown > 10 ft, and assault with depressed level of consciousness)?

(Select 1)( 1pts extra credit)

<5%

20%

35%

50%
The answer is B, 20%. The decision to image is challenging in the wellappearing patient
with a concerning mechanism or intermediate examination. Approximately 20% of
patients with no physical evidence of chest trauma but a concerning mechanism will have
injuries on thoracic CT. Alternatively, patients with a NEXUS chest score of 0 (see Table
2, page 5) have a much lower incidence of significant findings on CT
Some 10% to 23% of patients with minimal findings on examination may still have
significant thoracic injuries.54,59 However, it is unclear what percentage of these injuries
is clinically important. Point tenderness and ecchymosis on the chest wall should raise the
concern for intrathoracic injury; however, these findings are nonspecific.55 Injuries to the
lower ribs may also indicate the presence of intra-abdominal injuries. In a 2005
prospective observational study, 3% of patients with “isolated” subjective pain or point
tenderness to the lower left ribs as the only indication for computed tomography (CT) had
splenic injuries. If patients had other indications for CT (hypotension, abdominal or flank
tenderness, pelvic or femur fractures, or gross hematuria) the rate of splenic injury was
9.4%.Read more

Feedback on your answer


Collapse
5)
0/1
A 26-year-old male presents with severe penile pain that occurred while he was having
sex with his wife. Physical exam shows an uncircumcised, grossly swollen penis is
deviated to the right that resembles an eggplant. There is no blood at the urethral meatus.
Which of the following is the most appropriate next step in management?

(Select 1)( 1pts extra credit)

Surgical exploration of penis

Retrograde urethrogram, followed by surgical exploration of penis

Foley catheterization

Antibiotics, analgesics and anti-inflammatory and follow-up in 24 hours

The answer is B. This patient exhibits a classic presentation of penile fracture. The penis,
due to its mobility in the flaccid state, is typically not injured in cases of blunt trauma. It
is far more vulnerable to trauma in a T Rex state, such as during sexual intercourse, and
this is the setting where penile fractures most commonly countered. This injury most
often occurs during intercourse with a woman is situated on top of the man because the
penis may emerge from the vagina in this position can be subsequently injured in a
bending passion between its rigid fixation of the male in the downwardly moving female
perineum. Patients typically complain of the snapping sensation and/or sound when the
injury occurs, followed by severe pain. The injury and associated snapping sensation
results from tearing of the tunica albuginea, which invests the corpus cavernous. A
hematoma rapidly forms at the site of injury causing bending of the shaft of the penis at
the site of the fracture. Treatment is with an emergent urethrogram to assess for urethral
injury, as well as emergent surgery to evacuate the hematoma and repair the torn tunica
albuginea. Read more

Feedback on your answer


Collapse
Unit 2
6)
0/1
What is the half life of the tumor marker CEA?
(Select 1)(1pts)

12-24 hours

1-2 days

3-5 days

7-14 days

The answer is D, 7-14 days. CEA remains the prototypical solid tumor marker. Despite
its lack of specificity, if used correctly, CEA testing is a valuable addition to the process
of clinical decision making in patients diagnosed with colon or rectal carcinoma.
However, it is not an appropriate screening test. Whether sampled once or serially, CEA
cannot be used in the differential diagnosis of an unknown-but-suspected bowel problem
or malignancy. Nevertheless, when CEA concentrations are determined before primary
tumor resection, they may be of additional prognostic value; this is particularly true in
patients with stage II disease, for whom elevated preoperative CEA is a poor prognostic
marker and may influence the decision regarding whether to administer adjuvant
chemotherapy.
Serial CEA values obtained postoperatively are a potentially effective means of
monitoring response to therapy. A postoperative CEA titer serves as a measure of the
completeness of tumor resection. It should be remembered, however, that the half-life of
CEA is 7 to 14 days; therefore, postoperative baselines are best established several weeks
after resection. If a preoperatively elevated CEA value does not fall to normal within 2 to
3 weeks after surgery, it is likely that (1) the resection was incomplete or (2) occult
metastases are present. A rising trend in serial CEA values from a normal postoperative
baseline (< 5 ng/mL) may predate any other clinical or laboratory evidence of recurrent
disease by 6 to 9 months. Read more

Feedback on your answer


Collapse
7)
0/1
Which one of the following tumor markers is used to monitor for the recurrence colon
cancer?
(1pts)

Prostate-specific antigen (PSA)

Carcinoembryonic antigen (CEA)

Cancer antigen 125 (CA-125)

Cancer antigen 27.29 (CA 27-29)

The answer is D., Carcinoembryonic antigen (CEA). Prostate-specific antigen (PSA) is a


marker that is used to screen for prostate cancer. It is elevated in more than 70% of
organ-confined prostate cancers. Alpha-tetoprotein is a marker for hepatocellular
carcinoma and nonseminomatous germ cell tumor, and is elevated in 80% of
hepatocellular carcinomas. CA-125 is a marker for ovarian cancer. Although it is elevated
in 85% of ovarian cancers, it is elevated in only 50% of early-stage ovarian cancers.
Carcinoembryonic antigen (CEA) is a marker for colon, esophageal, and hepatic cancers.
It is expressed in normal mucosal cells and is overexpressed in adenocarcinoma,
especially colon cancer. Though not specific for colon cancer, levels above 10 ng/mL are
rarely due to benign disease. CEA levels typically return to normal within 4–6 weeks
after successful surgical resection. CEA elevation occurs in nearly half of patients with a
normal preoperative CEA level that have cancer recurrence. Cancer antigen 27.29 (CA
27-29) is a tumor marker for breast cancer. It is elevated in about 33% of early-stage
breast cancers and about 67% of late-stage breast cancers. Some tumor markers, such as
CEA, alpha-fetoprotein, and CA-125, may be more helpful in monitoring response to
therapy than in detecting the primary tumor. Ref: Perkins GL, Slater ED, Sanders GK, et
al: Serum tumor markers. Am Fam Physician 2003;68(6):1075-1082.
Feedback on your answer
Collapse
8)
0/1
You are seeing a 73-year old male who has a past medical history of
CHF, hypertension and type II diabetes. He initially presented to you
with changes in stool patterns, constipation and hematochezia. He was
found to have adenocarcinoma of the ascending colon along with
metastasis to his liver, brain and his right lung. For patient presenting
with metastatic colon cancer, which of the following treatments is not
routinely recommended?
(1pts)
Chemotherapy

Primary Resection of the colon cancer

Radiation

All are routinely recommended

All of the above

The answer is B. In treating Colorectal cancer, it is crucial to


understand that surgical extirpation of the primary tumor is done when
there is a realistic possibility of cure or for patients with symptomatic
tumors that cause acute obstruction or clinically significant bleeding.
For those who present with synchronous primary tumors and incurable
metastatic disease, resection is not routinely indicated. Advances in
systemic chemotherapy (outlined below) have significantly increased
the probability of managing the tumor medically, and chemotherapy
can begin immediately in the setting of an asymptomatic or minimally
symptomatic primary. In other words, there is no need to delay
initiation of systemic chemotherapy by palliative resection of a primary
tumor that is not actively symptomatic. In truth, resection of a primary
lesion in the setting of metastatic disease is likely to cause significant
morbidity and even mortality. A review of Medicare/SEER data
focusing on patients 65 and older reported a 30-day postoperative
mortality of 10% with resection of a synchronous primary tumor.
Furthermore, in a large retrospective series, Poultsides and colleagues
reported that 93% of patients presenting with synchronous stage IV
disease without overt obstruction never required specific intervention
on their primary tumor. Adenocarcinoma of the Colon and Rectum
Feedback on your answer
Collapse
9)
0/1
Which of the following statements regarding colorectal cancer (CRC) is not true?
(Select 1)(1pts)

The probability of CRC developing during an individual's lifetime is about


6%
Colon cancer is three times more common than rectal cancer

CRC ranks as the third most common malignancy in the United States

30% of colorectal cancer is diagnosed in individuals younger than 40

Nonhereditary CRCs are referred to as "sporadic" and comprise 75 to 80%


of all CRCs

Answer D is false. Worldwide, over 1 million people are diagnosed with CRC annually,
and there are more than 500,000 associated deaths.1 The highest rates of colorectal
carcinoma are found in industrialized countries. The rates are significantly lower in
eastern Europe, Asia, Africa, and South America.2 However, studies of Japanese
migration to the United States, Asiatic Jewish migration to Israel, and eastern European
migration to Australia show that migrants acquire the high rates of CRCs prevalent in
their adopted countries. There is little question that environmental factors, most likely
dietary, account for this.
Colon cancer is three times more common than rectal cancer. Interestingly,
epidemiologic studies indicate a rising proportion of right-sided colonic lesions. The
proximal migration of colon cancer may be associated with changing environmental
factors; however, there is no doubt that increased screening successfully detects early
lesions in an aging population. CRC ranks as the third most common malignancy in the
United States (behind prostate and lung cancer in men and breast and lung cancer in
women) and the second leading cause of cancer-related mortality. Approximately
143,000 patients are diagnosed with CRC in the United States each year, and 51,000 die
of disease.4,5 The probability of CRC developing during an individual's lifetime is about
6%. In contrast to the three previous decades, however, the overall incidence and
mortality of CRC have declined for both men and women. Age-adjusted incidence and
mortality are associated with race and ethnicity; however, the relationships are complex,
influenced by social and economic confounding factors more than tumor biology.
Clearly, CRC is associated with genetic and environmental influences. Overt risk factors
include a personal or family history of CRC or colorectal adenoma(s), a personal history
of colorectal polyps, inflammatory bowel disease (IBD), and age greater than 50. Age is
the most common risk factor. The risk of CRC increases after the fourth decade of life.13
Most individuals present with disease after the age of 60, and only 10% of CRCs are
diagnosed in individuals younger than 40.
Nonhereditary CRCs are referred to as "sporadic" and comprise 75 to 80% of all CRCs.
Genetic etiology may be identified in the remaining 20 to 25% of patients, including
family history (15 to 20%), Lynch syndrome (5%), and FAP (< 1%). Cancer can arise
within a polyp or at another site in the colon or rectum. Read more
Feedback on your answer
Collapse
Unit 3
10)
0/1
You are seeing a 61-year old female who presents with worsening shortness of breath
for two days. The day preceding her symptoms she was involved in an MVC and was
seen in your facility and diagnosed with 3 non displaced rib fractures on the right. Her x-
ray today is shown below. What is the most likely diagnosis?

(Select 1)(1pts)

Pneumothorax

Pulmonary Contusion

Fat embolism

Pneumonia

Congestive Heart Failure

The answer is B. Severe blunt trauma to the chest can cause rib fractures and other
immediate injuries but sometimes pathology doesn't show up until days later such as a
pulmonary contusion and myocardial contusion. The contused lung is very sensitive to
fluid overload which can lead to respiratory distress. Pulmonary contusions generally
develop over the first 24 hours and resolve in about one week. Irregular, nonlobular
opacification of the pulmonary parenchyma on chest radiograph is the diagnostic
hallmark. About one-third of the time the contusion is not evident on initial radiographs.
Chest CT provides better resolution, but rarely alters management, unless other injuries
are found. Contusions evident on CT but not plain radiograph have better outcomes.
Pain control and pulmonary toilet are the mainstays of treatment. Prophylactic
endotracheal intubation is unnecessary, but patients with hypoxia or difficulty ventilating
require airway management. While opinions vary, fluid resuscitation with crystalloid to
euvolemia appears appropriate. Common complications include pneumonia and acute
respiratory distress syndrome (ARDS). Read more

Feedback on your answer


Collapse
11)
0/1
You are seeing a 27-year old male in the trauma bay shortly after being stabbed in this
back with a 6 cm buck knife by his disgruntled neighbor after an argument.
He arrived via EMS sitting up, awake and alert with a GCS of 15. His BP is 140/90, HR
120 and O2 sat is 85% on a non re-breather.
On physical exam there is a 3 cm non bleeding wound just left of his mid thoracic spine.
Upon auscultation there is decreased breath sounds on his left side. There are no signs of
tracheal deviation or JVD.
What is the next best step in management?
(1pts)

Immediately to the OR for surgical exploration

Upright Chest Xray followed by a chest tube

Immediate Needle thoracotomy on the left

Pericardiocentesis under US guidance

The answer is A, Upright Chest Xray followed by a chest tube. This patient is awake and
talking. He is stable enough for a chest xray followed by chest tube placement. A needle
thoracotomy to should be done if he has evidence of tension pneumothorax which would
be respiratory distress and hypotension. He will likely then need a chest tube and can be
further managed. Read More
Feedback on your answer
Collapse
12)
0/1
A 65-year-old male comes in the emergency Department because of severe right-sided
chest pain following a fall off his ladder 10 feet high.
Examination shows decreased respiratory movements on the right side of the chest and
tenderness on palpation over the right mentions, his abdomen is soft and non tender.
An x-ray film of his chest shows a non displaced fracture of the right 6th and 7th ribs on
the right without signs of pneumothorax.
Which of the following is the most appropriate goal in management of the rib fracture in
this patient?
(1pts)

To achieve a tital volume of 500 ML with intubation

To ensure appropriate analgesia

To provide mechanical stabilization to the chest wall

To give prophylactic antibiotics

The answer is B. Once significant associated injuries have been evaluated and treated, the
cornerstone of rib fracture management is pain control [36-38]. Early and adequate pain
relief is essential to avoid complications from splinting and atelectasis, primarily
pneumonia. The choice of analgesia depends upon the injury, the clinician's comfort
performing nerve blocks with their potential complications, and the ease with which more
invasive treatments can be performed. Analgesia for severe and multiple rib fractures and
monitoring of admitted patients are discussed separately.
For isolated injuries (ie, single rib fracture), clinicians generally begin treatment with
nonsteroidal anti-inflammatory drugs (NSAIDs) with or without opioids. For more severe
injuries, particularly if ventilation is compromised, admission and invasive treatments,
such as intercostal nerve blocks, may be needed.
Respiratory care, including use of incentive spirometry to prevent atelectasis and its
complications, is often important. We do not recommend rib belts or binders because
they compromise respiratory function. Studies of rib belts involve small numbers of
patients and have reached contradictory conclusions [39,40]. Patients with a rib fracture
who are discharged home can also use incentive spirometry throughout the day, after
analgesics have taken effect. Holding a pillow or similar soft brace against the fracture
site reduces discomfort while using the spirometer, or when coughing.
Disposition — Several researchers recommend hospital admission for any patient with
three or more rib fractures, and ICU care for elderly patients with six or more rib
fractures [14,15]. They cite the significant correlation between these findings and
serious internal injuries, such as pneumothorax and pulmonary contusion. We suggest
hospitalization for the majority of patients with three or more rib fractures. Patients with
multiple rib fractures sustained from high-energy trauma are best evaluated at a trauma
center. Transfer should be arranged expeditiously.
Displaced rib fractures likely increase the risk of injury to the lung and proximate
intercostal blood vessels. Bleeding from such fractures can be delayed, and admission or
close observation and follow-up should be arranged for patients with displaced fractures,
depending upon clinical and social circumstances. Multiple case reports indicate that
delayed bleeding from intercostal vessels or other injuries can be life-threatening,
particularly in older patients [20,41,42].
Surgical fixation may be of benefit with some types of rib fractures, particularly those
associated with chest wall deformity, flail chest, or symptomatic nonunion. The
appropriate role of surgical fixation is discussed separately. (See "Inpatient
management of traumatic rib fractures", section on 'Surgical
management'.)
Rarely, younger healthy individuals with three rib fractures, having undergone a thorough
clinical and radiographic evaluation by clinicians experienced in trauma management,
and an appropriate period of observation (a minimum of six hours of observation,
including a follow-up chest radiograph, is needed to rule out pneumothorax), may be
discharged from the emergency department. Clinicians must also consider patient
comorbidities and clinical and social circumstances when determining disposition.
Patients with one or possibly two isolated rib fractures and no complicating factors may
be discharged home with appropriate follow-up and adequate analgesia. Uptodate
Feedback on your answer
Collapse
13)
0/1
You are seeing a 20-year old female that was stabbed in the chest by an attacker 15
minutes prior to arrival. EMS noted she was initially awake and talking but has since
become less talkative and more lethargic. Vitals signs show her blood pressure is
60/palp, heart rate 120 beats/min, respiratory rate 28/min and O2 92% on a non re-
breather. On physical exam her eyes are closed but arouses to painful stimuli and loud
voice. She has distended neck veins and muffled heart sounds. Chest exam reveals a 2-cm
wound just to left of her sternum. What is the next best step in management?
(Select 1)(1pts)

Emergenct chest Tube on left

Pericardiocentesis

Upright chest xray

Emergent Ultrasound FAST exam

This young lady has been stabbed in the heart and has evidence of cardiac tamponade.
She needs an immediate percicardiocentesis. A chest tube will not help with this problem
and she is too unstable for an emegent echo or upright xray.Read more on the
initial evaluation of chest trauma

Feedback on your answer


Collapse
Unit 4
14)
0/1
You are seeing a 32-year old male who presents after being kicked multiple times in his
right flank. He is complaining of severe flank pain and gross hematuria. What is the gold
standard for imaging of renal trauma?
(Select 1)(1pts)

MRI abdomen/pelvis

Ultrasound

CT abdomen/pelvis with contrast

Intravenous pyelography

None of the above

The answer is C, CT abdomen/pelvis with contrast. CT scanning with intravenous


contrast enhancement is the modality of choice for the identification and staging of renal
trauma in the hemodynamically stable patient (image 6). The initial CT images will
frequently miss injuries to the renal pelvis and ureters as sufficient contrast may not yet
be present in the collecting system. Additional delayed images are needed to assess for
contrast extravasation when these injuries are suspected (image 7). In patients
undergoing CT scanning of other organ systems, suspected bladder injuries may be
investigated with CT cystography after retrograde filling of the bladder, as described
above. Read more
 CT with IV contrast is the Gold standard, high sensitivity
 Immediate and delayed post-contrast images to view collecting system
 Images abdomen and retroperitoneum
 Allows diagnosis and staging
 Intravenous pyelography doesnt allow you to image the abdomen or
retroperitoneum.

Feedback on your answer


Collapse
15)
0/1
A 32-year-old man is brought to the emergency department after he drove his motorcycle
into a guardrail at a high speed and was riding without a helmet. On arrival, his pulse is
100/min, respirations are 14/min, and blood pressure is 120/80 mm Hg.
Physical examination shows a GCS of 14, on abdominal exam there is mild suprapubic
tenderness. GU exam shows a deep perineal laceration. He voids 25 mL of bloody urine.
An x-ray of the pelvis shows a widened pubic symphysis. Which of the following is the
most appropriate next step in diagnosis?
(Select 1)(1pts)

Urethrography (retrograde urethrogram)

Cystography

Placement of a urinary catheter

Intravenous pyelography

The answer is A. This injury often follow pelvic fractures and classically present with
blood at the urethral meatus, an ability to void and a high riding or a non palpable
prostate to an intern. If you suspect urethral injury a retrograde urethrogram needs to be
performed prior to foley insertion. This procedure will locate the damage of the urethra, if
present. Inserting a Foley before this procedure is contraindicated as this can worsen the
urethral tear and potentially cause infection or a hematoma (you do not want to cause or
worsen the chance of a urethral stricture). These injuries often need surgical repair,
especially anterior urethral injuries. Some are treated with urinary diversion via
suprapubic catheter while the primary injury heals. Read more
Feedback on your answer
Collapse
16)
0/1
You are seeing a 48-year old male in the trauma bay after shooting himself in his right
thigh while it was in its holster.
On exam there is one wound that is about 2cm in diameter on his posterolateral right
thigh. It has since stopped bleeding after pressure was applied. His pulses are equal and
symmetrical and his neuro exam in normal. You obtain an xray which shows bullet
fragments in his thigh without evidence of fracture.
What is the next best step in treatment?
(1pts)

Ct scan of his leg to evaluate the soft tissue damage

Ultrasound to evaluate arterial flow


Ct angiography of his leg

Irrigation and tetanus

This patient shows no sign of arterial injury as he has no hard or soft signs of vascular
injury (see below). He has a normal neuro exam and has no fracture on xray. The only
thing that this patient needs is a tetanus update and wound irrigation.

Hard signs of Vascular injury


 Pulsatile bleeding
 Expanding haematoma
 Absent distal pulses
 Cold, pale limb
 Palpable thrill
 Audible bruit
The presence of hard signs of vascular injury mandates immediate operative intervention.
Usually the site of injury is obvious, and angiography is unnecessary. If in doubt,
angiography can be performed emergently on the operating room table. Unnecessary
interventions and investigations should be avoided to minimise the delay to definitive
care. Read more
Feedback on your answer
Collapse
17)
1/1
You are seeing a 25-year old male in the trauma bay after he was involved in a motor
vehicle accident. He was thrown from his bike and is now complaining of right flank
pain. His GCS is 15 and he is awake and oriented. His BP is 125/75, HR 99, RR 18, O2
98% on room air. His physical exam is pertinent for right flank ecchymosis with mild
tenderness to palpation along with mild right upper quadrant tenderness. There is no
gross blood seen at his urethral meatus and his urinalysis shows no RBCs. True or False:
In the setting of potential renal trauma (blunt and penetrating trauma), gross or
microscopic hematuria is ALWAYS present.
(1pts)

True

False Correct

The answer is False: In the setting of renal trauma, gross or microscopic hematuria is
absent in up to 5% of cases and this finding alone should not be used to preclude in those
you are suspicious of renal trauma. Read more on renal trauma
Feedback on your answer
Collapse
Unit 5
18)
0/1
According to a large, randomized, multicenter Veterans Affairs (VA) study comparing
laparoscopic or open approach for the treatment of inguinal hernias, what was the overall
recurrence rate at 2 years for laparoscopic vs open mesh repair respectively?

(Select 1)(1pts)

10%/4%

20%/10%

4%/10%

10%/20%

The answer is A, 10%/4%. An analysis of nearly 18,000 herniorrhaphies in


Sweden reported that 15% of operations were performed to treat
recurrent hernias.46 This figure is remarkably consistent with the data
from other large population-based series and is influenced by the type
of repair, type of hernia (primary versus recurrent), patient
characteristics, and surgeon characteristics (hernia specialist or not).
The use of mesh is an important factor, with a Cochrane meta-analysis
of open mesh inguinal hernia repair versus open nonmesh repair
finding that tension-free mesh repair led to a significant reduction in
hernia recurrence of between 50 and 75%.19 Two randomized trials
using the Lichtenstein repair as the control operation documented 2-
year recurrence rates between 1 and 4%, setting a benchmark for
primary inguinal hernia repair.47,48
Similar low recurrence rates have been demonstrated regardless of the
technique of mesh placement, whether laparoscopic or open.49,50 On
the other hand, a large, randomized, multicenter Veterans Affairs (VA)
study found that for primary, unilateral hernias, the laparoscopic
approach was associated with a higher overall recurrence rate at 2
years (10%) when compared with open mesh repair (4%).48 Most
reported recurrences after laparoscopic
herniorrhaphy come at an early stage in the surgeon's experience with
these procedures and arise soon after operation.48,51 The majority
can be attributed to (1) inadequate preperitoneal dissection; (2) use of
an inadequately sized patch, which may migrate or fail to support the
entire inguinal area, including direct, indirect, and femoral spaces; or
(3) staple failure with TAPP repair. Read more
Feedback on your answer
Collapse
19)
0/1
Following non mesh hernia repair, how many patients would need to be treated to prevent
one infection at 30 days?
(Select 1)(1pts)

20

50

75

120

The answer is 50. Mesh infection is rare. In a 2003 Cochrane review of


antibiotic prophylaxis for nonmesh hernia repairs, the overall infection
rate was 4.69% in the control group and 3.08% in the treatment
group.65 Thus, to prevent one infection in 30 days, 50 patients would
have to be treated, and
these patients would then be at risk for antibiotic-associated
complications. Laparoscopic repairs were excluded from this review;
however, in a meta-analysis comparing postoperative complications
after laparoscopic inguinal hernia repair with those after open repair,
superficial infection was less frequent in the laparoscopic
groups.49 Deep mesh infection was rare in both groups. Mesh
infection usually responds to conservative treatment with antibiotics
and drainage. On rare occasions, the mesh must be removed; this
may be accomplished via an external approach. If, however, a
prosthesis composed of a hydrophobic material (e.g., expanded
polytetrafluoroethylene) becomes infected, it is very difficult to
sterilize and virtually always must be removed. It is noteworthy that
removal of the mesh does not always lead to recurrence of the hernia,
a finding that may be attributable to the resulting fibrosis. Read more
Feedback on your answer
Collapse
20)
0/1
A 24-year old male basketball player presents to you with a painful bulge into his right
inguinal area into his scrotum that he first noticed while he was playing basketball. He
asks if it could be a sports hernia and if this will limit his practicing and upcoming
tournament. What hernia does this patient most likely have?
(Select 1)(1pts)
Indirect

Direct

Femoral

Richter

This patient most likely has an indirect hernia as it is the most common hernia and given
the lack of other findings in the stem that is the best answer and most likely. Although a
sports hernia may lead to a traditional, abdominal hernia, it is a different injury. A sports
hernia is a strain or tear of any soft tissue (muscle, tendon, ligament) in the lower
abdomen or groin area. Because different tissues may be affected and a traditional hernia
may not exist, the medical community prefers the term "athletic pubalgia" to refer to this
type of injury.
An indirect inguinal hernia is one of the most common abdominal hernias. It is
five times more common than a direct inguinal hernia, and is seven times more frequent
in males, due to persistence of the process vaginalis during testicular descent. An indirect
hernia enters the inguinal canal at the deep ring, lateral to the inferior epigastric vessels.
It passes infromedially to emerge via the superficial ring and, if large enough, extend into
the scrotum. In children, the vast majority of inguinal hernias are indirect (see Case 3).
Incarceration represents the most common complication associated with inguinal hernias,
the incidence could be as high as 30% for infants younger than 2 months.
A direct inguinal hernia arises from protrusion of abdominal viscera through a
weakness of the posterior wall of the inguinal canal medial to the inferior epigastric
vessels, specifically through the Hasselbach's triangle. This type of hernia is termed
direct as the hernial sac directly protrudes through the inguinal wall in contrast to indirect
ones which arise through the deep ring and enter the inguinal canal. Since direct hernias
do not have a guiding path, they seldom extend into the scrotum unless very large and
chronic. Direct hernias arise usually as acquired weakness of the Hasselbalch's triangle.
Therefore, they are seen in the elderly with chronic conditions which increase intra-
abdominal pressure over a long period, e.g. COPD, bladder outflow obstruction, chronic
constipation etc. Increased abdominal pressure is transmitted to both sides and as a result,
direct hernias are usually bilateral. Compared to indirect hernia, they are less susceptible
to strangulation as they have a wide neck.
In contrast to the indirect hernia, a direct hernia is most often an acquired lesion. It
occurs when a weak spot develops in the lower abdominal musculature (the posterior
floor of the inguinal canal) due to the normal and/or abnormal stresses inflicted by living
and aging. In adults, stresses such as lifting heavy of objects, frequent coughing or
straining, pregnancy, and constipation can instigate hernia. Unlike indirect hernias, direct
hernias traverse medial to the inferior epigastric vessels and are not associated with the
processus vaginalis. The hernia consists primarily of retroperitoneal fat. Only rarely is a
peritoneal sac containing bowel encountered. Because there is typically no involvement
of a sac, they do not protrude with the spermatic cord, and as such, have a lower
incidence of incarceration or strangulation. Like indirect inguinal hernia, direct inguinal
hernias typically cause a bulge in the groin (at the top of or within the scrotum) and
usually with increased abdominal pressure. Like indirect hernias, they may or may not be
painful (usually not). By palpating the inguinal canal and asking the patient to cough
while standing, one can usually elicit the hernia. In fact, one can often times palpate an
inguinal hernia without invaginating the scrotum (as is typically taught in medical
school). Rather, by placing one's fingers over the inguinal canal and asking the patient to
cough, one can often feel the bulge against the lower abdominal wall. As direct and
indirect hernias are unreliably differentiated by physical exam alone, the need to
invaginate the scrotum to feel into the inguinal canal is often more uncomfortable to the
patient, than telling to the physician. Rarely, palpation is not even necessary, as the
hernia is large enough to be visualized. Read more on Inguinal hernias: A Brief
review

Feedback on your answer


Collapse
21)
0/1
Which of the following statements regarding hernias is not correct?
(Select 1)(1pts)

In female patients, indirect inguinal hernias are the most common type

The lifetime risk of having to undergo an inguinal hernia repair is greater


for men than in women

In male patients, direct inguinal hernias are the most common type

Femoral hernias account for fewer than 10% of all groin hernias; however,
40% present as emergencie

Answer C is the only incorrect statement. In the United States, approximately


1,000,000 abdominal wall herniorrhaphies are performed each year, of
which almost 80% are for inguinal or femoral hernias.1Worldwide,
some 20 million groin hernias are repaired each year.2 The lifetime
risk of having to undergo an inguinal hernia repair is 27% for men and
3% for women.3 In male patients, indirect inguinal hernias are the
most common type (more often located on the right), occurring
approximately twice as frequently as direct inguinal hernias; femoral
hernias account for a much smaller percentage. In female patients,
indirect inguinal hernias are also the most common type, but femoral
hernias are seen more frequently than direct hernias, which are rare in
this population. Femoral hernias account for fewer than 10% of all
groin hernias; however, 40% present as emergencies (i.e., with
incarceration or strangulation), and mortality is higher for emergency
repair than for elective repair. A two-peak theory has been described,
stating that a new diagnosis of an inguinal hernia is most likely in
patients younger than 1 year and in patients older than 55 years,
although hernias can be diagnosed across any given age group. Read
more
Feedback on your answer
Collapse
Unit 6
22)
0/1
Which of the following statements about duodenal or pancreatic injury is not true?
(Select 1)(1pts)

CT scan is the primary diagnostic modality

A bicycle handlebar to the epigastrium is the most common cause of injury

An elevated amylase level is a very sensitive but non specific finding

Duodenal hematomas causing obstruction usually present 2-3 after initial


injury

Answer C is the only false statement as No laboratory findings are particularly specific
for duodenal or pancreatic injury. Amylase has been proposed for both pancreas and
duodenal injuries but is not specific for either one. Some have noted that amylase can be
elevated in as many as 50% of duodenal injuries, but this has not been a consistent
finding.9 An elevated amylase should prompt an evaluation of the duodenum for injury
but alone is not diagnostic. Although the highest concentration of amylase in the human
body is in the pancreas, hyperamylasemia is also not a reliable indicator of pancreatic
trauma. In one series, only 8% of blunt abdominal injuries with hyperamylasemia had
pancreatic injury.10 Furthermore, as many as 40% of patients with a pancreatic injury
may initially have a normal serum amylase.
The history of a patient with a possible blunt pancreatic or duodenal injury usually
consists of a direct blow to the epigastrium. In children, this commonly involves a bicycle
handlebar to the epigastrium; in adults, more commonly the steering wheel or a motor
cycle handle bar is involved. However, any direct blow should raise suspicion. The
patient may complain of abdominal, back, or flank pain.
Outside of the pelvis x-ray, plain abdominal radiographs to evaluate blunt abdominal
trauma are less common in the era of focused abdominal sonography for trauma (FAST)
and computed tomographic (CT) scans. Reports suggest that signs of duodenal injury on
plain radiographs are identified less than one third of the time. Retroperitoneal air, free
intraperitoneal air, or obliteration of the psoas shadow should raise suspicion for
duodenal and other hollow viscous trauma.9 Upper gastrointestinal series have also been
used to evaluate the duodenum for injury and can add to the sensitivity and specificity of
plain films, but more recently, CT has become the primary diagnostic modality.
For optimal duodenal evaluation, intraluminal contrast administered via a nasogastric
tube soon but not immediately prior to the CT scan may aid by opacification of the lumen
of the duodenal "C loop." Visualization of contrast extravasation, retroperitoneal air,
adjacent fat stranding, and unexplained fluid, as well as duodenal wall thickening, are CT
findings suggestive of potential duodenal injury. The sensitivity of CT for duodenal
injury is related to the technology of the scanner (i.e., "number of slices") and the time
interval from the injury to imaging. In a recent review, CT was considered to have an
overall sensitivity of around 76% with new-generation scanners (16- or 64-slice), having
a higher sensitivity of around 82%. Additionally, if clinical suspicion remains high after
an initial negative CT scan, then repeat imaging is warranted and may improve diagnostic
yield.
At times, duodenal hematomas can cause duodenal obstruction. This usually presents 2 to
3 days after the trauma with evidence of gastric outlet obstruction. The common
nonoperative treatment of isolated obstructing duodenal hematomas is nasogastric tube
decompression of the stomach and duodenum, nutritional support, and time.38 Repeat
imaging in 7 to 14 days is reasonable to evaluate improvement or an unexpected finding
as most duodenal hematomas will have resolved after 7 to 14 days, and continued
obstruction may reveal additional injury, prompting intervention.39-41 Enteral nutrition
is preferred to parenteral nutrition, but in the setting of a duodenal obstruction, the
parenteral route (total parenteral nutrition [TPN]) is often required as passing a feeding
tube beyond the obstruction can be difficult and surgical feeding access defeats the
purpose of "nonoperative" care.
Feedback on your answer
Collapse
23)
0/1
Which of the following statements regarding hollow viscus injuries in patients suffering
from blunt trauma is not true?
(Select 1)(1pts)

Mortality is increased in parallel with time to operative intervention

Only 1.2% of blunt trauma admissions had an associated hollow viscus


injury
The "seat-belt" sign is associated with a more than doubled relative risk of
small bowel injury

Unexplained intraperitoneal fluid (i.e., fluid appearing in the absence of


solid-organ injury) was the least common radiographic finding associated
with blunt bowel or mesenteric injury

Answer D is false. Unexplained intraperitoneal fluid (i.e., fluid appearing in the absence
of solid-organ injury) was the most common radiographic finding associated with blunt
bowel or mesenteric injury but often proved to be a false positive finding.
Hollow viscus injury after blunt trauma, although uncommon, can have serious
consequences if the diagnosis is missed or delayed. In a multi-institutional study of 198
patients with blunt small bowel injury, delay of as little as 8 hours in making the
diagnosis resulted in increased morbidity and mortality.2 Mortality increased in parallel
with time to operative intervention (< 8 hours to operation, 2% mortality; 8 to 16 hours,
9%; 16 to 25 hours, 17%; > 24 hours, 31%), as did the complication rate. Consequently,
it is important to have an expedient approach to the diagnosis of blunt bowel injury.
Physical examination findings such as abdominal tenderness or tachycardia may suggest
the presence of hollow viscus injury. Distracting chest or long bone injury, closed-head
injury, spinal cord injury, or intoxication, however, may compromise reliability of the
examination. In addition, it is not uncommon for blunt bowel injury to have a latent
period from the time of injury, whereby the expected signs and symptoms of such injuries
take some time to develop. Laboratory abnormalities, including elevations in white blood
cell (WBC) count, amylase, and/or lactic acid, may also point toward the presence of
hollow viscus injury but are relatively nonspecific. Provided that the patient has suffered
a low-risk mechanism of injury (such as a fall from standing or a low-speed motor
vehicle collision), hollow viscus injury is extremely unlikely in the face of a normal,
reliable physical examination and normal laboratory results. With these conditions
present, the presence of blunt bowel injury can be effectively excluded. However, the
presence of abdominal complaints, an abnormal or unreliable physical examination,
abnormal laboratory results, or a high-risk mechanism of injury (such as a high-speed
motor vehicle collision) warrants further evaluation by imaging for the presence of bowel
injury.
Particular consideration should be given to lap-and shoulder-restraint injuries, which may
be associated with an increased risk of hollow viscus injury. The "seat-belt" sign (i.e.,
ecchymosis of the abdominal wall secondary to the compressive force of the lap belt) is
associated with a more than doubled relative risk of small bowel injury.3,4 Flexion-
distraction fractures of the spine (Chance fractures) are also associated with lap-belt use,
and the presence of such fractures should raise the index of suspicion for associated
hollow viscus injury.
Ultrasonography is routinely performed early in the evaluation of blunt abdominal
trauma. It is highly specific and moderately sensitive in identifying intra-abdominal fluid,
the presence of which in a hemodynamically unstable patient is an indication for
laparotomy (in that it strongly suggests the presence of significant intra-abdominal
hemorrhage).5 Ultrasonography does not, however, reliably distinguish solid-organ
injury from hollow viscus injury—a distinction that is critical for determining subsequent
management (i.e., operative versus nonoperative) in a hemodynamically stable patient.
Computed tomography (CT) is the imaging modality of choice in stable patients who
warrant evaluation by imaging as described above. We reviewed over 8,000 CT scans
performed to evaluate cases of blunt abdominal trauma and found that the number of
abnormal radiologic findings suggesting blunt injury to the bowel, the mesentery, or both
was correlated with the true presence of injury [see Table 1].6 A CT scan
demonstrating a solitary abnormality was associated with a true positive rate of 36%,
whereas a scan demonstrating more than one abnormality was associated with a true
positive rate of 83%. Unexplained intraperitoneal fluid (i.e., fluid appearing in the
absence of solid-organ injury) was the most common radiographic finding associated
with blunt bowel or mesenteric injury but often proved to be a false positive finding. On
the basis of this experience, we developed an algorithm for the evaluation of blunt hollow
viscus injury in patients with unreliable physical examinations [see Figure 1].
Most CT scans performed in this clinical setting, however, will be negative for evidence
of intra-abdominal injury. A prospective multi-institutional trial involving 3,822 blunt
trauma patients demonstrated that the negative predictive value of a normal abdominal
CT scan was 99.63%, leading the authors to conclude that patients with a normal scan do
not benefit from hospital admission and prolonged observation.7 However, a multi-
institutional review of 2,457 cases carried out by the Eastern Association for the Surgery
of Trauma (EAST) reported a 13% incidence of blunt small bowel injury in patients with
an initial negative CT scan. These results indicate that caution should be exercised in
dismissing the presence of hollow viscus injury on the basis of a negative scan.3 This
concern is echoed by our own institutional experience, in which the incidence of injury in
patients with an initial negative CT scan was 12%.6 If CT scanning demonstrates no
suspicious findings, no further diagnostic workup of hollow viscus injury is necessary,
but the duration of the observation period depends on both the overall condition of the
patient and clinical judgment. Most patients, as supported by the negative predictive
value of the study above, will not require ongoing further observation.
Feedback on your answer
Collapse
24)
0/1
What is the mean mortality rate of patients suffering from pancreatic trauma?
(Select 1)(1pts)

<3%

9%

19%

31%
The answer is C, 19%. Duodenal and pancreatic injury continues to
challenge the trauma surgeon. The relatively rare occurrence of these
injuries, the difficulty in making a timely diagnosis, and high morbidity
and mortality rates justify the anxiety these unforgiving injuries
invoke. Mortality rates for pancreatic trauma range from 9 to 34%,
with a mean rate of 19%. Duodenal injuries are similarly lethal, with
mortality rates ranging from 6 to 25%. Complications following
duodenal or pancreatic injuries are alarmingly frequent, occurring in
30 to 60% of patients.1-3 Recognized early, the operative treatment
of most duodenal and pancreatic injuries is straightforward, with low
morbidity and mortality. Read more
Feedback on your answer
Collapse
25)
0/1
What percentage of patients suffering from blunt abdominal trauma develop hollow
viscus injury?
(Select 1)(1pts)

1.2%

4.8%

8.9%

13.4%

The answer is A, 1.2%. Hollow viscus injury after blunt trauma, although
uncommon, can have serious consequences if the diagnosis is missed
or delayed.Hollow viscus injury is most often the consequence of
penetrating abdominal trauma. As a result of blunt force trauma,
bowel injury occurs with relative infrequency: in one multi-institutional
analysis, only 1.2% of blunt trauma admissions had an associated
hollow viscus injury. Read more

Unit 1
1)
0/1
What is the most common cancer among woman?
(Select 1)( 1pts extra credit)
Ovarian

Lung

Colon

Breast

The answer is D, Breast Cancer. Approximately 230,480 American women are diagnosed
with breast cancer annually, and 39,520 women die from this disease. Global cancer
statistics show that breast cancer is the most frequently diagnosed cancer and the leading
cause of cancer death among females, accounting for 23 percent of total cancer cases and
14 percent of cancer deaths. Breast cancer is now also the leading cause of cancer death
among females in economically developing countries.
Globally, breast cancer is the most frequently diagnosed cancer and the leading cause of
cancer death in women. In the United States, breast cancer is the most commonly
diagnosed cancer and the second most common cause of cancer death in women. In
addition, breast cancer is the leading cause of death in women ages 40 to 49 years.Breast
cancer is treated with a multidisciplinary approach involving surgical oncology, radiation
oncology, and medical oncology, which has been associated with a reduction in breast
cancer mortality. Read more from Up to date

Feedback on your answer


Collapse
2)
0/1
You are evaluating a 56-year old female for a suspicious breast mass in her right breast.
Her last mammogram 2 years ago was normal and you have sent her for another
mammogram. She denies family history of cancer and has no current medical problems.
Which of the following calcification patterns is more suspicious for a ductal carcinoma in
situ?

(Select 1)( 1pts extra credit)

Linear calcifications

Round calcifications

Popcorn calcification pattern

Egg shell calcification pattern


The answer is A, linear calcifications. The histologic hallmark of DCIS is the presence of
malignant-appearing cells confined to the lumen of the ductal system in the breast. DCIS
is most often diagnosed because of the presence of calcifications clustered in one area of
the breast. The calcifications suspicious for DCIS are often linear (not round), growing in
a ductal distribution. The DCIS may not be confined to the extent of calcifications, and
not all DCIS is manifested by calcification. Therefore, the size or extent of DCIS may not
be defined by the mammographic appearance. Although DCIS occasionally presents as a
mass, the presence of the mass even in the setting of a biopsy showing DCIS most often
signifies that the cancer is primarily invasive. Read more on Breast Cancer
 Popcorn calcification in the breast is the classical
description for the calcification seen in involuting
fibroadenomas, which as the name suggests has a pop
corn like appearance.
 Egg shell calcifications in the breast are benign peripheral
rim like calcifications
 Read More on Breast Calcification patterns
Feedback on your answer
Collapse
3)
0/1
A 55-year-old white female presents with pain and redness to her right breast at the scar
site from a lumpectomy that was performed for stage I cancer four months ago. She has
also since completed multiple rounds of radiation treatments to her breast as well.
Currently her vitals are normal and she is afebrile. Physical exam reveals 5-6 cm of
erythema surrounding her surgical scar that is painful to touch, there is no axillary
adenopathy, no wound drainage, crepitance, or bullous lesions. Which one of the
following organisms would be the most likely cause of cellulitis in this patient?

(Select 1)( 1pts extra credit)

Non–group A hemolytic Streptococcus

Pneumococcus pneumoniae

Clostridium perfringens

Escherichia coli

The answer is A. Cellulitis in patients after breast lumpectomy is thought to be related to


lymphedema. Axillary dissection and radiation predispose to these infections. Non–group
A hemolytic Streptococcus is the most common organism associated with this infection.
The onset is often several weeks to several months after surgery. Pneumococcus is more
frequently a cause of periorbital cellulitis. It is also seen in patients who have bacteremia
with immunocompromised status. Immunocompromising conditions would include
diabetes mellitus, alcoholism, lupus, nephritic syndrome, and some hematologic cancers.
Clostridium and Escherichia coli are more frequently associated with crepitant cellulitis
and tissue necrosis. Pasteurella multocida cellulitis is most frequently associated with
animal bites, especially cat bites. Ref: Swartz MN: Cellulitis. N Engl J Med
2004;350(9):904-912.
Feedback on your answer
Collapse
4)
0/1
You are seeing a 31-year-old woman who presents with worsening sore throat, dry
cough, fever, and severe neck pain over the last week. She is otherwise healthy and takes
no medications.
Her vitals show a temperature is 102.5 °F, blood pressure is 99/68 mm Hg, pulse rate is
125/min, and respiration rate is 24/min. On exam she appears ill and her neck is tender to
palpation along the left side with overlying erythema, mild induration, without
lymphadenopathy. Her pharynx is erythematous, with tonsillar enlargement and no
exudates or ulcers. Her lungs are clear to auscultation. The remainder of the examination
is normal. Her chest x-ray shows multiple bilateral infiltrates. Labs show a leukocyte
count is 19,700/µL with 16% band forms. Hgb 10.8 g/dl, BUN 34 mg/dL, serum
creatinine level is 1.8 mg/dL. Which of the following tests is most likely to establish the
diagnosis?

(Select 1)( 1pts extra credit)

Computed tomography (CT) angiography of the chest with contrast

CT of the neck with contrast

Soft Tissue Neck xray

Transthoracic echocardiography

The answer is B, The patient should undergo computed tomography (CT) of the neck
with contrast. She has fever, leukocytosis, sore throat, unilateral neck tenderness, and
multiple densities on chest radiograph, suggestive of septic emboli. The combination of
these factors points strongly toward Lemierre syndrome, which is septic thrombosis of
the internal jugular vein. The diagnosis should be suspected in anyone with pharyngitis,
persistent fever, neck pain, and septic pulmonary emboli. CT of the affected vessel with
contrast would confirm the diagnosis. Treatment includes intravenous antibiotics that
cover streptococci, anaerobes, and β-lactamase-producing organisms. Penicillin with a β-
lactamase inhibitor and carbapenem are both reasonable choices (eg, ampicillin-
sulbactam, piperacillin-tazobactam, ticarcillin-clavulanate).
Chest CT would better characterize the pulmonary infiltrates, but this information would
not provide specific diagnostic information that would guide therapy.
Soft tissue radiography of the neck cannot detect jugular vein filling defects or
thromboses, which are diagnostic of septic thrombophlebitis.
Echocardiography would be helpful to exclude right-sided endocarditis as a cause of
septic emboli. However, there is nothing in the history or on cardiac examination to
suggest a cardiac source of septic emboli. Centor RM, Samlowski R. Avoiding sore
throat morbidity and mortality: when is it not “just a sore throat?” Am Fam Physician.
2011;83:26, 28. PMID: 21888123
Feedback on your answer
Collapse
5)
0/1
You are seeing a 26-year old college student in the ER following a roll over MVC. He
was wearing a seat belt and was not ejected. Upon arrival his complaining of abdominal
pain. His vital signs show a heart rate of 120/min, blood pressure is 134/98, RR 20/min,
O2 98% on room air. Physical exam shows he has a seat belt sign across his abdomen and
has tenderness greatest in his upper abdomen with no guarding. His CT scan of the
abdomen/pelvis with IV contrast only shows a liver laceration.

True or False: Most liver lacerations/injuries are managed operatively as they


often fail non-operative management.
( 1pts extra credit)

True Incorrect

False

False: The current approach to hepatic trauma has evolved to nonoperative management
in more than 80% of cases. Several contributing factors have been recognized: realization
that more than 50% of liver injuries stop bleeding spontaneously, the precedent of
successful nonoperative management in pediatric patients, knowledge that the liver has
tremendous capacity to heal after injury, and improvements in liver imaging with CT.
Criteria for nonoperative management include foremost, hemodynamic stability, absence
of other abdominal injuries that require laparotomy, immediate availability of resources
including a fully staffed operating room, and a vigilant surgeon. In general, any patient
who is stable enough to have a CT scan performed is likely to be successfully managed
nonoperatively. Grade I and II hepatic injuries should be observed in a monitored setting
with serial hematocrit evaluations and bed rest. Higher-grade injuries in stable patients
should be observed in an intensive care unit setting with optimization of all coagulation
factors. Read More
Feedback on your answer
Collapse
Unit 2
6)
0/1
In cases of blunt trauma to the diaphragm, the injury is on the ____ side __% of the time?
(Select 1)(1pts)
Right; 90%

Left; 75%

Left; 25%

Right; 10%

The answer is B, left;75%. In cases of blunt trauma to the diaphragm, the injury is on the
left side 75% of the time, presumably because the liver diffuses some of the energy on
the right side. Blunt diaphragmatic injuries result in a linear tear in the central tendon,
whereas penetrating injuries are variable in size and location depending on the weapon. It
is important to identify the trajectory of penetrating injuries to determine the likelihood of
diaphragm injuries. With blunt and occasionally with penetrating injuries, the diagnosis is
suggested by an abnormality of the diaphragmatic shadow on a chest radiograph [see
Figure 5]. In patients without clear imaging results in the trauma bay, a CT scan may
identify a diaphragmatic injury.
Regardless of the etiology, acute injuries are repaired through an abdominal incision.
Thoracoscopy or laparoscopy may be used if concomitant injuries requiring laparotomy
have been ruled out. Following delineation of the injury, the chest should be evacuated of
all blood and particulate matter, and tube thoracostomy should be placed if not previously
done. Using Allis clamps to approximate the diaphragmatic edges, the defect can be
closed with a running permanent suture [see Figure 6]. Occasionally, large avulsions or
shotgun wounds with extensive tissue loss will require mesh to bridge the defect.
Alternatively, transposition of the diaphragm cephalad one to two intercostal spaces may
allow repair without undue tension. Read more
Feedback on your answer
Collapse
7)
1/1
Nonoperative Management of solid-organ injuries can be pursued in hemodynamically
stable patients who do not have overt peritonitis or other indications for laparotomy.
According to contemporary data, what percentage of patients with splenic injuries are
candidates for nonoperative management?
(Select 1)(1pts)

<10%

25%

45%
60%

The answer is D, 60%. NOM of solid-organ injuries can be pursued in hemodynamically


stable patients who do not have overt peritonitis or other indications for laparotomy.5-9
Contemporary data suggest that this will be approximately 60% of the overall splenic
injury population.6 Failure of NOM in this selected population has been reported at only
10 to 20%. It is important to understand that this rate represents an average and may vary
widely across differing grades of injury and across varying patient populations. For
example, the NOM failure rate reported in the Eastern Association for the Surgery of
Trauma multiinstitutional study from 2000 was 11%.10 However, those with highgrade
injuries (III, IV, V) had a NOM failure rate of 20%, 33%, and 75%, respectively. In
addition to increasing grade, contrast extravasation, older age, and high injury severity
score (> 25) have been linked to higher risk of failure of NOM.11-16 NOM may be time
and resource intensive and is not safe without the supporting resources such as a setting
and staff for adequate observation, rapid operating room mobilization if needed,
immediately available anesthesia, and an adequate blood bank. If such resources are
unavailable, patients should be transferred to a higher level of care or early operation
should be considered.
The role of angioembolization of the injured spleen continues to evolve. The
identification of contrast extravasation within the splenic parenchyma (pseudoaneurysm)
as a risk factor for failure of NOM led to liberal use of angioembolization and improved
rates of splenic salvage in this population [see Figure 1 and Figure 2].17 Contrast
extravasation into the peritoneal cavity outside the splenic parenchyma is also commonly
cited as an indication for angiography. However, the use of angioembolization in such
patients has met with less success.18 In our experience, such patients rarely remain stable
for long and often require laparotomy if angiography cannot be performed rapidly
In addition to use in contrast blush, angiography is increasingly used in all higher-grade
spleen injuries undergoing NOM. Multiple institutions have addressed this concept, and a
meta-analysis published in 2011 and a subsequent prospective study in 2014 highlighted
the higher splenic salvage rate during NOM when angioembolization is employed in all
grade IV and V injuries.19,20 This is echoed in a Level 2 recommendation in the Eastern
Association for the Surgery of Trauma practice management guidelines from 2012.10
Subcapsular hematomas also may be at higher risk for delayed rupture, and special
consideration should be given to patients with this injury pattern. Also currently
controversial is the superiority of proximal embolization versus distal embolization.18,21
With such emphasis placed on reporting techniques for increasing NOM success and
splenic salvage, it is important to keep in mind the role of early, rapid operation.
Approximately 20 to 30% of patients with splenic trauma deserve early splenectomy, and
failure of NOM often represents poor patient selection.22,23 In adults, indications for
prompt laparotomy include hemodynamic instability and initiation of blood transfusion
within the first 12 hours considered to be secondary to the splenic injury. In the pediatric
population, blood transfusions up to half of the patient's blood volume are used prior to
operative intervention. Following the first 12 postinjury hours, indications for laparotomy
are not as black and white. Determination of the patient's age, comorbidities, current
physiology, degree of anemia, and associated injuries will determine the use of
transfusion alone versus intervention with either embolization or operation. Delayed
hemorrhage or rupture of the spleen can occur up to weeks following injury, but the
majority who fail do so while still hospitalized.24 Overall, nonoperative treatment
obviates laparotomy in more than 90% of appropriately selected cases. Read more
Feedback on your answer
Collapse
8)
0/1
You are seeing a 20-year-old male in hospital 12 hours after he was admitted for
observation following an MVC. Initially he was complaining of left sided chest wall pain
after his car was t-boned on the passenger side. His initial physical examination showed
bruises on the anterior chest wall and upper abdominal wall. X-rays revealed fractures
of his sixth and seventh ribs on the left (but no pneumothorax or pleural effusion). A
FAST Exam did not show any free intraperitoneal fluid. Currently, he complains of
worsening epigastric pain, left shoulder pain and mild nausea. His current vitals show his
blood pressure is now 95/60 and pulse rate 115 beats/min, and O2 saturation is 96% on
room air. Which of the following is the next best step in treatment?
(Select 1)(1pts)

Repeat ECG and cardiac biomarkers

Repeat PA and lateral chest x-ray

Abdominal CT scan with intravenous contrast

Transesophageal echocardiogram

This patient presents with blunt abdominal trauma with the delayed onset of hypertension
and signs and symptoms worrisome for likely splenic injury. Most common injuries are
to the spleen, liver, and less common injuries are to the hollow viscous organs in the
abdomen. Symptoms and signs suggesting splenic injury include left upper quadrant pain,
abdominal wall contusion, left lower chest wall tenderness, hypotension, and left
shoulder. Pain referred from splenic hemorrhage, hitting the phrenic nerve and diaphragm
(Kehr sign). The initial examination after blunt abdominal trauma can be unremarkable
and the symptoms can occur hours later, indicating ongoing splenic injury. The best
choice here would be an abdominal CT scan with intravenous contrast, only (no oral
contrast is needed because as little utility). This will define organ injury, assess for
presence of bleeding in all abdominal compartments, determine the need for surgery.
The spleen is the second most commonly injured abdominal organ in blunt trauma
patients. Historical studies have reported a 10% mortality with all splenic injuries;
however, isolated splenic injury mortality is less than 1%. The mechanisms of injury are
similar to those seen with liver injuries: motor vehicle collisions, autopedestrian
accidents, and falls. Similar to penetrating trauma to the liver, stab wounds to the spleen
typically result in direct linear tears, whereas gunshot wounds result in significant
cavitary injuries.
Until the 1970s, splenectomy was considered mandatory for all splenic injuries.
Recognition of the immune function of the spleen refocused efforts on splenic salvage in
the 1980s.38,39Following success in pediatric patients, NOM of splenic injuries was
adopted in the adult population and has become the prevailing strategy for blunt splenic
trauma.40

INITIAL EVALUATION AND INJURY GRADING

Addressing the patient's ABC's, examining the patient's abdomen, and performing
adjunctive imaging with FAST and CT are the initial steps of diagnosing a patient's
splenic injury. Hypotension with a positive FAST scan should prompt emergent
laparotomy. For patients with an identified blunt splenic injury on a CT scan, the injury
should be graded according to the AAST injury grading scale [see Table 1].3

Similar to liver injuries, the grade of splenic injury predicts failure rates and complication
rates of NOM. Other findings that should be searched for on a CT scan include contrast
extravasation (is the contrast blush contained within the spleen, or does it spill into the
peritoneum?), the amount of intra-abdominal hemorrhage (is it isolated to the splenic
fossa, or does blood extend into the pelvis?), and the presence of pseudoaneurysms.

NONOPERATIVE MANAGEMENT

Indications for Nonoperative Management

NOM of solid-organ injuries is pursued in hemodynamically stable patients who do not


have overt peritonitis or other indications for laparotomy.41-45 There is no age cutoff for
patients for the NOM of solid-organ injuries.46,47 High-grade injuries, a large amount of
hemoperitoneum, contrast extravasation, and pseudoaneurysms are not absolute
contraindications for NOM; however, these patients are at high risk for failure.48-51 The
identification of contrast extravasation as a risk factor for failure of NOM led to liberal
use of angioembolization. The true value of angioembolization in splenic salvage has not
been rigorously evaluated. Patients with intraparenchymal splenic blushes who are
otherwise asymptomatic may be considered for a period of observation rather than
empiric angioembolization52; it is thought that the contained hemorrhage within the
splenic capsule may result in tamponade of the bleeding

It is clear, however, that 20 to 30% of patients with splenic trauma deserve early
splenectomy and that failure of NOM often represents poor patient selection.53,54 In
adults, indications for prompt laparotomy include initiation of blood transfusion within
the first 12 hours considered to be secondary to the splenic injury or hemodynamic
instability. In the pediatric population, blood transfusions up to half of the patient's blood
volume are used prior to operative intervention. Following the first 12 postinjury hours,
indications for laparotomy are not as black and white. Determination of the patient's age,
comorbidities, current physiology, degree of anemia, and associated injuries will
determine the use of transfusion alone versus intervention with either embolization or
operation. Unlike hepatic injuries, which rebleed in 24 to 48 hours, delayed hemorrhage
or rupture of the spleen can occur up to weeks following injury. Algorithms for the
management of pediatric splenic injuries exist,55 and the patient's physiologic status is
the key determinant. Rapid mobilization in patients who are hemodynamically stable with
a stable hematocrit and no abdominal pain is generally successful. Overall, nonoperative
treatment obviates laparotomy in more than 90% of cases.

Follow-up Imaging

Out of concern over the risk of delayed hemorrhage or other complications, follow-up CT
scans have often been recommended; unfortunately, there is no consensus as to when or
even whether they should be obtained. Patients with grade I or II splenic injuries rarely
show progression of the lesion or other complications on routine follow-up CT scans; it is
reasonable to omit such scans if patients' hematocrits remain stable and they are
otherwise well. Patients with more extensive injuries often have a less predictable course,
and CT may be necessary to evaluate possible complications. Routine CT before
discharge, however, is unwarranted. Outpatient CT, however, in patients who participate
in vigorous or contact sports should be performed at 6 weeks to document complete
healing before resuming those activities. A more convenient and less expensive
alternative to follow-up CT is ultrasonographic monitoring of lesions.

OPERATIVE EXPOSURE AND HEMORRHAGE CONTROL

In penetrating abdominal injuries not suitable for NOM and in blunt abdominal injuries
when NOM is contraindicated or has failed, exploratory laparotomy is performed.

To ensure safe removal or repair, the spleen should be mobilized to the point where it can
be brought to the surface of the abdominal wall without tension. An incision is made in
the peritoneum and the endoabdominal fascia, beginning at the white line of Toldt along
the descending colon and continuing cephalad 1 to 2 cm lateral to the posterior peritoneal
reflection of the spleen; this plane of dissection is continued superiorly until the
esophagus is encountered [see

Figure 15a]. Posteriorly, blunt dissection is performed to mobilize the spleen and
pancreas as a composite away from Gerota fascia and up and out of the retroperitoneum;
this posterior plane may be extended to the aorta if necessary [seeFigure 15b].
Additionally, the attachments between the spleen and the splenic flexure of the colon may
be divided to avoid avulsion of the inferior splenic capsule. Care must be taken not to
pull on the spleen; otherwise, it will tear along the posterior peritoneal reflection, causing
significant hemorrhage. It is often helpful to rotate the operating table 20° to the patient's
right so that the weight of the abdominal viscera facilitates viscera retraction. Any
ongoing hemorrhage from the splenic injury may be temporarily controlled with digital
occlusion of the splenic hilar vessels. Once mobilization is complete, the spleen can be
repaired or removed without any need to struggle to achieve adequate exposure.

DEFINITIVE MANAGEMENT OF INJURIES

Splenic injuries are treated operatively by splenectomy, partial splenectomy, or splenic


repair (splenorrhaphy), based on the extent of the injury and the physiologic condition of
the patient. Splenectomy is indicated for hilar injuries, pulverized splenic parenchyma, or
any grade II or higher injury in a coagulopathic or multiply injured patient. We employ
autotransplantation of splenic implants [see

Figure 16] for partial immunocompetence in younger patients.56 Drains are not used.
Partial splenectomy can be employed in patients in whom only the superior or inferior
pole has been injured. Hemorrhage from the raw splenic edge is controlled with a
horizontal mattress suture, with gentle compression of the parenchyma [see

Figure 17]. Similar to hepatic injuries, splenorrhaphy techniques to achieve hemostasis


include topical agents (electrocautery, argon beam coagulation, thrombin-soaked gelatin
foam sponge, fibrin glue, BioGlue), enveloping the injured spleen in absorbable mesh,
and pledgeted suture repair.

POSTOPERATIVE CARE AND COMPLICATIONS

Enthusiasm for splenic salvage was driven by the rare but often fatal complication of
overwhelming postsplenectomy sepsis (OPSS). OPSS is caused by encapsulated
bacteria,Streptococcus pneumoniae, Haemophilus influenzae,
and Neisseria meningitidis, which are resistant to antimicrobial treatment. In
patients undergoing splenectomy, prevention against these bacteria is provided via
vaccines administered optimally at 14 days but definitely prior to hospital
discharge.57 Vaccines to be administered include Pneumovax (Merck & Co., Inc.,
Whitehouse Station, NJ), Menactra (Sanofi Pasteur, Swiftwater, PA), and Fluvirin
(Novartis, East Hanover, NJ). Revaccination remains open to debate, but some argue for
revaccination every 6 years.

An immediate postsplenectomy increase in platelets and white blood cells (WBCs) is


normal; however, beyond postoperative day 5, a WBC count above 15,000/μL and a
platelet to WBC ratio less than 20 are highly associated with sepsis and should prompt a
thorough search for underlying infection.58,59 A common infectious complication
following splenectomy is a subphrenic abscess, which should be managed with
percutaneous drainage. Following splenectomy or splenorrhaphy, postoperative
hemorrhage may be attributable to loosening of a tie around the splenic vessels, a missed
short gastric artery, or recurrent bleeding from the spleen if splenic repair was used.
Additional sources of morbidity include a concurrent but unrecognized iatrogenic injury
to the pancreatic tail during rapid splenectomy, resulting in pancreatic ascites or fistula.
2014. Scientific American Surgery. Hamilton, Ontario & Philadelphia, PA. Decker
Intellectual Properties Inc. ISSN 2368-2744. STAT!Ref Online Electronic Medical
Library. http://online.statref.com/Document.aspx?fxId=61&docId=2297. 10/17/2014
4:45:39 AM CDT (UTC -05:00).

Feedback on your answer


Collapse
9)
0/1
You are seeing a 42-year-old female following in the trauma bay following a high-speed
motor vehicle collision. She was was a restrained passenger and her car flipped over after
she lost control. She had to be cut out of the car and is complaining of shortness of breath
and upper abdominal pain.
She is awake and alert and her GCS is 15. Her pulse is 120/min, respirations are 28/min,
and blood pressure is 80/40 mm Hg. Breath sounds are decreased at the left lung base. An
x-ray of the chest shows opacification of the left lower lung field. You decide to place an
emergent chest tube that yields a small amount of air followed by greenish fluid. Which
of the following is the most appropriate next step in management?
(Select 1)(1pts)

CT scan of the abdomen and pelvis

CT scan of the chest

Thoracoscopy

Laparotomy

The answer is D. This patient suffered a ruptured diaphragm and chest tube placement
appears to have ruptured the bowel as evidence from the greenish fluid that was returned.
She needs an emergent laparotomy to control bleeding and repair her diaphragm. Read
More
Feedback on your answer
Collapse
Unit 3
10)
0/1
Roughly what percentage of patients with blunt liver injury initially managed non-
operatively will go on to need surgical intervention?
(1pts)

<10%
25%

50%

75%

The answer is A, During the last century, the management of blunt force trauma to the
liver has changed from observation and expectant management in the early part of the
1900s to mainly operative intervention, to the current practice of selective operative and
nonoperative management. A 2008 study by Tinkoff et al.4 showed that 86.3% of hepatic
injuries are now managed without operative intervention. The current reported success
rate of nonoperative management of hepatic trauma ranges from 82% to 100%. Most
blunt liver trauma (80% in adults, 97% in children) patients are currently treated
conservatively. The success of non-operative management depends upon proper selection
of the patient. The patients, who are managed non-operatively, usually have grade I and
II liver injuries, hemoperitoneum less than 900 ml and blood transfusion of less than 3
units. The contraindications to non-operative management include refractory
hypotension, signinficant fall in haematocrit, the extravasations of intravenous contrast
agent, expanding haematoma and grade IV and V liver injury on CECT abdomen. The
patients of grade III liver injuries need very close observation as they may require
surgical intervention during first 24 hours. The failure rate of non-operative management
is not more than 5% inmost studies. It seems that patients with grade VI injuries rarely
reaches to the hospital alive and are not salvageable. Therefore, such injuries are usually
documented on autopsy. Mortality from blunt hepatic trauma is about 5% and is related
to uncontrolled hemorrhage.
Interventional radiology may be needed to perform an angiogram and embolization for
bleeding or to percutaneously drain an abscess or b iloma. An endoscopic retrograde
cholangiopancreatogram (ERCP) and stent placement may be required for biliary leak.
Even when such complications of the liver injury develop, only 15% require operative
intervention. Hepatic artery angiography with embolization is an important tool for the
stable patient with contrast extravasation who is being managed nonoperatively. It can
also be invaluable for the postoperative patient who has been stabilized by perihepatic
packing or who has rebled after an initial period of stability. Angioembolization has a
greater than 90% success rate in the control of bleeding with a low risk of rebleeding and
a reduction in required volume of transfusion. Read More
Feedback on your answer
Collapse
11)
0/1
The liver is the most commonly injured solid organ in blunt trauma, comprising 5% of all
trauma admissions. With that being said, over 80% of patients with liver injuries may be
managed nonoperatively. Which of the following is not a predictor of nonoperative
management?
(1pts)
Female sex

Increasing age

Decreasing Glasgow Coma Scale score

Hypotension

Increasing Injury Severity Score

The answer is A, female sex. Over 80% of patients with liver injuries may be managed
nonoperatively. One of the early studies to test the application of NOM in 1995 supported
its broad application, with an overall success rate greater than 85% in hemodynamically
stable patients, despite substantial hemoperitoneum documented by CT.11 Of the 8% of
patients who failed NOM, half required operation as a result of associated injuries (i.e.,
enteric or pancreatic injuries), whereas half underwent laparotomy for hepatic-related
hemorrhage. Patients who require intervention for hemorrhage typically fail NOM in the
first 24 to 48 hours.2,11,17 Patients who fail NOM due to associated enteric or pancreatic
injury have a more variable time frame to presentation17; half manifested symptoms
within 48 hours, with the remainder becoming symptomatic up to 3 weeks later. Perhaps
not surprisingly, those patients who failed NOM had failure rates associated with
increasing grades of hepatic injury, with grade V injuries having a greater than 20%
failure rate. Subsequent studies have reported failure rates of 14% in grade IV injuries
and 23% in grade V injuries.12 The most recent analysis of the National Trauma Data
Bank of severe blunt liver injuries (grade IV and V) identified that initial NOM occurred
in 73%, with a failure rate of 7%.16 Interestingly, failure of NOM was associated with
higher mortality. Predictors of failure of NOM included increasing age, male sex,
increasing Injury Severity Score, decreasing Glasgow Coma Scale score, and
hypotension. A similar study of high-grade liver injuries identified a similar pattern with
NOM initiated in 66% patients with a failure rate of 9%.18 The amount of
hemoperitoneum evident on a CT scan appears to correlate with successful management;
patients with a large amount of hemoperitoneum (i.e., blood extending into the pelvis) are
more likely to fail NOM. However, predicting which patients will ultimately require
laparotomy has yet to be accomplished. Read more
Feedback on your answer
Collapse
12)
0/1
You are seeing a 34-year old female in the trauma following an MVC. He is complaining
intense diffuse abdominal pain after his car was hit from behind causing him to spin out
and crash into a wall. Initially he had a GCS of 15. His vitals showed a BP of 125/76, HR
95, RR 18, O2 of 98% on room air. On physical exam his lungs are clear and his
abdomen shows diffuse tenderness to light palpation and ecchymosis over his
epigastrium. His initial fast exam is negative but technically limited and you order a CT
scan with IV contrast of his abdomen and pelvis as you suspect a possible liver
injury. What description from the options below accurately describes a grade IV liver
laceration on CT imaging?
(Select 1)(1pts)

Parenchymal distruption involving 25 - 75% hepatic lobes

Parenchymal distruption involving >75% helpatic lobe

Intraparenchymal laceration < 10cm diameter

Sub capsular hematoma, > 50% surface area, or ruptured with active
bleeding

The answer is A. Parenchymal distruption involving 25 - 75% hepatic lobes. Read


more
The AAST Liver injury grading system is as follows
 grade I :
 haematoma: sub capsular, < 10% surface area
 laceration: capsular tear, < 1cm depth
 grade II :
 haematoma: sub capsular, 10 - 50% surface area
 haematoma: intraparenchymal < 10cm diameter
 laceration: capsular tear, 1 - 3cm depth, < 10cm length
 grade III :
 haematoma: sub capsular, > 50% surface area, or ruptured with
active bleeding
 haematoma: intraparenchymal > 10 cm diameter
 laceration: capsular tear, > 3 cm depth
 grade IV :
 haematoma: ruptured intraparenchymal with active bleeding
 laceration: parenchymal distruption involving 25 - 75% hepatic
lobes or
 1 - 3 Couinaud segments (within one lobe)
 grade V :
 laceration: parenchymal distruption involving >75% helpatic
lobe or
 > 3 Couinaud segments (within one lobe)
 vascular: juxtahepatic venous injuries (IVC, major hepatic vein)
 grade VI : vascular: hepatic avulsion

Feedback on your answer


Collapse
13)
0/1
What is the most common abdominal organ injured in the setting of blunt abdominal
trauma?
(Select 1)(1pts)

Spleen

Liver

Kidneys

Small Intestines

The liver is the most commonly injured solid organ in blunt trauma, comprising 5% of all
trauma admissions, and because of its size is frequently involved in penetrating trauma.
Following blunt trauma, the most commonly injured structures are the parenchyma and
hepatic veins. Blunt forces dissipate along segments of the liver and along the fibrous
coverings of the portal triad structures; the hepatic veins, however, are not so insulated.
Given its size and location within the abdomen, the liver is also commonly involved in
penetrating trauma. Stab wounds typically result in direct linear tears, whereas gunshot
wounds or shotgun wounds result in significant cavitary injuries attributable to blast
effect and the "tumbling" of the missile within the liver parenchyma. Thus, arterial injury
is more common with penetrating trauma.
Over the past 20 years, nonoperative management (NOM) of liver injuries has evolved to
become the prevailing therapeutic strategy for blunt hepatic trauma. Several concurrent
changes resulted in this paradigm change. First was the realization that diagnostic
peritoneal lavage (DPL) was sensitive but not specific for identifying intraperitoneal
hemorrhage that necessitated operative management. Surgeons recognized that many
laparotomies undertaken for a positive DPL were associated with liver injuries that did
not require intervention for bleeding.1Second, trauma surgeons noted that nonbleeding
hepatic venous injuries, if manipulated at laparotomy, often resulted in more hemorrhage
and sometimes even death.2 Furthermore, it became conspicuous that with hemostasis
achieved in the operating room, recurrent postoperative bleeding was rare. Therefore,
surgeons queried whether hepatic venous injuries, which are low-pressure system
injuries, could heal without intervention. Finally, computed tomography (CT) provided a
reliable method for diagnosing and grading liver injuries.

The spleen is the second most commonly injured abdominal organ in blunt trauma
patients. Historical studies have reported a 10% mortality with all splenic injuries;
however, isolated splenic injury mortality is less than 1%. The mechanisms of injury are
similar to those seen with liver injuries: motor vehicle collisions, autopedestrian
accidents, and falls. Similar to penetrating trauma to the liver, stab wounds to the spleen
typically result in direct linear tears, whereas gunshot wounds result in significant
cavitary injuries.

Until the 1970s, splenectomy was considered mandatory for all splenic injuries.
Recognition of the immune function of the spleen refocused efforts on splenic salvage in
the 1980s.38,39Following success in pediatric patients, NOM of splenic injuries was
adopted in the adult population and has become the prevailing strategy for blunt splenic
trauma.

Duodenal and pancreatic injury continues to challenge the trauma surgeon. The relatively
rare occurrence of these injuries, the difficulty in making a timely diagnosis, and high
morbidity and mortality rates justify the anxiety these unforgiving injuries invoke.
Mortality rates for pancreatic trauma range from 9 to 34%, with a mean rate of 19%.
Duodenal injuries are similarly lethal, with mortality rates ranging from 6 to 25%.
Complications following duodenal or pancreatic injuries are alarmingly frequent,
occurring in 30 to 60% of patients.1-3 Recognized early, the operative treatment of most
duodenal and pancreatic injuries is straightforward, with low morbidity and mortality.

2014. Scientific American Surgery. Hamilton, Ontario & Philadelphia, PA. Decker
Intellectual Properties Inc. ISSN 2368-2744. STAT!Ref Online Electronic Medical
Library. http://online.statref.com/Document.aspx?fxId=61&docId=2329. 10/27/2014
1:39:15 PM CDT (UTC -05:00).

Feedback on your answer


Collapse
Unit 4
14)
0/1
A 30-year old white female presents to your office with right breast pain. She is currently
breast feeding her healthy 4-week old infant and has been having focal tenderness,
swelling and redness to her right breast near the nipple over the last 2 days. She reports
no past medical history and is a non smoker. Given the patient’s history and physical,
which of the following inflammatory disorders is the most likely diagnosis?
(Select 1)(1pts)

Acute mastitis

Granulomatous mastitis

Mammary duct ectasia


Benign severe breast engorgement

The answer is A. Lactational mastitis is a localized, painful inflammation of the breast


associated with fever and malaise that occurs in breastfeeding women and has been
estimated to occur in 2 to 10 percent of breastfeeding women Acute mastitis almost
always occurs during the first month of breastfeeding. The diagnosis of mastitis is made
clinically. Lactational mastitis typically presents as a hard, red, tender, swollen area of
one breast associated with fever >38.3ºC in a nursing mother. Other systemic complaints
may variably include myalgia, chills, malaise, and flu-like symptoms. In the early stages
of breast infection the presentation can be subtle with few clinical signs, while patients
with advanced infection may present with a large area of breast swelling with overlying
skin changes (eg, erythema). Reactive lymphadenopathy can also cause axillary pain and
swelling. Septic shock rarely occurs.In a lactating woman, severe engorgement can be
distinguished from mastitis because engorgement is bilateral with generalized
involvement [1]. Mastitis associated with erythema and edema during is uncommon and
a diagnosis of inflammatory breast cancer must be excluded The most common
organisms causing the infections are S. aureus and streptococcal species. It may also be a
complication of nipple piercing. “D” is incorrect because women often present with
inversion of the nipple and over 90% of women affected are smokers. Read more
Feedback on your answer
Collapse
15)
1/1
A 25 -ear obese black female presents to you with a painful breast lump along with
tenderness in both her breasts. She notes recurrent pain and multiple lumps on both
breasts that seem to “come and go” at different times during her menses. Now she has a
firm, round, mass that has not gone away for the last 5 weeks. What is the most likely
diagnosis?

(Select 1)(1pts)

Cystosarcoma Phyllodes

Intraductal papilloma

Fibrocystic disease

Fibroadenoma

The answer is C. This patient has fibrocystic disease which is a nonproliferative epithelial
lesions that are generally not associated with an increased risk of breast cancer [1]. It
should be noted that terms such as fibrocystic changes, fibrocystic disease, chronic cystic
mastitis, and mammary dysplasia refer to nonproliferative lesions and are not useful
clinically, as they encompass a heterogeneous group of diagnoses [5,11]. The most
common nonproliferative breast lesions are breast cysts. Other nonproliferative lesions
include papillary apocrine change, epithelial-related calcifications, and mild hyperplasia
of the usual type [5]. Apocrine metaplasia (also referred to as a "benign epithelial
alteration") is also a nonproliferative change that is secondary to some form of irritation,
typically associated with a breast cyst. Read More
Feedback on your answer
Collapse
16)
0/1
You are seeing a 25-year old white female who is presenting to your clinic with bloody
nipple discharge over the last 3 days. She denies any history of a breast lump, she is not
breast feeding and denies family history of breast cancer. Her exam shows no palpable
masses but there is scant bloody discharge coming from her left nipple. You order a
mammogram that does not show any suspicious lesions. What is the most likely
diagnosis?
(Select 1)(1pts)

Fibroadenoma

Intraductal papilloma

Cystosarcoma Phyllodes

Mammary dysplasia

The answer is B, Intraductal papilloma. The old concern over cancer is the issue, and the
way to detect cancer that is not palpable is with a mammogram. That should be the first
choice. If negative, one may still wish to find an resect the intraductal papilloma to
provide symptomatic relief. Intraductal papillomas consist of a monotonous array of
papillary cells that grow from the wall of a cyst into its lumen. Although they are not
concerning in and of themselves, they can harbor areas of atypia or ductal carcinoma in
situ (DCIS). Papillomas can occur as solitary or multiple lesions. The standard approach
to a papilloma diagnosed by core needle biopsy (CNB) is to perform a surgical excision,
particularly if atypical cells are identified [14,16-21]. In a meta-analysis of 34 studies
that included 2236 non-malignant breast papillary lesions, 346 (15.7 percent) were
upgraded to malignancy following a surgical excision [21]. Because of a risk of
malignancy, these require surgical excision.Read More
Feedback on your answer
Collapse
17)
0/1
You are seeing a 24-year old female who is 4-weeks post postpartum in your clinic. She
is complaining of bilateral nipple pain over the last two days along with a nipple rash.
She notes pain both with breastfeeding and even when she is not feeding. The pain is so
bad that she didn't breastfeed her baby this morning because of the pain. Physical exam
shows erythema to both breasts along with cracking around both areolas. There are no
signs of an abscess and no induration present. What is the most likely cause of this
patient's symptoms?
(Select 1)(1pts)

Engorgement

Mastitis

Deep space abscess

Candida infection

None of the above

The answer is D, Candida infection. In breastfeeding women, bilateral nipple pain with
and between feedings after initial soreness has resolved is usually due to Candida. Pain
from engorgement typically resolves after feeding. Mastitis is usually unilateral and is
associated with systemic symptoms and wedge-shaped erythema of the breast tissue.
Improper latch-on is painful only during feedings. Eczema isolated to the nipple, while a
reasonable part of the differential, would be much more unusual.

Ref: Jatoi I, Kaufman M (eds): Management of Breast Diseases. Springer, 2010,


pp 95-96.
Feedback on your answer
Collapse
Unit 5
18)
0/1
Which of the following is considered a benign lesion that usually does not require cancer
treatment?
(Select 1)(1pts)

Lobular carcinoma in situ

Ductal carcinoma in situ

Medullary carcinoma

Mucinous carcinoma
The answer is A, Cancer that is confined to the lumen of the duct or
lobule of the breast and has not penetrated the basement membrane
is termed in situ cancer. This generally refers to ductal carcinoma in
situ (DCIS) but also encompasses a benign entity called lobular
carcinoma in situ (LCIS). Small uniform cells confined to the lobule of
the breast characterize LCIS. It is generally a clinically and
mammographically occult lesion that is identified only incidentally
when a biopsy is performed for calcifications or a mass that proves to
be some other benign lesion. LCIS is actually not cancer but rather is a
benign lesion and does not require cancer treatment. The primary
issue with LCIS is that it conveys an increased lifelong risk of
subsequent invasive cancer quantified at 0.5 to 0.75% per year. In
addition, when LCIS is identified on a core-needle biopsy, there is a 10
to 20% chance of DCIS or invasive cancer in the surrounding tissue;
therefore, surgical excision is warranted. Long-term follow-up shows
that the large majority of women with LCIS never develop invasive
breast cancer. Therefore, ablative surgical therapy and radiation for
LCIS are not necessary. Previously, LCIS was considered in and of
itself an indication to consider bilateral mastectomy. However,
mastectomy is generally not indicated in women with LCIS and should
be performed only in the context of risk reduction for those at very
high risk related to factors such as inherited susceptibility. Because
women with a diagnosis of LCIS are at increased risk for subsequent
invasive cancer, they should be counseled regarding that risk and may
benefit from consultation with genetics professionals if they have a
family history of breast or ovarian cancer. Women with a biopsy
showing LCIS may also consider risk-reducing chemoprevention with
one of the selective estrogen receptor modulators (SERM's), tamoxifen
or raloxifene. These reduce the risk of subsequent invasive cancer by
about 50%, with an acceptable toxicity profile.20,21 Raloxifene is the
preferred agent in postmenopausal women. Read more on Breast
Cancer
Feedback on your answer
Collapse
19)
1/1
True or False: When approaching a patient with an abnormal mammogram for a
suspicious breast mass the next best step is a surgical excision is preferred over
percutaneous needle biopsy.
(1pts)

True

False Correct
False: Breast lesion suspicious for malignancy requires tissue biopsy.
Percutaneous needle biopsy is preferred over surgical excision in all
circumstances. Surgical excision as a diagnostic procedure is not a
justifiable alternative simply because of "patient choice" and should be
performed only when needle biopsy cannot be performed for specific
technical reasons, when a needle biopsy is either nondiagnostic, the
result is not concordant with the imaging findings (i.e., the needle
biopsy is benign, but the lesion is of high suspicion), or in highly select
other cases. Technical reasons that may preclude needle biopsy
include anatomic location of the lesion on mammography directly
opposed to the chest wall or in the far periphery of the breast so that
it cannot be visualized on stereotactic imaging devices. Breast Cancer
Feedback on your answer
Collapse
20)
0/1
You are seeing a 75-year old female in clinic because of an enlarging breast mass the
last two months. On examination of her right breast you palpate a firm 3cm non tender
firm non-mobile mass lateral to her right nipple. She denies past medical history and
takes no medication. What is the next best step?
(Select 1)(1pts)

Mammogram

Breast Ultrasound

Core Biopsy

Excisional Biopsy

If a palpable mass is appreciated, bilateral diagnostic mammography should be performed


prior to biopsy, even if the mass is clinically suspicious for cancer (algorithm
3 and algorithm 4) [27]. Fine needle aspiration or core needle biopsy may alter both
mammographic or US appearance. The goal of imaging in this setting is not to establish a
diagnosis of cancer but rather to identify other suspicious areas or calcifications in either
breast that might impact treatment. This evaluation should be performed prior to
percutaneous biopsy, so that biopsy of additional suspicious lesions can be undertaken at
one time. If the lesion is large and breast conservation is not an option, this is less
important. read more
Feedback on your answer
Collapse
21)
0/1
You are seeing a 43-year old Hispanic female who is presenting with an enlarging mass
in her right breast. It has been present for several years and growing to it’s present size.
On examination of her right breast you note a large 10 cm mass that is mobile, firm
and rubbery. There are no palpable axillary nodes. What is the most likely diagnosis?

(Select 1)(1pts)

Fibroadenoma

Fibrocystic disease

Intraductal papilloma.

Cystosarcoma Phyllodes

The answer is D. This is a Phyllodes tumors, which are an uncommon fibroepithelial


breast tumors that are capable of a diverse range of biologic behavior. In their least
aggressive form, they behave similarly to benign fibroadenomas, although with a
propensity to recur locally following excision without wide margins. At the other end of
the spectrum are tumors that metastasize distantly, sometimes degenerating histologically
into sarcomatous lesions that lack an epithelial component. Phyllodes tumors account for
fewer than 0.5 percent of all breast malignancies. The vast majority occur in women, in
whom the median age at presentation is 42 to 45 (range 10 to 82 years).and there was a
higher incidence in Latina whites, as compared to non-Latina whites, Asians, and African
American women. Read More
Feedback on your answer
Collapse
Unit 6
22)
0/1
A 23-year old male patient presents with a human bite to his right hand after an
altercation just prior to arrival. He has no past medical history and is up to date with
tetanus. Exam shows there are two small puncture wounds overlying the dorsal aspect of
his right hand over his 1stmetacarpal and does not seem to involve any
tendons or ligaments. The wound does not appear amenable to sutures for
closure and you decide to copiously irrigate it with saline and discharge him home. What
antibiotic should he receive for wound care prophylaxis?
(1pts)

Clindamycin
Flagyl

Oxacillin

Augmentin

The answer is D. Augmentin. The most common bacterial etiology in human bites is strep
viridans. You are however covering for Eikenella corrodens which is a gram negative rod
which is susceptible to pcn but resistant to flagyl, clindamycin, first generation
cephalosporins and erythromycin. It is susceptible to flouroquinolones, bactrim and
augmentin. The most important treatment is good irrigation and initial wound
cleaning. Read more
Feedback on your answer
Collapse
23)
0/1
You are seeing a 30-year-old male wrestler who is presenting with a painful lesion on his
upper back. He first noted a small painful area a few days ago, and the lesion has since
enlarged and became more red. He notes that others on his wrestling team have similar
lesions and have sought care elsewhere. He has no past medical history and takes no
medications.
Physical Examination of the upper back reveals a 1 × 1 cm erythematous, raised pustule
that is tender to palpation, with a 4 × 4 cm area of surrounding erythema. The remainder
of the physical examination is normal. You elect to perform an incision and drainage, and
culture. Of the following, which is the most appropriate empiric treatment?
(Select 1)(1pts)

Levofloxacin

Doxycycline

Dicloxacillin

Cephalexin

Ciprofloxacin

All of the above

None of the above

The answer is B, Purulence is more often present in staphylococcal cellulitis. Specimens


from wounds, carbuncles, or furuncles may reveal gram-positive cocci in clusters,
consistent with S. aureus. Clinicians must judge whether MRSA or MSSA is likely.
Appropriate choices for the oral treatment of MSSA include cloxacillin; dicloxacillin;
cephalexin; and, for penicillin-allergic patients, clindamycin or a macrolide
(erythromycin, clarithromycin, or azithromycin). However, patients treated with
cephalexin should be observed for treatment failure, as some studies have reported high
failure rates in adults (29), possibly due to poor absorption (30). Appropriate orally
administered choices for the treatment of MRSA include trimethoprim–
sulfamethoxazole, doxycycline, minocycline, or linezolid. This patient likely has a
community acquired MRSA as others on his wrestling team have similar lesions and thus
MRSA targeted therapy is best. Read More on Cellulitis and Soft-Tissue
Infections
Feedback on your answer
Collapse
24)
0/1
What is the most common causative pathogen of Lemierre disease, an infection of the
parapharyngeal space that leads to septic thrombophlebitis of the internal jugular vein
with bacteremia and metastatic pulmonary nodules.
(Select 1)(1pts)

C. diphtheriae

Fusobacterium necrophorum

Infectious mononucleosis

Streptococcus pyogenes

The answer is B, Fusobacterium necrophorum is the most common causative pathogen of


Lemierre disease, an infection of the parapharyngeal space that leads to septic
thrombophlebitis of the internal jugular vein with bacteremia and metastatic pulmonary
nodules (12). Lemierre disease may be complicated by septicemia; suppurative
intracranial complications; and erosion of the carotid artery, which may be life-
threatening. Lemierre disease is often suspected based on clinical grounds in a toxic-
appearing patient; however, several recent European reports also suggest a possible
causative role of F. necrophorum in uncomplicated pharyngitis (13–15). Read More
on Pharyngitis
Feedback on your answer
Collapse
25)
0/1
Which of the following choices best differentiates cellulitis from erysipelas?
(Select 1)(1pts)
Presence of fever

Presence of leukocytosis

Depth of inflammation

Presence of purulence

All of the above

None of the above

The answer is C, depth of inflammation. Cellulitis and erysipelas are diffuse spreading
skin infections that not associated with underlying suppurative foci. Clinically, there is
often some degree of overlap between the two different entities. Erysipelas is
differentiated from cellulitis by the depth of inflammation; erysipelas affects the upper
dermis, including the superficial lymphatics, whereas cellulitis affects the deeper dermis
and subcutaneous fat (Mayo Clin Proc, Vol. 89;1436). Read More onCellulitis and
erysipelas from Up to Date

1/1
You are seeing a 26-year old male who just suffered severe blunt trauma in a car
accident. He has multiple injuries to his arms and legs, has a large scalp laceration and
has a pneumothorax on the left. Shortly after initial examination it is noted that he is
developing progressive subcutaneous emphysema all over his upper chest and lower
neck. What is the likely cause of this exam finding?

( 1pts extra credit)

Ruptured Pericardium

Ruptured Major Bronchus/Trachea

Ruptured Aorta

Ruptured Alveoli

The answer is B. This patient has a pneumomediastinum from traumatic rupture of the
trachea or major bronchus. Fiberoptic bronchoscopy to confirm diagnosis and level of
injury and to secure an airway. Most patients undergo primary repair or possible lung
resection, although good results using selective nonoperative management have been
reported. The need for surgical repair is generally based on the risk for airway
obstruction, massive air leak, or mediastinitis. Read More
Feedback on your answer
Collapse
2)
0/1
Antibiotic prophylaxis for elective inguinal hernia repair is universally recommended
( 1pts extra credit)

True Incorrect

False

False: Most recent Cochrane meta-analysis on antibiotic prophylaxis in inguinal hernia


repair in which seven of thirteen trials were mesh repair series concluded that
"administration of antibiotic prophylaxis for elective inguinal hernia repair cannot be
universally recommended”. In addition, it has been stated that antibiotic prophylaxis
cannot either be recommended against when high rates of wound infection are observed.
European Hernia Society guideline for inguinal hernia repair also states that “In clinical
settings with low rates (<5%) of wound infec tion, there is no indication for the routine
use of antibiotic prophylaxis in elective open groin hernia repair in low risk patients.” 24
. However antibiotic prophylaxis in the centers with high rates of infection (>5%), and
also for high-risk patients (e.g., advanced age, recurrent hernia, steroid use,
immunosupression, expected long operating time, use of drains, emergency repair, etc.) is
still practically in use in many institutions. Yerdel et al found a 10-fold decrease in
overall wound infection rate when single-dose, intravenous ampicillinsulbactam was used
during Lichtenstein hernia repair 27 . However, a large number of evidences say that first
generation cephalosporins should be the choice when the surgeon decides prophylaxis. A
single dose intravenous administration 30 minutes before the incision is sufficient. Oral
route for antibiotic prophylaxis can be a safe alternative which decreases the costs 28,29 .
Antibiotics are not continued postoperatively. Read more

Feedback on your answer


Collapse
3)
1/1
You are seeing a 30-year old male in the trauma bay who was an unrestrained driver
involved in a rollover MVC who is complaining of lower abdominal pain. His GCS is
15, vitals are stable and he is awake and talking. The rest of his exam shows he has
moderate right lower abdomen and flank tenderness to palpation without guarding.
His iStat labs show a Hct 45.7, BUN 15, Cr 1.2. Upright CXR is shows no signs of
trauma and there is a normal sized medastinum. CT scan of his abd/pelvis demonstrates a
small renal subcapsular hematoma with 0.5 cm superficial parenchymal laceration that
does not involve the collecting system. According to the American Association for the
Surgery of Trauma (AAST), what grade injury does this demonstrate?
(Select 1)( 1pts extra credit)

Grade I

Grade II

Grade III

Grade IV

The patient above has a grade II renal laceration as he has a


<1cm renal laceration,Read more
Renal trauma grading is often done using theAmerican Association for
the Surgery of Trauma (AAST)3-4 according to depth of damage and
involvement of the urinary collecting system and renal vessels.
 grade I - contusion or non enlarging subcapsular haematoma, but
no laceration ;
 grade II - superfical laceration < 1cm depth and does not involve
the collecting system ; non expanding perirenal haematoma
 grade III - laceration > 1cm, without extension into the renal
pelvis or collecting system and with no evidence of urine extravasation
 grade IV - laceration extends to renal pelvis or urinary
extravasation.
 grade V - shattered kidney ; devascularisation of kidney due to hilar
injury.
Read more from Up to Date

Feedback on your answer


Collapse
4)
0/1
What percentage of abdominal wall hernias are found in the groin?

(Select 1)( 1pts extra credit)

10%

25%
50%

75%

Approximately 75% of all abdominal wall hernias are seen in the groin. An Inguinal
hernia is much more common in men than women. Although femoral and umbilical
hernias are more common in female population, indirect inguinal hernia is still the most
common type of hernia in women. Age is a factor for incidence and type of inguinal
hernia; incidence increases by age 2 . Indirect hernia is more common in young and direct
hernia in the elderly. Read more

Feedback on your answer


Collapse
5)
1/1
You are seeing a 64-year-old female comes in your office for chronic constipation.
She notes a history of chronic laxative use for most of her adult years. You schedule her
for a colonscopy and during the procedure you note that the anal and rectal mucosa
contain scattered areas of bluish-black discoloration. Which one of the following is the
most likely explanation for the sigmoidoscopic findings?

( 1pts extra credit)

Endometriosis

Collagenous colitis

Melanosis coli

Metastatic malignant melanoma

This patient has typical findings of melanosis coli, the term used to describe black or
brown discoloration of the mucosa of the colon. It results from the presence of dark
pigment in large mononuclear cells or macrophages in the lamina propria of the mucosa.
The coloration is usually most intense just inside the anal sphincter and is lighter higher
up in the sigmoid colon. The condition is thought to result from fecal stasis and the use of
anthracene cathartics such as cascara sagrada, senna, and danthron. Ectopic endometrial
tissue (endometriosis) most commonly involves the serosal layer of those parts of the
bowel adjacent to the uterus and fallopian tubes, particularly the rectosigmoid colon.
Collagenous colitis does not cause mucosal pigmentary changes. Melanoma rarely
metastasizes multicentrically to the bowel wall. Multiple arteriovenous malformations are
more common in the proximal bowel, and would not appear as described. Ref: Feldman
M, Friedman LS, Sleisenger MH (eds): Sleisenger & Fordtran’s Gastrointestinal and
Liver Disease, ed 7. WB Saunders Co, 2004, p 2305. 2) Hardman JG, Limbird LE,
Gilman AG (eds): Goodman & Gilman’s The Pharmacological Basis of Therapeutics, ed
10. McGraw-Hill, 2001, pp 1046-1047. 3) Kasper DL, Braunwald E, Fauci AS, et al
(eds): Harrison’s Principles of Internal Medicine, ed 16. McGraw-Hill, 2005, pp 231-233.
Feedback on your answer
Collapse
Unit 2
6)
1/1
Which of the following statements regarding colorectal cancer (CRC) is not true?
(Select 1)(1pts)

The probability of CRC developing during an individual's lifetime is about


6%

Colon cancer is three times more common than rectal cancer

CRC ranks as the third most common malignancy in the United States

30% of colorectal cancer is diagnosed in individuals younger than 40

Nonhereditary CRCs are referred to as "sporadic" and comprise 75 to 80%


of all CRCs

Answer D is false. Worldwide, over 1 million people are diagnosed with CRC annually,
and there are more than 500,000 associated deaths.1 The highest rates of colorectal
carcinoma are found in industrialized countries. The rates are significantly lower in
eastern Europe, Asia, Africa, and South America.2 However, studies of Japanese
migration to the United States, Asiatic Jewish migration to Israel, and eastern European
migration to Australia show that migrants acquire the high rates of CRCs prevalent in
their adopted countries. There is little question that environmental factors, most likely
dietary, account for this.
Colon cancer is three times more common than rectal cancer. Interestingly,
epidemiologic studies indicate a rising proportion of right-sided colonic lesions. The
proximal migration of colon cancer may be associated with changing environmental
factors; however, there is no doubt that increased screening successfully detects early
lesions in an aging population. CRC ranks as the third most common malignancy in the
United States (behind prostate and lung cancer in men and breast and lung cancer in
women) and the second leading cause of cancer-related mortality. Approximately
143,000 patients are diagnosed with CRC in the United States each year, and 51,000 die
of disease.4,5 The probability of CRC developing during an individual's lifetime is about
6%. In contrast to the three previous decades, however, the overall incidence and
mortality of CRC have declined for both men and women. Age-adjusted incidence and
mortality are associated with race and ethnicity; however, the relationships are complex,
influenced by social and economic confounding factors more than tumor biology.
Clearly, CRC is associated with genetic and environmental influences. Overt risk factors
include a personal or family history of CRC or colorectal adenoma(s), a personal history
of colorectal polyps, inflammatory bowel disease (IBD), and age greater than 50. Age is
the most common risk factor. The risk of CRC increases after the fourth decade of life.13
Most individuals present with disease after the age of 60, and only 10% of CRCs are
diagnosed in individuals younger than 40.
Nonhereditary CRCs are referred to as "sporadic" and comprise 75 to 80% of all CRCs.
Genetic etiology may be identified in the remaining 20 to 25% of patients, including
family history (15 to 20%), Lynch syndrome (5%), and FAP (< 1%). Cancer can arise
within a polyp or at another site in the colon or rectum. Read more
Feedback on your answer
Collapse
7)
0/1
What % of patients develop recurrent disease after curative resection of colon and rectal
carcinomas within 3 years?

(Select 1)(1pts)

20%

40%

60%

80%

The answer is D, 80%. Eighty percent of patients who recur after curative resection of
colon and rectal carcinomas do so within 3 years. Therefore, any posttreatment plan
should include regular follow-up during at least these 3 years. An additional biologic
precept in designing follow-up should take into account the efficacy of therapy once
recurrent disease is identified. It is important to note that the role of surveillance is to
identify recurrent disease that can be resected with true curative intent; early
identification of asymptomatic, incurable disease is exceedingly unlikely to improve
outcome as there are neither data nor a compelling rationale to believe that outcome
would be improved by earlier institution of noncurative treatment, such as systemic
chemotherapy.
In general, if a patient is a candidate for resection of recurrent disease (e.g., hepatic
resection), serum carcinoembryonic antigen (CEA) testing should be performed every 3
to 6 months for 2 years and then every 6 months for 5 years after resection of the primary
tumor. Chest, abdomen, and pelvis CT is recommended annually for 3 years for patients
at high risk for recurrence. PET-CT is not recommended in NCCN, ASCO, or CCO
guidelines and should not be used for the purpose of postoperative surveillance.
Colonoscopy should be performed 1 year after surgery or 3 to 6 months after surgery if
not performed preoperatively due to an obstructing lesion, and then 3 years later, and then
every 5 years, unless findings or specific risk factors dictate more frequent
evaluations. Read more

Feedback on your answer


Collapse
8)
0/1
What % of patients with colorectal cancer (CRC) have metastatic disease at presentation?
(1pts)

<5%

10%

20%

30%

The answer is C, 20%. As many as 20% of CRC patients have metastatic disease at the
time of the initial presentation. The need for surgical intervention in this group is not well
defined. Clearly, surgical resection or diversion is indicated in patients who present with
significant bleeding, perforation, or obstruction. In asymptomatic patients with
unresectable metastatic disease, the role of surgical resection of the primary lesion
remains controversial. In patients with resectable metastatic disease (e.g., isolated liver or
lung metastases), curative resection may be undertaken.
A recent review of 233 patients with synchronous stage IV colorectal cancer found that
217 patients (93%) never required surgical palliation of the primary tumor; 16 patients
(7%) needed emergency surgery for obstruction or perforation of the primary tumor; 10
patients (4%) were managed nonoperatively.68,122
Management of patients with synchronous resectable isolated liver metastases continues
to evolve. Multiple studies have documented improved survival after liver resection in
patients with metastatic disease confined to the liver. Patients presenting with
synchronous lesions have a worse prognosis than those presenting with metachronous
lesions.123 Many of these patients have been managed with staged resections of their
primary cancers and the liver metastases. Several groups have reported that such
combined procedures do not substantially increase surgical morbidity and mortality or
compromise cancer survival.124,125 These combined procedures should be done only in
carefully selected patients, at specialized centers where there is significant experience in
resection of both CRCs and liver tumors. Read more
Feedback on your answer
Collapse
9)
0/1
You are seeing a 73-year old male who has a past medical history of
CHF, hypertension and type II diabetes. He initially presented to you
with changes in stool patterns, constipation and hematochezia. He was
found to have adenocarcinoma of the ascending colon along with
metastasis to his liver, brain and his right lung. For patient presenting
with metastatic colon cancer, which of the following treatments is not
routinely recommended?
(1pts)

Chemotherapy

Primary Resection of the colon cancer

Radiation

All are routinely recommended

All of the above

The answer is B. In treating Colorectal cancer, it is crucial to


understand that surgical extirpation of the primary tumor is done when
there is a realistic possibility of cure or for patients with symptomatic
tumors that cause acute obstruction or clinically significant bleeding.
For those who present with synchronous primary tumors and incurable
metastatic disease, resection is not routinely indicated. Advances in
systemic chemotherapy (outlined below) have significantly increased
the probability of managing the tumor medically, and chemotherapy
can begin immediately in the setting of an asymptomatic or minimally
symptomatic primary. In other words, there is no need to delay
initiation of systemic chemotherapy by palliative resection of a primary
tumor that is not actively symptomatic. In truth, resection of a primary
lesion in the setting of metastatic disease is likely to cause significant
morbidity and even mortality. A review of Medicare/SEER data
focusing on patients 65 and older reported a 30-day postoperative
mortality of 10% with resection of a synchronous primary tumor.
Furthermore, in a large retrospective series, Poultsides and colleagues
reported that 93% of patients presenting with synchronous stage IV
disease without overt obstruction never required specific intervention
on their primary tumor. Adenocarcinoma of the Colon and Rectum
Feedback on your answer
Collapse
Unit 3
10)
0/1
You are seeing a 61-year old female who presents with worsening shortness of breath
for two days. The day preceding her symptoms she was involved in an MVC and was
seen in your facility and diagnosed with 3 non displaced rib fractures on the right. Her x-
ray today is shown below. What is the most likely diagnosis?

(Select 1)(1pts)

Pneumothorax

Pulmonary Contusion

Fat embolism

Pneumonia

Congestive Heart Failure

The answer is B. Severe blunt trauma to the chest can cause rib fractures and other
immediate injuries but sometimes pathology doesn't show up until days later such as a
pulmonary contusion and myocardial contusion. The contused lung is very sensitive to
fluid overload which can lead to respiratory distress. Pulmonary contusions generally
develop over the first 24 hours and resolve in about one week. Irregular, nonlobular
opacification of the pulmonary parenchyma on chest radiograph is the diagnostic
hallmark. About one-third of the time the contusion is not evident on initial radiographs.
Chest CT provides better resolution, but rarely alters management, unless other injuries
are found. Contusions evident on CT but not plain radiograph have better outcomes.
Pain control and pulmonary toilet are the mainstays of treatment. Prophylactic
endotracheal intubation is unnecessary, but patients with hypoxia or difficulty ventilating
require airway management. While opinions vary, fluid resuscitation with crystalloid to
euvolemia appears appropriate. Common complications include pneumonia and acute
respiratory distress syndrome (ARDS). Read more

Feedback on your answer


Collapse
11)
1/1
You are seeing a 27-year old male in the trauma bay shortly after being stabbed in this
back with a 6 cm buck knife by his disgruntled neighbor after an argument.
He arrived via EMS sitting up, awake and alert with a GCS of 15. His BP is 140/90, HR
120 and O2 sat is 85% on a non re-breather.
On physical exam there is a 3 cm non bleeding wound just left of his mid thoracic spine.
Upon auscultation there is decreased breath sounds on his left side. There are no signs of
tracheal deviation or JVD.
What is the next best step in management?
(1pts)

Immediately to the OR for surgical exploration

Upright Chest Xray followed by a chest tube

Immediate Needle thoracotomy on the left

Pericardiocentesis under US guidance

The answer is A, Upright Chest Xray followed by a chest tube. This patient is awake and
talking. He is stable enough for a chest xray followed by chest tube placement. A needle
thoracotomy to should be done if he has evidence of tension pneumothorax which would
be respiratory distress and hypotension. He will likely then need a chest tube and can be
further managed. Read More
Feedback on your answer
Collapse
12)
0/1
A 65-year-old male comes in the emergency Department because of severe right-sided
chest pain following a fall off his ladder 10 feet high.
Examination shows decreased respiratory movements on the right side of the chest and
tenderness on palpation over the right mentions, his abdomen is soft and non tender.
An x-ray film of his chest shows a non displaced fracture of the right 6th and 7th ribs on
the right without signs of pneumothorax.
Which of the following is the most appropriate goal in management of the rib fracture in
this patient?
(1pts)

To achieve a tital volume of 500 ML with intubation

To ensure appropriate analgesia

To provide mechanical stabilization to the chest wall

To give prophylactic antibiotics

The answer is B. Once significant associated injuries have been evaluated and treated, the
cornerstone of rib fracture management is pain control [36-38]. Early and adequate pain
relief is essential to avoid complications from splinting and atelectasis, primarily
pneumonia. The choice of analgesia depends upon the injury, the clinician's comfort
performing nerve blocks with their potential complications, and the ease with which more
invasive treatments can be performed. Analgesia for severe and multiple rib fractures and
monitoring of admitted patients are discussed separately.
For isolated injuries (ie, single rib fracture), clinicians generally begin treatment with
nonsteroidal anti-inflammatory drugs (NSAIDs) with or without opioids. For more severe
injuries, particularly if ventilation is compromised, admission and invasive treatments,
such as intercostal nerve blocks, may be needed.
Respiratory care, including use of incentive spirometry to prevent atelectasis and its
complications, is often important. We do not recommend rib belts or binders because
they compromise respiratory function. Studies of rib belts involve small numbers of
patients and have reached contradictory conclusions [39,40]. Patients with a rib fracture
who are discharged home can also use incentive spirometry throughout the day, after
analgesics have taken effect. Holding a pillow or similar soft brace against the fracture
site reduces discomfort while using the spirometer, or when coughing.
Disposition — Several researchers recommend hospital admission for any patient with
three or more rib fractures, and ICU care for elderly patients with six or more rib
fractures [14,15]. They cite the significant correlation between these findings and
serious internal injuries, such as pneumothorax and pulmonary contusion. We suggest
hospitalization for the majority of patients with three or more rib fractures. Patients with
multiple rib fractures sustained from high-energy trauma are best evaluated at a trauma
center. Transfer should be arranged expeditiously.
Displaced rib fractures likely increase the risk of injury to the lung and proximate
intercostal blood vessels. Bleeding from such fractures can be delayed, and admission or
close observation and follow-up should be arranged for patients with displaced fractures,
depending upon clinical and social circumstances. Multiple case reports indicate that
delayed bleeding from intercostal vessels or other injuries can be life-threatening,
particularly in older patients [20,41,42].
Surgical fixation may be of benefit with some types of rib fractures, particularly those
associated with chest wall deformity, flail chest, or symptomatic nonunion. The
appropriate role of surgical fixation is discussed separately. (See "Inpatient
management of traumatic rib fractures", section on 'Surgical
management'.)
Rarely, younger healthy individuals with three rib fractures, having undergone a thorough
clinical and radiographic evaluation by clinicians experienced in trauma management,
and an appropriate period of observation (a minimum of six hours of observation,
including a follow-up chest radiograph, is needed to rule out pneumothorax), may be
discharged from the emergency department. Clinicians must also consider patient
comorbidities and clinical and social circumstances when determining disposition.
Patients with one or possibly two isolated rib fractures and no complicating factors may
be discharged home with appropriate follow-up and adequate analgesia. Uptodate
Feedback on your answer
Collapse
13)
1/1

What is the estimated overall mortality from pulmonary contusions?

(Select 1)(1pts)

3% to 5%

5% to 10%

10% to 25%

25% to 40%

The answer is C, 10-25%. Pulmonary contusions, pneumothorax, and hemothorax occur


in 30% to 50% of patients with severe blunt chest trauma managed in trauma centers.
Pulmonary contusions commonly result from direct impact to the chest wall or from
concurrent chest wall injury, such as from a blast.20 Acute parenchymal lung injury
produces multiple physiological effects, all of which have an impact on outcomes. The
mortality associated with pulmonary contusion is difficult to predict, but is estimated to
be 10% to 25%.21,22 The clinical manifestations of pulmonary contusion are variable,
ranging from mild dyspnea to acute lung injury and acute respiratory distress syndrome.
The pathophysiology of pulmonary contusion includes alveolar hemorrhage and edema
resulting in decreased lung compliance, increased alveolar capillary permeability, and
increased intrapulmonary shunting. These processes result in hypoxemia, hypercarbia,
and decreased lung compliance. Lung contusions also result in acute local and systemic
inflammatory cascades that activate tissue macrophages and the production of
inflammatory mediators, cytokines, and chemokines. These result in pathophysiologic
changes that clinically manifest as immunosuppression, acute lung injury/acute
respiratory distress syndrome, and respiratory failure. Read more

Feedback on your answer


Collapse
Unit 4
14)
1/1
You are seeing a 25-year old male in the trauma bay after he was involved in a motor
vehicle accident. He was thrown from his bike and is now complaining of right flank
pain. His GCS is 15 and he is awake and oriented. His BP is 125/75, HR 99, RR 18, O2
98% on room air. His physical exam is pertinent for right flank ecchymosis with mild
tenderness to palpation along with mild right upper quadrant tenderness. There is no
gross blood seen at his urethral meatus and his urinalysis shows no RBCs. True or False:
In the setting of potential renal trauma (blunt and penetrating trauma), gross or
microscopic hematuria is ALWAYS present.
(1pts)

True

False Correct

The answer is False: In the setting of renal trauma, gross or microscopic hematuria is
absent in up to 5% of cases and this finding alone should not be used to preclude in those
you are suspicious of renal trauma. Read more on renal trauma
Feedback on your answer
Collapse
15)
0/1
You are seeing a 48-year old male in the trauma bay after shooting himself in his right
thigh while it was in its holster.
On exam there is one wound that is about 2cm in diameter on his posterolateral right
thigh. It has since stopped bleeding after pressure was applied. His pulses are equal and
symmetrical and his neuro exam in normal. You obtain an xray which shows bullet
fragments in his thigh without evidence of fracture.
What is the next best step in treatment?
(1pts)

Ct scan of his leg to evaluate the soft tissue damage

Ultrasound to evaluate arterial flow

Ct angiography of his leg

Irrigation and tetanus

This patient shows no sign of arterial injury as he has no hard or soft signs of vascular
injury (see below). He has a normal neuro exam and has no fracture on xray. The only
thing that this patient needs is a tetanus update and wound irrigation.

Hard signs of Vascular injury


 Pulsatile bleeding
 Expanding haematoma
 Absent distal pulses
 Cold, pale limb
 Palpable thrill
 Audible bruit
The presence of hard signs of vascular injury mandates immediate operative intervention.
Usually the site of injury is obvious, and angiography is unnecessary. If in doubt,
angiography can be performed emergently on the operating room table. Unnecessary
interventions and investigations should be avoided to minimise the delay to definitive
care. Read more
Feedback on your answer
Collapse
16)
1/1
You are seeing a 32-year old male who presents after being kicked multiple times in his
right flank. He is complaining of severe flank pain and gross hematuria. What is the gold
standard for imaging of renal trauma?
(Select 1)(1pts)

MRI abdomen/pelvis

Ultrasound

CT abdomen/pelvis with contrast

Intravenous pyelography
None of the above

The answer is C, CT abdomen/pelvis with contrast. CT scanning with intravenous


contrast enhancement is the modality of choice for the identification and staging of renal
trauma in the hemodynamically stable patient (image 6). The initial CT images will
frequently miss injuries to the renal pelvis and ureters as sufficient contrast may not yet
be present in the collecting system. Additional delayed images are needed to assess for
contrast extravasation when these injuries are suspected (image 7). In patients
undergoing CT scanning of other organ systems, suspected bladder injuries may be
investigated with CT cystography after retrograde filling of the bladder, as described
above. Read more
 CT with IV contrast is the Gold standard, high sensitivity
 Immediate and delayed post-contrast images to view collecting
system
 Images abdomen and retroperitoneum
 Allows diagnosis and staging
 Intravenous pyelography doesnt allow you to image the abdomen or
retroperitoneum.

Feedback on your answer


Collapse
17)
1/1
A 10-year-old male is brought to the emergency department abdominal pain after he fell
riding his bicycle. His parents say that he wiped out on some sand and
hit his epigastric region into his handle bars and has since had repeated vomiting.
Vital signs show a blood pressure of 109/71, HR 99, RR 16, O2 on RA is 99%. On
physical exam he is awake and alert with a GCS of 15. He appears in moderate pain and
his abdomen is tender in his epigastrum with some voluntary guarding. The rest of his
exam is unremarkable. Barium examination shows duodenal obstruction. CT scan of the
abdomen shows a 3-cm duodenal hematoma with no other abnormalities. Which of the
following is the most appropriate next step in management?

(Select 1)(1pts)

Exploratory laparotomy

Nasogastric suction with parenteral nutrition

Bowel rest and antibiotics


Endoscopic removal of the hematoma

The answer is B, Nasogastric suction with parenteral nutrition. In the treatment of


duodenal injuries, one cannot forget the overall patient. Embarking on a complex repair
in the unstable, cold, acidotic coagulopathy patient is ill advised and will likely not end
well. "Damage control" principles of hemorrhage and contamination control should not
be forgotten in these often multiply injured patients. Once the timing is right, the surgical
treatment of duodenal injuries has two parts. One is the repair itself, and the other is to
assess the need to "protect" the repair or preparation for a potential leak and the resultant
fistula. The grade of the injury and the location within the duodenum often dictate the
type of repair, whereas the grade, location, and potentially other associated injuries,
especially the pancreas, often dictate the possible benefit of adjunctive "preparation"
maneuvers. First, however, we should discuss the possibility of nonoperative treatment of
select duodenal injuries.
Nonoperative treatment of select duodenal injuries has been well described and is more
common with blunt mechanisms as penetrating injuries are often diagnosed at
laparotomy. Endoscopic injuries are another set of duodenal trauma that may be managed
nonoperatively as they are commonly of low grade. In one recent multicenter review, all
success with nonoperative treatment was with grade I or II injuries only.18 In isolated
duodenal trauma, the indications for laparotomy are not different from those for other
types of abdominal trauma, hemodynamic compromise and peritonitis being the most
common. Thus, the indication for attempting nonoperative "treatment" of duodenal
injuries requires a hemodynamically normal patient without peritonitis or other indication
for immediate surgical intervention.
At times, duodenal hematomas can cause duodenal obstruction. This usually presents 2 to
3 days after the trauma with evidence of gastric outlet obstruction. The common
nonoperative treatment of isolated obstructing duodenal hematomas is nasogastric tube
decompression of the stomach and duodenum, nutritional support, and time.38 Repeat
imaging in 7 to 14 days is reasonable to evaluate improvement or an unexpected finding
as most duodenal hematomas will have resolved after 7 to 14 days, and continued
obstruction may reveal additional injury, prompting intervention.39-41 Enteral nutrition is
preferred to parenteral nutrition, but in the setting of a duodenal obstruction, the
parenteral route (total parenteral nutrition [TPN]) is often required as passing a feeding
tube beyond the obstruction can be difficult and surgical feeding access defeats the
purpose of "nonoperative" care.

Endoscopic injury to the duodenum deserves special mention here as it is a specific type
of duodenal trauma that may be commonly encountered by surgeons, even when trauma
does not make up a large part of their practice. There is literature that reports many of
these injuries being treated nonoperatively.42This is consistent with similar nonoperative
management of other isolated grade I or II injuries. However, as with any other type of
trauma, the clinical status of the patient and standard indications for laparotomy should
dictate the need for surgery.43 CT has been reported as the imaging test of choice to
diagnose perforation from endoscopy if immediate indications for laparotomy do not
exist.44 Often the pressure can be high to pursue a nonoperative approach as surgery was
likely not on the patient's or endoscopist's schedule, but clinical parameters should dictate
the appropriate course of action. Again, endoscopic trauma to the duodenum should
follow the same principles of evaluation and treatment as other mechanisms and the
patient course dictate treatment.

As noted above, operative treatment of duodenal injuries has two major components: the
repair and evaluating the need for leak "preparation." The repair itself must follow the
basic principles of all anastomosis: approximation of well-vascularized, healthy, tension-
free tissue. "Preparation" maneuvers are not required for most injuries and are debated.
They are discussed below.

Types of Duodenal Repairs

If a duodenal hematoma is discovered at operation, there is some debate as to the optimal


management. Simple observation for most hematomas is advocated by some. Other
authors have argued for seromuscular incision and decompression to ensure no duodenal
obstruction or delayed rupture. A selective approach seems to be a common compromise
depending on the findings at the time of operation, specifically the size of the hematoma.

For lacerations, the location and extent of tissue damage or loss are critical in deciding
the best type of repair. As noted in the literature, simple approximation is the most
common repair selected.9,45-48 This is commonly described as being done in a running
continuous fashion with monofilament suture after the edges have been debrided of any
questionable tissue. This repair choice mandates that the basic principles of bowel
anastomosis be met: approximation of tension-free, healthy, vascularized tissue.
Adequate duodenal mobilization is often required to decrease tension and should have
been done already for appropriate diagnosis.

If any of the basic principles cannot be met with simple approximation or if the resultant
repair would narrow the lumen of the duodenum, other options are available for repair.
The next option considered is often resection of the involved portion of duodenum and
primary anastomosis.48 Again, for this to have a maximal chance of success, the
anastomosis must be between well-vascularized, healthy, tension-free tissues. Another
option is a Roux-en-Y duodenojejunostomy. This is more likely to be necessary with D2
injuries not amenable to simple approximation. The pancreaticoduodenectomy with
pancreaticojejunostomy, gastrojejunostomy, and hepaticojejunostomy reconstruction
(Whipple procedure) is rarely needed in duodenal trauma and is discussed below.
Randomized comparison of one repair versus another is absent, but most reviews
consider simple approximation to be the most commonly required and most commonly
used repair.

2014. Scientific American Surgery. Hamilton, Ontario & Philadelphia, PA. Decker
Intellectual Properties Inc. ISSN 2368-2744. STAT!Ref Online Electronic Medical
Library. http://online.statref.com/Document.aspx?fxId=61&docId=2329. 10/17/2014
5:10:16 AM CDT (UTC -05:00).

Feedback on your answer


Collapse
Unit 5
18)
1/1
Following non mesh hernia repair, how many patients would need to be treated to prevent
one infection at 30 days?
(Select 1)(1pts)

20

50

75

120

The answer is 50. Mesh infection is rare. In a 2003 Cochrane review of


antibiotic prophylaxis for nonmesh hernia repairs, the overall infection
rate was 4.69% in the control group and 3.08% in the treatment
group.65 Thus, to prevent one infection in 30 days, 50 patients would
have to be treated, and
these patients would then be at risk for antibiotic-associated
complications. Laparoscopic repairs were excluded from this review;
however, in a meta-analysis comparing postoperative complications
after laparoscopic inguinal hernia repair with those after open repair,
superficial infection was less frequent in the laparoscopic
groups.49 Deep mesh infection was rare in both groups. Mesh
infection usually responds to conservative treatment with antibiotics
and drainage. On rare occasions, the mesh must be removed; this
may be accomplished via an external approach. If, however, a
prosthesis composed of a hydrophobic material (e.g., expanded
polytetrafluoroethylene) becomes infected, it is very difficult to
sterilize and virtually always must be removed. It is noteworthy that
removal of the mesh does not always lead to recurrence of the hernia,
a finding that may be attributable to the resulting fibrosis. Read more
Feedback on your answer
Collapse
19)
1/1
Which of the following statements regarding hernias is not correct?
(Select 1)(1pts)
In female patients, indirect inguinal hernias are the most common type

The lifetime risk of having to undergo an inguinal hernia repair is greater


for men than in women

In male patients, direct inguinal hernias are the most common type

Femoral hernias account for fewer than 10% of all groin hernias; however,
40% present as emergencie

Answer C is the only incorrect statement. In the United States, approximately


1,000,000 abdominal wall herniorrhaphies are performed each year, of
which almost 80% are for inguinal or femoral hernias.1Worldwide,
some 20 million groin hernias are repaired each year.2 The lifetime
risk of having to undergo an inguinal hernia repair is 27% for men and
3% for women.3 In male patients, indirect inguinal hernias are the
most common type (more often located on the right), occurring
approximately twice as frequently as direct inguinal hernias; femoral
hernias account for a much smaller percentage. In female patients,
indirect inguinal hernias are also the most common type, but femoral
hernias are seen more frequently than direct hernias, which are rare in
this population. Femoral hernias account for fewer than 10% of all
groin hernias; however, 40% present as emergencies (i.e., with
incarceration or strangulation), and mortality is higher for emergency
repair than for elective repair. A two-peak theory has been described,
stating that a new diagnosis of an inguinal hernia is most likely in
patients younger than 1 year and in patients older than 55 years,
although hernias can be diagnosed across any given age group. Read
more
Feedback on your answer
Collapse
20)
0/1
A 24-year old male basketball player presents to you with a painful bulge into his right
inguinal area into his scrotum that he first noticed while he was playing basketball. He
asks if it could be a sports hernia and if this will limit his practicing and upcoming
tournament. What hernia does this patient most likely have?
(Select 1)(1pts)

Indirect

Direct
Femoral

Richter

This patient most likely has an indirect hernia as it is the most common hernia and given
the lack of other findings in the stem that is the best answer and most likely. Although a
sports hernia may lead to a traditional, abdominal hernia, it is a different injury. A sports
hernia is a strain or tear of any soft tissue (muscle, tendon, ligament) in the lower
abdomen or groin area. Because different tissues may be affected and a traditional hernia
may not exist, the medical community prefers the term "athletic pubalgia" to refer to this
type of injury.
An indirect inguinal hernia is one of the most common abdominal hernias. It is
five times more common than a direct inguinal hernia, and is seven times more frequent
in males, due to persistence of the process vaginalis during testicular descent. An indirect
hernia enters the inguinal canal at the deep ring, lateral to the inferior epigastric vessels.
It passes infromedially to emerge via the superficial ring and, if large enough, extend into
the scrotum. In children, the vast majority of inguinal hernias are indirect (see Case 3).
Incarceration represents the most common complication associated with inguinal hernias,
the incidence could be as high as 30% for infants younger than 2 months.
A direct inguinal hernia arises from protrusion of abdominal viscera through a
weakness of the posterior wall of the inguinal canal medial to the inferior epigastric
vessels, specifically through the Hasselbach's triangle. This type of hernia is termed
direct as the hernial sac directly protrudes through the inguinal wall in contrast to indirect
ones which arise through the deep ring and enter the inguinal canal. Since direct hernias
do not have a guiding path, they seldom extend into the scrotum unless very large and
chronic. Direct hernias arise usually as acquired weakness of the Hasselbalch's triangle.
Therefore, they are seen in the elderly with chronic conditions which increase intra-
abdominal pressure over a long period, e.g. COPD, bladder outflow obstruction, chronic
constipation etc. Increased abdominal pressure is transmitted to both sides and as a result,
direct hernias are usually bilateral. Compared to indirect hernia, they are less susceptible
to strangulation as they have a wide neck.
In contrast to the indirect hernia, a direct hernia is most often an acquired lesion. It
occurs when a weak spot develops in the lower abdominal musculature (the posterior
floor of the inguinal canal) due to the normal and/or abnormal stresses inflicted by living
and aging. In adults, stresses such as lifting heavy of objects, frequent coughing or
straining, pregnancy, and constipation can instigate hernia. Unlike indirect hernias, direct
hernias traverse medial to the inferior epigastric vessels and are not associated with the
processus vaginalis. The hernia consists primarily of retroperitoneal fat. Only rarely is a
peritoneal sac containing bowel encountered. Because there is typically no involvement
of a sac, they do not protrude with the spermatic cord, and as such, have a lower
incidence of incarceration or strangulation. Like indirect inguinal hernia, direct inguinal
hernias typically cause a bulge in the groin (at the top of or within the scrotum) and
usually with increased abdominal pressure. Like indirect hernias, they may or may not be
painful (usually not). By palpating the inguinal canal and asking the patient to cough
while standing, one can usually elicit the hernia. In fact, one can often times palpate an
inguinal hernia without invaginating the scrotum (as is typically taught in medical
school). Rather, by placing one's fingers over the inguinal canal and asking the patient to
cough, one can often feel the bulge against the lower abdominal wall. As direct and
indirect hernias are unreliably differentiated by physical exam alone, the need to
invaginate the scrotum to feel into the inguinal canal is often more uncomfortable to the
patient, than telling to the physician. Rarely, palpation is not even necessary, as the
hernia is large enough to be visualized. Read more on Inguinal hernias: A Brief
review

Feedback on your answer


Collapse
21)
0/1
What % of minimally symptomatic inguinal hernias end up requiring emergency
operation for strangulation over a one year period?

(Select 1)(1pts)

0.18%

1.18%

2.1%

6.5%

12.4%

The answer is A, The natural history of an untreated, minimally


symptomatic inguinal hernia was addressed in a randomized,
controlled trial in which 364 men were assigned to "watchful waiting"
(WW) and 356 men underwent routine operation.48 Only two patients
in the WW group required emergency operations for strangulation over the
follow-up period of 2 to 4.5 years. This result translated into a rate of 1.8 per 1,000
patient-years (0.18%), or about one fifth of 1% for each year that the hernia remains
unrepaired. The two patients who required emergency operations recovered uneventfully.
The question that remained to be answered was which group fared better overall: the WW
group or the group whose hernias were repaired immediately in accordance with
conventional teaching? The answer to this question was at variance with conventional
assumptions. At the conclusion of the study, functional status, as measured by quality of
life instruments and pain scales, was identical in the two groups. About one third of the
patients in the WW group crossed over to undergo operative treatment, principally
because of symptom progression. However, there appeared to be no penalty for delaying
surgery. Before this study, most surgeons assumed that a hernia would become harder to
repair the longer it remained (because of enlargement and buildup of scar tissue) and that
patients whose operations were delayed would experience more complications. The
investigators found, however, that postoperative complication rates were the same in
patients who underwent immediate surgery as in those who were assigned to WW but had
to cross over to surgical treatment. Read more

Feedback on your answer


Collapse
Unit 6
22)
1/1
Which of the following statements about duodenal or pancreatic injury is not true?
(Select 1)(1pts)

CT scan is the primary diagnostic modality

A bicycle handlebar to the epigastrium is the most common cause of injury

An elevated amylase level is a very sensitive but non specific finding

Duodenal hematomas causing obstruction usually present 2-3 after initial


injury

Answer C is the only false statement as No laboratory findings are particularly specific
for duodenal or pancreatic injury. Amylase has been proposed for both pancreas and
duodenal injuries but is not specific for either one. Some have noted that amylase can be
elevated in as many as 50% of duodenal injuries, but this has not been a consistent
finding.9 An elevated amylase should prompt an evaluation of the duodenum for injury
but alone is not diagnostic. Although the highest concentration of amylase in the human
body is in the pancreas, hyperamylasemia is also not a reliable indicator of pancreatic
trauma. In one series, only 8% of blunt abdominal injuries with hyperamylasemia had
pancreatic injury.10 Furthermore, as many as 40% of patients with a pancreatic injury
may initially have a normal serum amylase.
The history of a patient with a possible blunt pancreatic or duodenal injury usually
consists of a direct blow to the epigastrium. In children, this commonly involves a bicycle
handlebar to the epigastrium; in adults, more commonly the steering wheel or a motor
cycle handle bar is involved. However, any direct blow should raise suspicion. The
patient may complain of abdominal, back, or flank pain.
Outside of the pelvis x-ray, plain abdominal radiographs to evaluate blunt abdominal
trauma are less common in the era of focused abdominal sonography for trauma (FAST)
and computed tomographic (CT) scans. Reports suggest that signs of duodenal injury on
plain radiographs are identified less than one third of the time. Retroperitoneal air, free
intraperitoneal air, or obliteration of the psoas shadow should raise suspicion for
duodenal and other hollow viscous trauma.9 Upper gastrointestinal series have also been
used to evaluate the duodenum for injury and can add to the sensitivity and specificity of
plain films, but more recently, CT has become the primary diagnostic modality.
For optimal duodenal evaluation, intraluminal contrast administered via a nasogastric
tube soon but not immediately prior to the CT scan may aid by opacification of the lumen
of the duodenal "C loop." Visualization of contrast extravasation, retroperitoneal air,
adjacent fat stranding, and unexplained fluid, as well as duodenal wall thickening, are CT
findings suggestive of potential duodenal injury. The sensitivity of CT for duodenal
injury is related to the technology of the scanner (i.e., "number of slices") and the time
interval from the injury to imaging. In a recent review, CT was considered to have an
overall sensitivity of around 76% with new-generation scanners (16- or 64-slice), having
a higher sensitivity of around 82%. Additionally, if clinical suspicion remains high after
an initial negative CT scan, then repeat imaging is warranted and may improve diagnostic
yield.
At times, duodenal hematomas can cause duodenal obstruction. This usually presents 2 to
3 days after the trauma with evidence of gastric outlet obstruction. The common
nonoperative treatment of isolated obstructing duodenal hematomas is nasogastric tube
decompression of the stomach and duodenum, nutritional support, and time.38 Repeat
imaging in 7 to 14 days is reasonable to evaluate improvement or an unexpected finding
as most duodenal hematomas will have resolved after 7 to 14 days, and continued
obstruction may reveal additional injury, prompting intervention.39-41 Enteral nutrition
is preferred to parenteral nutrition, but in the setting of a duodenal obstruction, the
parenteral route (total parenteral nutrition [TPN]) is often required as passing a feeding
tube beyond the obstruction can be difficult and surgical feeding access defeats the
purpose of "nonoperative" care.
Feedback on your answer
Collapse
23)
0/1
Which of the following statements regarding hollow viscus injuries in patients suffering
from blunt trauma is not true?
(Select 1)(1pts)

Mortality is increased in parallel with time to operative intervention

Only 1.2% of blunt trauma admissions had an associated hollow viscus


injury
The "seat-belt" sign is associated with a more than doubled relative risk of
small bowel injury

Unexplained intraperitoneal fluid (i.e., fluid appearing in the absence of


solid-organ injury) was the least common radiographic finding associated
with blunt bowel or mesenteric injury

Answer D is false. Unexplained intraperitoneal fluid (i.e., fluid appearing in the absence
of solid-organ injury) was the most common radiographic finding associated with blunt
bowel or mesenteric injury but often proved to be a false positive finding.
Hollow viscus injury after blunt trauma, although uncommon, can have serious
consequences if the diagnosis is missed or delayed. In a multi-institutional study of 198
patients with blunt small bowel injury, delay of as little as 8 hours in making the
diagnosis resulted in increased morbidity and mortality.2 Mortality increased in parallel
with time to operative intervention (< 8 hours to operation, 2% mortality; 8 to 16 hours,
9%; 16 to 25 hours, 17%; > 24 hours, 31%), as did the complication rate. Consequently,
it is important to have an expedient approach to the diagnosis of blunt bowel injury.
Physical examination findings such as abdominal tenderness or tachycardia may suggest
the presence of hollow viscus injury. Distracting chest or long bone injury, closed-head
injury, spinal cord injury, or intoxication, however, may compromise reliability of the
examination. In addition, it is not uncommon for blunt bowel injury to have a latent
period from the time of injury, whereby the expected signs and symptoms of such injuries
take some time to develop. Laboratory abnormalities, including elevations in white blood
cell (WBC) count, amylase, and/or lactic acid, may also point toward the presence of
hollow viscus injury but are relatively nonspecific. Provided that the patient has suffered
a low-risk mechanism of injury (such as a fall from standing or a low-speed motor
vehicle collision), hollow viscus injury is extremely unlikely in the face of a normal,
reliable physical examination and normal laboratory results. With these conditions
present, the presence of blunt bowel injury can be effectively excluded. However, the
presence of abdominal complaints, an abnormal or unreliable physical examination,
abnormal laboratory results, or a high-risk mechanism of injury (such as a high-speed
motor vehicle collision) warrants further evaluation by imaging for the presence of bowel
injury.
Particular consideration should be given to lap-and shoulder-restraint injuries, which may
be associated with an increased risk of hollow viscus injury. The "seat-belt" sign (i.e.,
ecchymosis of the abdominal wall secondary to the compressive force of the lap belt) is
associated with a more than doubled relative risk of small bowel injury.3,4 Flexion-
distraction fractures of the spine (Chance fractures) are also associated with lap-belt use,
and the presence of such fractures should raise the index of suspicion for associated
hollow viscus injury.
Ultrasonography is routinely performed early in the evaluation of blunt abdominal
trauma. It is highly specific and moderately sensitive in identifying intra-abdominal fluid,
the presence of which in a hemodynamically unstable patient is an indication for
laparotomy (in that it strongly suggests the presence of significant intra-abdominal
hemorrhage).5 Ultrasonography does not, however, reliably distinguish solid-organ
injury from hollow viscus injury—a distinction that is critical for determining subsequent
management (i.e., operative versus nonoperative) in a hemodynamically stable patient.
Computed tomography (CT) is the imaging modality of choice in stable patients who
warrant evaluation by imaging as described above. We reviewed over 8,000 CT scans
performed to evaluate cases of blunt abdominal trauma and found that the number of
abnormal radiologic findings suggesting blunt injury to the bowel, the mesentery, or both
was correlated with the true presence of injury [see Table 1].6 A CT scan
demonstrating a solitary abnormality was associated with a true positive rate of 36%,
whereas a scan demonstrating more than one abnormality was associated with a true
positive rate of 83%. Unexplained intraperitoneal fluid (i.e., fluid appearing in the
absence of solid-organ injury) was the most common radiographic finding associated
with blunt bowel or mesenteric injury but often proved to be a false positive finding. On
the basis of this experience, we developed an algorithm for the evaluation of blunt hollow
viscus injury in patients with unreliable physical examinations [see Figure 1].
Most CT scans performed in this clinical setting, however, will be negative for evidence
of intra-abdominal injury. A prospective multi-institutional trial involving 3,822 blunt
trauma patients demonstrated that the negative predictive value of a normal abdominal
CT scan was 99.63%, leading the authors to conclude that patients with a normal scan do
not benefit from hospital admission and prolonged observation.7 However, a multi-
institutional review of 2,457 cases carried out by the Eastern Association for the Surgery
of Trauma (EAST) reported a 13% incidence of blunt small bowel injury in patients with
an initial negative CT scan. These results indicate that caution should be exercised in
dismissing the presence of hollow viscus injury on the basis of a negative scan.3 This
concern is echoed by our own institutional experience, in which the incidence of injury in
patients with an initial negative CT scan was 12%.6 If CT scanning demonstrates no
suspicious findings, no further diagnostic workup of hollow viscus injury is necessary,
but the duration of the observation period depends on both the overall condition of the
patient and clinical judgment. Most patients, as supported by the negative predictive
value of the study above, will not require ongoing further observation.
Feedback on your answer
Collapse
24)
1/1
What is the mean mortality rate of patients suffering from pancreatic trauma?
(Select 1)(1pts)

<3%

9%

19%

31%
The answer is C, 19%. Duodenal and pancreatic injury continues to
challenge the trauma surgeon. The relatively rare occurrence of these
injuries, the difficulty in making a timely diagnosis, and high morbidity
and mortality rates justify the anxiety these unforgiving injuries
invoke. Mortality rates for pancreatic trauma range from 9 to 34%,
with a mean rate of 19%. Duodenal injuries are similarly lethal, with
mortality rates ranging from 6 to 25%. Complications following
duodenal or pancreatic injuries are alarmingly frequent, occurring in
30 to 60% of patients.1-3 Recognized early, the operative treatment
of most duodenal and pancreatic injuries is straightforward, with low
morbidity and mortality. Read more
Feedback on your answer
Collapse
25)
0/1
What percentage of patients suffering from blunt abdominal trauma develop hollow
viscus injury?
(Select 1)(1pts)

1.2%

4.8%

8.9%

13.4%

The answer is A, 1.2%. Hollow viscus injury after blunt trauma, although
uncommon, can have serious consequences if the diagnosis is missed
or delayed.Hollow viscus injury is most often the consequence of
penetrating abdominal trauma. As a result of blunt force trauma,
bowel injury occurs with relative infrequency: in one multi-institutional
analysis, only 1.2% of blunt trauma admissions had an associated
hollow viscus injury. Read more
Feedback on your answer
Collapse

Unit 1
1)
0/1
You are seeing 45-year old black female again in the office who presented to you initially
for a right sided breast mass. She has no past medical history or family history of breast
cancer. On physical exam you palpated a 2 cm mass just lateral to her right nipple. You
ordered a mammogram which cam back normal. What percentage of patients who are
found to have a normal mammogram have breast cancer after biopsy of a suspicious
lesion?
(Select 1)( 1pts extra credit)

5%

15%

25%

45%

The answer is A. A diagnostic mammogram is the first imaging study performed for
a woman with a new, palpable breast mass, and should be performed even if a recent
mammogram was negative. While the false negative rate of mammograms is less than 5
percent for clinically palpable breast cancers [15], a normal mammogram does not
eliminate the need for further evaluation of a suspicious mass [5]. For women under age
30 years, the breasts are hypersensitive to radiation exposure [16]; however, if the
clinical findings are suspicious, a mammogram should be performed. Read More

Feedback on your answer


Collapse
2)
0/1
A 65-year-old white female comes to your office for a routine visit. She has no
complaints at this time. She has a past medical history of a cholecystectomy 30 years ago.
She does not smoke drink or use illicit drugs. She takes no medication and has no family
history of cancer. She had a normal mammogram two years ago as well as a normal Pap
smear (her third consecutive negative screening). She had a colonoscopy six years ago
that revealed no lesions and a fasting lipid panel two years ago that was normal. Which of
the following studies is most appropriate this time?

(Select 1)( 1pts extra credit)

Electrocardiogram

Mammography

Pap smear
Colonoscopy

The answer is B, mammography. The U.S. Preventive Services Task


Force (USPSTF) recommends screening mammography every two years for women
between ages of 50 and 74. Routine mammography is not necessary beyond 75 is no
longer emphasized or recommended. The women in this scenario needs a mammogram
once every two years until she is 75.
Electrocardiograms are not effective screening studies in patients pose no risk for
coronary artery disease and are not recommended.
The USPSTF recommends routine cholesterol screening in men at average risk for
coronary artery disease beginning age 35 and women at age 45 the exact stop date is not
specified. Many experts advocate checking lipids no more than every five years in
patients with normal risk of coronary artery disease and normal lipid panel in the past.
Woman beyond the age 65 to 72 of had three consecutive negative cervical smears no
longer require screening. Read more

Feedback on your answer


Collapse
3)
1/1
What is the most common cancer among woman?
(Select 1)( 1pts extra credit)

Ovarian

Lung

Colon

Breast

The answer is D, Breast Cancer. Approximately 230,480 American women are diagnosed
with breast cancer annually, and 39,520 women die from this disease. Global cancer
statistics show that breast cancer is the most frequently diagnosed cancer and the leading
cause of cancer death among females, accounting for 23 percent of total cancer cases and
14 percent of cancer deaths. Breast cancer is now also the leading cause of cancer death
among females in economically developing countries.
Globally, breast cancer is the most frequently diagnosed cancer and the leading cause of
cancer death in women. In the United States, breast cancer is the most commonly
diagnosed cancer and the second most common cause of cancer death in women. In
addition, breast cancer is the leading cause of death in women ages 40 to 49 years.Breast
cancer is treated with a multidisciplinary approach involving surgical oncology, radiation
oncology, and medical oncology, which has been associated with a reduction in breast
cancer mortality. Read more from Up to date

Feedback on your answer


Collapse
4)
0/1
You are seeing a 32-year old male who is presenting with right finger and hand pain over
the last 3 days. He is a mechanic and works with his hands on a daily basis and denies
recent trauma or injury. On exam his right 2nd digit is painful and swollen. He has pain
to passive extension and redness of the digit. A picture of his finger is shown
below. Which of the following options is true regarding his diagnosis?

(Select 1)( 1pts extra credit)

All patients with flexor tenosynovitis require hospital admission and


prompt consultation with a hand surgeon to determine whether open
drainage or closed tendon sheath irrigation is indicated.

Early recognition and treatment are essential because elevated pressure


within the enclosure of the tendon sheath can occlude the already tenuous
circulation to the tendon, resulting in necrosis and proximal spread.

Disseminated gonorrhea should be considered in all sexually active


individuals, especially if there is no apparent traumatic etiology of the
infection.

The most common pathogens implicated in tenosynovitis due to trauma


are skin flora (eg, gram-positive cocci such as Staphylococcus aureus and
streptococci)

All of the above

None of the above

This patient has flexor tenosynovitis which is inflammation of a tendon and its sheath.
Most acute cases of flexor tenosynovitis (FT)—which involves disruption of
normal flexor tendonfunction in the hand—result from infection. However, FT also can
develop secondary to acute or chronic inflammation from a noninfectious cause, such as
diabetes, overuse, or arthritis.
Patients with infectious FT can present at any time following a penetrating injury, with
complaints of pain, redness, and fever. Physical examination reveals Kanavel signs of
flexor tendon sheath infection, which are as follows:
Finger held in slight flexion
Fusiform swelling
Tenderness along the flexor tendon sheath
Pain with passive extension of the digit
Clinical features of gonococcal tenosynovitis include the following:
Erythema, tenderness to palpation, and painful range of

motion (ROM) of the involved tendon(s)
 Fever - A common sign
 Dermatitis - Also a common sign; it occurs in
approximately two thirds of disseminated gonococcal
infections; it is characterized by hemorrhagic macules or
papules on the distal extremities or trunk
Inflammatory flexor tenosynovitis
Usually the result of an underlying disease process

Presentation is indolent but progressive if therapy is not

initiated
 Similar findings to those found in infectious FT eventually
present
 Swelling is the most common initial finding
 Hallmark is a difference in active, versus passive, flexion
 As the tissue expands and impingement occurs, pain and
restricted motion ensue
Delayed presentations can have the appearance of fulminant FT with all Kanavel signs or
may involve tendon rupture if the patient delays seeking treatment long enough. Read
more

Here is a good you tube video of an examination of a patient with this FTS.
http://www.youtube.com/watch?v=qf9SW0ChsCU

Some anatomy: http://www.youtube.com/watch?v=dsYIzvm9Deo&feature=related

Feedback on your answer


Collapse
5)
1/1
What is the most common site of origin of breast cancer?

(Select 1)( 1pts extra credit)


central area

upper-outer quadrant

upper-inner quadrant

lower-outer quadrant

The answer is B. The most common site of origin of breast cancer is the upper-outer
quadrant (38.5%), central area (29%), upper-inner quadrant (14.2%), lower-outer
quadrant (8.8%), and the lower-inner quadrant (5%). These percentages correlate with the
amount of tissue that is present in these quadrants. Metachronous bilateral carcinoma of
the breast has been observed in 5% to 8% of patients. Read more on Disorders of the
Breast
Feedback on your answer
Collapse
Unit 2
6)
0/1
You are seeing a 42-year-old female following in the trauma bay following a high-speed
motor vehicle collision. She was was a restrained passenger and her car flipped over after
she lost control. She had to be cut out of the car and is complaining of shortness of breath
and upper abdominal pain.
She is awake and alert and her GCS is 15. Her pulse is 120/min, respirations are 28/min,
and blood pressure is 80/40 mm Hg. Breath sounds are decreased at the left lung base. An
x-ray of the chest shows opacification of the left lower lung field. You decide to place an
emergent chest tube that yields a small amount of air followed by greenish fluid. Which
of the following is the most appropriate next step in management?
(Select 1)(1pts)

CT scan of the abdomen and pelvis

CT scan of the chest

Thoracoscopy

Laparotomy

The answer is D. This patient suffered a ruptured diaphragm and chest tube placement
appears to have ruptured the bowel as evidence from the greenish fluid that was returned.
She needs an emergent laparotomy to control bleeding and repair her diaphragm. Read
More
Feedback on your answer
Collapse
7)
0/1
You are seeing a 20-year-old male in hospital 12 hours after he was admitted for
observation following an MVC. Initially he was complaining of left sided chest wall pain
after his car was t-boned on the passenger side. His initial physical examination showed
bruises on the anterior chest wall and upper abdominal wall. X-rays revealed fractures
of his sixth and seventh ribs on the left (but no pneumothorax or pleural effusion). A
FAST Exam did not show any free intraperitoneal fluid. Currently, he complains of
worsening epigastric pain, left shoulder pain and mild nausea. His current vitals show his
blood pressure is now 95/60 and pulse rate 115 beats/min, and O2 saturation is 96% on
room air. Which of the following is the next best step in treatment?
(Select 1)(1pts)

Repeat ECG and cardiac biomarkers

Repeat PA and lateral chest x-ray

Abdominal CT scan with intravenous contrast

Transesophageal echocardiogram

This patient presents with blunt abdominal trauma with the delayed onset of hypertension
and signs and symptoms worrisome for likely splenic injury. Most common injuries are
to the spleen, liver, and less common injuries are to the hollow viscous organs in the
abdomen. Symptoms and signs suggesting splenic injury include left upper quadrant pain,
abdominal wall contusion, left lower chest wall tenderness, hypotension, and left
shoulder. Pain referred from splenic hemorrhage, hitting the phrenic nerve and diaphragm
(Kehr sign). The initial examination after blunt abdominal trauma can be unremarkable
and the symptoms can occur hours later, indicating ongoing splenic injury. The best
choice here would be an abdominal CT scan with intravenous contrast, only (no oral
contrast is needed because as little utility). This will define organ injury, assess for
presence of bleeding in all abdominal compartments, determine the need for surgery.
The spleen is the second most commonly injured abdominal organ in blunt trauma
patients. Historical studies have reported a 10% mortality with all splenic injuries;
however, isolated splenic injury mortality is less than 1%. The mechanisms of injury are
similar to those seen with liver injuries: motor vehicle collisions, autopedestrian
accidents, and falls. Similar to penetrating trauma to the liver, stab wounds to the spleen
typically result in direct linear tears, whereas gunshot wounds result in significant
cavitary injuries.
Until the 1970s, splenectomy was considered mandatory for all splenic injuries.
Recognition of the immune function of the spleen refocused efforts on splenic salvage in
the 1980s.38,39Following success in pediatric patients, NOM of splenic injuries was
adopted in the adult population and has become the prevailing strategy for blunt splenic
trauma.40

INITIAL EVALUATION AND INJURY GRADING

Addressing the patient's ABC's, examining the patient's abdomen, and performing
adjunctive imaging with FAST and CT are the initial steps of diagnosing a patient's
splenic injury. Hypotension with a positive FAST scan should prompt emergent
laparotomy. For patients with an identified blunt splenic injury on a CT scan, the injury
should be graded according to the AAST injury grading scale [see Table 1].3

Similar to liver injuries, the grade of splenic injury predicts failure rates and complication
rates of NOM. Other findings that should be searched for on a CT scan include contrast
extravasation (is the contrast blush contained within the spleen, or does it spill into the
peritoneum?), the amount of intra-abdominal hemorrhage (is it isolated to the splenic
fossa, or does blood extend into the pelvis?), and the presence of pseudoaneurysms.

NONOPERATIVE MANAGEMENT

Indications for Nonoperative Management

NOM of solid-organ injuries is pursued in hemodynamically stable patients who do not


have overt peritonitis or other indications for laparotomy.41-45 There is no age cutoff for
patients for the NOM of solid-organ injuries.46,47 High-grade injuries, a large amount of
hemoperitoneum, contrast extravasation, and pseudoaneurysms are not absolute
contraindications for NOM; however, these patients are at high risk for failure.48-51 The
identification of contrast extravasation as a risk factor for failure of NOM led to liberal
use of angioembolization. The true value of angioembolization in splenic salvage has not
been rigorously evaluated. Patients with intraparenchymal splenic blushes who are
otherwise asymptomatic may be considered for a period of observation rather than
empiric angioembolization52; it is thought that the contained hemorrhage within the
splenic capsule may result in tamponade of the bleeding

It is clear, however, that 20 to 30% of patients with splenic trauma deserve early
splenectomy and that failure of NOM often represents poor patient selection.53,54 In
adults, indications for prompt laparotomy include initiation of blood transfusion within
the first 12 hours considered to be secondary to the splenic injury or hemodynamic
instability. In the pediatric population, blood transfusions up to half of the patient's blood
volume are used prior to operative intervention. Following the first 12 postinjury hours,
indications for laparotomy are not as black and white. Determination of the patient's age,
comorbidities, current physiology, degree of anemia, and associated injuries will
determine the use of transfusion alone versus intervention with either embolization or
operation. Unlike hepatic injuries, which rebleed in 24 to 48 hours, delayed hemorrhage
or rupture of the spleen can occur up to weeks following injury. Algorithms for the
management of pediatric splenic injuries exist,55 and the patient's physiologic status is
the key determinant. Rapid mobilization in patients who are hemodynamically stable with
a stable hematocrit and no abdominal pain is generally successful. Overall, nonoperative
treatment obviates laparotomy in more than 90% of cases.

Follow-up Imaging

Out of concern over the risk of delayed hemorrhage or other complications, follow-up CT
scans have often been recommended; unfortunately, there is no consensus as to when or
even whether they should be obtained. Patients with grade I or II splenic injuries rarely
show progression of the lesion or other complications on routine follow-up CT scans; it is
reasonable to omit such scans if patients' hematocrits remain stable and they are
otherwise well. Patients with more extensive injuries often have a less predictable course,
and CT may be necessary to evaluate possible complications. Routine CT before
discharge, however, is unwarranted. Outpatient CT, however, in patients who participate
in vigorous or contact sports should be performed at 6 weeks to document complete
healing before resuming those activities. A more convenient and less expensive
alternative to follow-up CT is ultrasonographic monitoring of lesions.

OPERATIVE EXPOSURE AND HEMORRHAGE CONTROL

In penetrating abdominal injuries not suitable for NOM and in blunt abdominal injuries
when NOM is contraindicated or has failed, exploratory laparotomy is performed.

To ensure safe removal or repair, the spleen should be mobilized to the point where it can
be brought to the surface of the abdominal wall without tension. An incision is made in
the peritoneum and the endoabdominal fascia, beginning at the white line of Toldt along
the descending colon and continuing cephalad 1 to 2 cm lateral to the posterior peritoneal
reflection of the spleen; this plane of dissection is continued superiorly until the
esophagus is encountered [see

Figure 15a]. Posteriorly, blunt dissection is performed to mobilize the spleen and
pancreas as a composite away from Gerota fascia and up and out of the retroperitoneum;
this posterior plane may be extended to the aorta if necessary [seeFigure 15b].
Additionally, the attachments between the spleen and the splenic flexure of the colon may
be divided to avoid avulsion of the inferior splenic capsule. Care must be taken not to
pull on the spleen; otherwise, it will tear along the posterior peritoneal reflection, causing
significant hemorrhage. It is often helpful to rotate the operating table 20° to the patient's
right so that the weight of the abdominal viscera facilitates viscera retraction. Any
ongoing hemorrhage from the splenic injury may be temporarily controlled with digital
occlusion of the splenic hilar vessels. Once mobilization is complete, the spleen can be
repaired or removed without any need to struggle to achieve adequate exposure.
DEFINITIVE MANAGEMENT OF INJURIES

Splenic injuries are treated operatively by splenectomy, partial splenectomy, or splenic


repair (splenorrhaphy), based on the extent of the injury and the physiologic condition of
the patient. Splenectomy is indicated for hilar injuries, pulverized splenic parenchyma, or
any grade II or higher injury in a coagulopathic or multiply injured patient. We employ
autotransplantation of splenic implants [see

Figure 16] for partial immunocompetence in younger patients.56 Drains are not used.
Partial splenectomy can be employed in patients in whom only the superior or inferior
pole has been injured. Hemorrhage from the raw splenic edge is controlled with a
horizontal mattress suture, with gentle compression of the parenchyma [see

Figure 17]. Similar to hepatic injuries, splenorrhaphy techniques to achieve hemostasis


include topical agents (electrocautery, argon beam coagulation, thrombin-soaked gelatin
foam sponge, fibrin glue, BioGlue), enveloping the injured spleen in absorbable mesh,
and pledgeted suture repair.

POSTOPERATIVE CARE AND COMPLICATIONS

Enthusiasm for splenic salvage was driven by the rare but often fatal complication of
overwhelming postsplenectomy sepsis (OPSS). OPSS is caused by encapsulated
bacteria,Streptococcus pneumoniae, Haemophilus influenzae,
and Neisseria meningitidis, which are resistant to antimicrobial treatment. In
patients undergoing splenectomy, prevention against these bacteria is provided via
vaccines administered optimally at 14 days but definitely prior to hospital
discharge.57 Vaccines to be administered include Pneumovax (Merck & Co., Inc.,
Whitehouse Station, NJ), Menactra (Sanofi Pasteur, Swiftwater, PA), and Fluvirin
(Novartis, East Hanover, NJ). Revaccination remains open to debate, but some argue for
revaccination every 6 years.

An immediate postsplenectomy increase in platelets and white blood cells (WBCs) is


normal; however, beyond postoperative day 5, a WBC count above 15,000/μL and a
platelet to WBC ratio less than 20 are highly associated with sepsis and should prompt a
thorough search for underlying infection.58,59 A common infectious complication
following splenectomy is a subphrenic abscess, which should be managed with
percutaneous drainage. Following splenectomy or splenorrhaphy, postoperative
hemorrhage may be attributable to loosening of a tie around the splenic vessels, a missed
short gastric artery, or recurrent bleeding from the spleen if splenic repair was used.
Additional sources of morbidity include a concurrent but unrecognized iatrogenic injury
to the pancreatic tail during rapid splenectomy, resulting in pancreatic ascites or fistula.

2014. Scientific American Surgery. Hamilton, Ontario & Philadelphia, PA. Decker
Intellectual Properties Inc. ISSN 2368-2744. STAT!Ref Online Electronic Medical
Library. http://online.statref.com/Document.aspx?fxId=61&docId=2297. 10/17/2014
4:45:39 AM CDT (UTC -05:00).

Feedback on your answer


Collapse
8)
1/1
In cases of blunt trauma to the diaphragm, the injury is on the ____ side __% of the time?
(Select 1)(1pts)

Right; 90%

Left; 75%

Left; 25%

Right; 10%

The answer is B, left;75%. In cases of blunt trauma to the diaphragm, the injury is on the
left side 75% of the time, presumably because the liver diffuses some of the energy on
the right side. Blunt diaphragmatic injuries result in a linear tear in the central tendon,
whereas penetrating injuries are variable in size and location depending on the weapon. It
is important to identify the trajectory of penetrating injuries to determine the likelihood of
diaphragm injuries. With blunt and occasionally with penetrating injuries, the diagnosis is
suggested by an abnormality of the diaphragmatic shadow on a chest radiograph [see
Figure 5]. In patients without clear imaging results in the trauma bay, a CT scan may
identify a diaphragmatic injury.
Regardless of the etiology, acute injuries are repaired through an abdominal incision.
Thoracoscopy or laparoscopy may be used if concomitant injuries requiring laparotomy
have been ruled out. Following delineation of the injury, the chest should be evacuated of
all blood and particulate matter, and tube thoracostomy should be placed if not previously
done. Using Allis clamps to approximate the diaphragmatic edges, the defect can be
closed with a running permanent suture [see Figure 6]. Occasionally, large avulsions or
shotgun wounds with extensive tissue loss will require mesh to bridge the defect.
Alternatively, transposition of the diaphragm cephalad one to two intercostal spaces may
allow repair without undue tension. Read more
Feedback on your answer
Collapse
9)
0/1
Nonoperative Management of solid-organ injuries can be pursued in hemodynamically
stable patients who do not have overt peritonitis or other indications for laparotomy.
According to contemporary data, what percentage of patients with splenic injuries are
candidates for nonoperative management?
(Select 1)(1pts)
<10%

25%

45%

60%

The answer is D, 60%. NOM of solid-organ injuries can be pursued in hemodynamically


stable patients who do not have overt peritonitis or other indications for laparotomy.5-9
Contemporary data suggest that this will be approximately 60% of the overall splenic
injury population.6 Failure of NOM in this selected population has been reported at only
10 to 20%. It is important to understand that this rate represents an average and may vary
widely across differing grades of injury and across varying patient populations. For
example, the NOM failure rate reported in the Eastern Association for the Surgery of
Trauma multiinstitutional study from 2000 was 11%.10 However, those with highgrade
injuries (III, IV, V) had a NOM failure rate of 20%, 33%, and 75%, respectively. In
addition to increasing grade, contrast extravasation, older age, and high injury severity
score (> 25) have been linked to higher risk of failure of NOM.11-16 NOM may be time
and resource intensive and is not safe without the supporting resources such as a setting
and staff for adequate observation, rapid operating room mobilization if needed,
immediately available anesthesia, and an adequate blood bank. If such resources are
unavailable, patients should be transferred to a higher level of care or early operation
should be considered.
The role of angioembolization of the injured spleen continues to evolve. The
identification of contrast extravasation within the splenic parenchyma (pseudoaneurysm)
as a risk factor for failure of NOM led to liberal use of angioembolization and improved
rates of splenic salvage in this population [see Figure 1 and Figure 2].17 Contrast
extravasation into the peritoneal cavity outside the splenic parenchyma is also commonly
cited as an indication for angiography. However, the use of angioembolization in such
patients has met with less success.18 In our experience, such patients rarely remain stable
for long and often require laparotomy if angiography cannot be performed rapidly
In addition to use in contrast blush, angiography is increasingly used in all higher-grade
spleen injuries undergoing NOM. Multiple institutions have addressed this concept, and a
meta-analysis published in 2011 and a subsequent prospective study in 2014 highlighted
the higher splenic salvage rate during NOM when angioembolization is employed in all
grade IV and V injuries.19,20 This is echoed in a Level 2 recommendation in the Eastern
Association for the Surgery of Trauma practice management guidelines from 2012.10
Subcapsular hematomas also may be at higher risk for delayed rupture, and special
consideration should be given to patients with this injury pattern. Also currently
controversial is the superiority of proximal embolization versus distal embolization.18,21
With such emphasis placed on reporting techniques for increasing NOM success and
splenic salvage, it is important to keep in mind the role of early, rapid operation.
Approximately 20 to 30% of patients with splenic trauma deserve early splenectomy, and
failure of NOM often represents poor patient selection.22,23 In adults, indications for
prompt laparotomy include hemodynamic instability and initiation of blood transfusion
within the first 12 hours considered to be secondary to the splenic injury. In the pediatric
population, blood transfusions up to half of the patient's blood volume are used prior to
operative intervention. Following the first 12 postinjury hours, indications for laparotomy
are not as black and white. Determination of the patient's age, comorbidities, current
physiology, degree of anemia, and associated injuries will determine the use of
transfusion alone versus intervention with either embolization or operation. Delayed
hemorrhage or rupture of the spleen can occur up to weeks following injury, but the
majority who fail do so while still hospitalized.24 Overall, nonoperative treatment
obviates laparotomy in more than 90% of appropriately selected cases. Read more
Feedback on your answer
Collapse
Unit 3
10)
1/1
What is the most common abdominal organ injured in the setting of blunt abdominal
trauma?
(Select 1)(1pts)

Spleen

Liver

Kidneys

Small Intestines

The liver is the most commonly injured solid organ in blunt trauma, comprising 5% of all
trauma admissions, and because of its size is frequently involved in penetrating trauma.
Following blunt trauma, the most commonly injured structures are the parenchyma and
hepatic veins. Blunt forces dissipate along segments of the liver and along the fibrous
coverings of the portal triad structures; the hepatic veins, however, are not so insulated.
Given its size and location within the abdomen, the liver is also commonly involved in
penetrating trauma. Stab wounds typically result in direct linear tears, whereas gunshot
wounds or shotgun wounds result in significant cavitary injuries attributable to blast
effect and the "tumbling" of the missile within the liver parenchyma. Thus, arterial injury
is more common with penetrating trauma.
Over the past 20 years, nonoperative management (NOM) of liver injuries has evolved to
become the prevailing therapeutic strategy for blunt hepatic trauma. Several concurrent
changes resulted in this paradigm change. First was the realization that diagnostic
peritoneal lavage (DPL) was sensitive but not specific for identifying intraperitoneal
hemorrhage that necessitated operative management. Surgeons recognized that many
laparotomies undertaken for a positive DPL were associated with liver injuries that did
not require intervention for bleeding.1Second, trauma surgeons noted that nonbleeding
hepatic venous injuries, if manipulated at laparotomy, often resulted in more hemorrhage
and sometimes even death.2 Furthermore, it became conspicuous that with hemostasis
achieved in the operating room, recurrent postoperative bleeding was rare. Therefore,
surgeons queried whether hepatic venous injuries, which are low-pressure system
injuries, could heal without intervention. Finally, computed tomography (CT) provided a
reliable method for diagnosing and grading liver injuries.

The spleen is the second most commonly injured abdominal organ in blunt trauma
patients. Historical studies have reported a 10% mortality with all splenic injuries;
however, isolated splenic injury mortality is less than 1%. The mechanisms of injury are
similar to those seen with liver injuries: motor vehicle collisions, autopedestrian
accidents, and falls. Similar to penetrating trauma to the liver, stab wounds to the spleen
typically result in direct linear tears, whereas gunshot wounds result in significant
cavitary injuries.

Until the 1970s, splenectomy was considered mandatory for all splenic injuries.
Recognition of the immune function of the spleen refocused efforts on splenic salvage in
the 1980s.38,39Following success in pediatric patients, NOM of splenic injuries was
adopted in the adult population and has become the prevailing strategy for blunt splenic
trauma.

Duodenal and pancreatic injury continues to challenge the trauma surgeon. The relatively
rare occurrence of these injuries, the difficulty in making a timely diagnosis, and high
morbidity and mortality rates justify the anxiety these unforgiving injuries invoke.
Mortality rates for pancreatic trauma range from 9 to 34%, with a mean rate of 19%.
Duodenal injuries are similarly lethal, with mortality rates ranging from 6 to 25%.
Complications following duodenal or pancreatic injuries are alarmingly frequent,
occurring in 30 to 60% of patients.1-3 Recognized early, the operative treatment of most
duodenal and pancreatic injuries is straightforward, with low morbidity and mortality.

2014. Scientific American Surgery. Hamilton, Ontario & Philadelphia, PA. Decker
Intellectual Properties Inc. ISSN 2368-2744. STAT!Ref Online Electronic Medical
Library. http://online.statref.com/Document.aspx?fxId=61&docId=2329. 10/27/2014
1:39:15 PM CDT (UTC -05:00).

Feedback on your answer


Collapse
11)
0/1
The liver is the most commonly injured solid organ in blunt trauma, comprising 5% of all
trauma admissions. With that being said, over 80% of patients with liver injuries may be
managed nonoperatively. Which of the following is not a predictor of nonoperative
management?
(1pts)
Female sex

Increasing age

Decreasing Glasgow Coma Scale score

Hypotension

Increasing Injury Severity Score

The answer is A, female sex. Over 80% of patients with liver injuries may be managed
nonoperatively. One of the early studies to test the application of NOM in 1995 supported
its broad application, with an overall success rate greater than 85% in hemodynamically
stable patients, despite substantial hemoperitoneum documented by CT.11 Of the 8% of
patients who failed NOM, half required operation as a result of associated injuries (i.e.,
enteric or pancreatic injuries), whereas half underwent laparotomy for hepatic-related
hemorrhage. Patients who require intervention for hemorrhage typically fail NOM in the
first 24 to 48 hours.2,11,17 Patients who fail NOM due to associated enteric or pancreatic
injury have a more variable time frame to presentation17; half manifested symptoms
within 48 hours, with the remainder becoming symptomatic up to 3 weeks later. Perhaps
not surprisingly, those patients who failed NOM had failure rates associated with
increasing grades of hepatic injury, with grade V injuries having a greater than 20%
failure rate. Subsequent studies have reported failure rates of 14% in grade IV injuries
and 23% in grade V injuries.12 The most recent analysis of the National Trauma Data
Bank of severe blunt liver injuries (grade IV and V) identified that initial NOM occurred
in 73%, with a failure rate of 7%.16 Interestingly, failure of NOM was associated with
higher mortality. Predictors of failure of NOM included increasing age, male sex,
increasing Injury Severity Score, decreasing Glasgow Coma Scale score, and
hypotension. A similar study of high-grade liver injuries identified a similar pattern with
NOM initiated in 66% patients with a failure rate of 9%.18 The amount of
hemoperitoneum evident on a CT scan appears to correlate with successful management;
patients with a large amount of hemoperitoneum (i.e., blood extending into the pelvis) are
more likely to fail NOM. However, predicting which patients will ultimately require
laparotomy has yet to be accomplished. Read more
Feedback on your answer
Collapse
12)
0/1
You are seeing a 34-year old female in the trauma following an MVC. He is complaining
intense diffuse abdominal pain after his car was hit from behind causing him to spin out
and crash into a wall. Initially he had a GCS of 15. His vitals showed a BP of 125/76, HR
95, RR 18, O2 of 98% on room air. On physical exam his lungs are clear and his
abdomen shows diffuse tenderness to light palpation and ecchymosis over his
epigastrium. His initial fast exam is negative but technically limited and you order a CT
scan with IV contrast of his abdomen and pelvis as you suspect a possible liver
injury. What description from the options below accurately describes a grade IV liver
laceration on CT imaging?
(Select 1)(1pts)

Parenchymal distruption involving 25 - 75% hepatic lobes

Parenchymal distruption involving >75% helpatic lobe

Intraparenchymal laceration < 10cm diameter

Sub capsular hematoma, > 50% surface area, or ruptured with active
bleeding

The answer is A. Parenchymal distruption involving 25 - 75% hepatic lobes. Read


more
The AAST Liver injury grading system is as follows
 grade I :
 haematoma: sub capsular, < 10% surface area
 laceration: capsular tear, < 1cm depth
 grade II :
 haematoma: sub capsular, 10 - 50% surface area
 haematoma: intraparenchymal < 10cm diameter
 laceration: capsular tear, 1 - 3cm depth, < 10cm length
 grade III :
 haematoma: sub capsular, > 50% surface area, or ruptured with
active bleeding
 haematoma: intraparenchymal > 10 cm diameter
 laceration: capsular tear, > 3 cm depth
 grade IV :
 haematoma: ruptured intraparenchymal with active bleeding
 laceration: parenchymal distruption involving 25 - 75% hepatic
lobes or
 1 - 3 Couinaud segments (within one lobe)
 grade V :
 laceration: parenchymal distruption involving >75% helpatic
lobe or
 > 3 Couinaud segments (within one lobe)
 vascular: juxtahepatic venous injuries (IVC, major hepatic vein)
 grade VI : vascular: hepatic avulsion

Feedback on your answer


Collapse
13)
0/1
Roughly what percentage of patients with blunt liver injury initially managed non-
operatively will go on to need surgical intervention?
(1pts)

<10%

25%

50%

75%

The answer is A, During the last century, the management of blunt force trauma to the
liver has changed from observation and expectant management in the early part of the
1900s to mainly operative intervention, to the current practice of selective operative and
nonoperative management. A 2008 study by Tinkoff et al.4 showed that 86.3% of hepatic
injuries are now managed without operative intervention. The current reported success
rate of nonoperative management of hepatic trauma ranges from 82% to 100%. Most
blunt liver trauma (80% in adults, 97% in children) patients are currently treated
conservatively. The success of non-operative management depends upon proper selection
of the patient. The patients, who are managed non-operatively, usually have grade I and
II liver injuries, hemoperitoneum less than 900 ml and blood transfusion of less than 3
units. The contraindications to non-operative management include refractory
hypotension, signinficant fall in haematocrit, the extravasations of intravenous contrast
agent, expanding haematoma and grade IV and V liver injury on CECT abdomen. The
patients of grade III liver injuries need very close observation as they may require
surgical intervention during first 24 hours. The failure rate of non-operative management
is not more than 5% inmost studies. It seems that patients with grade VI injuries rarely
reaches to the hospital alive and are not salvageable. Therefore, such injuries are usually
documented on autopsy. Mortality from blunt hepatic trauma is about 5% and is related
to uncontrolled hemorrhage.
Interventional radiology may be needed to perform an angiogram and embolization for
bleeding or to percutaneously drain an abscess or b iloma. An endoscopic retrograde
cholangiopancreatogram (ERCP) and stent placement may be required for biliary leak.
Even when such complications of the liver injury develop, only 15% require operative
intervention. Hepatic artery angiography with embolization is an important tool for the
stable patient with contrast extravasation who is being managed nonoperatively. It can
also be invaluable for the postoperative patient who has been stabilized by perihepatic
packing or who has rebled after an initial period of stability. Angioembolization has a
greater than 90% success rate in the control of bleeding with a low risk of rebleeding and
a reduction in required volume of transfusion. Read More
Feedback on your answer
Collapse
Unit 4
14)
0/1
You are seeing a 15-year-old African-American female in your office because she and
her mom are concerned about a non tender breast lump that she just noticed the other day.
On exam you note a rubbery, well-defined, nontender breast mass approximately 2 cm in
diameter. The patient denies any history of breast tenderness, nipple discharge, or skin
changes. What is most likely diagnosis?

(Select 1)(1pts)

Fibrocystic breast disease

Fibroadenoma

Benign breast cyst

Cystosarcoma phyllodes

The answer is B, Fibroadenoma. Most breast masses in adolescent girls are benign.
Fibroadenoma is the most common, accounting for approximately two-thirds of all
adolescent breast masses. It is characterized by a slow growing, nontender, rubbery, well-
defined mass, most commonly located in the upper, outer quadrant. Size varies, and is
most commonly in the range of 2–3 cm. Fibrocystic disease is found in older adolescents
and is characterized by bilateral nodularity and cyclic tenderness. Benign breast cysts are
characterized by a spongy, tender mass with symptoms exacerbated by menses. Cysts are
frequently multiple, and spontaneous regression occurs in 50% of patients. Cystosarcoma
phyllodes is a rare tumor with malignant potential, although most are benign. It presents
as a firm, rubbery mass that may enlarge rapidly. Skin necrosis is usually associated with
the tumor. Intraductal papillomas are usually benign but do have malignant potential.
They are commonly subareolar and are associated with nipple discharge. These tumors
are rare in the adolescent population. Ref: Hay WH (ed): Current Pediatric Diagnosis and
Treatment, ed 16. McGraw-Hill, 2003, pp 122-123.

Feedback on your answer


Collapse
15)
1/1
A 25 -ear obese black female presents to you with a painful breast lump along with
tenderness in both her breasts. She notes recurrent pain and multiple lumps on both
breasts that seem to “come and go” at different times during her menses. Now she has a
firm, round, mass that has not gone away for the last 5 weeks. What is the most likely
diagnosis?

(Select 1)(1pts)

Cystosarcoma Phyllodes

Intraductal papilloma

Fibrocystic disease

Fibroadenoma

The answer is C. This patient has fibrocystic disease which is a nonproliferative epithelial
lesions that are generally not associated with an increased risk of breast cancer [1]. It
should be noted that terms such as fibrocystic changes, fibrocystic disease, chronic cystic
mastitis, and mammary dysplasia refer to nonproliferative lesions and are not useful
clinically, as they encompass a heterogeneous group of diagnoses [5,11]. The most
common nonproliferative breast lesions are breast cysts. Other nonproliferative lesions
include papillary apocrine change, epithelial-related calcifications, and mild hyperplasia
of the usual type [5]. Apocrine metaplasia (also referred to as a "benign epithelial
alteration") is also a nonproliferative change that is secondary to some form of irritation,
typically associated with a breast cyst. Read More
Feedback on your answer
Collapse
16)
0/1
You are seeing a 25-year old white female who is presenting to your clinic with bloody
nipple discharge over the last 3 days. She denies any history of a breast lump, she is not
breast feeding and denies family history of breast cancer. Her exam shows no palpable
masses but there is scant bloody discharge coming from her left nipple. You order a
mammogram that does not show any suspicious lesions. What is the most likely
diagnosis?
(Select 1)(1pts)

Fibroadenoma

Intraductal papilloma
Cystosarcoma Phyllodes

Mammary dysplasia

The answer is B, Intraductal papilloma. The old concern over cancer is the issue, and the
way to detect cancer that is not palpable is with a mammogram. That should be the first
choice. If negative, one may still wish to find an resect the intraductal papilloma to
provide symptomatic relief. Intraductal papillomas consist of a monotonous array of
papillary cells that grow from the wall of a cyst into its lumen. Although they are not
concerning in and of themselves, they can harbor areas of atypia or ductal carcinoma in
situ (DCIS). Papillomas can occur as solitary or multiple lesions. The standard approach
to a papilloma diagnosed by core needle biopsy (CNB) is to perform a surgical excision,
particularly if atypical cells are identified [14,16-21]. In a meta-analysis of 34 studies
that included 2236 non-malignant breast papillary lesions, 346 (15.7 percent) were
upgraded to malignancy following a surgical excision [21]. Because of a risk of
malignancy, these require surgical excision.Read More
Feedback on your answer
Collapse
17)
0/1
You are seeing a 24-year old female who is 4-weeks post postpartum in your clinic. She
is complaining of bilateral nipple pain over the last two days along with a nipple rash.
She notes pain both with breastfeeding and even when she is not feeding. The pain is so
bad that she didn't breastfeed her baby this morning because of the pain. Physical exam
shows erythema to both breasts along with cracking around both areolas. There are no
signs of an abscess and no induration present. What is the most likely cause of this
patient's symptoms?
(Select 1)(1pts)

Engorgement

Mastitis

Deep space abscess

Candida infection

None of the above

The answer is D, Candida infection. In breastfeeding women, bilateral nipple pain with
and between feedings after initial soreness has resolved is usually due to Candida. Pain
from engorgement typically resolves after feeding. Mastitis is usually unilateral and is
associated with systemic symptoms and wedge-shaped erythema of the breast tissue.
Improper latch-on is painful only during feedings. Eczema isolated to the nipple, while a
reasonable part of the differential, would be much more unusual.

Ref: Jatoi I, Kaufman M (eds): Management of Breast Diseases. Springer, 2010,


pp 95-96.
Feedback on your answer
Collapse
Unit 5
18)
0/1
Which of the following is considered a benign lesion that usually does not require cancer
treatment?
(Select 1)(1pts)

Lobular carcinoma in situ

Ductal carcinoma in situ

Medullary carcinoma

Mucinous carcinoma

The answer is A, Cancer that is confined to the lumen of the duct or


lobule of the breast and has not penetrated the basement membrane
is termed in situ cancer. This generally refers to ductal carcinoma in
situ (DCIS) but also encompasses a benign entity called lobular
carcinoma in situ (LCIS). Small uniform cells confined to the lobule of
the breast characterize LCIS. It is generally a clinically and
mammographically occult lesion that is identified only incidentally
when a biopsy is performed for calcifications or a mass that proves to
be some other benign lesion. LCIS is actually not cancer but rather is a
benign lesion and does not require cancer treatment. The primary
issue with LCIS is that it conveys an increased lifelong risk of
subsequent invasive cancer quantified at 0.5 to 0.75% per year. In
addition, when LCIS is identified on a core-needle biopsy, there is a 10
to 20% chance of DCIS or invasive cancer in the surrounding tissue;
therefore, surgical excision is warranted. Long-term follow-up shows
that the large majority of women with LCIS never develop invasive
breast cancer. Therefore, ablative surgical therapy and radiation for
LCIS are not necessary. Previously, LCIS was considered in and of
itself an indication to consider bilateral mastectomy. However,
mastectomy is generally not indicated in women with LCIS and should
be performed only in the context of risk reduction for those at very
high risk related to factors such as inherited susceptibility. Because
women with a diagnosis of LCIS are at increased risk for subsequent
invasive cancer, they should be counseled regarding that risk and may
benefit from consultation with genetics professionals if they have a
family history of breast or ovarian cancer. Women with a biopsy
showing LCIS may also consider risk-reducing chemoprevention with
one of the selective estrogen receptor modulators (SERM's), tamoxifen
or raloxifene. These reduce the risk of subsequent invasive cancer by
about 50%, with an acceptable toxicity profile.20,21 Raloxifene is the
preferred agent in postmenopausal women. Read more on Breast
Cancer
Feedback on your answer
Collapse
19)
1/1
True or False: When approaching a patient with an abnormal mammogram for a
suspicious breast mass the next best step is a surgical excision is preferred over
percutaneous needle biopsy.
(1pts)

True

False Correct

False: Breast lesion suspicious for malignancy requires tissue biopsy.


Percutaneous needle biopsy is preferred over surgical excision in all
circumstances. Surgical excision as a diagnostic procedure is not a
justifiable alternative simply because of "patient choice" and should be
performed only when needle biopsy cannot be performed for specific
technical reasons, when a needle biopsy is either nondiagnostic, the
result is not concordant with the imaging findings (i.e., the needle
biopsy is benign, but the lesion is of high suspicion), or in highly select
other cases. Technical reasons that may preclude needle biopsy
include anatomic location of the lesion on mammography directly
opposed to the chest wall or in the far periphery of the breast so that
it cannot be visualized on stereotactic imaging devices. Breast Cancer
Feedback on your answer
Collapse
20)
1/1
True or False: A clinically suspicious breast mass should be biopsied regardless of
imaging findings such as a negative mammogram.
(1pts)

True Correct

False
True: Algorithms for clinical and imaging evaluation of palpable masses are stratified by
the age of the woman. Even in the setting of palpable masses, image guidance may
improve diagnostic accuracy. A clinically suspicious mass should be biopsied regardless
of imaging findings, as 10 to 15 percent of such lesions can be mammographically
occult. Read more

Feedback on your answer


Collapse
21)
0/1
You are seeing a 43-year old Hispanic female who is presenting with an enlarging mass
in her right breast. It has been present for several years and growing to it’s present size.
On examination of her right breast you note a large 10 cm mass that is mobile, firm
and rubbery. There are no palpable axillary nodes. What is the most likely diagnosis?

(Select 1)(1pts)

Fibroadenoma

Fibrocystic disease

Intraductal papilloma.

Cystosarcoma Phyllodes

The answer is D. This is a Phyllodes tumors, which are an uncommon fibroepithelial


breast tumors that are capable of a diverse range of biologic behavior. In their least
aggressive form, they behave similarly to benign fibroadenomas, although with a
propensity to recur locally following excision without wide margins. At the other end of
the spectrum are tumors that metastasize distantly, sometimes degenerating histologically
into sarcomatous lesions that lack an epithelial component. Phyllodes tumors account for
fewer than 0.5 percent of all breast malignancies. The vast majority occur in women, in
whom the median age at presentation is 42 to 45 (range 10 to 82 years).and there was a
higher incidence in Latina whites, as compared to non-Latina whites, Asians, and African
American women. Read More
Feedback on your answer
Collapse
Unit 6
22)
0/1
Which of the following statements regarding soft tissue infections is not true?
(Select 1)(1pts)
As many as 20% of necrotizing soft tissue infections are primary
(idiopathic) and occur in previously healthy patients who have no
predisposing factors and no known portal of entry for bacterial inoculation

Crepitus is noted in the majority of patients with necrotizing soft tissue


infections

Physical findings characteristic of a necrotizing infection include


tenderness beyond the area of erythema, crepitus and cutaneous
anesthesia

Fewer than 40% of patients with Patients with Necrotizing Soft Tissue
Infections exhibit the classic symptoms and signs described

The answer is B, crepitus is noted in only 30% of patients with necrotizing soft tissue
infections. Patients with necrotizing soft tissue infections often complain of severe pain
that is out of proportion to their physical findings. Compared with patients who have
nonnecrotizing infections, they are more likely to have fever, bullae, or blebs [see Figure
1]; signs of systemic toxicity; hyponatremia; and leukocytosis with a shift in immature
forms. Physical findings characteristic of a necrotizing infection include tenderness
beyond the area of erythema, crepitus, cutaneous anesthesia, and cellulitis that is
refractory to antibiotic therapy.6 Tenderness beyond the borders of the erythematous area
is an especially important clinical clue that develops as the infection in the deeper
cutaneous layers undermines the skin. Early in the course of a necrotizing soft tissue
infection, skin changes may be minimal despite extensive necrosis of the deeper
cutaneous layers. Bullae, blebs, cutaneous anesthesia, and skin necrosis occur as a result
of thrombosis of the nutrient vessels and destruction of the cutaneous nerves of the skin,
which typically occur late in the course of infection.
Clinicians should be mindful of certain diagnostic barriers that may delay recognition and
treatment of necrotizing soft tissue infections.7 In particular, these infections have a
variable clinical presentation. Although most patients present with an acute, rapidly
progressive illness and signs of systemic toxicity, a subset of patients may present with a
more indolent, slowly progressive infection. Patients with postoperative necrotizing
infections often have a more indolent course. Moreover, in the early stages, underlying
necrosis may be masked by normal-appearing overlying skin. As many as 20% of
necrotizing soft tissue infections are primary (idiopathic) and occur in previously healthy
patients who have no predisposing factors and no known portal of entry for bacterial
inoculation. Finally, crepitus is noted in only 30% of patients with necrotizing soft tissue
infections. Overall, fewer than 40% of patients exhibit the classic symptoms and signs
described.7,8Accordingly, it is imperative to maintain a high index of suspicion for this
disease in the appropriate setting.Read More on Soft Tissue Infection
Feedback on your answer
Collapse
23)
0/1
Which of the following choices best differentiates cellulitis from erysipelas?
(Select 1)(1pts)

Presence of fever

Presence of leukocytosis

Depth of inflammation

Presence of purulence

All of the above

None of the above

The answer is C, depth of inflammation. Cellulitis and erysipelas are diffuse spreading
skin infections that not associated with underlying suppurative foci. Clinically, there is
often some degree of overlap between the two different entities. Erysipelas is
differentiated from cellulitis by the depth of inflammation; erysipelas affects the upper
dermis, including the superficial lymphatics, whereas cellulitis affects the deeper dermis
and subcutaneous fat (Mayo Clin Proc, Vol. 89;1436). Read More onCellulitis and
erysipelas from Up to Date
Feedback on your answer
Collapse
24)
1/1
A 23-year old male patient presents with a human bite to his right hand after an
altercation just prior to arrival. He has no past medical history and is up to date with
tetanus. Exam shows there are two small puncture wounds overlying the dorsal aspect of
his right hand over his 1stmetacarpal and does not seem to involve any
tendons or ligaments. The wound does not appear amenable to sutures for
closure and you decide to copiously irrigate it with saline and discharge him home. What
antibiotic should he receive for wound care prophylaxis?
(1pts)

Clindamycin

Flagyl

Oxacillin
Augmentin

The answer is D. Augmentin. The most common bacterial etiology in human bites is strep
viridans. You are however covering for Eikenella corrodens which is a gram negative rod
which is susceptible to pcn but resistant to flagyl, clindamycin, first generation
cephalosporins and erythromycin. It is susceptible to flouroquinolones, bactrim and
augmentin. The most important treatment is good irrigation and initial wound
cleaning. Read more
Feedback on your answer
Collapse
25)
0/1
What is the most common causative pathogen of Lemierre disease, an infection of the
parapharyngeal space that leads to septic thrombophlebitis of the internal jugular vein
with bacteremia and metastatic pulmonary nodules.
(Select 1)(1pts)

C. diphtheriae

Fusobacterium necrophorum

Infectious mononucleosis

Streptococcus pyogenes

The answer is B, Fusobacterium necrophorum is the most common causative pathogen of


Lemierre disease, an infection of the parapharyngeal space that leads to septic
thrombophlebitis of the internal jugular vein with bacteremia and metastatic pulmonary
nodules (12). Lemierre disease may be complicated by septicemia; suppurative
intracranial complications; and erosion of the carotid artery, which may be life-
threatening. Lemierre disease is often suspected based on clinical grounds in a toxic-
appearing patient; however, several recent European reports also suggest a possible
causative role of F. necrophorum in uncomplicated pharyngitis (13–15). Read More
on Pharyngitis
Feedback on your answer
Collapse

Unit 1
1)
1/1

What percentage of patients will have thoracic injuries on CT with no physical


examination findings but with a significant mechanism of injury alone (ie, motor vehicle
crash at > 35 mph, fall from > 15 ft, automobile hitting a pedestrian with pedestrian being
thrown > 10 ft, and assault with depressed level of consciousness)?

(Select 1)( 1pts extra credit)

<5%

20%

35%

50%

The answer is B, 20%. The decision to image is challenging in the wellappearing patient
with a concerning mechanism or intermediate examination. Approximately 20% of
patients with no physical evidence of chest trauma but a concerning mechanism will have
injuries on thoracic CT. Alternatively, patients with a NEXUS chest score of 0 (see Table
2, page 5) have a much lower incidence of significant findings on CT
Some 10% to 23% of patients with minimal findings on examination may still have
significant thoracic injuries.54,59 However, it is unclear what percentage of these injuries
is clinically important. Point tenderness and ecchymosis on the chest wall should raise the
concern for intrathoracic injury; however, these findings are nonspecific.55 Injuries to the
lower ribs may also indicate the presence of intra-abdominal injuries. In a 2005
prospective observational study, 3% of patients with “isolated” subjective pain or point
tenderness to the lower left ribs as the only indication for computed tomography (CT) had
splenic injuries. If patients had other indications for CT (hypotension, abdominal or flank
tenderness, pelvic or femur fractures, or gross hematuria) the rate of splenic injury was
9.4%.Read more

Feedback on your answer


Collapse
2)
1/1
Which type of colon polyp is most likely to become malignant?

(Select 1)( 1pts extra credit)

Hamartomatous polyp
Tubular adenoma

Villous adenoma

Tubulovillous adenoma

The answer is C, Villous adenoma. Adenomatous polyps are the most common of the
classically neoplastic polyps. About two-thirds of all colonic polyps are adenomas.
Adenomas are by definition dysplastic and thus have malignant potential. Most colorectal
cancers arise from adenomas, but only a small minority of adenomas progress to cancer
(5 percent or less). Studies reporting the average age at presentation of patients with
adenomatous polyps versus colorectal cancer suggest the time for development of
adenomas to cancer is about 7 to 10 years. Hamartomatous (or juvenile) polyps and
hyperplastic polyps are benign lesions and are not considered to be premalignant.
Adenomas, on the other hand, have the potential to become malignant. Sessile adenomas
and lesions >1.0 cm have a higher risk for becoming malignant. Of the three types of
adenomas (tubular, tubulovillous, and villous), villous adenomas are the most likely to
develop into an adenocarcinoma.

An advanced adenoma is any adenoma with high-grade dysplasia, an
adenoma that is >10 mm in size, or an adenoma with a villous
component. Adenomas without villous components are also referred to as
tubular adenomas.
 A synchronous adenoma is an adenoma that is diagnosed at the same time
as an index colorectal neoplasm (a pathologically more advanced lesion).
Thirty to fifty percent of colons with one adenoma will contain at least
one other synchronous adenoma .
 A metachronous adenoma is an adenoma that is diagnosed at least six
months after the diagnosis of a previous adenoma.
Ref: Fauci AS, Braunwald E, Kasper DL, et al (eds):Harrison’s Principles of
Internal Medicine, ed 17. McGraw-Hill, 2008, p 574. Read more on Approach to
the patient with colonic polyps.

Feedback on your answer


Collapse
3)
1/1
You are seeing a 42-year old male who is presenting with flank pain following
an MVC. On physical exam he has a large degree of flank ecchymosis along with gross
hematuria. You are suspecting a kidney injury and he appears stable for CT at this time.
What renal injuries in the setting of trauma require surgical management?

(Select 1)( 1pts extra credit)

Vascular Injury
Expanding Hematoma

Shattered Kidney

All of the above

None of the above

The answer is D. All of the above. Generally about 80-90% renal injuries treated
conservatively with remarkable resolution! Injuries requiring surgery: vascular injury,
shattered kidney and an expanding hematoma.
The goal in the management of renal trauma is to safely preserve the maximal number of
renal units, avoiding unnecessary explorations, repairs, and nephrectomies. Increasing
numbers of renal injuries, including grade IV and V injuries, are being been managed
nonoperatively. The accuracy and rapidity of helical CT, combined with the
improvements achieved in resuscitation methods, have reduced the number of renal
explorations performed.8 Currently, 36% of all penetrating renal injuries and fewer than
5% of blunt injuries necessitate operative management.9 All grade I and II renal injuries,
regardless of the mechanism of injury, can be managed with observation alone because
the risk of delayed bleeding is extremely low. Most grade III and IV injuries, including
those with devitalized parenchymal fragments and urinary extravasation, can be managed
nonoperatively with close monitoring, serial hematocrit measurement, and repeat imaging
in selected cases. Active arterial bleeding, in the absence of other associated injuries, can
be treated with emergency arteriography and selective angioembolization.

The only absolute indications for renal exploration are pedicle avulsion, pulsatile or
expanding hematoma, and hemodynamic instability resulting from renal
injury.6 Significant numbers of shattered kidneys (grade V) and renal vascular injuries
(grades IV and V) are now managed nonoperatively. The strongest predictor for
nephrectomy is severity of renal injury; however, roughly one third of all penetrating and
44% of blunt grade IV and V renal injuries are now managed nonoperatively.9,13,14In
patients who require laparotomy for associated injuries, renal exploration and
reconstruction of grade III and IV injuries may reduce the likelihood of delayed
complications. However, the need for laparotomy and surgery for other intra-abdominal
organs is associated with a higher likelihood of nephrectomy. Thus, exploration of
suspected kidney injuries (as determined by previous imaging or on-table evaluation) in
patients undergoing laparotomy for major splenic or bowel injury should be attempted by
surgeons experienced in repairing an injured kidney.9 In reality, the success of
nonoperative management for most grade III and IV injuries means that operative
intervention in cases of blunt trauma is typically limited to patients with the most severe
renal injuries, in whom conservative management fails either because of bleeding or
because of ongoing urinary extravasation despite ureteral stenting.15

A significant number of patients with a penetrating injury and a minority of those with
blunt trauma require immediate laparotomy before radiographic
evaluation.9,13 Hematuria should alert the surgeon to the possibility of renal injury, and
the presence of a perinephric hematoma visible through the mesocolon should prompt
further evaluation. If a major renal injury is suspected on the basis of the size of the
hematoma or an abnormal intraoperative intravenous pyelogram (IVP), exploration is
indicated.
2014. Scientific American Surgery. Hamilton, Ontario & Philadelphia, PA. Decker
Intellectual Properties Inc. ISSN 2368-2744. STAT!Ref Online Electronic Medical
Library. http://online.statref.com/Document.aspx?fxId=61&docId=2366. 10/20/2014
1:04:32 PM CDT (UTC -05:00).

Feedback on your answer


Collapse
4)
1/1
You are seeing a 56-year old male who presents with a low-grade fever, high white blood
cell count, abdominal pain, and rectal bleeding following a colonscopy the 12 hours ago.
Per the record he had a 1.2-cm polyp removed after multiple attempts. His abdominal
exam shows mild tenderness in the left lower quadrant without signs of rigidity or
rebound. Abdominal x-ray shows no signs of free air. What is the most likely diagnosis in
this patient?

(Select 1)( 1pts extra credit)

Pancreatitis

Perforation

Postpolypectomy electrocoagulation syndrome

Residual infected polyp

Postpolypectomy electrocoagulation syndrome refers to the development of abdominal


pain, fever, leukocytosis, and peritoneal inflammation in the absence of frank perforation
following polypectomy with electrocoagulation. It occurs when electrocautery current is
transmitted beyond the mucosa into the muscularis propria and serosa of the colon.
Patients usually present within 12 hours, but symptoms may occur up to five days after
the procedure. Most patients complain of mild ache and tenderness in the region of the
polypectomy site and do not require hospitalization. Some patients have localized
abdominal tenderness, rigidity, fever, leukocytosis, and tachycardia, a symptom complex
that closely resembles colonic perforation.
The diagnosis should be suspected in patients who develop the clinical features described
above following polypectomy with electrocautery, particularly those who had large
sessile polyps removed. The distinction from frank perforation requires radiographic
evaluation, which is best accomplished with an abdominal CT scan.
For patients whose symptoms warrant hospitalization, we suggest conservative treatment
with intravenous fluids, nothing by mouth, bedrest, and antibiotics until symptoms
improve. Most patients with mild symptoms will not need any treatment beyond
reassurance. Read more from Uptodate

Feedback on your answer


Collapse
5)
1/1
You are seeing a 30-year old male in the trauma bay who was an unrestrained driver
involved in a rollover MVC who is complaining of lower abdominal pain. His GCS is
15, vitals are stable and he is awake and talking. The rest of his exam shows he has
moderate right lower abdomen and flank tenderness to palpation without guarding.
His iStat labs show a Hct 45.7, BUN 15, Cr 1.2. Upright CXR is shows no signs of
trauma and there is a normal sized medastinum. CT scan of his abd/pelvis demonstrates a
small renal subcapsular hematoma with 0.5 cm superficial parenchymal laceration that
does not involve the collecting system. According to the American Association for the
Surgery of Trauma (AAST), what grade injury does this demonstrate?

(Select 1)( 1pts extra credit)

Grade I

Grade II

Grade III

Grade IV

The patient above has a grade II renal laceration as he has a


<1cm renal laceration,Read more
Renal trauma grading is often done using theAmerican Association for
the Surgery of Trauma (AAST)3-4 according to depth of damage and
involvement of the urinary collecting system and renal vessels.
grade I - contusion or non enlarging subcapsular haematoma, but no

laceration ;
 grade II - superfical laceration < 1cm depth and does not involve the
collecting system ; non expanding perirenal haematoma
 grade III - laceration > 1cm, without extension into the renal pelvis
or collecting system and with no evidence of urine extravasation
 grade IV - laceration extends to renal pelvis or urinary extravasation.
 grade V - shattered kidney ; devascularisation of kidney due to hilar
injury.
Read more from Up to Date

Feedback on your answer


Collapse
Unit 2
6)
0/1
What is the half life of the tumor marker CEA?
(Select 1)(1pts)

12-24 hours

1-2 days

3-5 days

7-14 days

The answer is D, 7-14 days. CEA remains the prototypical solid tumor marker. Despite
its lack of specificity, if used correctly, CEA testing is a valuable addition to the process
of clinical decision making in patients diagnosed with colon or rectal carcinoma.
However, it is not an appropriate screening test. Whether sampled once or serially, CEA
cannot be used in the differential diagnosis of an unknown-but-suspected bowel problem
or malignancy. Nevertheless, when CEA concentrations are determined before primary
tumor resection, they may be of additional prognostic value; this is particularly true in
patients with stage II disease, for whom elevated preoperative CEA is a poor prognostic
marker and may influence the decision regarding whether to administer adjuvant
chemotherapy.
Serial CEA values obtained postoperatively are a potentially effective means of
monitoring response to therapy. A postoperative CEA titer serves as a measure of the
completeness of tumor resection. It should be remembered, however, that the half-life of
CEA is 7 to 14 days; therefore, postoperative baselines are best established several weeks
after resection. If a preoperatively elevated CEA value does not fall to normal within 2 to
3 weeks after surgery, it is likely that (1) the resection was incomplete or (2) occult
metastases are present. A rising trend in serial CEA values from a normal postoperative
baseline (< 5 ng/mL) may predate any other clinical or laboratory evidence of recurrent
disease by 6 to 9 months. Read more

Feedback on your answer


Collapse
7)
1/1
You are seeing a 73-year old male who has a past medical history of
CHF, hypertension and type II diabetes. He initially presented to you
with changes in stool patterns, constipation and hematochezia. He was
found to have adenocarcinoma of the ascending colon along with
metastasis to his liver, brain and his right lung. For patient presenting
with metastatic colon cancer, which of the following treatments is not
routinely recommended?
(1pts)

Chemotherapy

Primary Resection of the colon cancer

Radiation

All are routinely recommended

All of the above

The answer is B. In treating Colorectal cancer, it is crucial to


understand that surgical extirpation of the primary tumor is done when
there is a realistic possibility of cure or for patients with symptomatic
tumors that cause acute obstruction or clinically significant bleeding.
For those who present with synchronous primary tumors and incurable
metastatic disease, resection is not routinely indicated. Advances in
systemic chemotherapy (outlined below) have significantly increased
the probability of managing the tumor medically, and chemotherapy
can begin immediately in the setting of an asymptomatic or minimally
symptomatic primary. In other words, there is no need to delay
initiation of systemic chemotherapy by palliative resection of a primary
tumor that is not actively symptomatic. In truth, resection of a primary
lesion in the setting of metastatic disease is likely to cause significant
morbidity and even mortality. A review of Medicare/SEER data
focusing on patients 65 and older reported a 30-day postoperative
mortality of 10% with resection of a synchronous primary tumor.
Furthermore, in a large retrospective series, Poultsides and colleagues
reported that 93% of patients presenting with synchronous stage IV
disease without overt obstruction never required specific intervention
on their primary tumor. Adenocarcinoma of the Colon and Rectum
Feedback on your answer
Collapse
8)
0/1
Which one of the following tumor markers is used to monitor for the recurrence colon
cancer?
(1pts)

Carcinoembryonic antigen (CEA)

Cancer antigen 27.29 (CA 27-29)

Cancer antigen 125 (CA-125)

Prostate-specific antigen (PSA)

The answer is D., Carcinoembryonic antigen (CEA). Prostate-specific antigen (PSA) is a


marker that is used to screen for prostate cancer. It is elevated in more than 70% of
organ-confined prostate cancers. Alpha-tetoprotein is a marker for hepatocellular
carcinoma and nonseminomatous germ cell tumor, and is elevated in 80% of
hepatocellular carcinomas. CA-125 is a marker for ovarian cancer. Although it is elevated
in 85% of ovarian cancers, it is elevated in only 50% of early-stage ovarian cancers.
Carcinoembryonic antigen (CEA) is a marker for colon, esophageal, and hepatic cancers.
It is expressed in normal mucosal cells and is overexpressed in adenocarcinoma,
especially colon cancer. Though not specific for colon cancer, levels above 10 ng/mL are
rarely due to benign disease. CEA levels typically return to normal within 4–6 weeks
after successful surgical resection. CEA elevation occurs in nearly half of patients with a
normal preoperative CEA level that have cancer recurrence. Cancer antigen 27.29 (CA
27-29) is a tumor marker for breast cancer. It is elevated in about 33% of early-stage
breast cancers and about 67% of late-stage breast cancers. Some tumor markers, such as
CEA, alpha-fetoprotein, and CA-125, may be more helpful in monitoring response to
therapy than in detecting the primary tumor. Ref: Perkins GL, Slater ED, Sanders GK, et
al: Serum tumor markers. Am Fam Physician 2003;68(6):1075-1082.
Feedback on your answer
Collapse
9)
0/1
Which of the following statements regarding colorectal cancer (CRC) is not true?
(Select 1)(1pts)
The probability of CRC developing during an individual's lifetime is about
6%

Colon cancer is three times more common than rectal cancer

CRC ranks as the third most common malignancy in the United States

30% of colorectal cancer is diagnosed in individuals younger than 40

Nonhereditary CRCs are referred to as "sporadic" and comprise 75 to 80%


of all CRCs

Answer D is false. Worldwide, over 1 million people are diagnosed with CRC annually,
and there are more than 500,000 associated deaths.1 The highest rates of colorectal
carcinoma are found in industrialized countries. The rates are significantly lower in
eastern Europe, Asia, Africa, and South America.2 However, studies of Japanese
migration to the United States, Asiatic Jewish migration to Israel, and eastern European
migration to Australia show that migrants acquire the high rates of CRCs prevalent in
their adopted countries. There is little question that environmental factors, most likely
dietary, account for this.
Colon cancer is three times more common than rectal cancer. Interestingly,
epidemiologic studies indicate a rising proportion of right-sided colonic lesions. The
proximal migration of colon cancer may be associated with changing environmental
factors; however, there is no doubt that increased screening successfully detects early
lesions in an aging population. CRC ranks as the third most common malignancy in the
United States (behind prostate and lung cancer in men and breast and lung cancer in
women) and the second leading cause of cancer-related mortality. Approximately
143,000 patients are diagnosed with CRC in the United States each year, and 51,000 die
of disease.4,5 The probability of CRC developing during an individual's lifetime is about
6%. In contrast to the three previous decades, however, the overall incidence and
mortality of CRC have declined for both men and women. Age-adjusted incidence and
mortality are associated with race and ethnicity; however, the relationships are complex,
influenced by social and economic confounding factors more than tumor biology.
Clearly, CRC is associated with genetic and environmental influences. Overt risk factors
include a personal or family history of CRC or colorectal adenoma(s), a personal history
of colorectal polyps, inflammatory bowel disease (IBD), and age greater than 50. Age is
the most common risk factor. The risk of CRC increases after the fourth decade of life.13
Most individuals present with disease after the age of 60, and only 10% of CRCs are
diagnosed in individuals younger than 40.
Nonhereditary CRCs are referred to as "sporadic" and comprise 75 to 80% of all CRCs.
Genetic etiology may be identified in the remaining 20 to 25% of patients, including
family history (15 to 20%), Lynch syndrome (5%), and FAP (< 1%). Cancer can arise
within a polyp or at another site in the colon or rectum. Read more
Feedback on your answer
Collapse
Unit 3
10)
0/1
You are seeing a 61-year old female who presents with worsening shortness of breath
for two days. The day preceding her symptoms she was involved in an MVC and was
seen in your facility and diagnosed with 3 non displaced rib fractures on the right. Her x-
ray today is shown below. What is the most likely diagnosis?

(Select 1)(1pts)

Pneumothorax

Pulmonary Contusion

Fat embolism

Pneumonia

Congestive Heart Failure

The answer is B. Severe blunt trauma to the chest can cause rib fractures and other
immediate injuries but sometimes pathology doesn't show up until days later such as a
pulmonary contusion and myocardial contusion. The contused lung is very sensitive to
fluid overload which can lead to respiratory distress. Pulmonary contusions generally
develop over the first 24 hours and resolve in about one week. Irregular, nonlobular
opacification of the pulmonary parenchyma on chest radiograph is the diagnostic
hallmark. About one-third of the time the contusion is not evident on initial radiographs.
Chest CT provides better resolution, but rarely alters management, unless other injuries
are found. Contusions evident on CT but not plain radiograph have better outcomes.
Pain control and pulmonary toilet are the mainstays of treatment. Prophylactic
endotracheal intubation is unnecessary, but patients with hypoxia or difficulty ventilating
require airway management. While opinions vary, fluid resuscitation with crystalloid to
euvolemia appears appropriate. Common complications include pneumonia and acute
respiratory distress syndrome (ARDS). Read more

Feedback on your answer


Collapse
11)
0/1
You are seeing a 27-year old male in the trauma bay shortly after being stabbed in this
back with a 6 cm buck knife by his disgruntled neighbor after an argument.
He arrived via EMS sitting up, awake and alert with a GCS of 15. His BP is 140/90, HR
120 and O2 sat is 85% on a non re-breather.
On physical exam there is a 3 cm non bleeding wound just left of his mid thoracic spine.
Upon auscultation there is decreased breath sounds on his left side. There are no signs of
tracheal deviation or JVD.
What is the next best step in management?
(1pts)

Immediately to the OR for surgical exploration

Upright Chest Xray followed by a chest tube

Immediate Needle thoracotomy on the left

Pericardiocentesis under US guidance

The answer is A, Upright Chest Xray followed by a chest tube. This patient is awake and
talking. He is stable enough for a chest xray followed by chest tube placement. A needle
thoracotomy to should be done if he has evidence of tension pneumothorax which would
be respiratory distress and hypotension. He will likely then need a chest tube and can be
further managed. Read More
Feedback on your answer
Collapse
12)
1/1
What is the estimated overall mortality from pulmonary contusions?

(Select 1)(1pts)

3% to 5%

5% to 10%

10% to 25%

25% to 40%

The answer is C, 10-25%. Pulmonary contusions, pneumothorax, and hemothorax occur


in 30% to 50% of patients with severe blunt chest trauma managed in trauma centers.
Pulmonary contusions commonly result from direct impact to the chest wall or from
concurrent chest wall injury, such as from a blast.20 Acute parenchymal lung injury
produces multiple physiological effects, all of which have an impact on outcomes. The
mortality associated with pulmonary contusion is difficult to predict, but is estimated to
be 10% to 25%.21,22 The clinical manifestations of pulmonary contusion are variable,
ranging from mild dyspnea to acute lung injury and acute respiratory distress syndrome.
The pathophysiology of pulmonary contusion includes alveolar hemorrhage and edema
resulting in decreased lung compliance, increased alveolar capillary permeability, and
increased intrapulmonary shunting. These processes result in hypoxemia, hypercarbia,
and decreased lung compliance. Lung contusions also result in acute local and systemic
inflammatory cascades that activate tissue macrophages and the production of
inflammatory mediators, cytokines, and chemokines. These result in pathophysiologic
changes that clinically manifest as immunosuppression, acute lung injury/acute
respiratory distress syndrome, and respiratory failure. Read more

Feedback on your answer


Collapse
13)
0/1
A 65-year-old male comes in the emergency Department because of severe right-sided
chest pain following a fall off his ladder 10 feet high.
Examination shows decreased respiratory movements on the right side of the chest and
tenderness on palpation over the right mentions, his abdomen is soft and non tender.
An x-ray film of his chest shows a non displaced fracture of the right 6th and 7th ribs on
the right without signs of pneumothorax.
Which of the following is the most appropriate goal in management of the rib fracture in
this patient?
(1pts)

To achieve a tital volume of 500 ML with intubation

To ensure appropriate analgesia

To provide mechanical stabilization to the chest wall

To give prophylactic antibiotics

The answer is B. Once significant associated injuries have been evaluated and treated, the
cornerstone of rib fracture management is pain control [36-38]. Early and adequate pain
relief is essential to avoid complications from splinting and atelectasis, primarily
pneumonia. The choice of analgesia depends upon the injury, the clinician's comfort
performing nerve blocks with their potential complications, and the ease with which more
invasive treatments can be performed. Analgesia for severe and multiple rib fractures and
monitoring of admitted patients are discussed separately.
For isolated injuries (ie, single rib fracture), clinicians generally begin treatment with
nonsteroidal anti-inflammatory drugs (NSAIDs) with or without opioids. For more severe
injuries, particularly if ventilation is compromised, admission and invasive treatments,
such as intercostal nerve blocks, may be needed.
Respiratory care, including use of incentive spirometry to prevent atelectasis and its
complications, is often important. We do not recommend rib belts or binders because
they compromise respiratory function. Studies of rib belts involve small numbers of
patients and have reached contradictory conclusions [39,40]. Patients with a rib fracture
who are discharged home can also use incentive spirometry throughout the day, after
analgesics have taken effect. Holding a pillow or similar soft brace against the fracture
site reduces discomfort while using the spirometer, or when coughing.
Disposition — Several researchers recommend hospital admission for any patient with
three or more rib fractures, and ICU care for elderly patients with six or more rib
fractures [14,15]. They cite the significant correlation between these findings and
serious internal injuries, such as pneumothorax and pulmonary contusion. We suggest
hospitalization for the majority of patients with three or more rib fractures. Patients with
multiple rib fractures sustained from high-energy trauma are best evaluated at a trauma
center. Transfer should be arranged expeditiously.
Displaced rib fractures likely increase the risk of injury to the lung and proximate
intercostal blood vessels. Bleeding from such fractures can be delayed, and admission or
close observation and follow-up should be arranged for patients with displaced fractures,
depending upon clinical and social circumstances. Multiple case reports indicate that
delayed bleeding from intercostal vessels or other injuries can be life-threatening,
particularly in older patients [20,41,42].
Surgical fixation may be of benefit with some types of rib fractures, particularly those
associated with chest wall deformity, flail chest, or symptomatic nonunion. The
appropriate role of surgical fixation is discussed separately. (See "Inpatient
management of traumatic rib fractures", section on 'Surgical
management'.)
Rarely, younger healthy individuals with three rib fractures, having undergone a thorough
clinical and radiographic evaluation by clinicians experienced in trauma management,
and an appropriate period of observation (a minimum of six hours of observation,
including a follow-up chest radiograph, is needed to rule out pneumothorax), may be
discharged from the emergency department. Clinicians must also consider patient
comorbidities and clinical and social circumstances when determining disposition.
Patients with one or possibly two isolated rib fractures and no complicating factors may
be discharged home with appropriate follow-up and adequate analgesia. Uptodate
Feedback on your answer
Collapse
Unit 4
14)
1/1
You are seeing a 32-year old male who presents after being kicked multiple times in his
right flank. He is complaining of severe flank pain and gross hematuria. What is the gold
standard for imaging of renal trauma?
(Select 1)(1pts)

MRI abdomen/pelvis

Ultrasound

CT abdomen/pelvis with contrast

Intravenous pyelography

None of the above

The answer is C, CT abdomen/pelvis with contrast. CT scanning with intravenous


contrast enhancement is the modality of choice for the identification and staging of renal
trauma in the hemodynamically stable patient (image 6). The initial CT images will
frequently miss injuries to the renal pelvis and ureters as sufficient contrast may not yet
be present in the collecting system. Additional delayed images are needed to assess for
contrast extravasation when these injuries are suspected (image 7). In patients
undergoing CT scanning of other organ systems, suspected bladder injuries may be
investigated with CT cystography after retrograde filling of the bladder, as described
above. Read more
 CT with IV contrast is the Gold standard, high sensitivity
 Immediate and delayed post-contrast images to view collecting system
 Images abdomen and retroperitoneum
 Allows diagnosis and staging
 Intravenous pyelography doesnt allow you to image the abdomen or
retroperitoneum.

Feedback on your answer


Collapse
15)
0/1
You are seeing a 48-year old male in the trauma bay after shooting himself in his right
thigh while it was in its holster.
On exam there is one wound that is about 2cm in diameter on his posterolateral right
thigh. It has since stopped bleeding after pressure was applied. His pulses are equal and
symmetrical and his neuro exam in normal. You obtain an xray which shows bullet
fragments in his thigh without evidence of fracture.
What is the next best step in treatment?
(1pts)

Ct scan of his leg to evaluate the soft tissue damage

Ultrasound to evaluate arterial flow

Ct angiography of his leg

Irrigation and tetanus

This patient shows no sign of arterial injury as he has no hard or soft signs of vascular
injury (see below). He has a normal neuro exam and has no fracture on xray. The only
thing that this patient needs is a tetanus update and wound irrigation.

Hard signs of Vascular injury


 Pulsatile bleeding
 Expanding haematoma
 Absent distal pulses
 Cold, pale limb
 Palpable thrill
 Audible bruit
The presence of hard signs of vascular injury mandates immediate operative intervention.
Usually the site of injury is obvious, and angiography is unnecessary. If in doubt,
angiography can be performed emergently on the operating room table. Unnecessary
interventions and investigations should be avoided to minimise the delay to definitive
care. Read more
Feedback on your answer
Collapse
16)
0/1
A 32-year-old man is brought to the emergency department after he drove his motorcycle
into a guardrail at a high speed and was riding without a helmet. On arrival, his pulse is
100/min, respirations are 14/min, and blood pressure is 120/80 mm Hg.
Physical examination shows a GCS of 14, on abdominal exam there is mild suprapubic
tenderness. GU exam shows a deep perineal laceration. He voids 25 mL of bloody urine.
An x-ray of the pelvis shows a widened pubic symphysis. Which of the following is the
most appropriate next step in diagnosis?
(Select 1)(1pts)

Urethrography (retrograde urethrogram)

Cystography

Placement of a urinary catheter

Intravenous pyelography

The answer is A. This injury often follow pelvic fractures and classically present with
blood at the urethral meatus, an ability to void and a high riding or a non palpable
prostate to an intern. If you suspect urethral injury a retrograde urethrogram needs to be
performed prior to foley insertion. This procedure will locate the damage of the urethra, if
present. Inserting a Foley before this procedure is contraindicated as this can worsen the
urethral tear and potentially cause infection or a hematoma (you do not want to cause or
worsen the chance of a urethral stricture). These injuries often need surgical repair,
especially anterior urethral injuries. Some are treated with urinary diversion via
suprapubic catheter while the primary injury heals. Read more
Feedback on your answer
Collapse
17)
0/1
You are seeing a 25-year old male in the trauma bay after he was involved in a motor
vehicle accident. He was thrown from his bike and is now complaining of right flank
pain. His GCS is 15 and he is awake and oriented. His BP is 125/75, HR 99, RR 18, O2
98% on room air. His physical exam is pertinent for right flank ecchymosis with mild
tenderness to palpation along with mild right upper quadrant tenderness. There is no
gross blood seen at his urethral meatus and his urinalysis shows no RBCs. True or False:
In the setting of potential renal trauma (blunt and penetrating trauma), gross or
microscopic hematuria is ALWAYS present.
(1pts)
True Incorrect

False

The answer is False: In the setting of renal trauma, gross or microscopic hematuria is
absent in up to 5% of cases and this finding alone should not be used to preclude in those
you are suspicious of renal trauma. Read more on renal trauma
Feedback on your answer
Collapse
Unit 5
18)
1/1
Which of the following statements regarding hernias is not correct?
(Select 1)(1pts)

In female patients, indirect inguinal hernias are the most common type

The lifetime risk of having to undergo an inguinal hernia repair is greater


for men than in women

In male patients, direct inguinal hernias are the most common type

Femoral hernias account for fewer than 10% of all groin hernias; however,
40% present as emergencie

Answer C is the only incorrect statement. In the United States, approximately


1,000,000 abdominal wall herniorrhaphies are performed each year, of
which almost 80% are for inguinal or femoral hernias.1Worldwide,
some 20 million groin hernias are repaired each year.2 The lifetime
risk of having to undergo an inguinal hernia repair is 27% for men and
3% for women.3 In male patients, indirect inguinal hernias are the
most common type (more often located on the right), occurring
approximately twice as frequently as direct inguinal hernias; femoral
hernias account for a much smaller percentage. In female patients,
indirect inguinal hernias are also the most common type, but femoral
hernias are seen more frequently than direct hernias, which are rare in
this population. Femoral hernias account for fewer than 10% of all
groin hernias; however, 40% present as emergencies (i.e., with
incarceration or strangulation), and mortality is higher for emergency
repair than for elective repair. A two-peak theory has been described,
stating that a new diagnosis of an inguinal hernia is most likely in
patients younger than 1 year and in patients older than 55 years,
although hernias can be diagnosed across any given age group. Read
more
Feedback on your answer
Collapse
19)
0/1
Following non mesh hernia repair, how many patients would need to be treated to prevent
one infection at 30 days?
(Select 1)(1pts)

20

50

75

120

The answer is 50. Mesh infection is rare. In a 2003 Cochrane review of


antibiotic prophylaxis for nonmesh hernia repairs, the overall infection
rate was 4.69% in the control group and 3.08% in the treatment
group.65 Thus, to prevent one infection in 30 days, 50 patients would
have to be treated, and
these patients would then be at risk for antibiotic-associated
complications. Laparoscopic repairs were excluded from this review;
however, in a meta-analysis comparing postoperative complications
after laparoscopic inguinal hernia repair with those after open repair,
superficial infection was less frequent in the laparoscopic
groups.49 Deep mesh infection was rare in both groups. Mesh
infection usually responds to conservative treatment with antibiotics
and drainage. On rare occasions, the mesh must be removed; this
may be accomplished via an external approach. If, however, a
prosthesis composed of a hydrophobic material (e.g., expanded
polytetrafluoroethylene) becomes infected, it is very difficult to
sterilize and virtually always must be removed. It is noteworthy that
removal of the mesh does not always lead to recurrence of the hernia,
a finding that may be attributable to the resulting fibrosis. Read more
Feedback on your answer
Collapse
20)
0/1
What % of minimally symptomatic inguinal hernias end up requiring emergency
operation for strangulation over a one year period?

(Select 1)(1pts)

0.18%

1.18%

2.1%

6.5%

12.4%

The answer is A, The natural history of an untreated, minimally


symptomatic inguinal hernia was addressed in a randomized,
controlled trial in which 364 men were assigned to "watchful waiting"
(WW) and 356 men underwent routine operation.48 Only two patients
in the WW group required emergency operations for strangulation over the
follow-up period of 2 to 4.5 years. This result translated into a rate of 1.8 per 1,000
patient-years (0.18%), or about one fifth of 1% for each year that the hernia remains
unrepaired. The two patients who required emergency operations recovered uneventfully.
The question that remained to be answered was which group fared better overall: the WW
group or the group whose hernias were repaired immediately in accordance with
conventional teaching? The answer to this question was at variance with conventional
assumptions. At the conclusion of the study, functional status, as measured by quality of
life instruments and pain scales, was identical in the two groups. About one third of the
patients in the WW group crossed over to undergo operative treatment, principally
because of symptom progression. However, there appeared to be no penalty for delaying
surgery. Before this study, most surgeons assumed that a hernia would become harder to
repair the longer it remained (because of enlargement and buildup of scar tissue) and that
patients whose operations were delayed would experience more complications. The
investigators found, however, that postoperative complication rates were the same in
patients who underwent immediate surgery as in those who were assigned to WW but had
to cross over to surgical treatment. Read more

Feedback on your answer


Collapse
21)
0/1
A 24-year old male basketball player presents to you with a painful bulge into his right
inguinal area into his scrotum that he first noticed while he was playing basketball. He
asks if it could be a sports hernia and if this will limit his practicing and upcoming
tournament. What hernia does this patient most likely have?
(Select 1)(1pts)

Indirect

Direct

Femoral

Richter

This patient most likely has an indirect hernia as it is the most common hernia and given
the lack of other findings in the stem that is the best answer and most likely. Although a
sports hernia may lead to a traditional, abdominal hernia, it is a different injury. A sports
hernia is a strain or tear of any soft tissue (muscle, tendon, ligament) in the lower
abdomen or groin area. Because different tissues may be affected and a traditional hernia
may not exist, the medical community prefers the term "athletic pubalgia" to refer to this
type of injury.
An indirect inguinal hernia is one of the most common abdominal hernias. It is
five times more common than a direct inguinal hernia, and is seven times more frequent
in males, due to persistence of the process vaginalis during testicular descent. An indirect
hernia enters the inguinal canal at the deep ring, lateral to the inferior epigastric vessels.
It passes infromedially to emerge via the superficial ring and, if large enough, extend into
the scrotum. In children, the vast majority of inguinal hernias are indirect (see Case 3).
Incarceration represents the most common complication associated with inguinal hernias,
the incidence could be as high as 30% for infants younger than 2 months.
A direct inguinal hernia arises from protrusion of abdominal viscera through a
weakness of the posterior wall of the inguinal canal medial to the inferior epigastric
vessels, specifically through the Hasselbach's triangle. This type of hernia is termed
direct as the hernial sac directly protrudes through the inguinal wall in contrast to indirect
ones which arise through the deep ring and enter the inguinal canal. Since direct hernias
do not have a guiding path, they seldom extend into the scrotum unless very large and
chronic. Direct hernias arise usually as acquired weakness of the Hasselbalch's triangle.
Therefore, they are seen in the elderly with chronic conditions which increase intra-
abdominal pressure over a long period, e.g. COPD, bladder outflow obstruction, chronic
constipation etc. Increased abdominal pressure is transmitted to both sides and as a result,
direct hernias are usually bilateral. Compared to indirect hernia, they are less susceptible
to strangulation as they have a wide neck.
In contrast to the indirect hernia, a direct hernia is most often an acquired lesion. It
occurs when a weak spot develops in the lower abdominal musculature (the posterior
floor of the inguinal canal) due to the normal and/or abnormal stresses inflicted by living
and aging. In adults, stresses such as lifting heavy of objects, frequent coughing or
straining, pregnancy, and constipation can instigate hernia. Unlike indirect hernias, direct
hernias traverse medial to the inferior epigastric vessels and are not associated with the
processus vaginalis. The hernia consists primarily of retroperitoneal fat. Only rarely is a
peritoneal sac containing bowel encountered. Because there is typically no involvement
of a sac, they do not protrude with the spermatic cord, and as such, have a lower
incidence of incarceration or strangulation. Like indirect inguinal hernia, direct inguinal
hernias typically cause a bulge in the groin (at the top of or within the scrotum) and
usually with increased abdominal pressure. Like indirect hernias, they may or may not be
painful (usually not). By palpating the inguinal canal and asking the patient to cough
while standing, one can usually elicit the hernia. In fact, one can often times palpate an
inguinal hernia without invaginating the scrotum (as is typically taught in medical
school). Rather, by placing one's fingers over the inguinal canal and asking the patient to
cough, one can often feel the bulge against the lower abdominal wall. As direct and
indirect hernias are unreliably differentiated by physical exam alone, the need to
invaginate the scrotum to feel into the inguinal canal is often more uncomfortable to the
patient, than telling to the physician. Rarely, palpation is not even necessary, as the
hernia is large enough to be visualized. Read more on Inguinal hernias: A Brief
review

Feedback on your answer


Collapse
Unit 6
22)
0/1
Which of the following statements regarding hollow viscus injuries in patients suffering
from blunt trauma is not true?
(Select 1)(1pts)

Mortality is increased in parallel with time to operative intervention

Only 1.2% of blunt trauma admissions had an associated hollow viscus


injury

The "seat-belt" sign is associated with a more than doubled relative risk of
small bowel injury

Unexplained intraperitoneal fluid (i.e., fluid appearing in the absence of


solid-organ injury) was the least common radiographic finding associated
with blunt bowel or mesenteric injury
Answer D is false. Unexplained intraperitoneal fluid (i.e., fluid appearing in the absence
of solid-organ injury) was the most common radiographic finding associated with blunt
bowel or mesenteric injury but often proved to be a false positive finding.
Hollow viscus injury after blunt trauma, although uncommon, can have serious
consequences if the diagnosis is missed or delayed. In a multi-institutional study of 198
patients with blunt small bowel injury, delay of as little as 8 hours in making the
diagnosis resulted in increased morbidity and mortality.2 Mortality increased in parallel
with time to operative intervention (< 8 hours to operation, 2% mortality; 8 to 16 hours,
9%; 16 to 25 hours, 17%; > 24 hours, 31%), as did the complication rate. Consequently,
it is important to have an expedient approach to the diagnosis of blunt bowel injury.
Physical examination findings such as abdominal tenderness or tachycardia may suggest
the presence of hollow viscus injury. Distracting chest or long bone injury, closed-head
injury, spinal cord injury, or intoxication, however, may compromise reliability of the
examination. In addition, it is not uncommon for blunt bowel injury to have a latent
period from the time of injury, whereby the expected signs and symptoms of such injuries
take some time to develop. Laboratory abnormalities, including elevations in white blood
cell (WBC) count, amylase, and/or lactic acid, may also point toward the presence of
hollow viscus injury but are relatively nonspecific. Provided that the patient has suffered
a low-risk mechanism of injury (such as a fall from standing or a low-speed motor
vehicle collision), hollow viscus injury is extremely unlikely in the face of a normal,
reliable physical examination and normal laboratory results. With these conditions
present, the presence of blunt bowel injury can be effectively excluded. However, the
presence of abdominal complaints, an abnormal or unreliable physical examination,
abnormal laboratory results, or a high-risk mechanism of injury (such as a high-speed
motor vehicle collision) warrants further evaluation by imaging for the presence of bowel
injury.
Particular consideration should be given to lap-and shoulder-restraint injuries, which may
be associated with an increased risk of hollow viscus injury. The "seat-belt" sign (i.e.,
ecchymosis of the abdominal wall secondary to the compressive force of the lap belt) is
associated with a more than doubled relative risk of small bowel injury.3,4 Flexion-
distraction fractures of the spine (Chance fractures) are also associated with lap-belt use,
and the presence of such fractures should raise the index of suspicion for associated
hollow viscus injury.
Ultrasonography is routinely performed early in the evaluation of blunt abdominal
trauma. It is highly specific and moderately sensitive in identifying intra-abdominal fluid,
the presence of which in a hemodynamically unstable patient is an indication for
laparotomy (in that it strongly suggests the presence of significant intra-abdominal
hemorrhage).5 Ultrasonography does not, however, reliably distinguish solid-organ
injury from hollow viscus injury—a distinction that is critical for determining subsequent
management (i.e., operative versus nonoperative) in a hemodynamically stable patient.
Computed tomography (CT) is the imaging modality of choice in stable patients who
warrant evaluation by imaging as described above. We reviewed over 8,000 CT scans
performed to evaluate cases of blunt abdominal trauma and found that the number of
abnormal radiologic findings suggesting blunt injury to the bowel, the mesentery, or both
was correlated with the true presence of injury [see Table 1].6 A CT scan
demonstrating a solitary abnormality was associated with a true positive rate of 36%,
whereas a scan demonstrating more than one abnormality was associated with a true
positive rate of 83%. Unexplained intraperitoneal fluid (i.e., fluid appearing in the
absence of solid-organ injury) was the most common radiographic finding associated
with blunt bowel or mesenteric injury but often proved to be a false positive finding. On
the basis of this experience, we developed an algorithm for the evaluation of blunt hollow
viscus injury in patients with unreliable physical examinations [see Figure 1].
Most CT scans performed in this clinical setting, however, will be negative for evidence
of intra-abdominal injury. A prospective multi-institutional trial involving 3,822 blunt
trauma patients demonstrated that the negative predictive value of a normal abdominal
CT scan was 99.63%, leading the authors to conclude that patients with a normal scan do
not benefit from hospital admission and prolonged observation.7 However, a multi-
institutional review of 2,457 cases carried out by the Eastern Association for the Surgery
of Trauma (EAST) reported a 13% incidence of blunt small bowel injury in patients with
an initial negative CT scan. These results indicate that caution should be exercised in
dismissing the presence of hollow viscus injury on the basis of a negative scan.3 This
concern is echoed by our own institutional experience, in which the incidence of injury in
patients with an initial negative CT scan was 12%.6 If CT scanning demonstrates no
suspicious findings, no further diagnostic workup of hollow viscus injury is necessary,
but the duration of the observation period depends on both the overall condition of the
patient and clinical judgment. Most patients, as supported by the negative predictive
value of the study above, will not require ongoing further observation.
Feedback on your answer
Collapse
23)
0/1
What percentage of patients suffering from blunt abdominal trauma develop hollow
viscus injury?
(Select 1)(1pts)

1.2%

4.8%

8.9%

13.4%

The answer is A, 1.2%. Hollow viscus injury after blunt trauma, although
uncommon, can have serious consequences if the diagnosis is missed
or delayed.Hollow viscus injury is most often the consequence of
penetrating abdominal trauma. As a result of blunt force trauma,
bowel injury occurs with relative infrequency: in one multi-institutional
analysis, only 1.2% of blunt trauma admissions had an associated
hollow viscus injury. Read more
Feedback on your answer
Collapse
24)
1/1
What is the mean mortality rate of patients suffering from pancreatic trauma?
(Select 1)(1pts)

<3%

9%

19%

31%

The answer is C, 19%. Duodenal and pancreatic injury continues to


challenge the trauma surgeon. The relatively rare occurrence of these
injuries, the difficulty in making a timely diagnosis, and high morbidity
and mortality rates justify the anxiety these unforgiving injuries
invoke. Mortality rates for pancreatic trauma range from 9 to 34%,
with a mean rate of 19%. Duodenal injuries are similarly lethal, with
mortality rates ranging from 6 to 25%. Complications following
duodenal or pancreatic injuries are alarmingly frequent, occurring in
30 to 60% of patients.1-3 Recognized early, the operative treatment
of most duodenal and pancreatic injuries is straightforward, with low
morbidity and mortality. Read more
Feedback on your answer
Collapse
25)
1/1
Which of the following statements about duodenal or pancreatic injury is not true?
(Select 1)(1pts)

CT scan is the primary diagnostic modality

A bicycle handlebar to the epigastrium is the most common cause of injury

An elevated amylase level is a very sensitive but non specific finding


Duodenal hematomas causing obstruction usually present 2-3 after initial
injury

Answer C is the only false statement as No laboratory findings are particularly specific
for duodenal or pancreatic injury. Amylase has been proposed for both pancreas and
duodenal injuries but is not specific for either one. Some have noted that amylase can be
elevated in as many as 50% of duodenal injuries, but this has not been a consistent
finding.9 An elevated amylase should prompt an evaluation of the duodenum for injury
but alone is not diagnostic. Although the highest concentration of amylase in the human
body is in the pancreas, hyperamylasemia is also not a reliable indicator of pancreatic
trauma. In one series, only 8% of blunt abdominal injuries with hyperamylasemia had
pancreatic injury.10 Furthermore, as many as 40% of patients with a pancreatic injury
may initially have a normal serum amylase.
The history of a patient with a possible blunt pancreatic or duodenal injury usually
consists of a direct blow to the epigastrium. In children, this commonly involves a bicycle
handlebar to the epigastrium; in adults, more commonly the steering wheel or a motor
cycle handle bar is involved. However, any direct blow should raise suspicion. The
patient may complain of abdominal, back, or flank pain.
Outside of the pelvis x-ray, plain abdominal radiographs to evaluate blunt abdominal
trauma are less common in the era of focused abdominal sonography for trauma (FAST)
and computed tomographic (CT) scans. Reports suggest that signs of duodenal injury on
plain radiographs are identified less than one third of the time. Retroperitoneal air, free
intraperitoneal air, or obliteration of the psoas shadow should raise suspicion for
duodenal and other hollow viscous trauma.9 Upper gastrointestinal series have also been
used to evaluate the duodenum for injury and can add to the sensitivity and specificity of
plain films, but more recently, CT has become the primary diagnostic modality.
For optimal duodenal evaluation, intraluminal contrast administered via a nasogastric
tube soon but not immediately prior to the CT scan may aid by opacification of the lumen
of the duodenal "C loop." Visualization of contrast extravasation, retroperitoneal air,
adjacent fat stranding, and unexplained fluid, as well as duodenal wall thickening, are CT
findings suggestive of potential duodenal injury. The sensitivity of CT for duodenal
injury is related to the technology of the scanner (i.e., "number of slices") and the time
interval from the injury to imaging. In a recent review, CT was considered to have an
overall sensitivity of around 76% with new-generation scanners (16- or 64-slice), having
a higher sensitivity of around 82%. Additionally, if clinical suspicion remains high after
an initial negative CT scan, then repeat imaging is warranted and may improve diagnostic
yield.
At times, duodenal hematomas can cause duodenal obstruction. This usually presents 2 to
3 days after the trauma with evidence of gastric outlet obstruction. The common
nonoperative treatment of isolated obstructing duodenal hematomas is nasogastric tube
decompression of the stomach and duodenum, nutritional support, and time.38 Repeat
imaging in 7 to 14 days is reasonable to evaluate improvement or an unexpected finding
as most duodenal hematomas will have resolved after 7 to 14 days, and continued
obstruction may reveal additional injury, prompting intervention.39-41 Enteral nutrition
is preferred to parenteral nutrition, but in the setting of a duodenal obstruction, the
parenteral route (total parenteral nutrition [TPN]) is often required as passing a feeding
tube beyond the obstruction can be difficult and surgical feeding access defeats the
purpose of "nonoperative" care.

Unit 1
1)
0/1
You are seeing 45-year old black female again in the office who presented to you initially
for a right sided breast mass. She has no past medical history or family history of breast
cancer. On physical exam you palpated a 2 cm mass just lateral to her right nipple. You
ordered a mammogram which cam back normal. What percentage of patients who are
found to have a normal mammogram have breast cancer after biopsy of a suspicious
lesion?
(Select 1)( 1pts extra credit)

5%

15%

25%

45%

The answer is A. A diagnostic mammogram is the first imaging study performed for
a woman with a new, palpable breast mass, and should be performed even if a recent
mammogram was negative. While the false negative rate of mammograms is less than 5
percent for clinically palpable breast cancers [15], a normal mammogram does not
eliminate the need for further evaluation of a suspicious mass [5]. For women under age
30 years, the breasts are hypersensitive to radiation exposure [16]; however, if the
clinical findings are suspicious, a mammogram should be performed. Read More

Feedback on your answer


Collapse
2)
0/1
A 29-year-old white male presents to your office for evaluation of pain in the right index
finger that has been worsening over the last two days after he cut his finger while cutting
a piece of wood. He notes the pain has been getting progressively worse. On examination
the his right index finger is red, swollen and held in a flexed position. The pain increases
with passive extension of the finger, and there is tenderness to palpation from the tip of
the finger into the palm. Which one of the following is the most appropriate management
of this patient?

(Select 1)( 1pts extra credit)

Surgical drainage and IV antibiotics

Antiviral medication

Oral antibiotics and splinting

Needle aspiration

The answer is A, Surgical drainage and antibiotics. This patient has pyogenic
tenosynovitis. When early tenosynovitis (within 48 hours of onset) is suspected,
treatment with antibiotics and splinting may prevent the spread of the infection. However,
this patient’s infection is no longer in the early stages and is more severe, so it requires
surgical drainage and antibiotics. A delay in treatment of these infections can lead to
ischemia of the tendons and damage to the flexor tendon and sheath. This can lead to
impaired function of the finger. Needle aspiration would not adequately drain the
infection. Antiviral medication would not be appropriate, as this is a bacterial infection.
Corticosteroid injections are contraindicated in the presence of infection.

Ref: Canale ST, Beaty JH (eds): Campbell’s Operative Orthopaedics, ed 11.


Mosby Elsevier, 2007, pp 4351-4354.
Feedback on your answer
Collapse
3)
0/1
A 55-year-old white female presents with pain and redness to her right breast at the scar
site from a lumpectomy that was performed for stage I cancer four months ago. She has
also since completed multiple rounds of radiation treatments to her breast as well.
Currently her vitals are normal and she is afebrile. Physical exam reveals 5-6 cm of
erythema surrounding her surgical scar that is painful to touch, there is no axillary
adenopathy, no wound drainage, crepitance, or bullous lesions. Which one of the
following organisms would be the most likely cause of cellulitis in this patient?

(Select 1)( 1pts extra credit)

Non–group A hemolytic Streptococcus

Pneumococcus pneumoniae
Clostridium perfringens

Escherichia coli

The answer is A. Cellulitis in patients after breast lumpectomy is thought to be related to


lymphedema. Axillary dissection and radiation predispose to these infections. Non–group
A hemolytic Streptococcus is the most common organism associated with this infection.
The onset is often several weeks to several months after surgery. Pneumococcus is more
frequently a cause of periorbital cellulitis. It is also seen in patients who have bacteremia
with immunocompromised status. Immunocompromising conditions would include
diabetes mellitus, alcoholism, lupus, nephritic syndrome, and some hematologic cancers.
Clostridium and Escherichia coli are more frequently associated with crepitant cellulitis
and tissue necrosis. Pasteurella multocida cellulitis is most frequently associated with
animal bites, especially cat bites. Ref: Swartz MN: Cellulitis. N Engl J Med
2004;350(9):904-912.
Feedback on your answer
Collapse
4)
0/1
You are seeing a 31-year-old woman who presents with worsening sore throat, dry
cough, fever, and severe neck pain over the last week. She is otherwise healthy and takes
no medications.
Her vitals show a temperature is 102.5 °F, blood pressure is 99/68 mm Hg, pulse rate is
125/min, and respiration rate is 24/min. On exam she appears ill and her neck is tender to
palpation along the left side with overlying erythema, mild induration, without
lymphadenopathy. Her pharynx is erythematous, with tonsillar enlargement and no
exudates or ulcers. Her lungs are clear to auscultation. The remainder of the examination
is normal. Her chest x-ray shows multiple bilateral infiltrates. Labs show a leukocyte
count is 19,700/µL with 16% band forms. Hgb 10.8 g/dl, BUN 34 mg/dL, serum
creatinine level is 1.8 mg/dL. Which of the following tests is most likely to establish the
diagnosis?

(Select 1)( 1pts extra credit)

Computed tomography (CT) angiography of the chest with contrast

CT of the neck with contrast

Soft Tissue Neck xray

Transthoracic echocardiography
The answer is B, The patient should undergo computed tomography (CT) of the neck
with contrast. She has fever, leukocytosis, sore throat, unilateral neck tenderness, and
multiple densities on chest radiograph, suggestive of septic emboli. The combination of
these factors points strongly toward Lemierre syndrome, which is septic thrombosis of
the internal jugular vein. The diagnosis should be suspected in anyone with pharyngitis,
persistent fever, neck pain, and septic pulmonary emboli. CT of the affected vessel with
contrast would confirm the diagnosis. Treatment includes intravenous antibiotics that
cover streptococci, anaerobes, and β-lactamase-producing organisms. Penicillin with a β-
lactamase inhibitor and carbapenem are both reasonable choices (eg, ampicillin-
sulbactam, piperacillin-tazobactam, ticarcillin-clavulanate).
Chest CT would better characterize the pulmonary infiltrates, but this information would
not provide specific diagnostic information that would guide therapy.
Soft tissue radiography of the neck cannot detect jugular vein filling defects or
thromboses, which are diagnostic of septic thrombophlebitis.
Echocardiography would be helpful to exclude right-sided endocarditis as a cause of
septic emboli. However, there is nothing in the history or on cardiac examination to
suggest a cardiac source of septic emboli. Centor RM, Samlowski R. Avoiding sore
throat morbidity and mortality: when is it not “just a sore throat?” Am Fam Physician.
2011;83:26, 28. PMID: 21888123
Feedback on your answer
Collapse
5)
0/1
You are evaluating a 56-year old female for a suspicious breast mass in her right breast.
Her last mammogram 2 years ago was normal and you have sent her for another
mammogram. She denies family history of cancer and has no current medical problems.
Which of the following calcification patterns is more suspicious for a ductal carcinoma in
situ?

(Select 1)( 1pts extra credit)

Linear calcifications

Round calcifications

Popcorn calcification pattern

Egg shell calcification pattern

The answer is A, linear calcifications. The histologic hallmark of DCIS is the presence of
malignant-appearing cells confined to the lumen of the ductal system in the breast. DCIS
is most often diagnosed because of the presence of calcifications clustered in one area of
the breast. The calcifications suspicious for DCIS are often linear (not round), growing in
a ductal distribution. The DCIS may not be confined to the extent of calcifications, and
not all DCIS is manifested by calcification. Therefore, the size or extent of DCIS may not
be defined by the mammographic appearance. Although DCIS occasionally presents as a
mass, the presence of the mass even in the setting of a biopsy showing DCIS most often
signifies that the cancer is primarily invasive. Read more on Breast Cancer
 Popcorn calcification in the breast is the classical
description for the calcification seen in involuting
fibroadenomas, which as the name suggests has a pop
corn like appearance.
 Egg shell calcifications in the breast are benign peripheral
rim like calcifications
 Read More on Breast Calcification patterns
Feedback on your answer
Collapse
Unit 2
6)
0/1
Nonoperative Management of solid-organ injuries can be pursued in hemodynamically
stable patients who do not have overt peritonitis or other indications for laparotomy.
According to contemporary data, what percentage of patients with splenic injuries are
candidates for nonoperative management?
(Select 1)(1pts)

<10%

25%

45%

60%

The answer is D, 60%. NOM of solid-organ injuries can be pursued in hemodynamically


stable patients who do not have overt peritonitis or other indications for laparotomy.5-9
Contemporary data suggest that this will be approximately 60% of the overall splenic
injury population.6 Failure of NOM in this selected population has been reported at only
10 to 20%. It is important to understand that this rate represents an average and may vary
widely across differing grades of injury and across varying patient populations. For
example, the NOM failure rate reported in the Eastern Association for the Surgery of
Trauma multiinstitutional study from 2000 was 11%.10 However, those with highgrade
injuries (III, IV, V) had a NOM failure rate of 20%, 33%, and 75%, respectively. In
addition to increasing grade, contrast extravasation, older age, and high injury severity
score (> 25) have been linked to higher risk of failure of NOM.11-16 NOM may be time
and resource intensive and is not safe without the supporting resources such as a setting
and staff for adequate observation, rapid operating room mobilization if needed,
immediately available anesthesia, and an adequate blood bank. If such resources are
unavailable, patients should be transferred to a higher level of care or early operation
should be considered.
The role of angioembolization of the injured spleen continues to evolve. The
identification of contrast extravasation within the splenic parenchyma (pseudoaneurysm)
as a risk factor for failure of NOM led to liberal use of angioembolization and improved
rates of splenic salvage in this population [see Figure 1 and Figure 2].17 Contrast
extravasation into the peritoneal cavity outside the splenic parenchyma is also commonly
cited as an indication for angiography. However, the use of angioembolization in such
patients has met with less success.18 In our experience, such patients rarely remain stable
for long and often require laparotomy if angiography cannot be performed rapidly
In addition to use in contrast blush, angiography is increasingly used in all higher-grade
spleen injuries undergoing NOM. Multiple institutions have addressed this concept, and a
meta-analysis published in 2011 and a subsequent prospective study in 2014 highlighted
the higher splenic salvage rate during NOM when angioembolization is employed in all
grade IV and V injuries.19,20 This is echoed in a Level 2 recommendation in the Eastern
Association for the Surgery of Trauma practice management guidelines from 2012.10
Subcapsular hematomas also may be at higher risk for delayed rupture, and special
consideration should be given to patients with this injury pattern. Also currently
controversial is the superiority of proximal embolization versus distal embolization.18,21
With such emphasis placed on reporting techniques for increasing NOM success and
splenic salvage, it is important to keep in mind the role of early, rapid operation.
Approximately 20 to 30% of patients with splenic trauma deserve early splenectomy, and
failure of NOM often represents poor patient selection.22,23 In adults, indications for
prompt laparotomy include hemodynamic instability and initiation of blood transfusion
within the first 12 hours considered to be secondary to the splenic injury. In the pediatric
population, blood transfusions up to half of the patient's blood volume are used prior to
operative intervention. Following the first 12 postinjury hours, indications for laparotomy
are not as black and white. Determination of the patient's age, comorbidities, current
physiology, degree of anemia, and associated injuries will determine the use of
transfusion alone versus intervention with either embolization or operation. Delayed
hemorrhage or rupture of the spleen can occur up to weeks following injury, but the
majority who fail do so while still hospitalized.24 Overall, nonoperative treatment
obviates laparotomy in more than 90% of appropriately selected cases. Read more
Feedback on your answer
Collapse
7)
0/1
You are seeing a 42-year-old female following in the trauma bay following a high-speed
motor vehicle collision. She was was a restrained passenger and her car flipped over after
she lost control. She had to be cut out of the car and is complaining of shortness of breath
and upper abdominal pain.
She is awake and alert and her GCS is 15. Her pulse is 120/min, respirations are 28/min,
and blood pressure is 80/40 mm Hg. Breath sounds are decreased at the left lung base. An
x-ray of the chest shows opacification of the left lower lung field. You decide to place an
emergent chest tube that yields a small amount of air followed by greenish fluid. Which
of the following is the most appropriate next step in management?
(Select 1)(1pts)

CT scan of the abdomen and pelvis

CT scan of the chest

Thoracoscopy

Laparotomy

The answer is D. This patient suffered a ruptured diaphragm and chest tube placement
appears to have ruptured the bowel as evidence from the greenish fluid that was returned.
She needs an emergent laparotomy to control bleeding and repair her diaphragm. Read
More
Feedback on your answer
Collapse
8)
0/1
What is the reported mortality rate of isolated traumatic splenic injuries?
(Select 1)(1pts)

<1%

5-10-%

10-20%

20-30%

The answer is A, The spleen is one of the most commonly injured abdominal organs in
blunt trauma patients. Historical studies have reported 10% mortality with all splenic
injuries; however, isolated splenic injury mortality is less than 1%. The mechanisms of
injury are similar to those seen with liver injuries: motor vehicle collisions, automobile-
pedestrian collisions, falls, and any type of penetrating injury. Stab wounds to the
abdomen are less likely to cause spleen injury compared with liver injury due to the
spleen's protected location. Stab wounds to the abdomen are less likely to cause injury to
the spleen than to the liver, due to its protected location.
Until the 1970s, splenectomy was considered mandatory for all splenic injuries.
Recognition of the immune function of the spleen refocused efforts on splenic salvage in
the 1980s.1,2 Following success in pediatric patients, nonoperative management (NOM)
of splenic injuries was adopted in the adult population and has become the prevailing
strategy for blunt splenic trauma. Read more
Feedback on your answer
Collapse
9)
0/1
In cases of blunt trauma to the diaphragm, the injury is on the ____ side __% of the time?
(Select 1)(1pts)

Right; 90%

Left; 75%

Left; 25%

Right; 10%

The answer is B, left;75%. In cases of blunt trauma to the diaphragm, the injury is on the
left side 75% of the time, presumably because the liver diffuses some of the energy on
the right side. Blunt diaphragmatic injuries result in a linear tear in the central tendon,
whereas penetrating injuries are variable in size and location depending on the weapon. It
is important to identify the trajectory of penetrating injuries to determine the likelihood of
diaphragm injuries. With blunt and occasionally with penetrating injuries, the diagnosis is
suggested by an abnormality of the diaphragmatic shadow on a chest radiograph [see
Figure 5]. In patients without clear imaging results in the trauma bay, a CT scan may
identify a diaphragmatic injury.
Regardless of the etiology, acute injuries are repaired through an abdominal incision.
Thoracoscopy or laparoscopy may be used if concomitant injuries requiring laparotomy
have been ruled out. Following delineation of the injury, the chest should be evacuated of
all blood and particulate matter, and tube thoracostomy should be placed if not previously
done. Using Allis clamps to approximate the diaphragmatic edges, the defect can be
closed with a running permanent suture [see Figure 6]. Occasionally, large avulsions or
shotgun wounds with extensive tissue loss will require mesh to bridge the defect.
Alternatively, transposition of the diaphragm cephalad one to two intercostal spaces may
allow repair without undue tension. Read more
Feedback on your answer
Collapse
Unit 3
10)
0/1
You are seeing a 34-year old female in the trauma following an MVC. He is complaining
intense diffuse abdominal pain after his car was hit from behind causing him to spin out
and crash into a wall. Initially he had a GCS of 15. His vitals showed a BP of 125/76, HR
95, RR 18, O2 of 98% on room air. On physical exam his lungs are clear and his
abdomen shows diffuse tenderness to light palpation and ecchymosis over his
epigastrium. His initial fast exam is negative but technically limited and you order a CT
scan with IV contrast of his abdomen and pelvis as you suspect a possible liver
injury. What description from the options below accurately describes a grade IV liver
laceration on CT imaging?
(Select 1)(1pts)

Parenchymal distruption involving 25 - 75% hepatic lobes

Parenchymal distruption involving >75% helpatic lobe

Intraparenchymal laceration < 10cm diameter

Sub capsular hematoma, > 50% surface area, or ruptured with active
bleeding

The answer is A. Parenchymal distruption involving 25 - 75% hepatic lobes. Read


more
The AAST Liver injury grading system is as follows
 grade I :
 haematoma: sub capsular, < 10% surface area
 laceration: capsular tear, < 1cm depth
 grade II :
 haematoma: sub capsular, 10 - 50% surface area
 haematoma: intraparenchymal < 10cm diameter
 laceration: capsular tear, 1 - 3cm depth, < 10cm length
 grade III :
 haematoma: sub capsular, > 50% surface area, or ruptured with
active bleeding
 haematoma: intraparenchymal > 10 cm diameter
 laceration: capsular tear, > 3 cm depth
 grade IV :
 haematoma: ruptured intraparenchymal with active bleeding
 laceration: parenchymal distruption involving 25 - 75% hepatic
lobes or
 1 - 3 Couinaud segments (within one lobe)
 grade V :
laceration: parenchymal distruption involving >75% helpatic
lobe or
 > 3 Couinaud segments (within one lobe)
 vascular: juxtahepatic venous injuries (IVC, major hepatic vein)
 grade VI : vascular: hepatic avulsion

Feedback on your answer


Collapse
11)
1/1
According to contemporary data, what percentage of blunt hepatic injuries may be
managed nonoperatively?
(Select 1)(1pts)

<10%

25%

50%

>80%

The answer is D, > 80%. Over 80% of patients with liver injuries may be managed
nonoperatively. The liver is the most commonly injured solid organ in blunt trauma,
comprising 5% of all trauma admissions, and because of its size is frequently involved in
penetrating trauma. Following blunt trauma, the most commonly injured structures are
the parenchyma and hepatic veins. Blunt forces dissipate along segments of the liver and
along the fibrous coverings of the portal triad structures; the hepatic veins, however, are
not as resilient. Stab wounds typically result in direct linear tears, whereas gunshot
wounds or shotgun wounds result in significant cavitary injuries attributable to blast
effect and the "tumbling" of the missile within the liver parenchyma. Thus, arterial injury
is more common with penetrating trauma. Biliary trauma is more common with
central/hilar trauma, either blunt or penetrating.
Nonoperative management (NOM) of liver injuries is now the prevailing therapeutic
strategy for blunt hepatic trauma and can be employed for isolated right subcostal
penetrating wounds. Several concurrent changes resulted in this paradigm change. Over
80% of patients with liver injuries may be managed nonoperatively. One of the early
studies to test the application of NOM in 1995 supported its broad application, with an
overall success rate greater than 85% in hemodynamically stable patients, despite
substantial hemoperitoneum documented by CT.11 Of the 8% of patients who failed
NOM, half required operation as a result of associated injuries (i.e., enteric or pancreatic
injuries), whereas half underwent laparotomy for hepatic-related hemorrhage. Patients
who require intervention for hemorrhage typically fail NOM in the first 24 to 48
hours.2,11,17 Patients who fail NOM due to associated enteric or pancreatic injury have a
more variable time frame to presentation17; half manifested symptoms within 48 hours,
with the remainder becoming symptomatic up to 3 weeks later. Perhaps not surprisingly,
those patients who failed NOM had failure rates associated with increasing grades of
hepatic injury, with grade V injuries having a greater than 20% failure rate. Subsequent
studies have reported failure rates of 14% in grade IV injuries and 23% in grade V
injuries.12 The most recent analysis of the National Trauma Data Bank of severe blunt
liver injuries (grade IV and V) identified that initial NOM occurred in 73%, with a failure
rate of 7%.16 Interestingly, failure of NOM was associated with higher mortality.
Predictors of failure of NOM included increasing age, male sex, increasing Injury
Severity Score, decreasing Glasgow Coma Scale score, and hypotension. A similar study
of high-grade liver injuries identified a similar pattern with NOM initiated in 66%
patients with a failure rate of 9%.18 The amount of hemoperitoneum evident on a CT
scan appears to correlate with successful management; patients with a large amount of
hemoperitoneum (i.e., blood extending into the pelvis) are more likely to fail NOM.
However, predicting which patients will ultimately require laparotomy has yet to be
accomplished. Read more
Feedback on your answer
Collapse
12)
0/1
Roughly what percentage of patients with blunt liver injury initially managed non-
operatively will go on to need surgical intervention?
(1pts)

<10%

25%

50%

75%

The answer is A, During the last century, the management of blunt force trauma to the
liver has changed from observation and expectant management in the early part of the
1900s to mainly operative intervention, to the current practice of selective operative and
nonoperative management. A 2008 study by Tinkoff et al.4 showed that 86.3% of hepatic
injuries are now managed without operative intervention. The current reported success
rate of nonoperative management of hepatic trauma ranges from 82% to 100%. Most
blunt liver trauma (80% in adults, 97% in children) patients are currently treated
conservatively. The success of non-operative management depends upon proper selection
of the patient. The patients, who are managed non-operatively, usually have grade I and
II liver injuries, hemoperitoneum less than 900 ml and blood transfusion of less than 3
units. The contraindications to non-operative management include refractory
hypotension, signinficant fall in haematocrit, the extravasations of intravenous contrast
agent, expanding haematoma and grade IV and V liver injury on CECT abdomen. The
patients of grade III liver injuries need very close observation as they may require
surgical intervention during first 24 hours. The failure rate of non-operative management
is not more than 5% inmost studies. It seems that patients with grade VI injuries rarely
reaches to the hospital alive and are not salvageable. Therefore, such injuries are usually
documented on autopsy. Mortality from blunt hepatic trauma is about 5% and is related
to uncontrolled hemorrhage.
Interventional radiology may be needed to perform an angiogram and embolization for
bleeding or to percutaneously drain an abscess or b iloma. An endoscopic retrograde
cholangiopancreatogram (ERCP) and stent placement may be required for biliary leak.
Even when such complications of the liver injury develop, only 15% require operative
intervention. Hepatic artery angiography with embolization is an important tool for the
stable patient with contrast extravasation who is being managed nonoperatively. It can
also be invaluable for the postoperative patient who has been stabilized by perihepatic
packing or who has rebled after an initial period of stability. Angioembolization has a
greater than 90% success rate in the control of bleeding with a low risk of rebleeding and
a reduction in required volume of transfusion. Read More
Feedback on your answer
Collapse
13)
0/1
The liver is the most commonly injured solid organ in blunt trauma, comprising 5% of all
trauma admissions. With that being said, over 80% of patients with liver injuries may be
managed nonoperatively. Which of the following is not a predictor of nonoperative
management?
(1pts)

Female sex

Increasing age

Decreasing Glasgow Coma Scale score

Hypotension

Increasing Injury Severity Score

The answer is A, female sex. Over 80% of patients with liver injuries may be managed
nonoperatively. One of the early studies to test the application of NOM in 1995 supported
its broad application, with an overall success rate greater than 85% in hemodynamically
stable patients, despite substantial hemoperitoneum documented by CT.11 Of the 8% of
patients who failed NOM, half required operation as a result of associated injuries (i.e.,
enteric or pancreatic injuries), whereas half underwent laparotomy for hepatic-related
hemorrhage. Patients who require intervention for hemorrhage typically fail NOM in the
first 24 to 48 hours.2,11,17 Patients who fail NOM due to associated enteric or pancreatic
injury have a more variable time frame to presentation17; half manifested symptoms
within 48 hours, with the remainder becoming symptomatic up to 3 weeks later. Perhaps
not surprisingly, those patients who failed NOM had failure rates associated with
increasing grades of hepatic injury, with grade V injuries having a greater than 20%
failure rate. Subsequent studies have reported failure rates of 14% in grade IV injuries
and 23% in grade V injuries.12 The most recent analysis of the National Trauma Data
Bank of severe blunt liver injuries (grade IV and V) identified that initial NOM occurred
in 73%, with a failure rate of 7%.16 Interestingly, failure of NOM was associated with
higher mortality. Predictors of failure of NOM included increasing age, male sex,
increasing Injury Severity Score, decreasing Glasgow Coma Scale score, and
hypotension. A similar study of high-grade liver injuries identified a similar pattern with
NOM initiated in 66% patients with a failure rate of 9%.18 The amount of
hemoperitoneum evident on a CT scan appears to correlate with successful management;
patients with a large amount of hemoperitoneum (i.e., blood extending into the pelvis) are
more likely to fail NOM. However, predicting which patients will ultimately require
laparotomy has yet to be accomplished. Read more
Feedback on your answer
Collapse
Unit 4
14)
0/1
A 25 -ear obese black female presents to you with a painful breast lump along with
tenderness in both her breasts. She notes recurrent pain and multiple lumps on both
breasts that seem to “come and go” at different times during her menses. Now she has a
firm, round, mass that has not gone away for the last 5 weeks. What is the most likely
diagnosis?

(Select 1)(1pts)

Cystosarcoma Phyllodes

Intraductal papilloma

Fibrocystic disease

Fibroadenoma

The answer is C. This patient has fibrocystic disease which is a nonproliferative epithelial
lesions that are generally not associated with an increased risk of breast cancer [1]. It
should be noted that terms such as fibrocystic changes, fibrocystic disease, chronic cystic
mastitis, and mammary dysplasia refer to nonproliferative lesions and are not useful
clinically, as they encompass a heterogeneous group of diagnoses [5,11]. The most
common nonproliferative breast lesions are breast cysts. Other nonproliferative lesions
include papillary apocrine change, epithelial-related calcifications, and mild hyperplasia
of the usual type [5]. Apocrine metaplasia (also referred to as a "benign epithelial
alteration") is also a nonproliferative change that is secondary to some form of irritation,
typically associated with a breast cyst. Read More
Feedback on your answer
Collapse
15)
0/1
You are seeing a 24-year old female who is 4-weeks post postpartum in your clinic. She
is complaining of bilateral nipple pain over the last two days along with a nipple rash.
She notes pain both with breastfeeding and even when she is not feeding. The pain is so
bad that she didn't breastfeed her baby this morning because of the pain. Physical exam
shows erythema to both breasts along with cracking around both areolas. There are no
signs of an abscess and no induration present. What is the most likely cause of this
patient's symptoms?
(Select 1)(1pts)

Engorgement

Mastitis

Deep space abscess

Candida infection

None of the above

The answer is D, Candida infection. In breastfeeding women, bilateral nipple pain with
and between feedings after initial soreness has resolved is usually due to Candida. Pain
from engorgement typically resolves after feeding. Mastitis is usually unilateral and is
associated with systemic symptoms and wedge-shaped erythema of the breast tissue.
Improper latch-on is painful only during feedings. Eczema isolated to the nipple, while a
reasonable part of the differential, would be much more unusual.

Ref: Jatoi I, Kaufman M (eds): Management of Breast Diseases. Springer, 2010,


pp 95-96.
Feedback on your answer
Collapse
16)
1/1
You are seeing a 25-year old white female who is presenting to your clinic with bloody
nipple discharge over the last 3 days. She denies any history of a breast lump, she is not
breast feeding and denies family history of breast cancer. Her exam shows no palpable
masses but there is scant bloody discharge coming from her left nipple. You order a
mammogram that does not show any suspicious lesions. What is the most likely
diagnosis?
(Select 1)(1pts)

Fibroadenoma

Intraductal papilloma

Cystosarcoma Phyllodes

Mammary dysplasia

The answer is B, Intraductal papilloma. The old concern over cancer is the issue, and the
way to detect cancer that is not palpable is with a mammogram. That should be the first
choice. If negative, one may still wish to find an resect the intraductal papilloma to
provide symptomatic relief. Intraductal papillomas consist of a monotonous array of
papillary cells that grow from the wall of a cyst into its lumen. Although they are not
concerning in and of themselves, they can harbor areas of atypia or ductal carcinoma in
situ (DCIS). Papillomas can occur as solitary or multiple lesions. The standard approach
to a papilloma diagnosed by core needle biopsy (CNB) is to perform a surgical excision,
particularly if atypical cells are identified [14,16-21]. In a meta-analysis of 34 studies
that included 2236 non-malignant breast papillary lesions, 346 (15.7 percent) were
upgraded to malignancy following a surgical excision [21]. Because of a risk of
malignancy, these require surgical excision.Read More
Feedback on your answer
Collapse
17)
0/1
You are seeing a 15-year-old African-American female in your office because she and
her mom are concerned about a non tender breast lump that she just noticed the other day.
On exam you note a rubbery, well-defined, nontender breast mass approximately 2 cm in
diameter. The patient denies any history of breast tenderness, nipple discharge, or skin
changes. What is most likely diagnosis?

(Select 1)(1pts)

Fibrocystic breast disease

Fibroadenoma

Benign breast cyst


Cystosarcoma phyllodes

The answer is B, Fibroadenoma. Most breast masses in adolescent girls are benign.
Fibroadenoma is the most common, accounting for approximately two-thirds of all
adolescent breast masses. It is characterized by a slow growing, nontender, rubbery, well-
defined mass, most commonly located in the upper, outer quadrant. Size varies, and is
most commonly in the range of 2–3 cm. Fibrocystic disease is found in older adolescents
and is characterized by bilateral nodularity and cyclic tenderness. Benign breast cysts are
characterized by a spongy, tender mass with symptoms exacerbated by menses. Cysts are
frequently multiple, and spontaneous regression occurs in 50% of patients. Cystosarcoma
phyllodes is a rare tumor with malignant potential, although most are benign. It presents
as a firm, rubbery mass that may enlarge rapidly. Skin necrosis is usually associated with
the tumor. Intraductal papillomas are usually benign but do have malignant potential.
They are commonly subareolar and are associated with nipple discharge. These tumors
are rare in the adolescent population. Ref: Hay WH (ed): Current Pediatric Diagnosis and
Treatment, ed 16. McGraw-Hill, 2003, pp 122-123.

Feedback on your answer


Collapse
Unit 5
18)
0/1
You are seeing a 43-year old Hispanic female who is presenting with an enlarging mass
in her right breast. It has been present for several years and growing to it’s present size.
On examination of her right breast you note a large 10 cm mass that is mobile, firm
and rubbery. There are no palpable axillary nodes. What is the most likely diagnosis?

(Select 1)(1pts)

Fibroadenoma

Fibrocystic disease

Intraductal papilloma.

Cystosarcoma Phyllodes

The answer is D. This is a Phyllodes tumors, which are an uncommon fibroepithelial


breast tumors that are capable of a diverse range of biologic behavior. In their least
aggressive form, they behave similarly to benign fibroadenomas, although with a
propensity to recur locally following excision without wide margins. At the other end of
the spectrum are tumors that metastasize distantly, sometimes degenerating histologically
into sarcomatous lesions that lack an epithelial component. Phyllodes tumors account for
fewer than 0.5 percent of all breast malignancies. The vast majority occur in women, in
whom the median age at presentation is 42 to 45 (range 10 to 82 years).and there was a
higher incidence in Latina whites, as compared to non-Latina whites, Asians, and African
American women. Read More
Feedback on your answer
Collapse
19)
0/1
Which of the following is considered a benign lesion that usually does not require cancer
treatment?
(Select 1)(1pts)

Lobular carcinoma in situ

Ductal carcinoma in situ

Medullary carcinoma

Mucinous carcinoma

The answer is A, Cancer that is confined to the lumen of the duct or


lobule of the breast and has not penetrated the basement membrane
is termed in situ cancer. This generally refers to ductal carcinoma in
situ (DCIS) but also encompasses a benign entity called lobular
carcinoma in situ (LCIS). Small uniform cells confined to the lobule of
the breast characterize LCIS. It is generally a clinically and
mammographically occult lesion that is identified only incidentally
when a biopsy is performed for calcifications or a mass that proves to
be some other benign lesion. LCIS is actually not cancer but rather is a
benign lesion and does not require cancer treatment. The primary
issue with LCIS is that it conveys an increased lifelong risk of
subsequent invasive cancer quantified at 0.5 to 0.75% per year. In
addition, when LCIS is identified on a core-needle biopsy, there is a 10
to 20% chance of DCIS or invasive cancer in the surrounding tissue;
therefore, surgical excision is warranted. Long-term follow-up shows
that the large majority of women with LCIS never develop invasive
breast cancer. Therefore, ablative surgical therapy and radiation for
LCIS are not necessary. Previously, LCIS was considered in and of
itself an indication to consider bilateral mastectomy. However,
mastectomy is generally not indicated in women with LCIS and should
be performed only in the context of risk reduction for those at very
high risk related to factors such as inherited susceptibility. Because
women with a diagnosis of LCIS are at increased risk for subsequent
invasive cancer, they should be counseled regarding that risk and may
benefit from consultation with genetics professionals if they have a
family history of breast or ovarian cancer. Women with a biopsy
showing LCIS may also consider risk-reducing chemoprevention with
one of the selective estrogen receptor modulators (SERM's), tamoxifen
or raloxifene. These reduce the risk of subsequent invasive cancer by
about 50%, with an acceptable toxicity profile.20,21 Raloxifene is the
preferred agent in postmenopausal women. Read more on Breast
Cancer
Feedback on your answer
Collapse
20)
0/1
You are seeing a 75-year old female in clinic because of an enlarging breast mass the
last two months. On examination of her right breast you palpate a firm 3cm non tender
firm non-mobile mass lateral to her right nipple. She denies past medical history and
takes no medication. What is the next best step?
(Select 1)(1pts)

Mammogram

Breast Ultrasound

Core Biopsy

Excisional Biopsy

If a palpable mass is appreciated, bilateral diagnostic mammography should be performed


prior to biopsy, even if the mass is clinically suspicious for cancer (algorithm
3 and algorithm 4) [27]. Fine needle aspiration or core needle biopsy may alter both
mammographic or US appearance. The goal of imaging in this setting is not to establish a
diagnosis of cancer but rather to identify other suspicious areas or calcifications in either
breast that might impact treatment. This evaluation should be performed prior to
percutaneous biopsy, so that biopsy of additional suspicious lesions can be undertaken at
one time. If the lesion is large and breast conservation is not an option, this is less
important. read more
Feedback on your answer
Collapse
21)
0/1
True or False: When approaching a patient with an abnormal mammogram for a
suspicious breast mass the next best step is a surgical excision is preferred over
percutaneous needle biopsy.
(1pts)
True Incorrect

False

False: Breast lesion suspicious for malignancy requires tissue biopsy.


Percutaneous needle biopsy is preferred over surgical excision in all
circumstances. Surgical excision as a diagnostic procedure is not a
justifiable alternative simply because of "patient choice" and should be
performed only when needle biopsy cannot be performed for specific
technical reasons, when a needle biopsy is either nondiagnostic, the
result is not concordant with the imaging findings (i.e., the needle
biopsy is benign, but the lesion is of high suspicion), or in highly select
other cases. Technical reasons that may preclude needle biopsy
include anatomic location of the lesion on mammography directly
opposed to the chest wall or in the far periphery of the breast so that
it cannot be visualized on stereotactic imaging devices. Breast Cancer
Feedback on your answer
Collapse
Unit 6
22)
1/1
You are seeing a 30-year-old male wrestler who is presenting with a painful lesion on his
upper back. He first noted a small painful area a few days ago, and the lesion has since
enlarged and became more red. He notes that others on his wrestling team have similar
lesions and have sought care elsewhere. He has no past medical history and takes no
medications.
Physical Examination of the upper back reveals a 1 × 1 cm erythematous, raised pustule
that is tender to palpation, with a 4 × 4 cm area of surrounding erythema. The remainder
of the physical examination is normal. You elect to perform an incision and drainage, and
culture. Of the following, which is the most appropriate empiric treatment?
(Select 1)(1pts)

Levofloxacin

Doxycycline

Dicloxacillin

Cephalexin

Ciprofloxacin

All of the above


None of the above

The answer is B, Purulence is more often present in staphylococcal cellulitis. Specimens


from wounds, carbuncles, or furuncles may reveal gram-positive cocci in clusters,
consistent with S. aureus. Clinicians must judge whether MRSA or MSSA is likely.
Appropriate choices for the oral treatment of MSSA include cloxacillin; dicloxacillin;
cephalexin; and, for penicillin-allergic patients, clindamycin or a macrolide
(erythromycin, clarithromycin, or azithromycin). However, patients treated with
cephalexin should be observed for treatment failure, as some studies have reported high
failure rates in adults (29), possibly due to poor absorption (30). Appropriate orally
administered choices for the treatment of MRSA include trimethoprim–
sulfamethoxazole, doxycycline, minocycline, or linezolid. This patient likely has a
community acquired MRSA as others on his wrestling team have similar lesions and thus
MRSA targeted therapy is best. Read More on Cellulitis and Soft-Tissue
Infections
Feedback on your answer
Collapse
23)
0/1
Which of the following choices best differentiates cellulitis from erysipelas?
(Select 1)(1pts)

Presence of fever

Presence of leukocytosis

Depth of inflammation

Presence of purulence

All of the above

None of the above

The answer is C, depth of inflammation. Cellulitis and erysipelas are diffuse spreading
skin infections that not associated with underlying suppurative foci. Clinically, there is
often some degree of overlap between the two different entities. Erysipelas is
differentiated from cellulitis by the depth of inflammation; erysipelas affects the upper
dermis, including the superficial lymphatics, whereas cellulitis affects the deeper dermis
and subcutaneous fat (Mayo Clin Proc, Vol. 89;1436). Read More onCellulitis and
erysipelas from Up to Date
Feedback on your answer
Collapse
24)
0/1
A 23-year old male patient presents with a human bite to his right hand after an
altercation just prior to arrival. He has no past medical history and is up to date with
tetanus. Exam shows there are two small puncture wounds overlying the dorsal aspect of
his right hand over his 1stmetacarpal and does not seem to involve any
tendons or ligaments. The wound does not appear amenable to sutures for
closure and you decide to copiously irrigate it with saline and discharge him home. What
antibiotic should he receive for wound care prophylaxis?
(1pts)

Clindamycin

Flagyl

Oxacillin

Augmentin

The answer is D. Augmentin. The most common bacterial etiology in human bites is strep
viridans. You are however covering for Eikenella corrodens which is a gram negative rod
which is susceptible to pcn but resistant to flagyl, clindamycin, first generation
cephalosporins and erythromycin. It is susceptible to flouroquinolones, bactrim and
augmentin. The most important treatment is good irrigation and initial wound
cleaning. Read more
Feedback on your answer
Collapse
25)
0/1
Which of the following statements regarding soft tissue infections is not true?
(Select 1)(1pts)

As many as 20% of necrotizing soft tissue infections are primary


(idiopathic) and occur in previously healthy patients who have no
predisposing factors and no known portal of entry for bacterial inoculation

Crepitus is noted in the majority of patients with necrotizing soft tissue


infections

Physical findings characteristic of a necrotizing infection include


tenderness beyond the area of erythema, crepitus and cutaneous
anesthesia

Fewer than 40% of patients with Patients with Necrotizing Soft Tissue
Infections exhibit the classic symptoms and signs described
The answer is B, crepitus is noted in only 30% of patients with necrotizing soft tissue
infections. Patients with necrotizing soft tissue infections often complain of severe pain
that is out of proportion to their physical findings. Compared with patients who have
nonnecrotizing infections, they are more likely to have fever, bullae, or blebs [see Figure
1]; signs of systemic toxicity; hyponatremia; and leukocytosis with a shift in immature
forms. Physical findings characteristic of a necrotizing infection include tenderness
beyond the area of erythema, crepitus, cutaneous anesthesia, and cellulitis that is
refractory to antibiotic therapy.6 Tenderness beyond the borders of the erythematous area
is an especially important clinical clue that develops as the infection in the deeper
cutaneous layers undermines the skin. Early in the course of a necrotizing soft tissue
infection, skin changes may be minimal despite extensive necrosis of the deeper
cutaneous layers. Bullae, blebs, cutaneous anesthesia, and skin necrosis occur as a result
of thrombosis of the nutrient vessels and destruction of the cutaneous nerves of the skin,
which typically occur late in the course of infection.
Clinicians should be mindful of certain diagnostic barriers that may delay recognition and
treatment of necrotizing soft tissue infections.7 In particular, these infections have a
variable clinical presentation. Although most patients present with an acute, rapidly
progressive illness and signs of systemic toxicity, a subset of patients may present with a
more indolent, slowly progressive infection. Patients with postoperative necrotizing
infections often have a more indolent course. Moreover, in the early stages, underlying
necrosis may be masked by normal-appearing overlying skin. As many as 20% of
necrotizing soft tissue infections are primary (idiopathic) and occur in previously healthy
patients who have no predisposing factors and no known portal of entry for bacterial
inoculation. Finally, crepitus is noted in only 30% of patients with necrotizing soft tissue
infections. Overall, fewer than 40% of patients exhibit the classic symptoms and signs
described.7,8Accordingly, it is imperative to maintain a high index of suspicion for this
disease in the appropriate setting.Read More on Soft Tissue Infection

Unit 1
1)
1/1
You are seeing a 31-year-old woman who presents with worsening sore throat, dry
cough, fever, and severe neck pain over the last week. She is otherwise healthy and takes
no medications.
Her vitals show a temperature is 102.5 °F, blood pressure is 99/68 mm Hg, pulse rate is
125/min, and respiration rate is 24/min. On exam she appears ill and her neck is tender to
palpation along the left side with overlying erythema, mild induration, without
lymphadenopathy. Her pharynx is erythematous, with tonsillar enlargement and no
exudates or ulcers. Her lungs are clear to auscultation. The remainder of the examination
is normal. Her chest x-ray shows multiple bilateral infiltrates. Labs show a leukocyte
count is 19,700/µL with 16% band forms. Hgb 10.8 g/dl, BUN 34 mg/dL, serum
creatinine level is 1.8 mg/dL. Which of the following tests is most likely to establish the
diagnosis?

(Select 1)( 1pts extra credit)


Computed tomography (CT) angiography of the chest with contrast

CT of the neck with contrast

Soft Tissue Neck xray

Transthoracic echocardiography

The answer is B, The patient should undergo computed tomography (CT) of the neck
with contrast. She has fever, leukocytosis, sore throat, unilateral neck tenderness, and
multiple densities on chest radiograph, suggestive of septic emboli. The combination of
these factors points strongly toward Lemierre syndrome, which is septic thrombosis of
the internal jugular vein. The diagnosis should be suspected in anyone with pharyngitis,
persistent fever, neck pain, and septic pulmonary emboli. CT of the affected vessel with
contrast would confirm the diagnosis. Treatment includes intravenous antibiotics that
cover streptococci, anaerobes, and β-lactamase-producing organisms. Penicillin with a β-
lactamase inhibitor and carbapenem are both reasonable choices (eg, ampicillin-
sulbactam, piperacillin-tazobactam, ticarcillin-clavulanate).
Chest CT would better characterize the pulmonary infiltrates, but this information would
not provide specific diagnostic information that would guide therapy.
Soft tissue radiography of the neck cannot detect jugular vein filling defects or
thromboses, which are diagnostic of septic thrombophlebitis.
Echocardiography would be helpful to exclude right-sided endocarditis as a cause of
septic emboli. However, there is nothing in the history or on cardiac examination to
suggest a cardiac source of septic emboli. Centor RM, Samlowski R. Avoiding sore
throat morbidity and mortality: when is it not “just a sore throat?” Am Fam Physician.
2011;83:26, 28. PMID: 21888123
Feedback on your answer
Collapse
2)
0/1
What is the most reliable predictor of survival in patient's diagnosed with Breast Cancer?

(Select 1)( 1pts extra credit)

estrogen receptor status

cancer stage at the time of diagnosis

tumor grade
histologic type

The most reliable predictor of survival in breast cancer is the stage at the time of
diagnosis. Tumor size and lymph node involvement are the main factors to take into
account. Other prognostic parameters (tumor grade, histologic type, and lymphatic or
blood vessel involvement) have been proposed as important variables, but most
microscopic findings other than lymph node involvement correlate poorly with prognosis.
Estrogen receptor (ER) status may also predict survival, with ER-positive tumors
appearing to be less aggressive than ER-negative tumors. Ref: Abeloff MD, Armitage JO,
Niederhuber JE, et al (eds): Clinical Oncology, ed 3. Elsevier Churchill Livingstone,
2004, pp 2399-2401.
Feedback on your answer
Collapse
3)
0/1
You are seeing 45-year old black female again in the office who presented to you initially
for a right sided breast mass. She has no past medical history or family history of breast
cancer. On physical exam you palpated a 2 cm mass just lateral to her right nipple. You
ordered a mammogram which cam back normal. What percentage of patients who are
found to have a normal mammogram have breast cancer after biopsy of a suspicious
lesion?
(Select 1)( 1pts extra credit)

5%

15%

25%

45%

The answer is A. A diagnostic mammogram is the first imaging study performed for
a woman with a new, palpable breast mass, and should be performed even if a recent
mammogram was negative. While the false negative rate of mammograms is less than 5
percent for clinically palpable breast cancers [15], a normal mammogram does not
eliminate the need for further evaluation of a suspicious mass [5]. For women under age
30 years, the breasts are hypersensitive to radiation exposure [16]; however, if the
clinical findings are suspicious, a mammogram should be performed. Read More

Feedback on your answer


Collapse
4)
1/1
You are seeing a 48-year old female who presented with a breast lump in her right breast.
She denies pain or nipple discharge and denies family history of breast cancer. You order
a mammogram and are wondering if there is any benefit of ordering a breast ultrasound
as well.

True or False: The addition of breast ultrasound when evaluating breast


masses DOES NOTincrease the sensitivity in the evaluation of breast masses and thus
a mammography is sufficient.
( 1pts extra credit)

True

False Correct

The answer is false. The addition of ultrasound improves and helps make clinical
decisions. The sensitivity, specificity, positive and negative predictive
values for clinical examination plus mammography plus US were 96.9,
94.8, 39.2, and 99.9 percent, while the corresponding values for clinical
examination plus mammography were 91.5, 87, 19.7, and 99.7 percent,
respectivelyEvaluation of a palpable breast mass requires a systematic approach to the
history, physical examination, and radiographic imaging studies to ensure a correct
diagnosis. A missed diagnosis of breast cancer is one of the most frequent causes of
malpractice claims in the United States [1-3].
The clinical manifestations and diagnostic evaluation of women with a palpable breast
mass are reviewed here. Screening and epidemiology of breast cancer, benign breast
disease, breast pain, nipple discharge, breast cysts, and breast cancer are reviewed
separately.
Breast US is often added to the initial diagnostic evaluation for women with a suspected
breast cancer if there is a palpable mass or a density is seen on mammogram. The benefit
of this approach was suggested in a series of 2020 patients (470 with a palpable mass)
who underwent clinical exam, mammography, and breast US. The systematic addition of
breast US detected eight additional malignancies, and correctly downgraded 332 cases of
suspected malignancy to no suspected malignancy (predominantly cysts or
fibroadenoma). Thus, the main benefit of breast US was improved specificity when used
in a targeted manner. . From Up to Date
Feedback on your answer
Collapse
5)
0/1
You are evaluating a 56-year old female for a suspicious breast mass in her right breast.
Her last mammogram 2 years ago was normal and you have sent her for another
mammogram. She denies family history of cancer and has no current medical problems.
Which of the following calcification patterns is more suspicious for a ductal carcinoma in
situ?

(Select 1)( 1pts extra credit)


Linear calcifications

Round calcifications

Popcorn calcification pattern

Egg shell calcification pattern

The answer is A, linear calcifications. The histologic hallmark of DCIS is the presence of
malignant-appearing cells confined to the lumen of the ductal system in the breast. DCIS
is most often diagnosed because of the presence of calcifications clustered in one area of
the breast. The calcifications suspicious for DCIS are often linear (not round), growing in
a ductal distribution. The DCIS may not be confined to the extent of calcifications, and
not all DCIS is manifested by calcification. Therefore, the size or extent of DCIS may not
be defined by the mammographic appearance. Although DCIS occasionally presents as a
mass, the presence of the mass even in the setting of a biopsy showing DCIS most often
signifies that the cancer is primarily invasive. Read more on Breast Cancer
 Popcorn calcification in the breast is the classical
description for the calcification seen in involuting
fibroadenomas, which as the name suggests has a pop
corn like appearance.
 Egg shell calcifications in the breast are benign
peripheral rim like calcifications
 Read More on Breast Calcification patterns
Feedback on your answer
Collapse
Unit 2
6)
0/1
Nonoperative Management of solid-organ injuries can be pursued in hemodynamically
stable patients who do not have overt peritonitis or other indications for laparotomy.
According to contemporary data, what percentage of patients with splenic injuries are
candidates for nonoperative management?
(Select 1)(1pts)

<10%

25%

45%
60%

The answer is D, 60%. NOM of solid-organ injuries can be pursued in hemodynamically


stable patients who do not have overt peritonitis or other indications for laparotomy.5-9
Contemporary data suggest that this will be approximately 60% of the overall splenic
injury population.6 Failure of NOM in this selected population has been reported at only
10 to 20%. It is important to understand that this rate represents an average and may vary
widely across differing grades of injury and across varying patient populations. For
example, the NOM failure rate reported in the Eastern Association for the Surgery of
Trauma multiinstitutional study from 2000 was 11%.10 However, those with highgrade
injuries (III, IV, V) had a NOM failure rate of 20%, 33%, and 75%, respectively. In
addition to increasing grade, contrast extravasation, older age, and high injury severity
score (> 25) have been linked to higher risk of failure of NOM.11-16 NOM may be time
and resource intensive and is not safe without the supporting resources such as a setting
and staff for adequate observation, rapid operating room mobilization if needed,
immediately available anesthesia, and an adequate blood bank. If such resources are
unavailable, patients should be transferred to a higher level of care or early operation
should be considered.
The role of angioembolization of the injured spleen continues to evolve. The
identification of contrast extravasation within the splenic parenchyma (pseudoaneurysm)
as a risk factor for failure of NOM led to liberal use of angioembolization and improved
rates of splenic salvage in this population [see Figure 1 and Figure 2].17 Contrast
extravasation into the peritoneal cavity outside the splenic parenchyma is also commonly
cited as an indication for angiography. However, the use of angioembolization in such
patients has met with less success.18 In our experience, such patients rarely remain stable
for long and often require laparotomy if angiography cannot be performed rapidly
In addition to use in contrast blush, angiography is increasingly used in all higher-grade
spleen injuries undergoing NOM. Multiple institutions have addressed this concept, and a
meta-analysis published in 2011 and a subsequent prospective study in 2014 highlighted
the higher splenic salvage rate during NOM when angioembolization is employed in all
grade IV and V injuries.19,20 This is echoed in a Level 2 recommendation in the Eastern
Association for the Surgery of Trauma practice management guidelines from 2012.10
Subcapsular hematomas also may be at higher risk for delayed rupture, and special
consideration should be given to patients with this injury pattern. Also currently
controversial is the superiority of proximal embolization versus distal embolization.18,21
With such emphasis placed on reporting techniques for increasing NOM success and
splenic salvage, it is important to keep in mind the role of early, rapid operation.
Approximately 20 to 30% of patients with splenic trauma deserve early splenectomy, and
failure of NOM often represents poor patient selection.22,23 In adults, indications for
prompt laparotomy include hemodynamic instability and initiation of blood transfusion
within the first 12 hours considered to be secondary to the splenic injury. In the pediatric
population, blood transfusions up to half of the patient's blood volume are used prior to
operative intervention. Following the first 12 postinjury hours, indications for laparotomy
are not as black and white. Determination of the patient's age, comorbidities, current
physiology, degree of anemia, and associated injuries will determine the use of
transfusion alone versus intervention with either embolization or operation. Delayed
hemorrhage or rupture of the spleen can occur up to weeks following injury, but the
majority who fail do so while still hospitalized.24 Overall, nonoperative treatment
obviates laparotomy in more than 90% of appropriately selected cases. Read more
Feedback on your answer
Collapse
7)
0/1
You are seeing a 42-year-old female following in the trauma bay following a high-speed
motor vehicle collision. She was was a restrained passenger and her car flipped over after
she lost control. She had to be cut out of the car and is complaining of shortness of breath
and upper abdominal pain.
She is awake and alert and her GCS is 15. Her pulse is 120/min, respirations are 28/min,
and blood pressure is 80/40 mm Hg. Breath sounds are decreased at the left lung base. An
x-ray of the chest shows opacification of the left lower lung field. You decide to place an
emergent chest tube that yields a small amount of air followed by greenish fluid. Which
of the following is the most appropriate next step in management?
(Select 1)(1pts)

CT scan of the abdomen and pelvis

CT scan of the chest

Thoracoscopy

Laparotomy

The answer is D. This patient suffered a ruptured diaphragm and chest tube placement
appears to have ruptured the bowel as evidence from the greenish fluid that was returned.
She needs an emergent laparotomy to control bleeding and repair her diaphragm. Read
More
Feedback on your answer
Collapse
8)
0/1
You are seeing a 20-year-old male in hospital 12 hours after he was admitted for
observation following an MVC. Initially he was complaining of left sided chest wall pain
after his car was t-boned on the passenger side. His initial physical examination showed
bruises on the anterior chest wall and upper abdominal wall. X-rays revealed fractures
of his sixth and seventh ribs on the left (but no pneumothorax or pleural effusion). A
FAST Exam did not show any free intraperitoneal fluid. Currently, he complains of
worsening epigastric pain, left shoulder pain and mild nausea. His current vitals show his
blood pressure is now 95/60 and pulse rate 115 beats/min, and O2 saturation is 96% on
room air. Which of the following is the next best step in treatment?
(Select 1)(1pts)

Repeat ECG and cardiac biomarkers

Repeat PA and lateral chest x-ray

Abdominal CT scan with intravenous contrast

Transesophageal echocardiogram

This patient presents with blunt abdominal trauma with the delayed onset of hypertension
and signs and symptoms worrisome for likely splenic injury. Most common injuries are
to the spleen, liver, and less common injuries are to the hollow viscous organs in the
abdomen. Symptoms and signs suggesting splenic injury include left upper quadrant pain,
abdominal wall contusion, left lower chest wall tenderness, hypotension, and left
shoulder. Pain referred from splenic hemorrhage, hitting the phrenic nerve and diaphragm
(Kehr sign). The initial examination after blunt abdominal trauma can be unremarkable
and the symptoms can occur hours later, indicating ongoing splenic injury. The best
choice here would be an abdominal CT scan with intravenous contrast, only (no oral
contrast is needed because as little utility). This will define organ injury, assess for
presence of bleeding in all abdominal compartments, determine the need for surgery.
The spleen is the second most commonly injured abdominal organ in blunt trauma
patients. Historical studies have reported a 10% mortality with all splenic injuries;
however, isolated splenic injury mortality is less than 1%. The mechanisms of injury are
similar to those seen with liver injuries: motor vehicle collisions, autopedestrian
accidents, and falls. Similar to penetrating trauma to the liver, stab wounds to the spleen
typically result in direct linear tears, whereas gunshot wounds result in significant
cavitary injuries.
Until the 1970s, splenectomy was considered mandatory for all splenic injuries.
Recognition of the immune function of the spleen refocused efforts on splenic salvage in
the 1980s.38,39Following success in pediatric patients, NOM of splenic injuries was
adopted in the adult population and has become the prevailing strategy for blunt splenic
trauma.40

INITIAL EVALUATION AND INJURY GRADING

Addressing the patient's ABC's, examining the patient's abdomen, and performing
adjunctive imaging with FAST and CT are the initial steps of diagnosing a patient's
splenic injury. Hypotension with a positive FAST scan should prompt emergent
laparotomy. For patients with an identified blunt splenic injury on a CT scan, the injury
should be graded according to the AAST injury grading scale [see Table 1].3
Similar to liver injuries, the grade of splenic injury predicts failure rates and complication
rates of NOM. Other findings that should be searched for on a CT scan include contrast
extravasation (is the contrast blush contained within the spleen, or does it spill into the
peritoneum?), the amount of intra-abdominal hemorrhage (is it isolated to the splenic
fossa, or does blood extend into the pelvis?), and the presence of pseudoaneurysms.

NONOPERATIVE MANAGEMENT

Indications for Nonoperative Management

NOM of solid-organ injuries is pursued in hemodynamically stable patients who do not


have overt peritonitis or other indications for laparotomy.41-45 There is no age cutoff for
patients for the NOM of solid-organ injuries.46,47 High-grade injuries, a large amount of
hemoperitoneum, contrast extravasation, and pseudoaneurysms are not absolute
contraindications for NOM; however, these patients are at high risk for failure.48-51 The
identification of contrast extravasation as a risk factor for failure of NOM led to liberal
use of angioembolization. The true value of angioembolization in splenic salvage has not
been rigorously evaluated. Patients with intraparenchymal splenic blushes who are
otherwise asymptomatic may be considered for a period of observation rather than
empiric angioembolization52; it is thought that the contained hemorrhage within the
splenic capsule may result in tamponade of the bleeding

It is clear, however, that 20 to 30% of patients with splenic trauma deserve early
splenectomy and that failure of NOM often represents poor patient selection.53,54 In
adults, indications for prompt laparotomy include initiation of blood transfusion within
the first 12 hours considered to be secondary to the splenic injury or hemodynamic
instability. In the pediatric population, blood transfusions up to half of the patient's blood
volume are used prior to operative intervention. Following the first 12 postinjury hours,
indications for laparotomy are not as black and white. Determination of the patient's age,
comorbidities, current physiology, degree of anemia, and associated injuries will
determine the use of transfusion alone versus intervention with either embolization or
operation. Unlike hepatic injuries, which rebleed in 24 to 48 hours, delayed hemorrhage
or rupture of the spleen can occur up to weeks following injury. Algorithms for the
management of pediatric splenic injuries exist,55 and the patient's physiologic status is
the key determinant. Rapid mobilization in patients who are hemodynamically stable with
a stable hematocrit and no abdominal pain is generally successful. Overall, nonoperative
treatment obviates laparotomy in more than 90% of cases.

Follow-up Imaging

Out of concern over the risk of delayed hemorrhage or other complications, follow-up CT
scans have often been recommended; unfortunately, there is no consensus as to when or
even whether they should be obtained. Patients with grade I or II splenic injuries rarely
show progression of the lesion or other complications on routine follow-up CT scans; it is
reasonable to omit such scans if patients' hematocrits remain stable and they are
otherwise well. Patients with more extensive injuries often have a less predictable course,
and CT may be necessary to evaluate possible complications. Routine CT before
discharge, however, is unwarranted. Outpatient CT, however, in patients who participate
in vigorous or contact sports should be performed at 6 weeks to document complete
healing before resuming those activities. A more convenient and less expensive
alternative to follow-up CT is ultrasonographic monitoring of lesions.

OPERATIVE EXPOSURE AND HEMORRHAGE CONTROL

In penetrating abdominal injuries not suitable for NOM and in blunt abdominal injuries
when NOM is contraindicated or has failed, exploratory laparotomy is performed.

To ensure safe removal or repair, the spleen should be mobilized to the point where it can
be brought to the surface of the abdominal wall without tension. An incision is made in
the peritoneum and the endoabdominal fascia, beginning at the white line of Toldt along
the descending colon and continuing cephalad 1 to 2 cm lateral to the posterior peritoneal
reflection of the spleen; this plane of dissection is continued superiorly until the
esophagus is encountered [see

Figure 15a]. Posteriorly, blunt dissection is performed to mobilize the spleen and
pancreas as a composite away from Gerota fascia and up and out of the retroperitoneum;
this posterior plane may be extended to the aorta if necessary [seeFigure 15b].
Additionally, the attachments between the spleen and the splenic flexure of the colon may
be divided to avoid avulsion of the inferior splenic capsule. Care must be taken not to
pull on the spleen; otherwise, it will tear along the posterior peritoneal reflection, causing
significant hemorrhage. It is often helpful to rotate the operating table 20° to the patient's
right so that the weight of the abdominal viscera facilitates viscera retraction. Any
ongoing hemorrhage from the splenic injury may be temporarily controlled with digital
occlusion of the splenic hilar vessels. Once mobilization is complete, the spleen can be
repaired or removed without any need to struggle to achieve adequate exposure.

DEFINITIVE MANAGEMENT OF INJURIES

Splenic injuries are treated operatively by splenectomy, partial splenectomy, or splenic


repair (splenorrhaphy), based on the extent of the injury and the physiologic condition of
the patient. Splenectomy is indicated for hilar injuries, pulverized splenic parenchyma, or
any grade II or higher injury in a coagulopathic or multiply injured patient. We employ
autotransplantation of splenic implants [see

Figure 16] for partial immunocompetence in younger patients.56 Drains are not used.
Partial splenectomy can be employed in patients in whom only the superior or inferior
pole has been injured. Hemorrhage from the raw splenic edge is controlled with a
horizontal mattress suture, with gentle compression of the parenchyma [see

Figure 17]. Similar to hepatic injuries, splenorrhaphy techniques to achieve hemostasis


include topical agents (electrocautery, argon beam coagulation, thrombin-soaked gelatin
foam sponge, fibrin glue, BioGlue), enveloping the injured spleen in absorbable mesh,
and pledgeted suture repair.

POSTOPERATIVE CARE AND COMPLICATIONS

Enthusiasm for splenic salvage was driven by the rare but often fatal complication of
overwhelming postsplenectomy sepsis (OPSS). OPSS is caused by encapsulated
bacteria,Streptococcus pneumoniae, Haemophilus influenzae,
and Neisseria meningitidis, which are resistant to antimicrobial treatment. In
patients undergoing splenectomy, prevention against these bacteria is provided via
vaccines administered optimally at 14 days but definitely prior to hospital
discharge.57 Vaccines to be administered include Pneumovax (Merck & Co., Inc.,
Whitehouse Station, NJ), Menactra (Sanofi Pasteur, Swiftwater, PA), and Fluvirin
(Novartis, East Hanover, NJ). Revaccination remains open to debate, but some argue for
revaccination every 6 years.

An immediate postsplenectomy increase in platelets and white blood cells (WBCs) is


normal; however, beyond postoperative day 5, a WBC count above 15,000/μL and a
platelet to WBC ratio less than 20 are highly associated with sepsis and should prompt a
thorough search for underlying infection.58,59 A common infectious complication
following splenectomy is a subphrenic abscess, which should be managed with
percutaneous drainage. Following splenectomy or splenorrhaphy, postoperative
hemorrhage may be attributable to loosening of a tie around the splenic vessels, a missed
short gastric artery, or recurrent bleeding from the spleen if splenic repair was used.
Additional sources of morbidity include a concurrent but unrecognized iatrogenic injury
to the pancreatic tail during rapid splenectomy, resulting in pancreatic ascites or fistula.

2014. Scientific American Surgery. Hamilton, Ontario & Philadelphia, PA. Decker
Intellectual Properties Inc. ISSN 2368-2744. STAT!Ref Online Electronic Medical
Library. http://online.statref.com/Document.aspx?fxId=61&docId=2297. 10/17/2014
4:45:39 AM CDT (UTC -05:00).

Feedback on your answer


Collapse
9)
0/1
What is the reported mortality rate of isolated traumatic splenic injuries?
(Select 1)(1pts)

<1%

5-10-%

10-20%
20-30%

The answer is A, The spleen is one of the most commonly injured abdominal organs in
blunt trauma patients. Historical studies have reported 10% mortality with all splenic
injuries; however, isolated splenic injury mortality is less than 1%. The mechanisms of
injury are similar to those seen with liver injuries: motor vehicle collisions, automobile-
pedestrian collisions, falls, and any type of penetrating injury. Stab wounds to the
abdomen are less likely to cause spleen injury compared with liver injury due to the
spleen's protected location. Stab wounds to the abdomen are less likely to cause injury to
the spleen than to the liver, due to its protected location.
Until the 1970s, splenectomy was considered mandatory for all splenic injuries.
Recognition of the immune function of the spleen refocused efforts on splenic salvage in
the 1980s.1,2 Following success in pediatric patients, nonoperative management (NOM)
of splenic injuries was adopted in the adult population and has become the prevailing
strategy for blunt splenic trauma. Read more
Feedback on your answer
Collapse
Unit 3
10)
0/1
The liver is the most commonly injured solid organ in blunt trauma, comprising 5% of all
trauma admissions. With that being said, over 80% of patients with liver injuries may be
managed nonoperatively. Which of the following is not a predictor of nonoperative
management?
(1pts)

Female sex

Increasing age

Decreasing Glasgow Coma Scale score

Hypotension

Increasing Injury Severity Score

The answer is A, female sex. Over 80% of patients with liver injuries may be managed
nonoperatively. One of the early studies to test the application of NOM in 1995 supported
its broad application, with an overall success rate greater than 85% in hemodynamically
stable patients, despite substantial hemoperitoneum documented by CT.11 Of the 8% of
patients who failed NOM, half required operation as a result of associated injuries (i.e.,
enteric or pancreatic injuries), whereas half underwent laparotomy for hepatic-related
hemorrhage. Patients who require intervention for hemorrhage typically fail NOM in the
first 24 to 48 hours.2,11,17 Patients who fail NOM due to associated enteric or pancreatic
injury have a more variable time frame to presentation17; half manifested symptoms
within 48 hours, with the remainder becoming symptomatic up to 3 weeks later. Perhaps
not surprisingly, those patients who failed NOM had failure rates associated with
increasing grades of hepatic injury, with grade V injuries having a greater than 20%
failure rate. Subsequent studies have reported failure rates of 14% in grade IV injuries
and 23% in grade V injuries.12 The most recent analysis of the National Trauma Data
Bank of severe blunt liver injuries (grade IV and V) identified that initial NOM occurred
in 73%, with a failure rate of 7%.16 Interestingly, failure of NOM was associated with
higher mortality. Predictors of failure of NOM included increasing age, male sex,
increasing Injury Severity Score, decreasing Glasgow Coma Scale score, and
hypotension. A similar study of high-grade liver injuries identified a similar pattern with
NOM initiated in 66% patients with a failure rate of 9%.18 The amount of
hemoperitoneum evident on a CT scan appears to correlate with successful management;
patients with a large amount of hemoperitoneum (i.e., blood extending into the pelvis) are
more likely to fail NOM. However, predicting which patients will ultimately require
laparotomy has yet to be accomplished. Read more
Feedback on your answer
Collapse
11)
0/1
You are seeing a 34-year old female in the trauma following an MVC. He is complaining
intense diffuse abdominal pain after his car was hit from behind causing him to spin out
and crash into a wall. Initially he had a GCS of 15. His vitals showed a BP of 125/76, HR
95, RR 18, O2 of 98% on room air. On physical exam his lungs are clear and his
abdomen shows diffuse tenderness to light palpation and ecchymosis over his
epigastrium. His initial fast exam is negative but technically limited and you order a CT
scan with IV contrast of his abdomen and pelvis as you suspect a possible liver
injury. What description from the options below accurately describes a grade IV liver
laceration on CT imaging?
(Select 1)(1pts)

Parenchymal distruption involving 25 - 75% hepatic lobes

Parenchymal distruption involving >75% helpatic lobe

Intraparenchymal laceration < 10cm diameter

Sub capsular hematoma, > 50% surface area, or ruptured with active
bleeding

The answer is A. Parenchymal distruption involving 25 - 75% hepatic lobes. Read


more
The AAST Liver injury grading system is as follows
 grade I :
 haematoma: sub capsular, < 10% surface area
 laceration: capsular tear, < 1cm depth
 grade II :
 haematoma: sub capsular, 10 - 50% surface area
 haematoma: intraparenchymal < 10cm diameter
 laceration: capsular tear, 1 - 3cm depth, < 10cm length
 grade III :
 haematoma: sub capsular, > 50% surface area, or ruptured with
active bleeding
 haematoma: intraparenchymal > 10 cm diameter
 laceration: capsular tear, > 3 cm depth
 grade IV :
 haematoma: ruptured intraparenchymal with active bleeding
 laceration: parenchymal distruption involving 25 - 75% hepatic
lobes or
 1 - 3 Couinaud segments (within one lobe)
 grade V :
 laceration: parenchymal distruption involving >75% helpatic
lobe or
 > 3 Couinaud segments (within one lobe)
 vascular: juxtahepatic venous injuries (IVC, major hepatic
vein)
 grade VI : vascular: hepatic avulsion

Feedback on your answer


Collapse
12)
0/1
What is the most common abdominal organ injured in the setting of blunt abdominal
trauma?
(Select 1)(1pts)

Spleen

Liver

Kidneys

Small Intestines
The liver is the most commonly injured solid organ in blunt trauma, comprising 5% of all
trauma admissions, and because of its size is frequently involved in penetrating trauma.
Following blunt trauma, the most commonly injured structures are the parenchyma and
hepatic veins. Blunt forces dissipate along segments of the liver and along the fibrous
coverings of the portal triad structures; the hepatic veins, however, are not so insulated.
Given its size and location within the abdomen, the liver is also commonly involved in
penetrating trauma. Stab wounds typically result in direct linear tears, whereas gunshot
wounds or shotgun wounds result in significant cavitary injuries attributable to blast
effect and the "tumbling" of the missile within the liver parenchyma. Thus, arterial injury
is more common with penetrating trauma.
Over the past 20 years, nonoperative management (NOM) of liver injuries has evolved to
become the prevailing therapeutic strategy for blunt hepatic trauma. Several concurrent
changes resulted in this paradigm change. First was the realization that diagnostic
peritoneal lavage (DPL) was sensitive but not specific for identifying intraperitoneal
hemorrhage that necessitated operative management. Surgeons recognized that many
laparotomies undertaken for a positive DPL were associated with liver injuries that did
not require intervention for bleeding.1Second, trauma surgeons noted that nonbleeding
hepatic venous injuries, if manipulated at laparotomy, often resulted in more hemorrhage
and sometimes even death.2 Furthermore, it became conspicuous that with hemostasis
achieved in the operating room, recurrent postoperative bleeding was rare. Therefore,
surgeons queried whether hepatic venous injuries, which are low-pressure system
injuries, could heal without intervention. Finally, computed tomography (CT) provided a
reliable method for diagnosing and grading liver injuries.

The spleen is the second most commonly injured abdominal organ in blunt trauma
patients. Historical studies have reported a 10% mortality with all splenic injuries;
however, isolated splenic injury mortality is less than 1%. The mechanisms of injury are
similar to those seen with liver injuries: motor vehicle collisions, autopedestrian
accidents, and falls. Similar to penetrating trauma to the liver, stab wounds to the spleen
typically result in direct linear tears, whereas gunshot wounds result in significant
cavitary injuries.

Until the 1970s, splenectomy was considered mandatory for all splenic injuries.
Recognition of the immune function of the spleen refocused efforts on splenic salvage in
the 1980s.38,39Following success in pediatric patients, NOM of splenic injuries was
adopted in the adult population and has become the prevailing strategy for blunt splenic
trauma.

Duodenal and pancreatic injury continues to challenge the trauma surgeon. The relatively
rare occurrence of these injuries, the difficulty in making a timely diagnosis, and high
morbidity and mortality rates justify the anxiety these unforgiving injuries invoke.
Mortality rates for pancreatic trauma range from 9 to 34%, with a mean rate of 19%.
Duodenal injuries are similarly lethal, with mortality rates ranging from 6 to 25%.
Complications following duodenal or pancreatic injuries are alarmingly frequent,
occurring in 30 to 60% of patients.1-3 Recognized early, the operative treatment of most
duodenal and pancreatic injuries is straightforward, with low morbidity and mortality.
2014. Scientific American Surgery. Hamilton, Ontario & Philadelphia, PA. Decker
Intellectual Properties Inc. ISSN 2368-2744. STAT!Ref Online Electronic Medical
Library. http://online.statref.com/Document.aspx?fxId=61&docId=2329. 10/27/2014
1:39:15 PM CDT (UTC -05:00).

Feedback on your answer


Collapse
13)
0/1
Roughly what percentage of patients with blunt liver injury initially managed non-
operatively will go on to need surgical intervention?
(1pts)

<10%

25%

50%

75%

The answer is A, During the last century, the management of blunt force trauma to the
liver has changed from observation and expectant management in the early part of the
1900s to mainly operative intervention, to the current practice of selective operative and
nonoperative management. A 2008 study by Tinkoff et al.4 showed that 86.3% of hepatic
injuries are now managed without operative intervention. The current reported success
rate of nonoperative management of hepatic trauma ranges from 82% to 100%. Most
blunt liver trauma (80% in adults, 97% in children) patients are currently treated
conservatively. The success of non-operative management depends upon proper selection
of the patient. The patients, who are managed non-operatively, usually have grade I and
II liver injuries, hemoperitoneum less than 900 ml and blood transfusion of less than 3
units. The contraindications to non-operative management include refractory
hypotension, signinficant fall in haematocrit, the extravasations of intravenous contrast
agent, expanding haematoma and grade IV and V liver injury on CECT abdomen. The
patients of grade III liver injuries need very close observation as they may require
surgical intervention during first 24 hours. The failure rate of non-operative management
is not more than 5% inmost studies. It seems that patients with grade VI injuries rarely
reaches to the hospital alive and are not salvageable. Therefore, such injuries are usually
documented on autopsy. Mortality from blunt hepatic trauma is about 5% and is related
to uncontrolled hemorrhage.
Interventional radiology may be needed to perform an angiogram and embolization for
bleeding or to percutaneously drain an abscess or b iloma. An endoscopic retrograde
cholangiopancreatogram (ERCP) and stent placement may be required for biliary leak.
Even when such complications of the liver injury develop, only 15% require operative
intervention. Hepatic artery angiography with embolization is an important tool for the
stable patient with contrast extravasation who is being managed nonoperatively. It can
also be invaluable for the postoperative patient who has been stabilized by perihepatic
packing or who has rebled after an initial period of stability. Angioembolization has a
greater than 90% success rate in the control of bleeding with a low risk of rebleeding and
a reduction in required volume of transfusion. Read More
Feedback on your answer
Collapse
Unit 4
14)
0/1
A 30-year old white female presents to your office with right breast pain. She is currently
breast feeding her healthy 4-week old infant and has been having focal tenderness,
swelling and redness to her right breast near the nipple over the last 2 days. She reports
no past medical history and is a non smoker. Given the patient’s history and physical,
which of the following inflammatory disorders is the most likely diagnosis?
(Select 1)(1pts)

Acute mastitis

Granulomatous mastitis

Mammary duct ectasia

Benign severe breast engorgement

The answer is A. Lactational mastitis is a localized, painful inflammation of the breast


associated with fever and malaise that occurs in breastfeeding women and has been
estimated to occur in 2 to 10 percent of breastfeeding women Acute mastitis almost
always occurs during the first month of breastfeeding. The diagnosis of mastitis is made
clinically. Lactational mastitis typically presents as a hard, red, tender, swollen area of
one breast associated with fever >38.3ºC in a nursing mother. Other systemic complaints
may variably include myalgia, chills, malaise, and flu-like symptoms. In the early stages
of breast infection the presentation can be subtle with few clinical signs, while patients
with advanced infection may present with a large area of breast swelling with overlying
skin changes (eg, erythema). Reactive lymphadenopathy can also cause axillary pain and
swelling. Septic shock rarely occurs.In a lactating woman, severe engorgement can be
distinguished from mastitis because engorgement is bilateral with generalized
involvement [1]. Mastitis associated with erythema and edema during is uncommon and
a diagnosis of inflammatory breast cancer must be excluded The most common
organisms causing the infections are S. aureus and streptococcal species. It may also be a
complication of nipple piercing. “D” is incorrect because women often present with
inversion of the nipple and over 90% of women affected are smokers. Read more
Feedback on your answer
Collapse
15)
0/1
You are seeing a 15-year-old African-American female in your office because she and
her mom are concerned about a non tender breast lump that she just noticed the other day.
On exam you note a rubbery, well-defined, nontender breast mass approximately 2 cm in
diameter. The patient denies any history of breast tenderness, nipple discharge, or skin
changes. What is most likely diagnosis?

(Select 1)(1pts)

Fibrocystic breast disease

Fibroadenoma

Benign breast cyst

Cystosarcoma phyllodes

The answer is B, Fibroadenoma. Most breast masses in adolescent girls are benign.
Fibroadenoma is the most common, accounting for approximately two-thirds of all
adolescent breast masses. It is characterized by a slow growing, nontender, rubbery, well-
defined mass, most commonly located in the upper, outer quadrant. Size varies, and is
most commonly in the range of 2–3 cm. Fibrocystic disease is found in older adolescents
and is characterized by bilateral nodularity and cyclic tenderness. Benign breast cysts are
characterized by a spongy, tender mass with symptoms exacerbated by menses. Cysts are
frequently multiple, and spontaneous regression occurs in 50% of patients. Cystosarcoma
phyllodes is a rare tumor with malignant potential, although most are benign. It presents
as a firm, rubbery mass that may enlarge rapidly. Skin necrosis is usually associated with
the tumor. Intraductal papillomas are usually benign but do have malignant potential.
They are commonly subareolar and are associated with nipple discharge. These tumors
are rare in the adolescent population. Ref: Hay WH (ed): Current Pediatric Diagnosis and
Treatment, ed 16. McGraw-Hill, 2003, pp 122-123.

Feedback on your answer


Collapse
16)
1/1
You are seeing a 25-year old white female who is presenting to your clinic with bloody
nipple discharge over the last 3 days. She denies any history of a breast lump, she is not
breast feeding and denies family history of breast cancer. Her exam shows no palpable
masses but there is scant bloody discharge coming from her left nipple. You order a
mammogram that does not show any suspicious lesions. What is the most likely
diagnosis?
(Select 1)(1pts)

Fibroadenoma

Intraductal papilloma

Cystosarcoma Phyllodes

Mammary dysplasia

The answer is B, Intraductal papilloma. The old concern over cancer is the issue, and the
way to detect cancer that is not palpable is with a mammogram. That should be the first
choice. If negative, one may still wish to find an resect the intraductal papilloma to
provide symptomatic relief. Intraductal papillomas consist of a monotonous array of
papillary cells that grow from the wall of a cyst into its lumen. Although they are not
concerning in and of themselves, they can harbor areas of atypia or ductal carcinoma in
situ (DCIS). Papillomas can occur as solitary or multiple lesions. The standard approach
to a papilloma diagnosed by core needle biopsy (CNB) is to perform a surgical excision,
particularly if atypical cells are identified [14,16-21]. In a meta-analysis of 34 studies
that included 2236 non-malignant breast papillary lesions, 346 (15.7 percent) were
upgraded to malignancy following a surgical excision [21]. Because of a risk of
malignancy, these require surgical excision.Read More
Feedback on your answer
Collapse
17)
0/1
A 25 -ear obese black female presents to you with a painful breast lump along with
tenderness in both her breasts. She notes recurrent pain and multiple lumps on both
breasts that seem to “come and go” at different times during her menses. Now she has a
firm, round, mass that has not gone away for the last 5 weeks. What is the most likely
diagnosis?

(Select 1)(1pts)

Cystosarcoma Phyllodes

Intraductal papilloma

Fibrocystic disease
Fibroadenoma

The answer is C. This patient has fibrocystic disease which is a nonproliferative epithelial
lesions that are generally not associated with an increased risk of breast cancer [1]. It
should be noted that terms such as fibrocystic changes, fibrocystic disease, chronic cystic
mastitis, and mammary dysplasia refer to nonproliferative lesions and are not useful
clinically, as they encompass a heterogeneous group of diagnoses [5,11]. The most
common nonproliferative breast lesions are breast cysts. Other nonproliferative lesions
include papillary apocrine change, epithelial-related calcifications, and mild hyperplasia
of the usual type [5]. Apocrine metaplasia (also referred to as a "benign epithelial
alteration") is also a nonproliferative change that is secondary to some form of irritation,
typically associated with a breast cyst. Read More
Feedback on your answer
Collapse
Unit 5
18)
0/1
You are seeing a 43-year old Hispanic female who is presenting with an enlarging mass
in her right breast. It has been present for several years and growing to it’s present size.
On examination of her right breast you note a large 10 cm mass that is mobile, firm
and rubbery. There are no palpable axillary nodes. What is the most likely diagnosis?

(Select 1)(1pts)

Fibroadenoma

Fibrocystic disease

Intraductal papilloma.

Cystosarcoma Phyllodes

The answer is D. This is a Phyllodes tumors, which are an uncommon fibroepithelial


breast tumors that are capable of a diverse range of biologic behavior. In their least
aggressive form, they behave similarly to benign fibroadenomas, although with a
propensity to recur locally following excision without wide margins. At the other end of
the spectrum are tumors that metastasize distantly, sometimes degenerating histologically
into sarcomatous lesions that lack an epithelial component. Phyllodes tumors account for
fewer than 0.5 percent of all breast malignancies. The vast majority occur in women, in
whom the median age at presentation is 42 to 45 (range 10 to 82 years).and there was a
higher incidence in Latina whites, as compared to non-Latina whites, Asians, and African
American women. Read More
Feedback on your answer
Collapse
19)
0/1
You are seeing a 75-year old female in clinic because of an enlarging breast mass the
last two months. On examination of her right breast you palpate a firm 3cm non tender
firm non-mobile mass lateral to her right nipple. She denies past medical history and
takes no medication. What is the next best step?
(Select 1)(1pts)

Mammogram

Breast Ultrasound

Core Biopsy

Excisional Biopsy

If a palpable mass is appreciated, bilateral diagnostic mammography should be performed


prior to biopsy, even if the mass is clinically suspicious for cancer (algorithm
3 and algorithm 4) [27]. Fine needle aspiration or core needle biopsy may alter both
mammographic or US appearance. The goal of imaging in this setting is not to establish a
diagnosis of cancer but rather to identify other suspicious areas or calcifications in either
breast that might impact treatment. This evaluation should be performed prior to
percutaneous biopsy, so that biopsy of additional suspicious lesions can be undertaken at
one time. If the lesion is large and breast conservation is not an option, this is less
important. read more
Feedback on your answer
Collapse
20)
1/1
True or False: When approaching a patient with an abnormal mammogram for a
suspicious breast mass the next best step is a surgical excision is preferred over
percutaneous needle biopsy.
(1pts)

True

False Correct

False: Breast lesion suspicious for malignancy requires tissue biopsy.


Percutaneous needle biopsy is preferred over surgical excision in all
circumstances. Surgical excision as a diagnostic procedure is not a
justifiable alternative simply because of "patient choice" and should be
performed only when needle biopsy cannot be performed for specific
technical reasons, when a needle biopsy is either nondiagnostic, the
result is not concordant with the imaging findings (i.e., the needle
biopsy is benign, but the lesion is of high suspicion), or in highly select
other cases. Technical reasons that may preclude needle biopsy
include anatomic location of the lesion on mammography directly
opposed to the chest wall or in the far periphery of the breast so that
it cannot be visualized on stereotactic imaging devices. Breast Cancer
Feedback on your answer
Collapse
21)
0/1
Which of the following is considered a benign lesion that usually does not require cancer
treatment?
(Select 1)(1pts)

Lobular carcinoma in situ

Ductal carcinoma in situ

Medullary carcinoma

Mucinous carcinoma

The answer is A, Cancer that is confined to the lumen of the duct or


lobule of the breast and has not penetrated the basement membrane
is termed in situ cancer. This generally refers to ductal carcinoma in
situ (DCIS) but also encompasses a benign entity called lobular
carcinoma in situ (LCIS). Small uniform cells confined to the lobule of
the breast characterize LCIS. It is generally a clinically and
mammographically occult lesion that is identified only incidentally
when a biopsy is performed for calcifications or a mass that proves to
be some other benign lesion. LCIS is actually not cancer but rather is a
benign lesion and does not require cancer treatment. The primary
issue with LCIS is that it conveys an increased lifelong risk of
subsequent invasive cancer quantified at 0.5 to 0.75% per year. In
addition, when LCIS is identified on a core-needle biopsy, there is a 10
to 20% chance of DCIS or invasive cancer in the surrounding tissue;
therefore, surgical excision is warranted. Long-term follow-up shows
that the large majority of women with LCIS never develop invasive
breast cancer. Therefore, ablative surgical therapy and radiation for
LCIS are not necessary. Previously, LCIS was considered in and of
itself an indication to consider bilateral mastectomy. However,
mastectomy is generally not indicated in women with LCIS and should
be performed only in the context of risk reduction for those at very
high risk related to factors such as inherited susceptibility. Because
women with a diagnosis of LCIS are at increased risk for subsequent
invasive cancer, they should be counseled regarding that risk and may
benefit from consultation with genetics professionals if they have a
family history of breast or ovarian cancer. Women with a biopsy
showing LCIS may also consider risk-reducing chemoprevention with
one of the selective estrogen receptor modulators (SERM's), tamoxifen
or raloxifene. These reduce the risk of subsequent invasive cancer by
about 50%, with an acceptable toxicity profile.20,21 Raloxifene is the
preferred agent in postmenopausal women. Read more on Breast
Cancer
Feedback on your answer
Collapse
Unit 6
22)
0/1
Which of the following choices best differentiates cellulitis from erysipelas?
(Select 1)(1pts)

Presence of fever

Presence of leukocytosis

Depth of inflammation

Presence of purulence

All of the above

None of the above

The answer is C, depth of inflammation. Cellulitis and erysipelas are diffuse spreading
skin infections that not associated with underlying suppurative foci. Clinically, there is
often some degree of overlap between the two different entities. Erysipelas is
differentiated from cellulitis by the depth of inflammation; erysipelas affects the upper
dermis, including the superficial lymphatics, whereas cellulitis affects the deeper dermis
and subcutaneous fat (Mayo Clin Proc, Vol. 89;1436). Read More onCellulitis and
erysipelas from Up to Date
Feedback on your answer
Collapse
23)
1/1
What is the most common causative pathogen of Lemierre disease, an infection of the
parapharyngeal space that leads to septic thrombophlebitis of the internal jugular vein
with bacteremia and metastatic pulmonary nodules.
(Select 1)(1pts)

C. diphtheriae

Fusobacterium necrophorum

Infectious mononucleosis

Streptococcus pyogenes

The answer is B, Fusobacterium necrophorum is the most common causative pathogen of


Lemierre disease, an infection of the parapharyngeal space that leads to septic
thrombophlebitis of the internal jugular vein with bacteremia and metastatic pulmonary
nodules (12). Lemierre disease may be complicated by septicemia; suppurative
intracranial complications; and erosion of the carotid artery, which may be life-
threatening. Lemierre disease is often suspected based on clinical grounds in a toxic-
appearing patient; however, several recent European reports also suggest a possible
causative role of F. necrophorum in uncomplicated pharyngitis (13–15). Read More
on Pharyngitis
Feedback on your answer
Collapse
24)
0/1
You are seeing a 30-year-old male wrestler who is presenting with a painful lesion on his
upper back. He first noted a small painful area a few days ago, and the lesion has since
enlarged and became more red. He notes that others on his wrestling team have similar
lesions and have sought care elsewhere. He has no past medical history and takes no
medications.
Physical Examination of the upper back reveals a 1 × 1 cm erythematous, raised pustule
that is tender to palpation, with a 4 × 4 cm area of surrounding erythema. The remainder
of the physical examination is normal. You elect to perform an incision and drainage, and
culture. Of the following, which is the most appropriate empiric treatment?
(Select 1)(1pts)

Levofloxacin

Doxycycline

Dicloxacillin

Cephalexin
Ciprofloxacin

All of the above

None of the above

The answer is B, Purulence is more often present in staphylococcal cellulitis. Specimens


from wounds, carbuncles, or furuncles may reveal gram-positive cocci in clusters,
consistent with S. aureus. Clinicians must judge whether MRSA or MSSA is likely.
Appropriate choices for the oral treatment of MSSA include cloxacillin; dicloxacillin;
cephalexin; and, for penicillin-allergic patients, clindamycin or a macrolide
(erythromycin, clarithromycin, or azithromycin). However, patients treated with
cephalexin should be observed for treatment failure, as some studies have reported high
failure rates in adults (29), possibly due to poor absorption (30). Appropriate orally
administered choices for the treatment of MRSA include trimethoprim–
sulfamethoxazole, doxycycline, minocycline, or linezolid. This patient likely has a
community acquired MRSA as others on his wrestling team have similar lesions and thus
MRSA targeted therapy is best. Read More on Cellulitis and Soft-Tissue
Infections
Feedback on your answer
Collapse
25)
0/1
Which of the following statements regarding soft tissue infections is not true?
(Select 1)(1pts)

As many as 20% of necrotizing soft tissue infections are primary


(idiopathic) and occur in previously healthy patients who have no
predisposing factors and no known portal of entry for bacterial inoculation

Crepitus is noted in the majority of patients with necrotizing soft tissue


infections

Physical findings characteristic of a necrotizing infection include


tenderness beyond the area of erythema, crepitus and cutaneous
anesthesia

Fewer than 40% of patients with Patients with Necrotizing Soft Tissue
Infections exhibit the classic symptoms and signs described

The answer is B, crepitus is noted in only 30% of patients with necrotizing soft tissue
infections. Patients with necrotizing soft tissue infections often complain of severe pain
that is out of proportion to their physical findings. Compared with patients who have
nonnecrotizing infections, they are more likely to have fever, bullae, or blebs [see Figure
1]; signs of systemic toxicity; hyponatremia; and leukocytosis with a shift in immature
forms. Physical findings characteristic of a necrotizing infection include tenderness
beyond the area of erythema, crepitus, cutaneous anesthesia, and cellulitis that is
refractory to antibiotic therapy.6 Tenderness beyond the borders of the erythematous area
is an especially important clinical clue that develops as the infection in the deeper
cutaneous layers undermines the skin. Early in the course of a necrotizing soft tissue
infection, skin changes may be minimal despite extensive necrosis of the deeper
cutaneous layers. Bullae, blebs, cutaneous anesthesia, and skin necrosis occur as a result
of thrombosis of the nutrient vessels and destruction of the cutaneous nerves of the skin,
which typically occur late in the course of infection.
Clinicians should be mindful of certain diagnostic barriers that may delay recognition and
treatment of necrotizing soft tissue infections.7 In particular, these infections have a
variable clinical presentation. Although most patients present with an acute, rapidly
progressive illness and signs of systemic toxicity, a subset of patients may present with a
more indolent, slowly progressive infection. Patients with postoperative necrotizing
infections often have a more indolent course. Moreover, in the early stages, underlying
necrosis may be masked by normal-appearing overlying skin. As many as 20% of
necrotizing soft tissue infections are primary (idiopathic) and occur in previously healthy
patients who have no predisposing factors and no known portal of entry for bacterial
inoculation. Finally, crepitus is noted in only 30% of patients with necrotizing soft tissue
infections. Overall, fewer than 40% of patients exhibit the classic symptoms and signs
described.7,8Accordingly, it is imperative to maintain a high index of suspicion for this
disease in the appropriate setting.Read More on Soft Tissue Infection
Feedback on your answer
Collapse

Unit 1
1)
0/1
A 43-year old obese male presents with worsening pain around his umbilicus over the last
2 days. He the bulging goes away when he lies down but it has continued to be stuck out
and is severely painful. Which of the following signs is an indication for immediate
surgery?

(Select 1)( 1pts extra credit)

erythematous

warmth

edematous

non reducible

All of the above


The answer is E. all of the above. A hernia that is incarcerated, tender, erythematous,
warm, or edematous is an indication for immediate operation. Primary or incisional
hernias may not be palpable in obese patients, in which case, advanced imaging should be
performed. 2014. Scientific American Surgery. Hamilton, Ontario & Philadelphia, PA.
Decker Intellectual Properties Inc. ISSN 2368-2744. STAT!Ref Online Electronic
Medical Library. http://online.statref.com/Document.aspx?fxId=61&docId=823.
10/20/2014 12:18:15 PM CDT (UTC -05:00).

Feedback on your answer


Collapse
2)
0/1
Which type of colon polyp is most likely to become malignant?

(Select 1)( 1pts extra credit)

Hamartomatous polyp

Tubular adenoma

Villous adenoma

Tubulovillous adenoma

The answer is C, Villous adenoma. Adenomatous polyps are the most common of the
classically neoplastic polyps. About two-thirds of all colonic polyps are adenomas.
Adenomas are by definition dysplastic and thus have malignant potential. Most colorectal
cancers arise from adenomas, but only a small minority of adenomas progress to cancer
(5 percent or less). Studies reporting the average age at presentation of patients with
adenomatous polyps versus colorectal cancer suggest the time for development of
adenomas to cancer is about 7 to 10 years. Hamartomatous (or juvenile) polyps and
hyperplastic polyps are benign lesions and are not considered to be premalignant.
Adenomas, on the other hand, have the potential to become malignant. Sessile adenomas
and lesions >1.0 cm have a higher risk for becoming malignant. Of the three types of
adenomas (tubular, tubulovillous, and villous), villous adenomas are the most likely to
develop into an adenocarcinoma.
 An advanced adenoma is any adenoma with high-grade dysplasia, an
adenoma that is >10 mm in size, or an adenoma with a villous
component. Adenomas without villous components are also referred to
as tubular adenomas.
A synchronous adenoma is an adenoma that is diagnosed at the same
time as an index colorectal neoplasm (a pathologically more advanced
lesion). Thirty to fifty percent of colons with one adenoma will contain
at least one other synchronous adenoma .
 A metachronous adenoma is an adenoma that is diagnosed at least six
months after the diagnosis of a previous adenoma.
Ref: Fauci AS, Braunwald E, Kasper DL, et al (eds):Harrison’s Principles of
Internal Medicine, ed 17. McGraw-Hill, 2008, p 574. Read more on Approach to
the patient with colonic polyps.

Feedback on your answer


Collapse
3)
0/1
You are seeing a 64-year-old female comes in your office for chronic constipation.
She notes a history of chronic laxative use for most of her adult years. You schedule her
for a colonscopy and during the procedure you note that the anal and rectal mucosa
contain scattered areas of bluish-black discoloration. Which one of the following is the
most likely explanation for the sigmoidoscopic findings?

( 1pts extra credit)

Endometriosis

Collagenous colitis

Melanosis coli

Metastatic malignant melanoma

This patient has typical findings of melanosis coli, the term used to describe black or
brown discoloration of the mucosa of the colon. It results from the presence of dark
pigment in large mononuclear cells or macrophages in the lamina propria of the mucosa.
The coloration is usually most intense just inside the anal sphincter and is lighter higher
up in the sigmoid colon. The condition is thought to result from fecal stasis and the use of
anthracene cathartics such as cascara sagrada, senna, and danthron. Ectopic endometrial
tissue (endometriosis) most commonly involves the serosal layer of those parts of the
bowel adjacent to the uterus and fallopian tubes, particularly the rectosigmoid colon.
Collagenous colitis does not cause mucosal pigmentary changes. Melanoma rarely
metastasizes multicentrically to the bowel wall. Multiple arteriovenous malformations are
more common in the proximal bowel, and would not appear as described. Ref: Feldman
M, Friedman LS, Sleisenger MH (eds): Sleisenger & Fordtran’s Gastrointestinal and
Liver Disease, ed 7. WB Saunders Co, 2004, p 2305. 2) Hardman JG, Limbird LE,
Gilman AG (eds): Goodman & Gilman’s The Pharmacological Basis of Therapeutics, ed
10. McGraw-Hill, 2001, pp 1046-1047. 3) Kasper DL, Braunwald E, Fauci AS, et al
(eds): Harrison’s Principles of Internal Medicine, ed 16. McGraw-Hill, 2005, pp 231-233.
Feedback on your answer
Collapse
4)
0/1
What percentage of abdominal wall hernias are found in the groin?

(Select 1)( 1pts extra credit)

10%

25%

50%

75%

Approximately 75% of all abdominal wall hernias are seen in the groin. An Inguinal
hernia is much more common in men than women. Although femoral and umbilical
hernias are more common in female population, indirect inguinal hernia is still the most
common type of hernia in women. Age is a factor for incidence and type of inguinal
hernia; incidence increases by age 2 . Indirect hernia is more common in young and direct
hernia in the elderly. Read more

Feedback on your answer


Collapse
5)
1/1
You are seeing a 30-year old male in the trauma bay who was an unrestrained driver
involved in a rollover MVC who is complaining of lower abdominal pain. His GCS is
15, vitals are stable and he is awake and talking. The rest of his exam shows he has
moderate right lower abdomen and flank tenderness to palpation without guarding.
His iStat labs show a Hct 45.7, BUN 15, Cr 1.2. Upright CXR is shows no signs of
trauma and there is a normal sized medastinum. CT scan of his abd/pelvis demonstrates a
small renal subcapsular hematoma with 0.5 cm superficial parenchymal laceration that
does not involve the collecting system. According to the American Association for the
Surgery of Trauma (AAST), what grade injury does this demonstrate?

(Select 1)( 1pts extra credit)


Grade I

Grade II

Grade III

Grade IV

The patient above has a grade II renal laceration as he has a


<1cm renal laceration,Read more
Renal trauma grading is often done using theAmerican Association for
the Surgery of Trauma (AAST)3-4 according to depth of damage and
involvement of the urinary collecting system and renal vessels.
 grade I - contusion or non enlarging subcapsular haematoma, but
no laceration ;
 grade II - superfical laceration < 1cm depth and does not involve
the collecting system ; non expanding perirenal haematoma
 grade III - laceration > 1cm, without extension into the renal
pelvis or collecting system and with no evidence of urine extravasation
 grade IV - laceration extends to renal pelvis or urinary
extravasation.
 grade V - shattered kidney ; devascularisation of kidney due to hilar
injury.
Read more from Up to Date

Feedback on your answer


Collapse
Unit 2
6)
1/1
What % of patients with colorectal cancer (CRC) have metastatic disease at presentation?
(1pts)

<5%

10%

20%
30%

The answer is C, 20%. As many as 20% of CRC patients have metastatic disease at the
time of the initial presentation. The need for surgical intervention in this group is not well
defined. Clearly, surgical resection or diversion is indicated in patients who present with
significant bleeding, perforation, or obstruction. In asymptomatic patients with
unresectable metastatic disease, the role of surgical resection of the primary lesion
remains controversial. In patients with resectable metastatic disease (e.g., isolated liver or
lung metastases), curative resection may be undertaken.
A recent review of 233 patients with synchronous stage IV colorectal cancer found that
217 patients (93%) never required surgical palliation of the primary tumor; 16 patients
(7%) needed emergency surgery for obstruction or perforation of the primary tumor; 10
patients (4%) were managed nonoperatively.68,122
Management of patients with synchronous resectable isolated liver metastases continues
to evolve. Multiple studies have documented improved survival after liver resection in
patients with metastatic disease confined to the liver. Patients presenting with
synchronous lesions have a worse prognosis than those presenting with metachronous
lesions.123 Many of these patients have been managed with staged resections of their
primary cancers and the liver metastases. Several groups have reported that such
combined procedures do not substantially increase surgical morbidity and mortality or
compromise cancer survival.124,125 These combined procedures should be done only in
carefully selected patients, at specialized centers where there is significant experience in
resection of both CRCs and liver tumors. Read more

Feedback on your answer


Collapse
7)
0/1
What is the half life of the tumor marker CEA?
(Select 1)(1pts)

12-24 hours

1-2 days

3-5 days

7-14 days

The answer is D, 7-14 days. CEA remains the prototypical solid tumor marker. Despite
its lack of specificity, if used correctly, CEA testing is a valuable addition to the process
of clinical decision making in patients diagnosed with colon or rectal carcinoma.
However, it is not an appropriate screening test. Whether sampled once or serially, CEA
cannot be used in the differential diagnosis of an unknown-but-suspected bowel problem
or malignancy. Nevertheless, when CEA concentrations are determined before primary
tumor resection, they may be of additional prognostic value; this is particularly true in
patients with stage II disease, for whom elevated preoperative CEA is a poor prognostic
marker and may influence the decision regarding whether to administer adjuvant
chemotherapy.
Serial CEA values obtained postoperatively are a potentially effective means of
monitoring response to therapy. A postoperative CEA titer serves as a measure of the
completeness of tumor resection. It should be remembered, however, that the half-life of
CEA is 7 to 14 days; therefore, postoperative baselines are best established several weeks
after resection. If a preoperatively elevated CEA value does not fall to normal within 2 to
3 weeks after surgery, it is likely that (1) the resection was incomplete or (2) occult
metastases are present. A rising trend in serial CEA values from a normal postoperative
baseline (< 5 ng/mL) may predate any other clinical or laboratory evidence of recurrent
disease by 6 to 9 months. Read more

Feedback on your answer


Collapse
8)
1/1
What % of patients develop recurrent disease after curative resection of colon and rectal
carcinomas within 3 years?

(Select 1)(1pts)

20%

40%

60%

80%

The answer is D, 80%. Eighty percent of patients who recur after curative resection of
colon and rectal carcinomas do so within 3 years. Therefore, any posttreatment plan
should include regular follow-up during at least these 3 years. An additional biologic
precept in designing follow-up should take into account the efficacy of therapy once
recurrent disease is identified. It is important to note that the role of surveillance is to
identify recurrent disease that can be resected with true curative intent; early
identification of asymptomatic, incurable disease is exceedingly unlikely to improve
outcome as there are neither data nor a compelling rationale to believe that outcome
would be improved by earlier institution of noncurative treatment, such as systemic
chemotherapy.
In general, if a patient is a candidate for resection of recurrent disease (e.g., hepatic
resection), serum carcinoembryonic antigen (CEA) testing should be performed every 3
to 6 months for 2 years and then every 6 months for 5 years after resection of the primary
tumor. Chest, abdomen, and pelvis CT is recommended annually for 3 years for patients
at high risk for recurrence. PET-CT is not recommended in NCCN, ASCO, or CCO
guidelines and should not be used for the purpose of postoperative surveillance.
Colonoscopy should be performed 1 year after surgery or 3 to 6 months after surgery if
not performed preoperatively due to an obstructing lesion, and then 3 years later, and then
every 5 years, unless findings or specific risk factors dictate more frequent
evaluations. Read more

Feedback on your answer


Collapse
9)
0/1
You are seeing a 73-year old male who has a past medical history of
CHF, hypertension and type II diabetes. He initially presented to you
with changes in stool patterns, constipation and hematochezia. He was
found to have adenocarcinoma of the ascending colon along with
metastasis to his liver, brain and his right lung. For patient presenting
with metastatic colon cancer, which of the following treatments is not
routinely recommended?
(1pts)

Chemotherapy

Primary Resection of the colon cancer

Radiation

All are routinely recommended

All of the above

The answer is B. In treating Colorectal cancer, it is crucial to


understand that surgical extirpation of the primary tumor is done when
there is a realistic possibility of cure or for patients with symptomatic
tumors that cause acute obstruction or clinically significant bleeding.
For those who present with synchronous primary tumors and incurable
metastatic disease, resection is not routinely indicated. Advances in
systemic chemotherapy (outlined below) have significantly increased
the probability of managing the tumor medically, and chemotherapy
can begin immediately in the setting of an asymptomatic or minimally
symptomatic primary. In other words, there is no need to delay
initiation of systemic chemotherapy by palliative resection of a primary
tumor that is not actively symptomatic. In truth, resection of a primary
lesion in the setting of metastatic disease is likely to cause significant
morbidity and even mortality. A review of Medicare/SEER data
focusing on patients 65 and older reported a 30-day postoperative
mortality of 10% with resection of a synchronous primary tumor.
Furthermore, in a large retrospective series, Poultsides and colleagues
reported that 93% of patients presenting with synchronous stage IV
disease without overt obstruction never required specific intervention
on their primary tumor. Adenocarcinoma of the Colon and Rectum
Feedback on your answer
Collapse
Unit 3
10)
0/1

What is the estimated overall mortality from pulmonary contusions?

(Select 1)(1pts)

3% to 5%

5% to 10%

10% to 25%

25% to 40%

The answer is C, 10-25%. Pulmonary contusions, pneumothorax, and hemothorax occur


in 30% to 50% of patients with severe blunt chest trauma managed in trauma centers.
Pulmonary contusions commonly result from direct impact to the chest wall or from
concurrent chest wall injury, such as from a blast.20 Acute parenchymal lung injury
produces multiple physiological effects, all of which have an impact on outcomes. The
mortality associated with pulmonary contusion is difficult to predict, but is estimated to
be 10% to 25%.21,22 The clinical manifestations of pulmonary contusion are variable,
ranging from mild dyspnea to acute lung injury and acute respiratory distress syndrome.
The pathophysiology of pulmonary contusion includes alveolar hemorrhage and edema
resulting in decreased lung compliance, increased alveolar capillary permeability, and
increased intrapulmonary shunting. These processes result in hypoxemia, hypercarbia,
and decreased lung compliance. Lung contusions also result in acute local and systemic
inflammatory cascades that activate tissue macrophages and the production of
inflammatory mediators, cytokines, and chemokines. These result in pathophysiologic
changes that clinically manifest as immunosuppression, acute lung injury/acute
respiratory distress syndrome, and respiratory failure. Read more

Feedback on your answer


Collapse
11)
1/1
You are seeing a 27-year old male in the trauma bay shortly after being stabbed in this
back with a 6 cm buck knife by his disgruntled neighbor after an argument.
He arrived via EMS sitting up, awake and alert with a GCS of 15. His BP is 140/90, HR
120 and O2 sat is 85% on a non re-breather.
On physical exam there is a 3 cm non bleeding wound just left of his mid thoracic spine.
Upon auscultation there is decreased breath sounds on his left side. There are no signs of
tracheal deviation or JVD.
What is the next best step in management?
(1pts)

Immediately to the OR for surgical exploration

Upright Chest Xray followed by a chest tube

Immediate Needle thoracotomy on the left

Pericardiocentesis under US guidance

The answer is A, Upright Chest Xray followed by a chest tube. This patient is awake and
talking. He is stable enough for a chest xray followed by chest tube placement. A needle
thoracotomy to should be done if he has evidence of tension pneumothorax which would
be respiratory distress and hypotension. He will likely then need a chest tube and can be
further managed. Read More
Feedback on your answer
Collapse
12)
0/1
You are seeing a 20-year old female that was stabbed in the chest by an attacker 15
minutes prior to arrival. EMS noted she was initially awake and talking but has since
become less talkative and more lethargic. Vitals signs show her blood pressure is
60/palp, heart rate 120 beats/min, respiratory rate 28/min and O2 92% on a non re-
breather. On physical exam her eyes are closed but arouses to painful stimuli and loud
voice. She has distended neck veins and muffled heart sounds. Chest exam reveals a 2-cm
wound just to left of her sternum. What is the next best step in management?
(Select 1)(1pts)

Emergenct chest Tube on left

Pericardiocentesis

Upright chest xray

Emergent Ultrasound FAST exam

This young lady has been stabbed in the heart and has evidence of cardiac tamponade.
She needs an immediate percicardiocentesis. A chest tube will not help with this problem
and she is too unstable for an emegent echo or upright xray.Read more on the
initial evaluation of chest trauma

Feedback on your answer


Collapse
13)
0/1
A 65-year-old male comes in the emergency Department because of severe right-sided
chest pain following a fall off his ladder 10 feet high.
Examination shows decreased respiratory movements on the right side of the chest and
tenderness on palpation over the right mentions, his abdomen is soft and non tender.
An x-ray film of his chest shows a non displaced fracture of the right 6th and 7th ribs on
the right without signs of pneumothorax.
Which of the following is the most appropriate goal in management of the rib fracture in
this patient?
(1pts)

To achieve a tital volume of 500 ML with intubation

To ensure appropriate analgesia

To provide mechanical stabilization to the chest wall

To give prophylactic antibiotics


The answer is B. Once significant associated injuries have been evaluated and treated, the
cornerstone of rib fracture management is pain control [36-38]. Early and adequate pain
relief is essential to avoid complications from splinting and atelectasis, primarily
pneumonia. The choice of analgesia depends upon the injury, the clinician's comfort
performing nerve blocks with their potential complications, and the ease with which more
invasive treatments can be performed. Analgesia for severe and multiple rib fractures and
monitoring of admitted patients are discussed separately.
For isolated injuries (ie, single rib fracture), clinicians generally begin treatment with
nonsteroidal anti-inflammatory drugs (NSAIDs) with or without opioids. For more severe
injuries, particularly if ventilation is compromised, admission and invasive treatments,
such as intercostal nerve blocks, may be needed.
Respiratory care, including use of incentive spirometry to prevent atelectasis and its
complications, is often important. We do not recommend rib belts or binders because
they compromise respiratory function. Studies of rib belts involve small numbers of
patients and have reached contradictory conclusions [39,40]. Patients with a rib fracture
who are discharged home can also use incentive spirometry throughout the day, after
analgesics have taken effect. Holding a pillow or similar soft brace against the fracture
site reduces discomfort while using the spirometer, or when coughing.
Disposition — Several researchers recommend hospital admission for any patient with
three or more rib fractures, and ICU care for elderly patients with six or more rib
fractures [14,15]. They cite the significant correlation between these findings and
serious internal injuries, such as pneumothorax and pulmonary contusion. We suggest
hospitalization for the majority of patients with three or more rib fractures. Patients with
multiple rib fractures sustained from high-energy trauma are best evaluated at a trauma
center. Transfer should be arranged expeditiously.
Displaced rib fractures likely increase the risk of injury to the lung and proximate
intercostal blood vessels. Bleeding from such fractures can be delayed, and admission or
close observation and follow-up should be arranged for patients with displaced fractures,
depending upon clinical and social circumstances. Multiple case reports indicate that
delayed bleeding from intercostal vessels or other injuries can be life-threatening,
particularly in older patients [20,41,42].
Surgical fixation may be of benefit with some types of rib fractures, particularly those
associated with chest wall deformity, flail chest, or symptomatic nonunion. The
appropriate role of surgical fixation is discussed separately. (See "Inpatient
management of traumatic rib fractures", section on 'Surgical
management'.)
Rarely, younger healthy individuals with three rib fractures, having undergone a thorough
clinical and radiographic evaluation by clinicians experienced in trauma management,
and an appropriate period of observation (a minimum of six hours of observation,
including a follow-up chest radiograph, is needed to rule out pneumothorax), may be
discharged from the emergency department. Clinicians must also consider patient
comorbidities and clinical and social circumstances when determining disposition.
Patients with one or possibly two isolated rib fractures and no complicating factors may
be discharged home with appropriate follow-up and adequate analgesia. Uptodate
Feedback on your answer
Collapse
Unit 4
14)
0/1
A 10-year-old male is brought to the emergency department abdominal pain after he fell
riding his bicycle. His parents say that he wiped out on some sand and
hit his epigastric region into his handle bars and has since had repeated vomiting.
Vital signs show a blood pressure of 109/71, HR 99, RR 16, O2 on RA is 99%. On
physical exam he is awake and alert with a GCS of 15. He appears in moderate pain and
his abdomen is tender in his epigastrum with some voluntary guarding. The rest of his
exam is unremarkable. Barium examination shows duodenal obstruction. CT scan of the
abdomen shows a 3-cm duodenal hematoma with no other abnormalities. Which of the
following is the most appropriate next step in management?

(Select 1)(1pts)

Exploratory laparotomy

Nasogastric suction with parenteral nutrition

Bowel rest and antibiotics

Endoscopic removal of the hematoma

The answer is B, Nasogastric suction with parenteral nutrition. In the treatment of


duodenal injuries, one cannot forget the overall patient. Embarking on a complex repair
in the unstable, cold, acidotic coagulopathy patient is ill advised and will likely not end
well. "Damage control" principles of hemorrhage and contamination control should not
be forgotten in these often multiply injured patients. Once the timing is right, the surgical
treatment of duodenal injuries has two parts. One is the repair itself, and the other is to
assess the need to "protect" the repair or preparation for a potential leak and the resultant
fistula. The grade of the injury and the location within the duodenum often dictate the
type of repair, whereas the grade, location, and potentially other associated injuries,
especially the pancreas, often dictate the possible benefit of adjunctive "preparation"
maneuvers. First, however, we should discuss the possibility of nonoperative treatment of
select duodenal injuries.
Nonoperative treatment of select duodenal injuries has been well described and is more
common with blunt mechanisms as penetrating injuries are often diagnosed at
laparotomy. Endoscopic injuries are another set of duodenal trauma that may be managed
nonoperatively as they are commonly of low grade. In one recent multicenter review, all
success with nonoperative treatment was with grade I or II injuries only.18 In isolated
duodenal trauma, the indications for laparotomy are not different from those for other
types of abdominal trauma, hemodynamic compromise and peritonitis being the most
common. Thus, the indication for attempting nonoperative "treatment" of duodenal
injuries requires a hemodynamically normal patient without peritonitis or other indication
for immediate surgical intervention.
At times, duodenal hematomas can cause duodenal obstruction. This usually presents 2 to
3 days after the trauma with evidence of gastric outlet obstruction. The common
nonoperative treatment of isolated obstructing duodenal hematomas is nasogastric tube
decompression of the stomach and duodenum, nutritional support, and time.38 Repeat
imaging in 7 to 14 days is reasonable to evaluate improvement or an unexpected finding
as most duodenal hematomas will have resolved after 7 to 14 days, and continued
obstruction may reveal additional injury, prompting intervention.39-41 Enteral nutrition is
preferred to parenteral nutrition, but in the setting of a duodenal obstruction, the
parenteral route (total parenteral nutrition [TPN]) is often required as passing a feeding
tube beyond the obstruction can be difficult and surgical feeding access defeats the
purpose of "nonoperative" care.

Endoscopic injury to the duodenum deserves special mention here as it is a specific type
of duodenal trauma that may be commonly encountered by surgeons, even when trauma
does not make up a large part of their practice. There is literature that reports many of
these injuries being treated nonoperatively.42This is consistent with similar nonoperative
management of other isolated grade I or II injuries. However, as with any other type of
trauma, the clinical status of the patient and standard indications for laparotomy should
dictate the need for surgery.43 CT has been reported as the imaging test of choice to
diagnose perforation from endoscopy if immediate indications for laparotomy do not
exist.44 Often the pressure can be high to pursue a nonoperative approach as surgery was
likely not on the patient's or endoscopist's schedule, but clinical parameters should dictate
the appropriate course of action. Again, endoscopic trauma to the duodenum should
follow the same principles of evaluation and treatment as other mechanisms and the
patient course dictate treatment.

As noted above, operative treatment of duodenal injuries has two major components: the
repair and evaluating the need for leak "preparation." The repair itself must follow the
basic principles of all anastomosis: approximation of well-vascularized, healthy, tension-
free tissue. "Preparation" maneuvers are not required for most injuries and are debated.
They are discussed below.

Types of Duodenal Repairs

If a duodenal hematoma is discovered at operation, there is some debate as to the optimal


management. Simple observation for most hematomas is advocated by some. Other
authors have argued for seromuscular incision and decompression to ensure no duodenal
obstruction or delayed rupture. A selective approach seems to be a common compromise
depending on the findings at the time of operation, specifically the size of the hematoma.
For lacerations, the location and extent of tissue damage or loss are critical in deciding
the best type of repair. As noted in the literature, simple approximation is the most
common repair selected.9,45-48 This is commonly described as being done in a running
continuous fashion with monofilament suture after the edges have been debrided of any
questionable tissue. This repair choice mandates that the basic principles of bowel
anastomosis be met: approximation of tension-free, healthy, vascularized tissue.
Adequate duodenal mobilization is often required to decrease tension and should have
been done already for appropriate diagnosis.

If any of the basic principles cannot be met with simple approximation or if the resultant
repair would narrow the lumen of the duodenum, other options are available for repair.
The next option considered is often resection of the involved portion of duodenum and
primary anastomosis.48 Again, for this to have a maximal chance of success, the
anastomosis must be between well-vascularized, healthy, tension-free tissues. Another
option is a Roux-en-Y duodenojejunostomy. This is more likely to be necessary with D2
injuries not amenable to simple approximation. The pancreaticoduodenectomy with
pancreaticojejunostomy, gastrojejunostomy, and hepaticojejunostomy reconstruction
(Whipple procedure) is rarely needed in duodenal trauma and is discussed below.
Randomized comparison of one repair versus another is absent, but most reviews
consider simple approximation to be the most commonly required and most commonly
used repair.

2014. Scientific American Surgery. Hamilton, Ontario & Philadelphia, PA. Decker
Intellectual Properties Inc. ISSN 2368-2744. STAT!Ref Online Electronic Medical
Library. http://online.statref.com/Document.aspx?fxId=61&docId=2329. 10/17/2014
5:10:16 AM CDT (UTC -05:00).

Feedback on your answer


Collapse
15)
0/1
A 32-year-old man is brought to the emergency department after he drove his motorcycle
into a guardrail at a high speed and was riding without a helmet. On arrival, his pulse is
100/min, respirations are 14/min, and blood pressure is 120/80 mm Hg.
Physical examination shows a GCS of 14, on abdominal exam there is mild suprapubic
tenderness. GU exam shows a deep perineal laceration. He voids 25 mL of bloody urine.
An x-ray of the pelvis shows a widened pubic symphysis. Which of the following is the
most appropriate next step in diagnosis?
(Select 1)(1pts)

Urethrography (retrograde urethrogram)

Cystography
Placement of a urinary catheter

Intravenous pyelography

The answer is A. This injury often follow pelvic fractures and classically present with
blood at the urethral meatus, an ability to void and a high riding or a non palpable
prostate to an intern. If you suspect urethral injury a retrograde urethrogram needs to be
performed prior to foley insertion. This procedure will locate the damage of the urethra, if
present. Inserting a Foley before this procedure is contraindicated as this can worsen the
urethral tear and potentially cause infection or a hematoma (you do not want to cause or
worsen the chance of a urethral stricture). These injuries often need surgical repair,
especially anterior urethral injuries. Some are treated with urinary diversion via
suprapubic catheter while the primary injury heals. Read more
Feedback on your answer
Collapse
16)
0/1
You are seeing a 25-year old male in the trauma bay after he was involved in a motor
vehicle accident. He was thrown from his bike and is now complaining of right flank
pain. His GCS is 15 and he is awake and oriented. His BP is 125/75, HR 99, RR 18, O2
98% on room air. His physical exam is pertinent for right flank ecchymosis with mild
tenderness to palpation along with mild right upper quadrant tenderness. There is no
gross blood seen at his urethral meatus and his urinalysis shows no RBCs. True or False:
In the setting of potential renal trauma (blunt and penetrating trauma), gross or
microscopic hematuria is ALWAYS present.
(1pts)

True Incorrect

False

The answer is False: In the setting of renal trauma, gross or microscopic hematuria is
absent in up to 5% of cases and this finding alone should not be used to preclude in those
you are suspicious of renal trauma. Read more on renal trauma
Feedback on your answer
Collapse
17)
0/1
You are seeing a 32-year old male who presents after being kicked multiple times in his
right flank. He is complaining of severe flank pain and gross hematuria. What is the gold
standard for imaging of renal trauma?
(Select 1)(1pts)

MRI abdomen/pelvis

Ultrasound
CT abdomen/pelvis with contrast

Intravenous pyelography

None of the above

The answer is C, CT abdomen/pelvis with contrast. CT scanning with intravenous


contrast enhancement is the modality of choice for the identification and staging of renal
trauma in the hemodynamically stable patient (image 6). The initial CT images will
frequently miss injuries to the renal pelvis and ureters as sufficient contrast may not yet
be present in the collecting system. Additional delayed images are needed to assess for
contrast extravasation when these injuries are suspected (image 7). In patients
undergoing CT scanning of other organ systems, suspected bladder injuries may be
investigated with CT cystography after retrograde filling of the bladder, as described
above. Read more
 CT with IV contrast is the Gold standard, high sensitivity
 Immediate and delayed post-contrast images to view collecting
system
 Images abdomen and retroperitoneum
 Allows diagnosis and staging
 Intravenous pyelography doesnt allow you to image the abdomen or
retroperitoneum.

Feedback on your answer


Collapse
Unit 5
18)
0/1
A 24-year old male basketball player presents to you with a painful bulge into his right
inguinal area into his scrotum that he first noticed while he was playing basketball. He
asks if it could be a sports hernia and if this will limit his practicing and upcoming
tournament. What hernia does this patient most likely have?
(Select 1)(1pts)

Indirect

Direct

Femoral

Richter
This patient most likely has an indirect hernia as it is the most common hernia and given
the lack of other findings in the stem that is the best answer and most likely. Although a
sports hernia may lead to a traditional, abdominal hernia, it is a different injury. A sports
hernia is a strain or tear of any soft tissue (muscle, tendon, ligament) in the lower
abdomen or groin area. Because different tissues may be affected and a traditional hernia
may not exist, the medical community prefers the term "athletic pubalgia" to refer to this
type of injury.
An indirect inguinal hernia is one of the most common abdominal hernias. It is
five times more common than a direct inguinal hernia, and is seven times more frequent
in males, due to persistence of the process vaginalis during testicular descent. An indirect
hernia enters the inguinal canal at the deep ring, lateral to the inferior epigastric vessels.
It passes infromedially to emerge via the superficial ring and, if large enough, extend into
the scrotum. In children, the vast majority of inguinal hernias are indirect (see Case 3).
Incarceration represents the most common complication associated with inguinal hernias,
the incidence could be as high as 30% for infants younger than 2 months.
A direct inguinal hernia arises from protrusion of abdominal viscera through a
weakness of the posterior wall of the inguinal canal medial to the inferior epigastric
vessels, specifically through the Hasselbach's triangle. This type of hernia is termed
direct as the hernial sac directly protrudes through the inguinal wall in contrast to indirect
ones which arise through the deep ring and enter the inguinal canal. Since direct hernias
do not have a guiding path, they seldom extend into the scrotum unless very large and
chronic. Direct hernias arise usually as acquired weakness of the Hasselbalch's triangle.
Therefore, they are seen in the elderly with chronic conditions which increase intra-
abdominal pressure over a long period, e.g. COPD, bladder outflow obstruction, chronic
constipation etc. Increased abdominal pressure is transmitted to both sides and as a result,
direct hernias are usually bilateral. Compared to indirect hernia, they are less susceptible
to strangulation as they have a wide neck.
In contrast to the indirect hernia, a direct hernia is most often an acquired lesion. It
occurs when a weak spot develops in the lower abdominal musculature (the posterior
floor of the inguinal canal) due to the normal and/or abnormal stresses inflicted by living
and aging. In adults, stresses such as lifting heavy of objects, frequent coughing or
straining, pregnancy, and constipation can instigate hernia. Unlike indirect hernias, direct
hernias traverse medial to the inferior epigastric vessels and are not associated with the
processus vaginalis. The hernia consists primarily of retroperitoneal fat. Only rarely is a
peritoneal sac containing bowel encountered. Because there is typically no involvement
of a sac, they do not protrude with the spermatic cord, and as such, have a lower
incidence of incarceration or strangulation. Like indirect inguinal hernia, direct inguinal
hernias typically cause a bulge in the groin (at the top of or within the scrotum) and
usually with increased abdominal pressure. Like indirect hernias, they may or may not be
painful (usually not). By palpating the inguinal canal and asking the patient to cough
while standing, one can usually elicit the hernia. In fact, one can often times palpate an
inguinal hernia without invaginating the scrotum (as is typically taught in medical
school). Rather, by placing one's fingers over the inguinal canal and asking the patient to
cough, one can often feel the bulge against the lower abdominal wall. As direct and
indirect hernias are unreliably differentiated by physical exam alone, the need to
invaginate the scrotum to feel into the inguinal canal is often more uncomfortable to the
patient, than telling to the physician. Rarely, palpation is not even necessary, as the
hernia is large enough to be visualized. Read more on Inguinal hernias: A Brief
review

Feedback on your answer


Collapse
19)
0/1
What % of minimally symptomatic inguinal hernias end up requiring emergency
operation for strangulation over a one year period?

(Select 1)(1pts)

0.18%

1.18%

2.1%

6.5%

12.4%

The answer is A, The natural history of an untreated, minimally


symptomatic inguinal hernia was addressed in a randomized,
controlled trial in which 364 men were assigned to "watchful waiting"
(WW) and 356 men underwent routine operation.48 Only two patients
in the WW group required emergency operations for strangulation over the
follow-up period of 2 to 4.5 years. This result translated into a rate of 1.8 per 1,000
patient-years (0.18%), or about one fifth of 1% for each year that the hernia remains
unrepaired. The two patients who required emergency operations recovered uneventfully.
The question that remained to be answered was which group fared better overall: the WW
group or the group whose hernias were repaired immediately in accordance with
conventional teaching? The answer to this question was at variance with conventional
assumptions. At the conclusion of the study, functional status, as measured by quality of
life instruments and pain scales, was identical in the two groups. About one third of the
patients in the WW group crossed over to undergo operative treatment, principally
because of symptom progression. However, there appeared to be no penalty for delaying
surgery. Before this study, most surgeons assumed that a hernia would become harder to
repair the longer it remained (because of enlargement and buildup of scar tissue) and that
patients whose operations were delayed would experience more complications. The
investigators found, however, that postoperative complication rates were the same in
patients who underwent immediate surgery as in those who were assigned to WW but had
to cross over to surgical treatment. Read more

Feedback on your answer


Collapse
20)
0/1
Which of the following statements regarding hernias is not correct?
(Select 1)(1pts)

In female patients, indirect inguinal hernias are the most common type

The lifetime risk of having to undergo an inguinal hernia repair is greater


for men than in women

In male patients, direct inguinal hernias are the most common type

Femoral hernias account for fewer than 10% of all groin hernias; however,
40% present as emergencie

Answer C is the only incorrect statement. In the United States, approximately


1,000,000 abdominal wall herniorrhaphies are performed each year, of
which almost 80% are for inguinal or femoral hernias.1Worldwide,
some 20 million groin hernias are repaired each year.2 The lifetime
risk of having to undergo an inguinal hernia repair is 27% for men and
3% for women.3 In male patients, indirect inguinal hernias are the
most common type (more often located on the right), occurring
approximately twice as frequently as direct inguinal hernias; femoral
hernias account for a much smaller percentage. In female patients,
indirect inguinal hernias are also the most common type, but femoral
hernias are seen more frequently than direct hernias, which are rare in
this population. Femoral hernias account for fewer than 10% of all
groin hernias; however, 40% present as emergencies (i.e., with
incarceration or strangulation), and mortality is higher for emergency
repair than for elective repair. A two-peak theory has been described,
stating that a new diagnosis of an inguinal hernia is most likely in
patients younger than 1 year and in patients older than 55 years,
although hernias can be diagnosed across any given age group. Read
more
Feedback on your answer
Collapse
21)
0/1
According to a large, randomized, multicenter Veterans Affairs (VA) study comparing
laparoscopic or open approach for the treatment of inguinal hernias, what was the overall
recurrence rate at 2 years for laparoscopic vs open mesh repair respectively?

(Select 1)(1pts)

10%/4%

20%/10%

4%/10%

10%/20%

The answer is A, 10%/4%. An analysis of nearly 18,000 herniorrhaphies in


Sweden reported that 15% of operations were performed to treat
recurrent hernias.46 This figure is remarkably consistent with the data
from other large population-based series and is influenced by the type
of repair, type of hernia (primary versus recurrent), patient
characteristics, and surgeon characteristics (hernia specialist or not).
The use of mesh is an important factor, with a Cochrane meta-analysis
of open mesh inguinal hernia repair versus open nonmesh repair
finding that tension-free mesh repair led to a significant reduction in
hernia recurrence of between 50 and 75%.19 Two randomized trials
using the Lichtenstein repair as the control operation documented 2-
year recurrence rates between 1 and 4%, setting a benchmark for
primary inguinal hernia repair.47,48
Similar low recurrence rates have been demonstrated regardless of the
technique of mesh placement, whether laparoscopic or open.49,50 On
the other hand, a large, randomized, multicenter Veterans Affairs (VA)
study found that for primary, unilateral hernias, the laparoscopic
approach was associated with a higher overall recurrence rate at 2
years (10%) when compared with open mesh repair (4%).48 Most
reported recurrences after laparoscopic
herniorrhaphy come at an early stage in the surgeon's experience with
these procedures and arise soon after operation.48,51 The majority
can be attributed to (1) inadequate preperitoneal dissection; (2) use of
an inadequately sized patch, which may migrate or fail to support the
entire inguinal area, including direct, indirect, and femoral spaces; or
(3) staple failure with TAPP repair. Read more
Feedback on your answer
Collapse
Unit 6
22)
1/1
What is the mean mortality rate of patients suffering from pancreatic trauma?
(Select 1)(1pts)

<3%

9%

19%

31%

The answer is C, 19%. Duodenal and pancreatic injury continues to


challenge the trauma surgeon. The relatively rare occurrence of these
injuries, the difficulty in making a timely diagnosis, and high morbidity
and mortality rates justify the anxiety these unforgiving injuries
invoke. Mortality rates for pancreatic trauma range from 9 to 34%,
with a mean rate of 19%. Duodenal injuries are similarly lethal, with
mortality rates ranging from 6 to 25%. Complications following
duodenal or pancreatic injuries are alarmingly frequent, occurring in
30 to 60% of patients.1-3 Recognized early, the operative treatment
of most duodenal and pancreatic injuries is straightforward, with low
morbidity and mortality. Read more
Feedback on your answer
Collapse
23)
0/1
Which of the following statements regarding hollow viscus injuries in patients suffering
from blunt trauma is not true?
(Select 1)(1pts)

Mortality is increased in parallel with time to operative intervention

Only 1.2% of blunt trauma admissions had an associated hollow viscus


injury
The "seat-belt" sign is associated with a more than doubled relative risk of
small bowel injury

Unexplained intraperitoneal fluid (i.e., fluid appearing in the absence of


solid-organ injury) was the least common radiographic finding associated
with blunt bowel or mesenteric injury

Answer D is false. Unexplained intraperitoneal fluid (i.e., fluid appearing in the absence
of solid-organ injury) was the most common radiographic finding associated with blunt
bowel or mesenteric injury but often proved to be a false positive finding.
Hollow viscus injury after blunt trauma, although uncommon, can have serious
consequences if the diagnosis is missed or delayed. In a multi-institutional study of 198
patients with blunt small bowel injury, delay of as little as 8 hours in making the
diagnosis resulted in increased morbidity and mortality.2 Mortality increased in parallel
with time to operative intervention (< 8 hours to operation, 2% mortality; 8 to 16 hours,
9%; 16 to 25 hours, 17%; > 24 hours, 31%), as did the complication rate. Consequently,
it is important to have an expedient approach to the diagnosis of blunt bowel injury.
Physical examination findings such as abdominal tenderness or tachycardia may suggest
the presence of hollow viscus injury. Distracting chest or long bone injury, closed-head
injury, spinal cord injury, or intoxication, however, may compromise reliability of the
examination. In addition, it is not uncommon for blunt bowel injury to have a latent
period from the time of injury, whereby the expected signs and symptoms of such injuries
take some time to develop. Laboratory abnormalities, including elevations in white blood
cell (WBC) count, amylase, and/or lactic acid, may also point toward the presence of
hollow viscus injury but are relatively nonspecific. Provided that the patient has suffered
a low-risk mechanism of injury (such as a fall from standing or a low-speed motor
vehicle collision), hollow viscus injury is extremely unlikely in the face of a normal,
reliable physical examination and normal laboratory results. With these conditions
present, the presence of blunt bowel injury can be effectively excluded. However, the
presence of abdominal complaints, an abnormal or unreliable physical examination,
abnormal laboratory results, or a high-risk mechanism of injury (such as a high-speed
motor vehicle collision) warrants further evaluation by imaging for the presence of bowel
injury.
Particular consideration should be given to lap-and shoulder-restraint injuries, which may
be associated with an increased risk of hollow viscus injury. The "seat-belt" sign (i.e.,
ecchymosis of the abdominal wall secondary to the compressive force of the lap belt) is
associated with a more than doubled relative risk of small bowel injury.3,4 Flexion-
distraction fractures of the spine (Chance fractures) are also associated with lap-belt use,
and the presence of such fractures should raise the index of suspicion for associated
hollow viscus injury.
Ultrasonography is routinely performed early in the evaluation of blunt abdominal
trauma. It is highly specific and moderately sensitive in identifying intra-abdominal fluid,
the presence of which in a hemodynamically unstable patient is an indication for
laparotomy (in that it strongly suggests the presence of significant intra-abdominal
hemorrhage).5 Ultrasonography does not, however, reliably distinguish solid-organ
injury from hollow viscus injury—a distinction that is critical for determining subsequent
management (i.e., operative versus nonoperative) in a hemodynamically stable patient.
Computed tomography (CT) is the imaging modality of choice in stable patients who
warrant evaluation by imaging as described above. We reviewed over 8,000 CT scans
performed to evaluate cases of blunt abdominal trauma and found that the number of
abnormal radiologic findings suggesting blunt injury to the bowel, the mesentery, or both
was correlated with the true presence of injury [see Table 1].6 A CT scan
demonstrating a solitary abnormality was associated with a true positive rate of 36%,
whereas a scan demonstrating more than one abnormality was associated with a true
positive rate of 83%. Unexplained intraperitoneal fluid (i.e., fluid appearing in the
absence of solid-organ injury) was the most common radiographic finding associated
with blunt bowel or mesenteric injury but often proved to be a false positive finding. On
the basis of this experience, we developed an algorithm for the evaluation of blunt hollow
viscus injury in patients with unreliable physical examinations [see Figure 1].
Most CT scans performed in this clinical setting, however, will be negative for evidence
of intra-abdominal injury. A prospective multi-institutional trial involving 3,822 blunt
trauma patients demonstrated that the negative predictive value of a normal abdominal
CT scan was 99.63%, leading the authors to conclude that patients with a normal scan do
not benefit from hospital admission and prolonged observation.7 However, a multi-
institutional review of 2,457 cases carried out by the Eastern Association for the Surgery
of Trauma (EAST) reported a 13% incidence of blunt small bowel injury in patients with
an initial negative CT scan. These results indicate that caution should be exercised in
dismissing the presence of hollow viscus injury on the basis of a negative scan.3 This
concern is echoed by our own institutional experience, in which the incidence of injury in
patients with an initial negative CT scan was 12%.6 If CT scanning demonstrates no
suspicious findings, no further diagnostic workup of hollow viscus injury is necessary,
but the duration of the observation period depends on both the overall condition of the
patient and clinical judgment. Most patients, as supported by the negative predictive
value of the study above, will not require ongoing further observation.
Feedback on your answer
Collapse
24)
0/1
Which of the following statements about duodenal or pancreatic injury is not true?
(Select 1)(1pts)

CT scan is the primary diagnostic modality

A bicycle handlebar to the epigastrium is the most common cause of injury

An elevated amylase level is a very sensitive but non specific finding


Duodenal hematomas causing obstruction usually present 2-3 after initial
injury

Answer C is the only false statement as No laboratory findings are particularly specific
for duodenal or pancreatic injury. Amylase has been proposed for both pancreas and
duodenal injuries but is not specific for either one. Some have noted that amylase can be
elevated in as many as 50% of duodenal injuries, but this has not been a consistent
finding.9 An elevated amylase should prompt an evaluation of the duodenum for injury
but alone is not diagnostic. Although the highest concentration of amylase in the human
body is in the pancreas, hyperamylasemia is also not a reliable indicator of pancreatic
trauma. In one series, only 8% of blunt abdominal injuries with hyperamylasemia had
pancreatic injury.10 Furthermore, as many as 40% of patients with a pancreatic injury
may initially have a normal serum amylase.
The history of a patient with a possible blunt pancreatic or duodenal injury usually
consists of a direct blow to the epigastrium. In children, this commonly involves a bicycle
handlebar to the epigastrium; in adults, more commonly the steering wheel or a motor
cycle handle bar is involved. However, any direct blow should raise suspicion. The
patient may complain of abdominal, back, or flank pain.
Outside of the pelvis x-ray, plain abdominal radiographs to evaluate blunt abdominal
trauma are less common in the era of focused abdominal sonography for trauma (FAST)
and computed tomographic (CT) scans. Reports suggest that signs of duodenal injury on
plain radiographs are identified less than one third of the time. Retroperitoneal air, free
intraperitoneal air, or obliteration of the psoas shadow should raise suspicion for
duodenal and other hollow viscous trauma.9 Upper gastrointestinal series have also been
used to evaluate the duodenum for injury and can add to the sensitivity and specificity of
plain films, but more recently, CT has become the primary diagnostic modality.
For optimal duodenal evaluation, intraluminal contrast administered via a nasogastric
tube soon but not immediately prior to the CT scan may aid by opacification of the lumen
of the duodenal "C loop." Visualization of contrast extravasation, retroperitoneal air,
adjacent fat stranding, and unexplained fluid, as well as duodenal wall thickening, are CT
findings suggestive of potential duodenal injury. The sensitivity of CT for duodenal
injury is related to the technology of the scanner (i.e., "number of slices") and the time
interval from the injury to imaging. In a recent review, CT was considered to have an
overall sensitivity of around 76% with new-generation scanners (16- or 64-slice), having
a higher sensitivity of around 82%. Additionally, if clinical suspicion remains high after
an initial negative CT scan, then repeat imaging is warranted and may improve diagnostic
yield.
At times, duodenal hematomas can cause duodenal obstruction. This usually presents 2 to
3 days after the trauma with evidence of gastric outlet obstruction. The common
nonoperative treatment of isolated obstructing duodenal hematomas is nasogastric tube
decompression of the stomach and duodenum, nutritional support, and time.38 Repeat
imaging in 7 to 14 days is reasonable to evaluate improvement or an unexpected finding
as most duodenal hematomas will have resolved after 7 to 14 days, and continued
obstruction may reveal additional injury, prompting intervention.39-41 Enteral nutrition
is preferred to parenteral nutrition, but in the setting of a duodenal obstruction, the
parenteral route (total parenteral nutrition [TPN]) is often required as passing a feeding
tube beyond the obstruction can be difficult and surgical feeding access defeats the
purpose of "nonoperative" care.
Feedback on your answer
Collapse
25)
0/1
What percentage of patients suffering from blunt abdominal trauma develop hollow
viscus injury?
(Select 1)(1pts)

1.2%

4.8%

8.9%

13.4%

The answer is A, 1.2%. Hollow viscus injury after blunt trauma, although
uncommon, can have serious consequences if the diagnosis is missed
or delayed.Hollow viscus injury is most often the consequence of
penetrating abdominal trauma. As a result of blunt force trauma,
bowel injury occurs with relative infrequency: in one multi-institutional
analysis, only 1.2% of blunt trauma admissions had an associated
hollow viscus injury. Read more
Feedback on your answer
Collapse

You might also like